You are on page 1of 756

Collections Of

General Surgery MCQS


2018
For undergraduate surgical training students

Prepared by:
Naser Radwan,MD
FEBS,PMCCS.
Consultant of general surgery
Head of surgical departments Indonesia Hospital,
Ass. Prof Al Azhar Univ.
Surgery MCQS
1. These ABGs (pH-7.51, pCO2-28, HCO3-26) show

A. Uncompensated respiratory acidosis


B. Partially compensated metabolic acidosis
C. Normal acid-base balance
D. Uncompensated respiratory alkalosis
E. A combined metabolic and respiratory alkalosis

2. These ABGs (pH-7.13, PCO2-36, HCO3-14) are interpreted as

A. Partially compensated respiratory acidosis


B. Uncompensated metabolic acidosis
C. Combined respiratory and metabolic acidosis
D. Uncompensated metabolic alkalosis
E. Normal acid-base balance
3. A patient has the following ABGs :PH -7.43, PCO2-28 mmHg, and HCO3-18mEq/L
This is interpreted as
A. Partially compensated respiratory alkalosis
B. fully compensated metabolic acidosis
C. Partially compensated respiratory acidosis
D. Fully compensated respiratory alkalosis

4.A patient has the following ABGs :PH -7.50, PCO2-49 mmHg, and HCO3-30mEq/L
This is interpreted as
A. Fully compensated respiratory acidosis
B. partially compensated metabolic alkalosis
C. partially compensated metabolic acidosis
D. fully compensated metabolic alkalosis

5. Indications for nutritional support include All the following except


A. Severe anorexia
B. Moderate or severe malnutrition but unable to eat sufficient orally.
C. Pre-op patient with wt loss ≥ 10% BW
D. Unable to eat or swallow.
E. Oral diet not anticipated for ≥ 5 days

6. Contraindications to enteral nutrition include All the following except

A. Low output intestinal fistula


B. Intractable vomiting
C. Intractable diarrhea
D. Severe malabsorption
E. Ischemic intestine

7. ALL are Contraindications to PEG tube except

A. Coagulopathy
B. Neuro disorders
C. Peritonitis
D. ileus
E. Anorexia nervosa

8. indications for parenteral nutrition include All the following except

A. Short bowel
B. Loss of 5% of BW in 4 months
C. Prolonged ileus
D. Pancreatitis
E. Sepsis

9. All the following are causes of metabolic acidosis except

A. Shock
B. Diabetes
C. small bowel fistula
D. pancreatic fistula
E. Repeated vomiting

10. Chloride resistance metabolic alkalosis include all the following except

A. Loss of acid from GIT


B. Excessive alkali Administration
C. Massive blood transfusion
D. Severe K depletion
E. Mineralocorticoid excess

11. Regarding 1st Degree Burns , All are true except

A. Superficial burns involving epidermis


B. Local pain and erythema
C. Multiple blister formation
D. Spontaneous healing without treatment
E. Skin pealing may occur
12. Major Severe Burns

A. Second degree burns of greater than 25% BSA in adults or greater than 20%
in children or elderly
B. Second-degree burns of less than 15% BSA in adults or 10% BSA in children &
elderly
C. Second-degree burns of 15%-25% BSA in adults or 10%-20% BSA in children or
elderly
D. Less than 10% BSA full-thickness burns
E. More than 10% BSA full-thickness burns

13. Early surgical excision of the eschar and grafting is best performed on

A. 2nd day
B. 5th day
C. 7th day
D. 10th day
E. 14th day

14. One of the following is not an Indication of Hospital admission in burns

A. Second degree burns affecting 10% of the body surface area in adults
B. 5% Third degree burns.
C. Most burns of the head and face
D. Electric burns, severe chemical burns
E. inhalation injuries.

15.One of the following is not a cause of SIADH (syndrome of inappropriate anti-


diuretic hormone)

A. Early Post operative period.


B. Lung neoplasms, infections.
C. Trauma, Head injuries.
D. Hypothyroid.
E. Excess water loss.

16. During laparotomy insensible losses reach

A. 700-1000 ml/H
B. 200-300 ml/H
C. 100-200 ml/H
D. 400-600 ml/H
E. 70-100 ml/H

17. The highest bicarbonate concentration is found in the secretion of

A. Salivary glands.
B. Pancreas
C. stomach.
D. Bile
E. Intestine

18- Staphylococcal infections:


 Do not cause cellutitis.
 Do not produce septicaemia.
 Do not produce fever.
 Produce yellow odourless pus.
 A&C only

19- Streptococcal infections:


 Are characterised by abscess formation.
 Rarely produce lymphadenitis.
 Do not produce baceraemia.
 Can produce a gangrenous skin infection.
 All of the above.

20- Tatanus prophylaxis in a patient with a badly contaminated wound:(all correct


except one)
 Depends, even in an actively immunised patient, on meticulous immediate
debridement of the wound.
 Should include the administration of tetanus toxoid.
 Is more safely achieved with human antitoxin.
 Is unnecessary in patients who have been recently actively immunised.
 Is necessary for the actively immunised patient to have a booster dose of
toxiod.

21- Tetanus:
 May have an incubation period of over 20 days.
 Can be prevented by the immediate administration of tetanus toxoid.
 Is more common after scalp lacerations than wounds of the extremities.
 Is usually associated with stupor or coma.
 B&C only.

22- In clostridial infections:


 A spreading cellutitis may be present.
 Gram-positive cocci can be isolated from the discharge.
 Surgical treatment has a minor part to play.
 Gas production is often present.
 C&D only.

23- Actinomycosis is characterised by:


 Chronic abscesses of the foot.
 A granulomatous abcess.
 Red coloured pus.
 Its resistance to antibiotics.
 Blake coloured pus

24- A subphrenic abscess:


 Is usually accompanied by consederable systemic effects.
 Is associated with local rib tenderness.
 Rarely produces abnormal signs in the chest.
 Usually can be diagnosed by a barium meal examinations.
 A&C only.

25- A pelvic abscess:


 Lies extrapeitoneally.
 May be a complication of abdominal surgery.
 Rarely presents with diarrhoea.
 Should be treated with antibiotics alone.
 A&C only

26- Paralytic ileus:


 Is associated with electrolyte imbalance.
 May be associated with mechanical intestinal obstruction.
 Requires treatment with nasogastric suction and intravenous fluids.
 Is associated with retroperitoneal haematoma
 All of the above.

27- Acute postoperative gastric dilatation:


 Usually cause postoperative vascular collapse.
 Can be prevented by regular nasogastric aspiration.
 Characteristically occurs on the first postoperative day.
 Is a relatively common problem after surgery on the gastrointetinal tract.
 A&D only.
28- The appearance of jaundice in the postoperative period: (all correct except one)
 May indicate an intraperitoneal haemorrhage.
 Is usually due to the toxic effects of anaesthetic agents.
 May be due to septicaemia.
 May indicate chronic liver disease.
 May be due acute liver insalt

29- A clean incised skin wound:


 Undergoes an inflammatory phase during the processes of repair.
 Commences epithelialization after 7 to 10 days.
 Regains the full strength of normal skin within 10 days.
 Regains its strength as the result of fibroblast activity.
 A&D only.

30- Wound healing:


 Is impaired in anaemic patients.
 Is impaired by haematoma formation.
 Is not impaired by hypoproteinaemia.
 Is stimulated by steroids.
 A&D only.

31- The principles of wound care include: (All except one)


 Early skin cover.
 Removal of foreign material.
 Routine administration of antibiotics.
 Close apposition of uninfected wounded tissues.
 Wound toilet if possible.

32-Heavily contaminated and dirty wounds:


 Require surgical toilet and delayed closure.
 Require the administration of systemic antibiotics.
 Can usually be treated by wound toilet and primary closure.
 Should be totally excised.
 A&B only.

33- surgical drainage of abscesses:


 Should be via a small incision with minimal disturbance of the adjacent tissues.
 Should be dependent wherever possible.
 Has been outmoded by antibiotic therapy.
 Should be undertaken before the signs of fluctuation appear.
 Usually performed under local anaethesia.

34- In localised surgical infections:


 An elevated leucocyte count is usually present.
 Fever and tenderness are usually present.
 The presence of glycosuria usually indicates metastatic pancreatic abscesses.
 Pus is frequently absent.
 A&B only.

35- A distal small bowel fistula:


 Loses intestinal fluid rich in potassium and sodium.
 May give rise to a metabolic acidosis.
 Rapidly results in dehydration.
 May be managed conservatively
 All of the above.
36- A pancreatic fistula:
 Loses fluids with high potassium and lower sodium levels than plasma.
 May give rise to a metabolic acidosis.
 Requires long term treatment with intravenous normal saline.
 Will usually close if oral feeding is temporarily suspended.
 All of the above.

37- Bed sores:


 Can be prevented by sheepskin blankets.
 Can be prevented by changing the patient’s position four times each 24 hours.
 Are the consequence of local ischemia.
 Only occur over the sacrum.
 A&B only.
38-Regarding tension pneumothorax, the first step in the management
is:
A. Obtaining a stat chest x-ray.
B. Cricothyroidotomy
C. Pass in an endotracheal tube
D. Starting oxygen by a valve-mask device
E. Chest decompression needle.

39- Regarding central facial bone injury in polytrauma victim Who has
labored breathing the first step in management is:
A. Perform tracheostomy
B. Perform chricothyroidotomy
C. Pass endotracheal tube by oral root.
D. Pass endotracheal tube by nasal root
E. Obtain a cross table x-ray of cervical spine.

40- A young man is admitted to Emergency department with a stab


wound to upper third of his right thigh with severe bleeding, the first
thing to do is:
A. Apply compression to the bleeding vessel with gloved finger.
B. Apply bandage and splint to the right thigh.
C. Apply tourniguet on right side above the wound.
D. Use clamps to control the bleeding.
E. Start blood transfusion.

41- Regarding the management of polytrauma:


A. Death follow a trimodal distribution.
B. X-ray after primary survey should be AP cervical spine, chest and pelvis.
C. Cardiac tamponade is characterized by raised B.p, a low JUP.
D. Assessment of uncomplicated limb fractures should occur during the primary
survey.
E. A and B only.
42- Blunt injuries to the abdomen
(all are correct except)
A. May cause shock
B. May cause peritonitis
C. May cause acute gastroduodenal ulceration
D. May treated conservativly
E. Rarely need urgent laparatomy

43- A serious intra-abdominal injury in a comatose patient may be


diagnosed by: (all are correct except one)
A. Abdominal paracentesis.
B. The observation of pattern bruising on the abdominal wall.
C. Falling of heamoglobin values.
D. The presence of marked abdominal distetion
E. The presence of diarrhea.

44- Injuries to the urethra (all are correct except one)


A. Are more common in male.
B. Are often caused by road traffic accidents.
C. Are easily diagnosed on intra venous pyelography.
D. Require urgent surgical treatment.
E. Diagnosed by retrograde urethragraphy.

45- Regarding injury to the spleen (all are correct except one )
A. Is the commonest organ injured in blunt abdominal trauma.
B. Usually continues to bleed once its capsule and its pulp lacerated.
C. Usually managed conservatively in polytrauma patient.
D. Should be removed if there is severe laceration of its capsule.
E. The risk of infection is increase after its removal.

46- Regarding urinary tract injuries (all are correct except one)
A. Are usually accompanied by haematuria.
B. Require urgent intravenous pyelogram.
C. Involving the kidney require urgent surgery.
D. May cause by RTA.
E. Which demonstrate extravasations of urine from urinary bladder require
surgery.

47- Patient with renal trauma


A. Usually present with hematuria
B. Usually require surgical management.
C. May require an intravenous pyelogram.
D. Often present with acute renal failure.
E. B & C only.
48- Regarding traumatic rupture of the spleen (all are correct except
one )
A. Frequently presents more than 7 days after the causative injury.
B. Frequently presents with shoulder tip pain
C. May be diagnosed by paracentesis.
D. May be accompanied by other abdominal organ injuries.
E. Usually manage conservatively

49- Car seat belts when properly adjusted


A. Prevent injuries to abdominal organs.
B. May cause small bowel injuries.
C. Do not reduce the incidence of head injuries of passengers involving in RTA.
D. Protect the cervical spine during sudden acceleration
D. A & D only.

50- Penetrating wounds of the abdomen.


A. Can be adequately explored by a probe.
B. Frequently result in acquired abdominal wall hernia.
C. May be managed by careful observation, laparatomy being indicated if signs of
peritonitis occur.
D. Can be treated conservatively if cause by gun shot.
E. A&D only.

51- In head trauma patient after control of air way , the first diagnostic
study
A. X-rays of the skull.
B. CT scan of the head.
C. X-rays of cervical spine
D. Carotid angiography.
E. Lumber puncture.

52- In abdominal trauma with signs of hypovolamic shock the first thing
to do after resuscitation of the patient is
A. Obtain lateral x-rays of cervical spine.
B. Obtain supine x-rays of the abdomen.
C. Obtain abdominal CT scan.
D. Obtain an abdominal aortogram.
E. Explore the abdomen.

53. An early signs of anterior compartment syndrome in the calf is:


A. Absence of pulse in the foot.
B. Firm calf muscle
C. Foot drop
D. Paraethesia between the big and second toes.
E. Painful calf muscle.

54- Haematuria:
• At the beginning of micturition is usually indicative of urethral pathology.
• At the end of micturition is usually due to bladder neck pathology.
• Throughout the urinary stream is typical of renal pathology.
• In elderly males is usually related to benign prostatic hypertrophy.
• All of the above.

55- An intravenous pyelogram:


• Yields most diagnostic information when performed on a slightly hydrated
patient.
• Should be preceded by a plain film of the abdomen.
• Normally shows incomplete filling of the ureter in any one exposure.
• Should provide evidence of the presence, if any , of lower urinary tract
obstruction.
• All of the above.

56- Wilms’ tumours:


• Metastasise readily to the lungs.
• Metastasise rarely to the bones.
• Are usually bilateral.
• Have the worst prognosis of all childhood abdominal tumours.
• B&C only.

57- In renal transplantation:


• A donor kidney may be used from a patient with malignancy provided there is
no abdominal involvement.
• ABO compatibility between donor and recipient does not have to be
considered.
• Satisfactory renal function can be expected with a warm ischaemic time of up
to 200 minutes.
• The characteristic signs of acute rejection include pyrexia, hypertension
and leucocytosis.
• A&B only.

58- An adenocarcinoma of the kidney:


• Usually occurs in the 35 to 45 age group.
• Usually presents with a urinary infection.
• Is often distinguishable from a renal cyst radiologically.
• Frequently invades and grows along the renal vein.
• A&C only.

59- Tumours of the renal pelvis )All correct except one)


• Rarely present as a mass in the loin.
• Are possibly due to a urinary carcinogen.
• Resemble those of the bladder in their histology.
• Are best treated by a partial or total nephrectomy.
• Usually cause hematuria and clot colic.

60- Ureteric calculi:


• Often result from urinary tract infection.
• Rarely cause haematuria.
• Are not usually radio-opaque.
• producing ureteric colic should be surgically removed.
• B&C only.

61- Cancer of the penis:


• Is more common in the circumcised.
• Commonly arises from the corona of glans penis.
• Is usually an adenocarcinoma.
• Rarely metastasises.
• C&D only.

62- Carcinoma of the prostate:


• Is commonly of squamous cell origin.
• Usually originates in the periphery of the gland.
• Usually presents relatively early with lower urinary tract symptoms.
• Rarely can be diagnosed on rectal examination.
• All other above.

63- Carcinoma of the prostate:


• Does not usually metastasise.
• Usually produces an elevated serum acid phosphatase.
• Can be effectively treated by hormones.
• Is most effectively treated by surgery.
• B&C only.

64- Benign prostatic hypertrophy: )all correct except one)


• Is the result of hyperplasia of the fibromuscular capsule of the gland.
• Results in diminished power of urination.
• Results in terminal dribbling of urine.
• Often presents with haematuria.
• Might cause mild elevation of serum PSA.

65- Benign prostatic hypertrophy:


• Can readily be assessed on rectal examination.
• Can be effectively treated with hormones.
• Is most effectively treated by surgery.
• Is a premalignant condition.
• Rarely presented with macroscopic hematuria

66- Acute prostatitis: (all correct except one)


• Is most commonly due to coliform organisms.
• Often presents as an ache in the perineum.
• May be diagnosed by rectal examination.
• Requires bladder catheterisation as part of the treatment .
• May be lead to a prostate abscess formation.

67-Bladder cancer: ( all correct except one)


• May follow exposure to beta-naphthylamine.
• Is more common in heavy smokers.
• Is more common in females.
• Is frequently associated with bladder schistosomiasis.
• May caused by chronic urinary bladder inflamation.

68- Bladder cancers:


• Are usually adenocarcinomas.
• Are usually ulcerating.
• Usually present with suprapubic pain radiating to the perineum.
• Are usually diagnosed on cystoscopy.
• All of the above.

69- Undescended testes:


• Are often associated with inguinal hernia.
• Usually descend at puberty.
• Can usually be made to descend by the examiner with warm hands.
• Should be treated by orchidopexy at puberty.
• Can be exist without any complication.
70- The spermatic cord contains:
• The inferior epigastric vein.
• The deep circumflex iliac artery.
• The pudendal nerve.
• The subcostal nerve.
• Vas deference.

71- Torsion of the spermatic cord:


• Often presents with vomiting and lower abdominal pain.
• Often produces gangrene of the testis.
• May be diagnosed clinically.
• Always requires surgical treatment.
• All of the above.

72- Seminomas of the testis:


• Most commonly occur before the age of 40.
• Are usually sensitive to radiotherapy.
• Rarely metastasise via the blood stream.
• Generally carry a good prognosis.
• All of the above.

73- Hypospadias:
• Is the result of failure of scrotal development.
• Results in the abnormal urethra opening on to the dorsum of the penis.
• Is associated with chordee.
• Is associated with maldescent of the testis.
• C&D only.

74-Which is the shortest phase of the normal cell cycle?


a) G1 phase
b) S phase
c) G2 phase
d) M phase
e) All phases approximately equal in length

75- A 67 year-old women with rectal cancer is admitted to gereral surgical floor which of
the following laboratory studies should be included in the surgeon’s initial nutritional
assessment :
a) Transferrin
b) Prealbumin
c) Albumin
d) Glutamine
e) All of above

76- In which of the following conditions is the entral route appropriate for nutrition :
a) Upper gastrointestinal obstruction
b) Complete small bowel obstruction
c) Acute flare-up of Crohn’s disease
d) Low out put colonic fistula
e) Non of the above

77-Which is the most commonly cultured hospital acquired organism in critical care with
aspiration pneumonia:
a) Staphylococcus aureus
b) Streptococcus pneumonia
c) Anaerobic species
d) Pseudomonas aeroginosa
e) Haemophlus influenzae

78-Which is the most appropriate single agent for empiric coverage of the above patient :
a) Metranidazole
b) Clindamycin
c) Pipracillin_tazobactam
d) Vancomycin
e) First generation penicillin

79-All of the following are true about neurogenic shock except:

a) There is a decrease in systemic vascular resistance and an increase in venous


capacitance.
b) Tachycardia or bradycardia may be observed, along with hypotension.
c) The use of an alpha agonist such as phenylephrine is the mainstay of
treatment.
d) Severe head injury, spinal cord injury, and high spinal anesthesia may all cause
neurogenic shock.
e) A and B

80-Which of the following statements about head injury and concomitant hyponatremia
are true?

a) There are no primary alterations in cardiovascular signs.


b) Signs of increased intracranial pressure may be masked by the hyponatremia.
c) Oliguric renal failure is an unlikely complication.
d) Rapid correction of the hyponatremia may prevent central pontine injury.
e) This patient is best treated by restriction of water intake

81- Regarding Cushing's Syndrome


a) 20% of cases are due to pituitary adenomas (Cushing's Disease)
b) Most ACTH secreting pituitary adenomas are more than 2 cm in diameter
c) Is characterised by loss of the diurnal rhythm of cortisol secretion
d) Cortisol production is suppressed by low-dose dexamethasone
e) Adrenal carcinomas are more common than adrenal adenomas
82-Which of the following statements about extracellular fluid are true?

a) The total extracellular fluid volume represents 40% of the body weight.
b) The plasma volume constitutes one fourth of the total extracellular fluid
volume.
c) Potassium is the principal cation in extracellular fluid.
d) The protein content of the plasma produces a lower concentration of cations than
in the interstitial fluid.
e) The interstitial fluid equilibrates slowly with the other body compartments.

83-In patients receiving massive blood transfusion for acute blood loss, which of the
following is/are correct?

a) Packed red blood cells and crystalloid solution should be infused to restore
oxygen-carrying capacity and intravascular volume.
b) Two units of FFP should be given with every 5 units of packed red blood cells in
most cases.
c) A “six pack” of platelets should be administered with every 10 units of packed red
blood cells in most cases.
d) One to two ampules of sodium bicarbonate should be administered with every 5
units of packed red blood cells to avoid acidosis.
e) One ampule of calcium chloride should be administered with every 5 units of
packed red blood cells to avoid hypocalcemia.

84- Which of the following statements about the presence of gallstones in diabetes
patients is/are correct?

a) Gallstones occur with the same frequency in diabetes patients as in the healthy
population.
b) The presence of gallstones, regardless of the presence of symptoms, is an
indication for cholecystectomy in a diabetes patient.
c) Diabetes patients with gallstones and chronic biliary pain should be managed
nonoperatively with chemical dissolution and/or lithotripsy because of severe
complicating medical conditions and a high operative risk.
d) The presence of diabetes and gallstones places the patient at high risk for
pancreatic cancer.
e) Diabetes patients with symptomatic gallstones should have prompt elective
cholecystectomy, to avoid the complications of acute cholecystitis and
gallbladder necrosis.

85. concerning Tetanus all true except:


a. Is due to an infection with a gram-negative spore forming rod
b. The organism produces a powerful exdotoxin
c. The toxin prevents the release of inhibitory neurotransmitter
d. Clostridium tetani is sensitive to penicillin
e. Risus sardonicus is the typical facial spasm
86-when should parentral antibiotics be given perioperatively?

a) The night before


b) 6 hr prior to surgery
c) 60 minutes prior to incision .
d) at the time of incision
e) 30 minutes after incision

87- Which of the following statements about esophageal anatomy is correct?

a) The esophagus has a poor blood supply, which is segmental in distribution and
accounts for the high incidence of anastomotic leakage.
b) The esophageal serosa consists of a thin layer of fibroareolar tissue.
c) The esophagus has two distinct muscle layers, an outer, longitudinal one and
an inner, circular one, which are striated in the upper third and smooth in the
distal two thirds.
d) Injury to the recurrent laryngeal nerve results in vocal cord dysfunction but does
not affect swallowing.
e) The lymphatic drainage of the esophagus is relatively sparse, localized primarily
to adjacent paraesophageal lymph nodes.

88- Which of the following medication should be given in preparation of a pation with
pheochromocytoma?

a) Phnoxybenzamine
b) Nifedipine
c) Linsinopril
d) Hydrochlorothiazide
e) Propranolol

89-Which of the following statement is true concerning excessive scarring processes?

a) Keloids occur randomly regardless of gender or race


b) Hypertrophic scars and keloid are histologically different
c) Keloids tend to develop early and hypertrophic scars late after the surgical injury
d) Simple reexcision and closure of a hypertrophic scar can be useful in certain
situations such as a wound closed by secondary intention
e) Non of the above

90-A 22-year-old man sustains a single stab wound to the left chest and presents to the
emergency room with hypotension. Which of the following statement is true concerning
his diagnosis and management?

a) The patient likely is suffering from hypovolemic shock and should respond
quickly to fluid resuscitation
b) Beck’s triad will likely be an obvious indication of compressive cardiogenic shock
due to pericardial tamponade
c) Echocardiography is the most sensitive noninvasive approach for diagnosis of
pericardial tamponade
d) The placement of bilateral chest tubes will likely resolve the problem

91-Which of the following statement is true concerning septic shock?

a) The clinical picture of gram negative septic shock is specifically different than
shock associated with other infectious agents
b) The circulatory derangements of septic shock precede the development of
metabolic abnormalities
c) Splanchnic vascular resistance falls in similar fashion to overall systemic vascular
resistance
d) Despite normal mechanisms of intrinsic expansion of the circulating blood
volume, exogenous volume resuscitation is necessary

92- During surgery on the submandibular gland


a) An incision on the lower border of the mandible is safe
b) The submandibular gland is seen to wrap around the posterior border of
mylohyoid
c) The facial artery and vein are divided as they course through the deep part of the
gland
d) The hypoglossal nerve is seen to loop under the submandibular duct
e) Damage to the lingual nerve will cause loss of sensation to the posterior third of
the tongue

93- Regarding benign breast disease


a) Cyclical mastalgia is the commonest reason for referral to the breast clinic
b) Fibroadenomas are derived from the breast lobule
c) Lactational breast abscesses are usually due to Staph aureus
d) Duct ectasia is more common in smokers
e) Atypical lobular hyperplasia is associated with an decreased risk of breast
cancer

94-. Which of the following statements regarding unusual hernias is incorrect?

a) An obturator hernia may produce nerve compression diagnosed by a positive


Howship-Romberg sign.
b) Grynfeltt's hernia appears through the superior lumbar triangle, whereas Petit's
hernia occurs through the inferior lumbar triangle.
c) Sciatic hernias usually present with a painful groin mass below the inguinal
ligament.
d) Littre's hernia is defined by a Meckel's diverticulum presenting as the sole
component of the hernia sac.
e) Richter's hernia involves the antimesenteric surface of the intestine within the
hernia sac and may present with partial intestinal obstruction

95- Staples may safely be placed during laparoscopic hernia repair in each of the
following structures except:

a) Cooper's ligament.
b) Tissues superior to the lateral iliopubic tract.
c) The transversus abdominis aponeurotic arch.
d) Tissues inferior to the lateral iliopubic tract.
e) The iliopubic tract at its insertion onto Cooper's ligament.

96-The following statements about the repair of inguinal hernias are true except:

a) The conjoined tendon is sutured to Cooper's ligament in the Bassini hernia


repair.
b) The McVay repair is a suitable option for the repair of femoral hernias.
c) The Shouldice repair involves a multilayer, imbricated repair of the floor of the
inguinal canal.
d) The Lichtenstein repair is accomplished by prosthetic mesh repair of the inguinal
canal floor in a tension-free manner.
e) The laparoscopic transabdominal preperitoneal (TAPP) and totally extraperitoneal
approach (TEPA) repairs are based on the preperitoneal repairs of Cheattle, Henry,
Nyhus, and Stoppa.

97-A number of special circumstances exist in the repair of inguinal hernias. The
following statement(s) is correct.

a) Simultaneous repair of bilateral direct inguinal hernias can be performed with no


significant increased risk of recurrence
b) The preperitoneal approach may be appropriate for repair of a multiple
recurrent hernia
c) A femoral hernia repair can best be accomplished using a Bassini or Shouldice
repair
d) Management of an incarcerated inguinal hernia with obstruction is best
approached via laparotomy incision
e) All are correct.

98- Which of the following statement(s) is true about benign lesions of the liver?

a) Adenomas are true neoplasms with a predisposition for complications and


should usually be resected.
b) Focal nodular hyperplasia (FNH) is a neoplasm related to birth control pills (BCPs)
and usually requires resection.
c) Hemangiomas are the most common benign lesions of the liver that come to the
surgeon's attention.
d) Nodular regenerative hyperplasia does not usually accompany cirrhosis
99- Ligation of all of the following arteries usually causes significant hepatic enzyme
abnormalities except:

a) Ligation of the right hepatic artery.


b) Ligation of the left hepatic artery.
c) Ligation of the hepatic artery distal to the gastro-duodenal branch.
d) Ligation of the hepatic artery proximal to the gastroduodenal artery.

100- Which of the following is the most effective definitive therapy for both prevention
of recurrent variceal hemorrhage and control of ascites?

a) Endoscopic sclerotherapy.
b) Distal splenorenal shunt.
c) Esophagogastric devascularization (Sugiura procedure).
d) Side-to-side portacaval shunt.
e) End-to-side portacaval shunt.

28-which of the following is associated with best prognosis for patient with breast
cancer?

a) Male sex
b) Estrogen receptor positive
c) Patient age <35 years
d) Pregnant patient
e) Tumor with overexpression of HER/ner.

29-A 49-year-old women has a palpable breast mass in the upper outer quadrant. The size
of the mass has increased over the last month . exicisional biopsy reveals cystic
carcinoma with invasion .appropriate management now would be :

a) Re-excision with wide margins


b) Axillary node dissection and hormonal therapy
c) Simple mastectomy
d) Modified radical mastectomy
e) Bilateral mastectomies

30-A contraindication to stereotactic core biopsy of the breast is the mammographic


presence of:

a) Microcacification
b) A radial scar
c) A nonpalpable mass lesion
d) Lesions<8 mm in diameter
e) Mutifocal lesions.
31-. Which of the following statements about epiphrenic diverticula of the esophagus
is/are correct?

a) They are traction diverticula that arise close to the tracheobronchial tree.
b) They characteristically arise proximal to an esophageal reflux stricture.
c) The degree of dysphagia correlates with the size of the pouch.
d) They are best approached surgically through a right thoracotomy.
e) The operation of choice is a stapled diverticulectomy, long esophagomyotomy,
and partial fundoplication.

32- Which of the following statements about Schatzki's ring is correct?

a) The ring represents a panmural fibrotic stricture resulting from gastroesophageal


reflux.
b) Dysphagia occurs when the ring diameter is 13 mm. or less.
c) The ring occurs within 1 to 2 cm. of the squamocolumnar epithelial junction.
d) Schatzki's ring indicates reflux esophagitis.
e) Schatzki's ring signifies the need for an antireflux operation.

33. Which of the following statements about pathology encountered at esophagoscopy


is/are correct?

a) Reflux esophagitis should be graded as mild, moderate, or severe, to promote


consistency among different observers.
b) An esophageal reflux stricture with a 2-mm. lumen is not dilatable and is best
treated with resection.
c) A newly diagnosed radiographic distal esophageal stricture warrants dilation and
antireflux medical therapy.
d) In patients with Barrett's mucosa, the squamocolumnar epithelial junction occurs 3
cm. or more proximal to the anatomic esophagogastric junction.
e) After fasting at least 12 hours, a patient with megaesophagus of achalasia can
safely undergo flexible fiberoptic esophagoscopy.

34. Infantile hypertrophic pyloric stenosis


a) Occurs with a female : male ratio of 4:1.
b) Sons of affected mothers have a 20% risk of developing the lesion
c) Invariably presents between six and eights months of age
d) Typically presents with bile stained projectile vomiting
e) Surgical treatment is by Heller's Cardiomyotomy

35-. Which of the following statements regarding the pathology of esophageal carcinoma
is/are correct?

a) Worldwide, adenocarcinoma is the most common esophageal malignancy.


b) Squamous cell carcinoma is most common in the distal esophagus, whereas
adenocarcinoma predominates in the middle third.
c) Patients with Barrett's metaplasia are 40 times more likely than the general
population to develop adenocarcinoma.
d) Metastases from esophageal carcinoma are characteristically localized to regional
mediastinal lymph nodes adjacent to the tumor.
e) Achalasia, radiation esophagitis, caustic esophageal stricture, Barrett's mucosa,
and Plummer-Vinson syndrome are all premalignant esophageal lesions that
predispose to the development of squamous cell carcinoma.

36-45-year-old with isolated 6-cm colorectal metastasis in the liver 2 years after
colectomy, otherwise healthy pest treatment would be:

a) Radiofrequency ablation
b) Systemic chemotherapy
c) Hepatic lobectomy
d) Liver transplantation
e) Cryosurgical ablation

37- Oesophageal atresia all true except:


a) Is often associated with a distal trachea-oesophageal fistula
b) Polyhydramnios is often present late in pregnancy
c) 50% have other associated congenital abnormalities
d) Contrast X-ray studies are necessary to confirm the diagnosis
e) Post-operatively over 30% develop oesophageal strictures

38-All are true about the dumping syndrome except:

a) Symptoms can be controlled with a somatostatin analog.


b) Diarrhea is always part of the dumping syndrome.
c) Flushing and tachycardia are common features of the syndrome.
d) Separating solids and liquids in the patient's oral intake alleviates some of the
symptoms of the syndrome.
e) Early postoperative dumping after vagotomy often resolves spontaneously.

39-Which of the following statements about gastric polyps is/are true?

a) Like their colonic counterparts, gastric epithelial polyps are common tumors.
b) They are analogous to colorectal polyps in natural history.
c) Endoscopy can uniformly predict the histology of a polyp based on location and
appearance.
d) In a given patient, multiple polyps are generally of a multiple histologic type.
e) Gastric adenomatous polyps greater than 2 cm. in diameter should be excised
because of the risk of malignant transformation.

40-All of the following statements about surgical management of gastric lymphomas are
true except:

a) Stage I gastric lymphomas (small lesions confined to the stomach wall) can be
cured completely with surgical therapy alone.
b) Extensive gastric lymphomas that initially are treated with radiation and/or
chemotherapy occasionally perforate during treatment and require secondary
resection.
c) Patients explored with a presumptive diagnosis of gastric lymphoma should
undergo an attempt at curative resection when this is safe and feasible.
d) Without a preoperative diagnosis resection for gastric mass should not be
attempted unless lymphoma can be excluded.
e) Appropriate staging for primary gastric lymphoma includes bone marrow biopsy.

41-the most accurate test to confirm diagnosis of infected necrotizing pancreases is:

a) Abdominal ultrasound study


b) Indium-labeled leeukocte scan
c) Cimputed tomographic scan
d) Elevated serum level of interleukain 6 and 8
e) Percutaneous needle aspiration

42- Which of the following variables best predicts prognosis for patients with a recent
diagnosis of cutaneous melanoma and no clinical evidence of metastatic disease?

a) Breslow thickness.
b) Clark's level.
c) Ulceration.
d) Gender.
e) Celtic complexion.

43-the following are true about intracranial tumors except:

a) The most common location of brain tumors of childhood is the posterior cranial
fossa.
b) With few exceptions, examination of the CSF is of no value in the diagnosis of an
intracranial tumor.
c) Even the most malignant of primary brain tumors seldom spread outside the
confines of the central nervous system (CNS).
d) The majority of astrocytomas can be cured surgically.
e) Primary neoplasms of astrocytic, oligodendroglial, or ependymal origin represent
gradations of a spectrum from slowly growing to rapidly growing neoplasms.

44 A right-sided disc herniation at the L5–S1 level typically may cause:

a) Low back pain and left sciatica.


b) Weakness of dorsiflexion of the right foot.
c) A diminished or absent right ankle jerk.
d) Diminution of sensation over the medial aspect of the right foot, including the
great toe.
e) Weakness of dorsiflexion of the left foot.

45-. The preferred operation for initial management of a thyroid nodule that is considered
suspicious for malignancy by FNAB is:

a) Excision.
b) Partial lobectomy.
c) Total lobectomy and isthmusectomy.
d) Total thyroidectomy.
e) All methods are correct

46-the most common presentation of Meckel,s diverticulum in an adult is:

a) GIT bleeding
b) GIT obstruction
c) Intussuception
d) Litter,s hernia
e) Diverticulitis

47-Optimal front-line treatment of squamous cell carcinoma of the rectum includes:

a) Abdominal perineal resection.


b) Low anterior resection when technically feasible.
c) Radiation therapy.
d) Chemotherapy.
e) Combined radiation and chemotherapy.

48 -65-year-old man presents with complaints of mucous discharge and perianal


discomfort. Physical examination reveals a fistulous opening lateral to the anus.
Anoscopic examination permits passage of a probe through the fistula tract. The fistula
traverses the internal anal sphincter, the intersphincteric plane, and a portion of the
external anal sphincter. The fistula is categorized as which type?

a) Intersphincteric
b) Transsphincteric
c) Suprasphincteric
d) Extrasphincteric
e) Non of the above

49-Warthin's tumor:

a) Is a pleomorphic adenoma of salivary gland


b) Should be treated by total paritidectomy
c) Is considered a benign salivary gland neoplasia
d) Respond well to preoperative radiotherapy
e) Often present with facial nerve compression

50-A 38 year old woman presents with right upper quadrant pain and bouts of vomiting.
She is known to have gallstones and has had similar episodes in the past. Which of the
following might support a diagnosis of acute cholecystitis rather than biliary colic

a) duration of symptoms
b) Severity of vomiting
c) Presence of Murphy's sign
d) Presence of gas under right hemidiaphragm on erect CXR.

Hemorrhoids MCQS

1. Which of the following is accurate about hemorrhoids?

A. Most symptoms arise from enlarged internal hemorrhoids as opposed to external


hemorrhoids
B. Of the three main hemorrhoidal venous cushions, hemorrhoids are most
commonly associated with the right anterior area of the anal canal, compared with
the left lateral and right posterior areas
C. External hemorrhoids are more common in older adults
D. Portal hypertension is associated with more frequent hemorrhoidal symptoms and
massive bleeding

2. Which of the following is accurate about the presentation and physical


examination of hemorrhoids?

A. The identification of anal fissures and perianal dermatitis requires internal


probing
B. Pain truly caused by hemorrhoids typically arises only with acute thrombus
formation and peaks at 48-72 hours
C. Internal hemorrhoids that are not thrombosed are often palpable during digital
examination
D. If rectal bleeding is bright red, anoscopy and/or flexible sigmoidoscopy is not
required for evaluation

3. Which of the following is accurate about the grading of internal hemorrhoids?

A. Grade IV hemorrhoids usually contain both internal and external components


B. Grade II hemorrhoids often bleed but do not prolapse
C. Grade I hemorrhoids protrude spontaneously and require manual reduction
D. Grade III hemorrhoids protrude with straining or defecating but reduce
spontaneously

4. Which of the following is accurate about the workup of hemorrhoids?


A. Barium enema is routinely indicated for patients with suspected hemorrhoids
B. Histologic examination of hemorrhoidal tissue is routinely indicated
C. Proctoscopy may be used in place of anoscopy in the assessment of internal
hemorrhoids
D. The presence of anemia on laboratory testing should raise suspicion of an
alternate diagnosis to hemorrhoids

5. Which of the following is accurate about the treatment of hemorrhoids?


A. External hemorrhoid symptoms respond well to enucleation
B. Although initially effective, most external hemorrhoids treated with operative
hemorrhoidectomy recur
C. Grade II and III hemorrhoids are initially treated with nonsurgical procedures
D. When treating acutely thrombosed external hemorrhoids in an emergency setting,
simple incision and clot evacuation is indicated
In a normal adult patient, a red blood cell travelling from the aorta to the portal

vein is most likely to pass through which structures?

A. Inferior mesenteric artery, superior rectal artery, rectal veins

B. Coeliac trunk, left gastro-omental artery, splenic vein

C. Right gastric artery, short gastric vein, splenic vein

D. Superior mesenteric artery, right colic vein, inferior mesenteric vein

E. Coeliac trunk, gastroduodenal artery, epigastric vein

1. The radial nerve.


A. Is a branch of the anterior cord of the brachial plexus.
B. Is derived from the posterior primary rami of C5 to C7.
C. Supplies the flexors of the arm.
D. Gives rise to the anterior interosseous nerve.
E. Injury above elbow produces a classical wrist drop.

2. Regarding femoral canal all are true except:


A. Lies lateral to the femoral vein.
B. Has the inguinal ligament as its anterior border.
C. Has the lacunar ligament as its medial border.
D. Has the pectineal ligament as its posterior border.
E. Contains the lymph node of Cloquet.

3. The following causes hypercalcaemia except:


A. Sarcoidosis.
B. Primary hyperparathyroidism.
C. Acute pancreatitis.
D. Metastatic bronchial carcinoma.
E. Milk-Alkali syndrome.

4. Number of human chromosomes in human female are:


A. 23 pairs +XX.
B. 21 pairs +XY.
C. 22 pairs +XY.
D. 22 pairs +XX.
E. 23 pairs +XY.

5. Potassium deficiency should be suspected in all the following except:


A. In cases of paralytic ileus.
B. When the patient's reflexes are exaggerated.
C. If there is a decrease in height and peaking of the T waves of an ECG.
D. In alkalotic states.

6. Active immunization in case of tetanus:


A. Antitetanus human serum.
B. Gives short period of protection.
C. Given in case proved tetanus.
D. Use of immunoglobulin.
E. None of the above.

7. All of the following are signs of rised intracranial pressure except:


A. Headache.
B. Vomiting.
C. Papilledema.
D. Aphasia.
E. Bradycardia.

8. All of the following are extra cranial hematomas except:


A. Subcutaneos haematoma.
B. Extra dural haematoma.
C. Cephalohaematoma.
D. Subgaleal hematoma.
E. Subperiostial haematoma.

9. Glasgow coma scale all the following are true except:


A. Used for evaluation of comatose patient.
B. It ranges from ( 3 to 15).
C. Useful for neurological follow up.
D. Useful for pupils evaluation.
E. Best motor response given 6 point.

10. All the followings are Indications for central line insertion EXCPET:
A. Massive fluid replacement
B. Massive blood replacement
C. Measurement of central venous pressure
D. Prolonged Intervenes fluid therapy

11. Most common early complication of central venous line is:


A. Sepsis
B. Pneumothorax
C. Thoracic duct injury
D. Thrombosis
12. The following are Complications of shock:
A. Acute Respiratory failure
B. Acute myocardial infarction
C. Acute renal failure
D. A&C only
E. All the above

13. Causes of delayed union of fractures includes all the following EXCPET:
A. Compound fracture
B. Infection
C. Adequate immobilization
D. Poor blood supply

14. In Head injury C.T. scan is indicated in the following


A. Aphasia after the injury
B. Deterioration of level of consciousness
C. Skull fracture with persistent headache
D. A&B only
E. All the above
15. Most important steps in management of head injury include:
A. Prevent hypoxia
B. Prevent Dehydration
C. Assure Brain Metabolism
D. Prevent secondary brain injury
E. All the above

16. Tension pneumothorax


A. is the commonest type of chest injuries
B. Needs urgent X-Ray chest
C. Is a clinical Diagnosis
D. Causes flat neek viens
E. Treated by thoracotomy tube after chest X-ray.

17. Calcitonin hormon is secreted to the blood circulations from:


A. Parathyroid gland.
B. Parafollicular cells of thyroid gland.
C. Supra renal gland.
D. Pituitary gland.
E. Gonads.
18. Regarding tension pneumothorax, the first step in the management is:
A. Obtaining a stat chest x-ray.
B. Cricothyroidectomy.
C. Pass an endotracheal tube.
D. Starting oxygen by a valve-mask device.
E. Chest decompression needle.

19. The following are features of thyrotoxicosis except:


A. Weight gain.
B. Palpitations.
C. Proximal myopathy.
D. Increased skin pigmentation.
E. Pretibial myxoedema.

20. The following is a clinical feature of Horner's syndrome:


A. Miosis
B. Failure of abduction of the orbit
C. Increased sweating on the contralateral side of the forehead
D. Exophthalmos.
E. All are true

21. In role of nine extent of burn if entire trunk is burned it will be equal to:
A. 9% body surface area.
B. 18% body surface area.
C. 36% body surface area.
D. 27% body surface area.
E. 45% body surface area.
22. Trachlea (4th) cranial nerve supply :
A. Lateral rectus muscle of th eye.
B. Medial rectus muscle of the eye.
C. Superior obligue rectus muscle of the eye.
D. Superior oblique muscle of the eye.
E. Muscles of the upper eye lid.

23. Regarding pathological terms :


A. Hypertrophy is an increase in tissue size due to increased cell number.
B. Hyperplasia is an increase in tissue size due to an increase in cell size.
C. Atrophy is an increase in tissue size due to disuse.
D. Metaplasia is a change form one abnormal tissue type to another.
E. A hamartoma is a developmental abnormality.

24. Regarding nephroblastomas:


A. They are otherwise known as a Wilm's tumour.
B. Account for 10% of childhood tumours.
C. The commonest presentation is with an abdominal mass.
D. Most commonly present between 2nd and 4th year of life.
E. All are true.

25. Regarding fluid losses in a major burn all are true except:
A. Are maximal between 12 and 24 hours after the injury.
B. Are related to the age of the patient.
C. Are not related to the weight of the patient.
D. Are related to the area burnt.
E. Are not related to the burn duration.

26. In obstructive jaundice :


A. Urinary conjugated bilirubin is increased.
B. Serum unconjugated bilirubin is increased.
C. Urinary urobilinogen is increased.
D. Serum conjugated bilirubin is reduced.
E. Faecal stercobilinogen is increased.

27. Regarding Hydatid disease:


A. Is due to Ecchinococcus granulosa.
B. Man is an accidental intermediate host.
C. The liver is the commonest site of infection.
D. Can be diagnosed by the Casoni test.
E. All are true.

28. The first aid of treatment in fracture of cervical spine should be:
A. Cervical spine x-ray.
B. Analgesia.
C. Neck immoblization.
D. Cervical traction.
E. Non of teh above.

29. The first aid of treatment in fracture of cervical spine should be:
A. Cervical spine x-ray.
B. Analgesia.
C. Neck immoblization.
D. Cervical traction.

30. Regarding local anaesthesia:


A. Local anaesthetics act on small before large nerve fibres
B. Adrenaline reduces absorption and prolongs the local effects
C. Preparations containing adrenaline are safe to use on digits and appendages
D. Lignocaine has a longer duration of action than bupivicaine.
E. All are false.

31. Small bowel obstruction often results in: (all correct except one)
A. Hyperkalaemia.
B. Metabolic alkalosis.
C. Oliguria.
D. Hypovolaemia.
E. Severe dehydration.

32. A serious intra-abdominal injury in a comatose patient may be diagnosed by (all


are correct except one):
A. Abdominal paracentesis.
B. The observation of bruising pattern on the abdominal wall.
C. Falling of heamoglobin values.
D. The presence of marked abdominal distetion.
E. The presence of diarrhea.

33. A perforated duodenal ulcer, all are true except:


A. Usually lies on the anterior or superior surface of the duodenum.
B. Usually presents with the acute onset of severe back pain.
C. Produces radiological evidence of free gas in the peritoneum in over 90 percent of
the patients.
D. Is usually treated by vagotomy and pyloroplasty.
E. Is usually treated conservatively.

34. Acute pancreatitis typically: (all correct except one)


A. Is accompanied by hypocalcaemia.
B. Produces paralytic ileus.
C. Is associated with a pleural effusion.
D. Produces pyloric stenosis.
E. Upper abdominal pain and vomiting.

35. Biliary colic typically:


A. Occurs 3 to 4 hours after meals.
B. Lasts 5 to 20 minutes.
C. Radiates from the upper abdomen to the right subscapular region.
D. Is made better by deep inspiration.
E. B&C only.
36. In post operative DVT, the following are true except:
A. Clinical DVT occures in the 4th post operative day.
B. If complicated by pulmonary embolism, it occures usually after the 7th post
operative day.
C. The process of DVT starts preoperatively with the induction of anaesthesia .
D. When discovered we should start the patient on coumadin "Warfarin
anticoagulation".
E. It may lead to chronic venous in suffering as a complication of DVT.

37. In acute appendicitis all of the following are true except:


A. Anorexia.
B. Abdominal pain usually preceedes vomiting.
C. Pain after begins in the paraumbilical region.
D. Constipation diarrhea may occur.
E. Dysuria excludes the diagnosis.

38. The most common cause of massive haemorrhage in the lower gastroinfestinal
tract is :
A. Carcinoma.
B. Diverticulosis
C. Diverticulitis
D. Polyp.
E. Ulcerative colitis.

39. Painless haematuria is the leading presentation of :


A. Renal cell carcinoma.
B. Transitional cell carcinoma of the bladder .
C. Ureteric stone.
D. Pelvi-ureteric obstruction.
E. Ureterocele.

40. All of the following are complications of massive blood transfusion except:
A. Acute congestive heart failure.
B. Transmission of infection.
C. Hypercalcaemia.
D. Hyperkalaemia.
E. Transfusion reactions.

41. Complication of undescended testis include all of the following except :


A. Malignant degeneration.
B. Increased susceptability to trauma.
C. Increased spermatogenesis.
D. More liable to testiculer torsion.
E. Psychological complication.
42. The recurrent laryngoeal nerve is branch of :
A. Facial nerve.
B. Glosso-pharyngeal nerve.
C. Cervical plexus.
D. Vagus nerve.
E. Brachial plexus.

43. The thyroid tumor which is may be associated with pheochromocytoma is :


A. Papillary carcinoma.
B. Medullary carcinoma.
C. Follicular carcinoma.
D. Anaplastic carcinoma.
E. Malignant lymphoma.

44. The most common pancreatic cyst is :


A. Dermoid cyst of the pancreas.
B. Hydatid cyst of the pancreas.
C. Pancreas pseudocyst.
D. Pancreatic cystadenoma.
E. Congenital cystic disease of the pancreas.

45. The anatomical division between the anus and rectum :


A. Lateral haemorrhoidal groove.
B. Inter haemorrhoidal groove.
C. Dentate line.
D. Arcuate line
E. Ano-rectal ring.

46. The comments type of Anorectal abscess is:


A. Ischio rectal
B. Perianal
C. Submucons
D. Pelvirectal

47. Anal Fissure:


A. Usually anterior
B. May be caused by previous anal surgery
C. Can cause dark bleeding PR.
D. Sometimes is painful
E. Treated by steroids

1. The effective osmotic pressure between the plasma and interstitial fluid
compartments is primarily controlled by:
A. Bicarbonate.
B. Chloride ion.
C. Potassium ion.
D. Protein.
E. Sodium ion.

2. Symptoms and signs of extracellular fluid volume deficit include all of the
following except:
A. Anorexia.
B. Apathy.
C. Decreased body temperature.
D. High pulse pressure.
E. Orthostatic hypotension.

3. The osmolarity of the extracellular fluid space is determined primarily by the


concentration of:
A. Bicarbonate
B. Chloride ion
C. Phosphate radicals
D. Sodium ion
E. Sulfate radicals

4. Potassium deficiency should be suspected in all except:


A. In cases of paralytic ileus.
B. When the patient's reflexes are exaggerated.
C. If there is a decrease in height and peaking of the T waves of an ECG.
D. In alkalotic states.

5. The sodium ion


A. Is the principal regulator of the intracellular volume.
B. Is the major ionic component of the intracellular fluid volume.
C. Is present in greater concentration in intracellular fluid than extracellular fluid.
D. Is excreted in larger amounts than normal in the early postoperative period.
E. C&D only

6. Acute post traumatic renal failure:


A. May be due to hypovolaemia and poor tissue perfusion.
B. Is particularly associated with crush injuries.
C. May be due to kidney damage following tubular obstruction.
D. Should initially be treated by fluid restriction.
E. Should be treated initially by fluid restriction.

7. Intravenous parenteral feeding:


A. Should deliver at least 2500 calories/day to an adult.
B. Should deliver at least 10g of nitrogen (i.e. 66g of protein)/day to an adult.
C. Can be effectively achieved with isotonic solutions.
D. Is with complications with present day solutions and methods of administration.
E. Can be given by central intravenous infusion line(CVL)

8. Acute abdominal pain which is (all correct except one)


A. Colicky in nature indicates obstruction of hollow viscus.
B. In right upper quadrant increased by inspiration is typical of cholecystitis
C. Continuous is typical of inflammation
D. Maximal in the right loin is typical with duodenal ulcer.
E. Maximal in epigastrium and related to meal is typical of gastric problem.

9. Faeculent vomiting:
A. Is commonly seen after upper gastrointestinal tract.
B. Indicates large bowel obstruction.
C. Indicates bacterial proliteration in the upper intestinal.
D. Suggests a gastro-colic fistula.
E. Suggests small bowel fistula

10. Perforated duodenal ulcers: (all correct except one)


A. Occur most frequently in female
B. Are usually preceded by an exacerbation of ulcer symptoms.
C. Are usually accompanied by a leucocytosis.
D. Produce abdominal tenderness which is most marked in the epigastrium.
E. May cause septicemia.

11. A perforated duodenal ulcer:


A. Usually lies on the anterior or superior surface of the duodenum.
B. Usually presents with the acute onset of severe back pain.
C. Produces radiological evidence of free gas in the peritoneum in over 90 percent of
the patients.
D. Is usually treated by vagotomy and pyloroplasty
E. Is usually treated conservatively.

12. Appendicitis is all true except:


A. More common in females
B. Distributed evenly thoughout the world's population
C. More likely to occur if the appendix is in the retrocaecal position.
D. Commonly the result of appendicular obstruction.

13. Patients with early appendicitis:


A. Usually present with central abdominal pain.
B. Rarely present with anorexia.
C. Have usually vomited many time.
D. Usually complain of similar attacks of pain in the previous few weeks.
E. B&C only.

14. Obstruction of the lumen of the appendix may lead to: (all correct except one)
A. Mucosal ulceration
B. Gangrenous appendicitis.
C. A perforated appendix.
D. Intussusception of the appendix.
E. Acute appendicitis.

15. In the differential diagnosis of appendicitis in an infant it's important to


consider: (all correct except)
A. Ileo-ileal instussusception.
B. Basal pneumonia.
C. Henoch-schoenlein purpura.
D. Torsion of an ovarian cyst.
E. Gastroenteritis.

16. Acute non-specific mesenteric lymphadenitis: (all correct except one)


A. Is commonest between 5 and 12 years of age.
B. Is usually associated with an upper respiratory tract infection.
C. Is usually associated with cervical lymphadenopathy
D. Is characterized by enlarged mesenteric lymph nodes which are infected by gram-
negative organisms.
E. Is one of the differential diagnosis of acute appendicitis.

17. The level of intestinal obstruction can be determined by:(all correct except one)
A. Questioning the patient.
B. Examining the patient.
C. Radiological examination of the patient.
D. Repeated measurements of the patient's girth.
E. Upper & lower contrast studies.

18. Acute small bowel obstruction: (all correct except one)


A. Is commonly caused by postoperative adhesions.
B. Accompanied by the signs of peritonitis, suggests bowel strangulation.
C. Is often associated with a raised serum amylase.
D. Generally produces abdominal distension within 2 to 3 hours of onset.
E. Can be diagnosed by history and clinical examination

19. Strangulation of the bowel: (all correct except one)


A. commonly complicates closed loop obstruction.
B. Is difficult to distinguish from simple intestinal obstruction.
C. Is accompanied by bleeding into the affected bowel.
D. Frequently causes peritonitis.
E. Is characterized by severe abdominal pain.
20. Large bowel obstruction:
A. Is most commonly caused by colonic cancer
B. Has its maximum incidence before the age of 50.
C. Frequently presents with nausea and vomiting.
D. Usually heralds its onset with constant suprapubic pain.
E. Frequently treated conservatively.

21. Acute pancreatitis typically: (all correct except one)


A. Is accompanied by hypercalcaemia.
B. Produces paralytic ileus.
C. Is associated with a pleural effusion.
D. Produces pyloric stenosis.
E. Upper abdominal pain and vomiting.

22. Patients with acute colonic diverticulitis:


A. Often give a history of recent lower abdominal colic.
B. Often present with pyrexia.
C. Can be frequently diagnosed on sigmoidoscopic appearances.
D. Frequently develop faecal peritonitis.
E. All the above.

23. Acute pancreatitis: (all correct except one)


A. Often simulates a perforated peptic ulcer in its presentation.
B. Often presents with the signs of hypovolaemia.
C. Can readily be distinguished from other causes of acute abdominal pain by the
presence of a raised serum amylase.
D. Frequently has a raised concentration of urinary amylase.
E. Most commonly caused as a complication of GB stones.

24. Childhood intussusception: (all correct except one)


A. Usually presents during the first year of life.
B. Is frequently ileocolic.
C. Can usually be diagnosed without x-ray examination of the abdomen.
D. Rarely requires surgical treatment.
E. Can be diagnosed by abdominal US.

25. Neonatal duodenal obstruction:


A. May be associated with down's syndrome.
B. Is more frequently found in premature infants.
C. Typically presents with gross abdominal distension.
D. Usually presents with vomiting of non-bile stained fluid
E. B&C only.

26. acute superior mesenteric artery occlusion: (all correct except one)
A. Characteristically presents with sudden pain and tenderness of increasing intensity.
B. Is frequently accompanied by overt or occult blood loss in the stools.
C. Frequently produces peritonitis.
D. Can usually be diagnosed on plain abdominal x-rays.
E. Can be diagnosed by mesenteric artery ongiography.

27. Biliary colic typically:


A. Occurs 3 to 4 hours after meals.
B. Lasts 5 to 20 minutes.
C. Radiates from the upper abdomen to the right subscapular region.
D. Is made better by deep inspiration.
E. B&C only.

28. Regarding tension pneumothorax, the first step in the management is:
A. Obtaining a stat chest x-ray.
B. Cricothyroidectomy
C. Passin on endotracheal tube
D. Starting oxygen by a valve-mask device
E. Chest decompression needle.

29. Regarding the management of polytrauma:


A. Death follow a trimodal distribution.
B. X-ray after primary survey should be AP cervical spine, chest and pelvis.
C. Cardiac tamponade is characterized by raised B.p, a low JUP.
D. Assessment of uncomplicated limb fractures should occur during the primary
survey.
E. A and B only.

30. Blunt injuries to the abdomen (all are correct except)


A. May cause shock
B. May cause peritonitis
C. May cause acute gastroduodenal ulceration
D. May treated conservativly
E. Rarely need urgent laparatomy
31.Injuries to the urethra (all are correct except one)
A. Are more common in male.
B. Are often caused by road traffic accidents.
C. Are easily diagnosed on intra venous pyelography.
D. Require urgent surgical treatment.
E. Diagnosed by antegrade urethragraphy.
32.Car seat belts when properly adjusted
A. Prevent injuries to abdominal organs.
B. May cause small bowel injuries.
C. Do not reduce the incidence of head injuries of passengers involving in RTA.
D. Protect the cervical spine during sudden acceleration .
E. A & D only.

33. In head trauma patient after control of air way , the first diagnostic study
A. X-rays of the skull.
B. CT scan of the head.
C. X-rays of cervical spine
D. Carotid angiography.
E. Lumber puncture.

34. Estimation of the area of a burn: ( all correct except one)


A. Is of very little clinical significance.
B. Provides important prognostic information.
C. Is an important factor in the estimation of the fluid required.
D. Can be based on a formula which states that the adult trunk is 36 per cent of the
whole body surface area.
E. Should be recorded in each chart of burned patient.

35. Patients with major burns:


A. Are in a negative nitrogen balance.
B. Have normal calorie requirements.
C. Do not generally become anaemic.
D. Are resistant to septicaemia.
E. All of the above.

36. Scalds:
A. Are more frequent in children.
B. Commonly cause full thickness skin loss.
C. Should be skin grafted within 48 hours of the injury.
D. Need routine antibiotic treatment.
E. All of the above.

37. Major burns are sometimes complicated by: (all correct except one)
A. Acute gastric and duodenal ulcers.
B. Paralytic ileus.
C. Cerebral oedema.
D. Mesenteric thrombosis.
E. Septicemia.

38.In all forms of shock there is:


A. An impairment of cellular oxygenation.
B. A decreased cardiac output.
C. An increased effective circulating fluid volume.
D. A low central venous pressure (CVP).
E. An increased pulse rate.

39.The metabolic acidosis of shock can be effectively treated by:


A. Warming the patient.
B. Administering ammonia chloride.
C. Artificial ventilation.
D. Restoring normal tissue perfusion.

40.. septic shock is associated with a hypodynamic cardiovascular state: ( all correct
except one)
A. if preceded by existing hypovolaemia.
B. In generalized peritonitis.
C. When there is a gram-positive bacteraemia.
D. In elderly patients.
E. In late gram negative septicemia.

41. In cardiogenic shock:


A. The central venous pressure is high.
B. The difference in the arteriovenous oxygen tension is increased.
C. The haematocrit is raised.
D. The blood pressure is unaffected.
E. C&D only.

42. A blood transfusion reaction: (all correct except one)


A. May be due to incompatibility of the recipient serum and donor cells.
B. Is manifest by thrombophlebitis of the infusion site.
C. Occurs within the first 30 minutes of transfusion.
D. May produce renal damage.
E. May produce anaphylactic shock.

43. Massive blood transfusions may be complicated by all true except:


A. Hyperkalaemia.
B. Hypocalcaemia.
C. Coagulopathy.
D. Leucopenia.
E. DIC.

44. In a healing fracture: (All correct except one)


A. The haematoma is initially invaded by osteoblasts.
B. The tissue formed by the invading osteoblasts is termed osteoid.
C. Calcium salts are laid down in the osteoid tissue.
D. The final stage of repair is the remodelling of the callus.
E. The callus formation is related to the amount of stress at fracture side.

45. Non-union is often seen in:


A. Fractures of the 4th metatarsal.
B. Fractures of the neck of the femur.
C. Fractures of the condyle of the mandible.
D. Colles’ fractures.
E. Oblique fracture line of femur.

46. In a colles’ fracture the distal radial fragment:


A. Is dorsally angulated on the proximal radius.
B. Is usually torn from the intra-articular triangular disc.
C. Is deviated to the ulnar side.
D. Is rarely impacted.
E. Is ventrally displaced.

47. A malignant melanoma:


A. Frequently arises from hair-bearing naevi.
B. Frequently arises from junctional naevi.
C. Has a worse prognosis when it areses on the leg.
D. Should be suspected in any big pigmented lesion.
E. Non of the above is correct.

48. Squamous cancer of the lip:


A. Is most common in early adult life.
B. Is more common in fair skinned subjects.
C. Metastasises readily by the blood stream.
D. Is preferably treated by radiotherapy once lymph node deposits are present.
E. All of the above are correct.

49. Basal cell carcinomas:


A. Usually metastasise to regional lymph nodes.
B. Are less common than squamous cell carcinomas.
C. Are characterised histologically by epithelial pearls.
D. Are particularly common in oriental races.
E. Non of the above is correct.
50. Fiboadenomata of the breast:
A. Are commonest in early adult life.
B. Are indiscrete and difficult to distinguish.
C. Are usually painless.
D. Resolve without treatment.
E. A&C only.

51. Paget’s disease of the nipple:


A. Usually presents as abilateral eczema of the nipple.
B. Is always related to an underlying breast cancer.
C. Indicates incurable breast cancer.
D. Has non-specific histological characteristics.
E. A&C only

52. Gallstones: (all correct except one)


A. Have an incidence which increases with age.
B. Are more frequent in females.
C. Usually contain a predominance of cholesterol.
D. Are formed in bile which is supersaturated with bile acids.
E. Are formed in bile which is supersaturated with cholesterol.

53. stones in the common bile duct:


A. Are present in nearly 50 per cent of cases of cholecystitis.
B. Often give rise to jaundice, fever and biliary colic.
C. Are usually accompanied by progressive jaundice.
D. Are usually associated with a distended gallbladder.
E. A&D only

54. Ulcerative colitis:


A. Is more common in female than males.
B. Appears most commonly between the ages of 20 and 30.
C. Usually presents with abdominal discomfort and diarrhoea.
D. Can usually be diagnosed on sigmoidoscopic examination.
E. All of the above correct.

55. Diverticular disease of the colon:


A. Is usually asymptomatic.
B. Often presents with lower abdominal pain.
C. May present with severe rectal haemorrhage.
D. May present with peritonitis.
E. All of the above are correct.
56. Colonic polyps: (all correct except one)
A. Are associated with colonic cancer.
B. May be hereditary.
C. Should not be removed if they are asymptomatic.
D. May be hyperplastic.
E. Are commonly adenomatous.

57. Signs and symptoms of hyperthytoidism include: (All correct except one)
A. Decreased sweating.
B. An irregular pulse rate.
C. Cardiac failure.
D. Diplopia.
E. Loss of weight.

58. a multinodular (adenomatous) goitre:


A. Is more common in those patients having a deficient iodine intake.
B. Is usually preceded by a diffuse goitre in early adult life
C. Is rarely a precancerous condition.
D. Is effectively treated in early stage with thyroid hormones to prevent further
adenomatous changes.
E. All of the above.

59. Hashimoto’s disease:


A. Is often associated with increased levels of circulating thyroid antibodies.
B. Is characterized by lymphocytic infiltration and fibrosis of the thyroid gland.
C. Is usually treated by sub-total thyroidectomy.
D. Should be treated by anti-thyroid drugs.
E. A&B only.

60. The surgical treatment of thyroid cancer:


A. Should be by total thyroidectomy in the potentially curable patient.
B. Should include block dissection of the neighboring lymph nodes whether or not
they appear to contain tumour.
C. Is most successful in the papillary type of cancer.
D. Should be preceded by a therapeutic dose of iodine 131.
E. A& C only.

61. The earliest symptoms of hyperparathyroidism include:


A. Diarrhea.
B. Polydipsia and polyuria.
C. Unexplained weight gain.
D. Muscle spasm
E. C&D only.
62. Phaeochromocytomata:
A. Are tumors of the spinal nerve roots.
B. Are frequently multiple.
C. Characteristically present with a yellow skin discoloration.
D. Can be effectively managed by long term medical therapy.
E. C&D only.
63. In cushing’s syndrome:
A. There is usually an increased deposition of fat over the face and trunk.
B. Hypotension and hypokalaemia are frequently present.
C. There is an increased production of adrenal hormones.
D. The cause is most frequently an adrenal tumor.
E. A&C only.

64. Streptococcal infections:


A. Are characterised by abscess formation.
B. Rarely produce lymphadenitis.
C. Do not produce baceraemia.
D. Can produce a gangrenous skin infection.
E. All of the above.

65. A pelvic abscess:


A. Lies extrapeitoneally.
B. May be a complication of abdominal surgery.
C. Rarely presents with diarrhoea.
D. Should be treated with antibiotics alone.
E. A&C only

66. Heavily contaminated and dirty wounds:


A. Require surgical toilet and delayed closure.
B. Require the administration of systemic antibiotics.
C. Can usually be treated by wound toilet and primary closure.
D. Should be totally excised.
E. A&B only.

67. Inguinal herniae:


A. In children are usually of the direct type.
B. Of the indirect type are congenital in origin.
C. Will regress spontaneously in children.
D. In young adults are most commonly of the direct type.
E. C&D only.
68. Strangulated contents of hernial sacs:
A. Are always accompanied by intestinal obstruction.
B. Are more common in direct than indirect inguinal herniae.
C. Are usually reducible.
D. Produce local pain and tenderness.
E. All of the above
69. Incisional herniae are related to:(ِAll correct except one)
A. Wound infections.
B. Anaemia and malnutrition.
C. Obesity.
D. The use of absorbable suture materials.
E. To the technique of wound closure.
70. True statements concerning a femoral hernia include which of the following?
A. It is common in male.
B. It is usually results from a defect in lateral part of transversalis fascia.
C. It is common in children.
D. It is less common in female.
E. It may be confused with inguinal lymphadenopathy.
71. Acute arterial occlusion:
A. Should be treated conservatively if the site of the occlusion is above the inguinal
ligament.
B. Demands the urgent use of vasodilator drugs.
C. Of a limb is usually painless due to the anoxic damage produced in the peripheral
nerves.
D. May produce irreversible muscle necrosis after 6 hours.
E. B&C only.

72. Common sites for atheromatous arterial aneurysms are:


A. The femoral artery.
B. The middle cerebral artery.
C. The abdominal aorta.
D. Intrarenal.
E. A&C only
73. The long saphenous vein:
A. Arises on the medial aspect of the sole of the foot.
B. Passes 1cm in front of the medial malleolus.
C. Passes in front the knee joint.
D. Enters the femoral sheath by piercing the fascia lata.
E. Related to the sural nerve

74. In deep venous thrombosis of the lower limb:


A. One of the most common sites of origin is the short saphenous vein.
B. One of the common sites of origin is in the iliofemoral segment.
C. The diagnosis can usually be made by clinical examination.
D. Tender swollen thrombosed veins are usually palpable.
E. Usually associated with varicose vein.
75. In head injuries the causes of a rising intracranial pressure: (all correct except
one)
A. Intracerebral haemorrhage.
B. Cerebral oedema.
C. Rhinorrhoea.
D. Meningitis.
E. Extradural hemorrhage.

76. Birth injuries involving the fifth and sixth cervicaal nerve roots of the brachial
plexus:
A. Are known as klumpke’s palsy.
B. Are rarely followed by full recovery.
C. Are characterised by the arm being held in the pronated and internally rotated
position.
D. Show weakness and wasting of the small muscles of the hand.
E. Non of the above correct.
77. Following a peripheral nerve injury:
A. Loss of axon continuity is described as neuropraxia.
B. Due to gunshot wounds primary nerve repair is desirable.
C. Delayed suture is best performed one week after the injury.
D. Delayed suture is best performed three months after the injury.
E. None of the above is correct.

1. The best test to monitor adequacy of L-Thyroxine therapy is :


A. radioactive isotope thyroid scan
B. thyroglobulin blood level
C. total T4 blood level
D. free T3 blood level
E. blood level of TSH

2. Hashimoto s thyroiditis :
A. is of viral origin
B. presents usually by nodular goiter
C. can be diagnosed by thyroid antibodies and fine needle aspiration of thyroid
D. is often premalignant
E. must be treated always by surgery

3. Thyrotoxicosis usually manifests by all of the following EXCEPT:


A. loss of weight inspite of good appetite
B. a recent preference to cold
C. palpitation
D. more common in male than in female
E. excitability of the patient

4. Exophthalmos :
A. is always present in thyrotoxicosis
B. is always bilateral
C. can cause diplopia in severe cases
D. decreases by hypothyroidism
E. is cured by surgery to the thyrotoxic goiter

5. Complications after thyroidectomy include all the following EXCEPT:


A. parathyroid insufficiency
B. recurrent laryngeal nerve paralysis
C. tracheomalacia
D. thyrotoxic crisis(storm) on operating on inadequitly prepared thyrotoxic patient
E. hypercalcemia

6. Regardig Carcinoma of the thyroid gland:


A. common diffrenciated types are the papillary and the follicular carcinoma
B. medullary carcinoma of the thyroid is treated only by resection of the involved
lobe
C. papillary carcinoma never metastasize to cervical lymph nodes
D. anaplastic carcinoma is more common in the young age group

7. Tuberculous cervical lymphadenitis:


A. is caused by bovine tuberculous bacillus rather than the human bacillus
B. usually occurs through the tonsil of the corresponding side
C. collar-stud abscess is never a feature of T. B adenitis
D. is treated always by surgery
E. biopsy is not necessary for diagnosis
8. Branchial cyst:
A. develops from the third branchial cleft
B. presence of cholesterol crystals in aspirated fluid from the cyst is
pathognomonic to it
C. should be treated with repeated aspiration
D. is not related to carotid vessles,hypoglossal and spinal accessory nerves
E. is never inflamed

9. Cystic hygroma:
A. develops as a sequestration of a portion of a jugular lymph sac
B. typically occupies the upper one third of the neck
C. occasionally may occur in cheek ,axilla and groin but exceptionally in tongue and
mediastinum
D. repeated aspiration is the only treatment
E. A and C are correct

10. Carotid body tumour:


A. is chromffin paragranuloma
B. is usually bilateral
C. is called chemodactoma or potato tumour
D. is situated ot the bifurcation of the carotid artery
E. C and D are correct

11. Differential diagnosis of lateral sided neck swelling:


A. cannot be diagnosed by fine needle asoiration
B. can be thyroglssal cyst
C. can be a lymphoma when having other characteristic features
D. cannot be diffrenciated into solid or cystic by ultrasound
E. is never a reactive lymphadenitis or secondary malignant disease

12. The breast :


A. is a modified apocrine sweat gland
B. overlies the third to the sixth rib
C. is having usually more than 30 lacteferous ducts
D. is drained only by six groups of axillary lymph nodes
E. is having no attachment to skin

13. The commonest cause of bloody nipple discharge is :


A. mammary ductectasia
B. carcinoma of the breast
C. lactational mastitis
D. duct papilloma
E. fibrocystic disease of the breast

14. All of the following may be manifestations of breast carcinoma EXCEPT


A. peau d orange of covering skin
B. no mass may be felt clinically
C. enlargement of the ipsilateral axillary lymph nodes
D. cervical spine metastasis
E. none of the above

15. Bacterial mastitis:


A. never occurs in lactating mothers
B. is usually caused by streptococcus hemolyticus
C. is mostly caused by staphylococcus aureus
D. is treated only by incision and drainage
E. is the same as mastitis of puberty

16. Signs of malignancy on mammogram may include all of the following EXCEPT :
A. mass lesions with poorly defined irregular margins
B. fine stipped soft tissue with periductal and not vascular microcalcifications
C. thickening and retraction of the overlying skin
D. dysplastic ductal pattern
E. well circumscribed , homogenous,and often surrounded by a zone of fatty tissue

17. Early detection of carcinoma of the breast includes all of the following EXCEPT:
A. self-examination just after menstruation
B. screening mammography
C. frequent consultations among the high risk group patients
D. presence of fixed breast mass to chest wall with skin changes on clinical
examination
E. A, B, C, are correct

18. Regarding Paget s disease of the nipple :


A. It is a benign condition
B. It is simply an eczematous lesion of the nipple
C. It is treated by excision of the nipple
D. It is usually diagnosed by biopsy of the suspected lesion
E. the areola and the surrounding skin are never involved

19. in a patient with fibroadenosis of the breast ;


A. cyst formation, adenosis, fibrosis,epitheliosis and papillomatosis are invariably
present
B. is premalignant
C. pregnancy usually produces relief
D. presentation may include pain, nipple discharge, and/ or breast lump
E. all of the above are usually characteristics
20. Regarding gynaecomastia :
A. it is hypertrophy of female breast
B. it is hypertrophy of male breast
C. it may associate leprosy and liver failure
D. it may associate cimetidine, spironolactone, INH, or digitalis thrapy
E. B, C, and D are correct

21. Regarding modified radical mastectomy all are correct EXCEPT :


A. it is indicated recently for T1N0M0
B. pectoralis major muscle is excised
C. axillary clearance is mandatory
D. adjuvant radiotherapy must be given for all patients post-operatively
E. the long thoracic nerve of Bell (nerve to serratous anterior) must be preserved but
nerve to latismus dorsi might be sacrified

22. Mammary duct ectasia is characterized by the following EXCEPT :


A. is defiened as primary dilatation of major ducts of breast in middle aged women
B. may present with nipple retraction and Peau d orange picture
C. is treated usually by simple mastectomy
D. anaerobic superinfection commonly occurs in this recurrent periductal plasma cell
mastitis
E. is commonly pre-malignant
23. Regarding carcinoma of the breast :
A. invasive intraductal carcinoma is the commonest form
B. lobular carcinoma may present bilateral
C. clinical staging is always correct and definit
D. the medullary (anaplastic) type feels hard
E. A, and B, are correct

A pregnant woman in her 32nd wk of gestation is given magnesium sulfate for .1


pre-eclampsia . The earliest clinical indication of hypermagnesemia is
A. Loss of deep tendon reflexes
B. Flaccid paralysis
C. Respiratory arrest
D. Hypotension
The chief surgical risk to which patients with polycythemia vera are exposed is .2
that due to
A. Anemic disturbances
B. Hemorrhage
C. Infection
D. Renal dysfunction

The surgeon should be particularly concerned about which coagulation function .3


?in patients receiving anti-inflammatory or analgesic medications
A. APTT
B. PT
C. Reptilase time
D. Bleeding time

The substrate depleted earliest in the postoperative period is .4


A. Branched-chain amino acids
B. Non-branched-chain amino acids
C. Ketone
D. Glycogen
E. Glucose
Signs and symptoms of hemolytic transfusion reactions include .5
A. Hypothermia
B. Hypertension
C. Polyuria
D. Abnormal bleeding

A patient suspected of having a hemolytic transfusion reaction should be managed .6


with
A. Removal of nonessential foreign body irritants, e.g., Foley catheter
B. Fluid restriction
C. 0.1 M HC1 infusion
D. Steroids
E. Fluids and mannitol

In a hemolytic reaction caused by an incompatible blood transfusion, the .7


treatment that is most likely to be helpful is
A. Promoting a diuresis with 250 ml of 50% mannitol
B. Treating anuria with fluid and potassium replacement
C. Acidifying the urine to prevent hemoglobin precipitation in the renal tubules
D. Removing foreign bodies, such as Foley catheters, which may cause hemorrhagic
complications
E. Stopping the transfusion immediately

The enteric fluid with an electrolyte (Na+, K+, C1 ) content similar to that of .8
Ringer’slactate is
A. Saliva
B. Contents of small intestine
C. Contents of right colon
D. Pancreatic secretions
E. Gastric juice

Which of the following medications administered for hyperkalemia counteracts .9


?the myocardial effects of potassium without reducing the serum potassium level
A. Sodium polystyrene sulfonate(Kayexalate)
B. Sodium bicarbonate
C. 50% dextrose
D. Calcium gluconate
E. Insulin
Hypocalcemia is associated with .10
A. Acidosis
B. Shortened QT interval
C. Hypomagnesemia
D. Myocardial irritability

?Which of the following might increase the risk of a wound infection .11
A. History of colon surgery
B. Hypertension
C. Male sex
D. Receipt of chemotherapy
E. Asthma

A correct statement regarding 5% dextrose in 0.9% saline is which of the .12


?following
A. It contains the same concentration of sodium ions as does plasma
B. It can be given in large quantities without seriously affecting acid-base balance
C. It is isosmotic with plasma
D. It has a pH of 7.4
E. It may cause a dilutional acidosis

Correct statements regarding lactated Ringer’s solution include which of the .13
?following
A. It contains a higher concentration of sodium ions than does plasma
B. It is most appropriate for replacement of nasogastric tube losses
C. It is isosmotic with plasma
D. It has a pH of less than 7.0
E. It may induce a significant metabolic acidosis

Prophylactic regimens of documented benefit in decreasing the risk of .14


postoperative thromboembolism include
A. Early ambulation
B. External pneumatic compression devices placed on the upper extremities
C. Elastic stockings
D. Leg elevation for 24 h postoperatively

Signs and symptoms associated with early sepsis include .15


A. Respiratory acidosis
B. Decreased cardiac output
C. Hypoglycemia
E. Cutaneous vasodilation

The most common physiologic cause of hypoxemia is .16


A. Hypoventilation
B. Incomplete alveolar oxygen diffusion
C. Ventilation-perfusion inequality
D. Pulmonary shunt flow

Generally accepted indications for mechanical ventilatory support include .17


A. PaO2 of less than 70 kPa and Pa greater than 50 kPa while breathing room air
B. Alveolar-arterial oxygen tension difference of 150 kPa while breathing 100% O2
C. Vital capacity of 40–60 mL/kg
D. Respiratory rate greater than 35 breaths/min
E. A dead space:tidal volume ratio (VD /VT) less than 0.6 CO2 of

Which of the following inhalation anesthetics accumulates in air-filled cavities .18


?during general anesthesia
A. Diethyl ether
B. Nitrous oxide
C. Halothane
D. Methoxyflurane
E. Trichloroethylene

Major alterations in pulmonary function associated with adult respiratory .19


distress syndrome (ARDS) include
A. Hypoxemia
B. Increased pulmonary compliance
C. Increased resting lung volume
E. Decreased dead space ventilation

Dopamine is a frequently used drug in critically ill patients because .20


A. At high doses it increases splanchnic flow
B. At high doses it increases coronary flow
C. At low doses it decreases heart rate
D. At low doses it lowers peripheral resistance

If end-diastolic pressure is held constant, increasing which of the following will .21
?increase the cardiac index
A. Peripheral vascular resistance
B. Pulmonary wedge pressure
C. Heart rate
D. Systemic diastolic pressure

The preoperative characteristics of patients likely to experience postoperative .22


ischemia after non cardiac surgery include
A. Angina
B. More than three premature ventricular contractions (PVCs) per minute
C. Dyspnea on exertion
D. Tricuspid regurgitation

Compensatory mechanisms during acute hemorrhage include .23


A. Decreased cerebral and coronary blood flow
B. Decreased myocardial contractility
C. Renal and splanchnic vasodilation
D. Increased respiratory rate
E. Decreased renal sodium resorption

An 18-year-old woman develops urticaria and wheezing after an injection of .24


penicillin. Her blood pressure is 120/60 mm Hg, heart rate is 155 beats/min, and
respiratory rate is 30 breaths/min. Immediate therapy should include
A. Intubation
B. Epinephrine
C. Beta blockers
D. Iodine
E. Fluid challenge

During blood transfusion, clotting of transfused blood is associated with .25


A. ABO incompatibility
B. Minor blood group incompatibility
C. Rh incompatibility
D. Transfusion through Ringer’s lactate

When an arterial blood gas determination of PCO2 40 kPa is obtained .26


A. There is probably a paradoxical aciduria
B. Alveolar ventilation is adequate
C. Arterial P will indicate the adequacy of alveolar ventilation
D. Arterial PO2 will indicate the degree of ventilation-perfusion mismatch
E. Arterial PO2 can be safely predicted to exceed 90 kPa on room air
Among patients who require nutritional resuscitation in an intensive care unit, .27
the best evidence that nutritional support is adequate is
A. Urinary nitrogen excretion levels
B. Total serum protein level
C. Serum albumin level
D. Serum transferrin levels

Paradoxical aciduria (the excretion of acid urine in the presence of metabolic .28
alkalosis) may occur in the presence of
A. Release of inappropriate antidiuretic hormone
B. Severe crush injury
C. Acute tubular necrosis
D. Gastric outlet obstruction

If a patient suffered a pulmonary arterial air embolism during an open .29


thoracotomy, the anesthesiologist’s most likely observation would be
A. Unexpected systemic hypertension
B. Rising right atrial filling pressures
C. Reduced systemic arterial oxygen saturation
D. Rising systemic CO partial pressures
E. Falling end-tidal CO2

A 72-year-old man undergoes resection of an abdominal aneurysm. He arrives in .30


the ICU with a core temperature of 33°C (91.4°F) and shivering. The physiologic
consequence of the shivering is
A. Rising mixed venous oxygen saturation
B. Increased production of carbon dioxide
C. Decreased consumption of oxygen
D. Rising base excess
E. Decreased minute ventilation

To prepare for operating on a patient with a bleeding history diagnosed as von .31
Willebrand’s disease (recessive), you would give
A. High-purity factor VIII:C concentrates
B. Low-molecular-weight dextran
C. Fresh frozen plasma (FFP)
D. Cryoprecipitate
E. Whole blood
The accidental aspiration of gastric contents into the tracheobronchial tree .32
should be initially treated by
A. Tracheal intubation and suctioning
B. Steroids
C. Intravenous fluid bolus
D. Cricothyroidotomy

In performing a tracheostomy .33


A. The strap muscles should be divided
B. The thyroid isthmus should be preserved
C. The trachea should be entered at the second or third cartilaginous ring
D. Only horizontal incisions should be used

If malignant hyperthermia is suspected intraoperatively .34


A. Complete the procedure but pretreat with dantrolene prior to future elective surgery
B. Administer inhalational anesthetic agents
C. Administer succinylcholine
D. Hyperventilate with 100% oxygen
E. Acidify the urine to prevent myoglobin precipitation in the renal tubules

Central venous pressure (CVP) may be decreased by .35


A. Pulmonary embolism
B. Hypervolemia
C. Positive-pressure ventilation
D. Pneumothorax
E. Gram-negative sepsis

Characteristics of continuous arteriovenous hemofiltration (CAVH) in the .36


treatment of surgical patients with acute renal failure include
A. CAVH is useful only in hemodynamically stable patients
B. CAVH requires placement of large bore(8 French) arterial and venous catheters,
usually in the femoral vessels
C. CAVH is not effective in treating hypervolemia
D. Continuous heparinization of the patient who undergoes CAVH is unnecessary
E. During CAVH, blood flow is maintained by a
mechanical extracorporeal pump–oxygenator

Signs and symptoms of unsuspected Addison’s disease include .37


A. Hypothermia
B. Hypokalemia
C. Hyperglycemia
D. Hyponatremia
E. Hypervolemia

The etiologic factor implicated in the development of pulmonary insufficiency .38


following major non thoracic trauma is
A. Aspiration
B. Atelectasis
C. Fat embolism syndrome
D. Fluid overload

For the severely traumatized patient requiring airway management .39


A. Awake endotracheal intubation is indicated in patients with penetrating ocular
injury
B. Steroids have been shown to be of value in the treatment of aspiration of acidic
gastric secretions
C. The stomach may be assumed to be empty only if a history is obtained indicating
no ingestion of food or liquid during the prior 8 h
D. Intubation should be performed in the emergency room if the patient is unstable
E. Cricothyroidotomy is contraindicated in the presence of maxillofacial injuries

Treatment for clostridial myonecrosis (gas gangrene) Includes which of the .40
?following measures
A. Administration of an antifungal agent
B. Administration of antitoxin
C. Wide debridement
D. Administration of hyperbaric oxygen

An abnormal ventilatioperfusion ratio (Qs/Qr) in the postoperative patient has .41


been associated with
A. Pulmonary thromboembolism
B. Lower abdominal surgery
C. starvation
D. The upright position
E. Increased cardiac output

Correct statements concerning drowning or near-drowning include which of the .42


?following
A. The prognosis for recovery of cerebral function in affected persons is better if
submersion occurs in warm water rather than extremely cold water
B. A majority of victims will demonstrate a severe metabolic alkalosis
C. Prompt administration of corticosteroids to affected persons has been shown to
decrease the extent of pulmonary membrane damage
D. Renal damage may occur in affected persons as a result of hemoglobinuria
E. The most important initial treatment of drowning victims is emptying the stomach
of swallowed water
Spontaneous retroperitoneal hemorrhage during anticoagulant therapy .43
A. Is best confirmed by bleeding scan
B. Is equally likely with parenteral and oral anticoagulants
C. May mimic an acute surgical abdomen
D. Frequently requires laparotomy for ligation of the bleeding site

Indications for surgical intervention to remove smuggled drug packets that have .44
been ingested include
A. Refusal to take high doses of laxatives
B. Refusal to allow endoscopic retrieval
C. Refusal to allow digital rectal disimpaction
D. Intraintestinal drug packets evident on abdominal x-ray in an asymptomatic
smuggler

1. Wasting of the intrinsic muscles of the hand can be expected to follow injury of
the
A. Ulnar nerve
B. Radial nerve
C. Brachial nerve
D. Axillary nerve

2. A 30-year-old man is stabbed in the arm. There is no evidence of vascular injury,


but he cannot flex his three radial digits. He has injured the
C. Median nerve

3. Although wide surgical excision is the traditional treatment for malignant


melanoma, narrow excision of thin (less than 1 mm deep) stage I melanomas has
been found to be equally safe and effective when the margin of resection is as small
as
C. 1 cm

4. With regard to wound healing, which one of the following statements is correct?
B. Monocytes are essential for normal wound healing
5. While you are on duty in the emergency room, a 12-year-old boy arrives with pain
and inflammation over the ball of his left foot and red streaks extending up the inner
aspect of his leg. He remembers removing a wood splinter from the sole of his foot
on the previous day. The most likely infecting organism is
E. Streptococcus
6. The appropriate antibiotic to prescribe while awaiting specific culture verification
is
A. Penicillin
7. Proper treatment for frostbite consists of
D. Immersion of the affected part in water at 40–44°C (104–111.2°F)
8. The true statement regarding tendon injuries in the hand is
E. The process of healing a tendon injury involves formation of a tenoma

9. Which one of the following cases is considered a clean contaminated wound


A. Open cholecystectomy for cholelithiasis
B. Herniorrhaphy with mesh repair
C. Lumpectomy with axillary node dissection
D. Appendectomy with walled-off abscess
10. True statements regarding squamous cell carcinoma of the lip include
A. The lesion often arises in areas of persistent hyperkeratosis
B. More than 90% of cases occur on the upper lip
C. The lesion constitutes 30% of all cancers of the oral cavity
D. Radiotherapy is considered inappropriate treatment for these lesions

11. Which of the following statements regarding carpal tunnel syndrome is correct?
B. It may be associated with pregnancy

12. Which of the following is true with regard to wound contraction?


D. It is based on specialized fibroblasts that contain actin myofilaments

13. Management of leukoplakia of the oral cavity includes


A. Excisional biopsy of all lesions
B. Application of topical antibiotics
C. Low-dose radiation therapy
D. Ascertaining that dentures fit properly

14. An 8-lb infant, born following uncomplicated labor and delivery, is noted to have
a unilateral cleft lip and palate .The parents should be advised that
A. The child almost certainly has other congenital anomalies
B. Rehabilitation requires adjunctive speech therapy
C. Lip repair is indicated at 1 year of age
D. Palate repair is indicated prior to 6 mo of age

15. A teenage boy falls from his bicycle and is run over by a truck. On arrival in the
emergency room, he is awake and alert and appears frightened but in no distress.
The chest radiograph suggests an airfluid level in the left lower lung field and the
nasogastric tube seems to coil upward into the left chest. The next best step in
management is
C. Immediate celiotomy

16. Which of the following conditions is most likely to follow a compression-type


abdominal injury?
A. Renal vascular injury
B. Superior mesenteric thrombosis
C. Mesenteric vascular injury
D. Avulsion of the splenic pedicle
E. Diaphragmatic hernia

17. A 65-year-old man who smokes cigarettes and has chronic obstructive pulmonary
disease falls and fractures the 7th, 8th, and 9th ribs in the left anterolateral chest.
Chest x-ray is otherwise normal. Appropriate treatment might include
D. Peritoneal lavage

18. A 27-year-old man sustains a single gunshot wound to the left thigh. In the
emergency room he is noted to have a large hematoma of his medial thigh. He
complains of paresthesias in his foot. On examination there are weak pulses palpable
distal to the injury and the patient is unable to move his foot. The appropriate initial
management of this patient would be
B. Immediate exploration and repair

Items 19–20 A 25-year-old woman arrives in the emergency room


following an automobile accident. She is acutely dyspneic with a
respiratory rate of 60 breaths/min. Breath sounds are markedly
diminished on the right side.

19. The first step in managing the patient should be to


C. Decompress the right pleural space

20. A chest x-ray of this woman Before therapy would probably reveal
A. Air in the right pleural space

21. Among the physiologic responses to acute injury is


A. Increased secretion of insulin
B. Increased secretion of thyroxine
C. Decreased secretion of vasopressin (ADH)
D. Decreased secretion of glucagon

22. In a stable patient, the management of a complete transection of the common


bile duct distal to the insertion of the cystic duct would be optimally performed with
a
D. Roux-en-Y choledochojejunostomy

23. Non operative management of penetrating neck injuries has been advocated as
an alternative to mandatory exploration in asymptomatic patients. Which of the
following findings would constitute a relative, rather than an absolute, indication for
formal neck exploration?
A. Expanding hematoma
B. Dysphagia
C. Dysphonia
D. Pneumothorax
E. Hemoptysis

24. Following blunt abdominal trauma, a 12-year-old girl develops upper abdominal
pain, nausea, and vomiting. An upper gastrointestinal series reveals a total
obstruction of the duodenum with a “coiled spring” appearance in the second and
third portions. Appropriate management is
B. Nasogastric suction and observation

25. Following traumatic peripheral nerve transection, regrowth usually occurs at


which of the following rates?
A. 0.1 mm per day
B. 1 mm per day
C. 5 mm per day
D. 1 cm per day
Items 26–27 A 28-year-old man is brought to the emergency room for a
severe head injury after a fall. Initially lethargic, he becomes comatose
and does not move his right side .His left pupil is dilated and responds
only sluggishly.

26. The most common initial manifestation of increasing intracranial pressure in the
victim of head trauma is
A. Change in level of consciousness

27. Initial emergency reduction of intracranial pressure is most rapidly


accomplished by
E. Hyperventilation

28. In the patient described, compression of the affected nerve is produced by


B. Herniation of the uncal process of the temporal lobe

29. Regarding high-voltage electrical burns to an extremity


D. Evaluation for fracture of the other extremities and visceral injury is indicated

30. Which of the following fractures or dislocations of the extremities induced by


blunt trauma is associated with
significant vascular injuries?
A. Knee dislocation
31. Protein metabolism after trauma is characterized by
A. Decreased liver gluconeogenesis
B. Inhibition of skeletal muscle breakdown)
C. Decreased urinary nitrogen loss
D. Hepatic synthesis of acute-phase reactants

32. The response to shock includes which of the following metabolic effects?
A. Increase in sodium and water excretion
B. Increase in renal perfusion
C. Decrease in cortisol levels
D. Hyperkalemia
E. Hypoglycemia

33. Appropriate treatment for an acute stable hematoma of the pinna of the ear
includes which of the following measures?
A. Ice packs and prophylactic antibiotics
B. Excision of the hematoma
C. Needle aspiration
D. Incision, drainage, and pressure bandage

34. Animal and clinical studies have shown that administration of lactated Ringer’s
solution to patients with hypovolemic shock may
A. Increase serum lactate concentration
B. Impair liver function
C. Improve hemodynamics by alleviating the deficit in the interstitial fluid
compartment

35. Which of the following situations would be an indication for performance of a


thoracotomy in the emergency room?
C. Rapidly deteriorating patient with cardiac tamponade from penetrating thoracic
trauma

Items 36–37 An 18-year-old high school football player is kicked in the


left flank. Three hours later he develops hematuria.His vital signs are
stable.

36. The diagnostic tests performed reveal extravasation of contrast into the renal
parenchyma .Treatment should consist of
E. Antibiotics and serial monitoring of blood count and vital signs
37. Initial diagnostic tests in the emergency room should include which of the
following?
D. Intravenous pyelogram

38. True statements concerning penetrating pancreatic trauma include


E. The major cause of death is exsanguination from associated vascular injuries

39. A 26-year-old man sustains a gunshot wound to the left thigh. Exploration
reveals that a 5-cm portion of superficial femoral artery is destroyed .Appropriate
management includes
A. Debridement and end-to-end anastomosis
B. Debridement and repair with an interposition prosthetic graft
C. Debridement and repair with an interposition arterial graft
D. Debridement and repair with an interposition vein graft

1. Estrogen receptor activity is clinically useful in predicting


A. The presence of ovarian cancer
B. The presence of metastatic disease
C. Response to chemotherapy
D. Response to hormonal manipulation

2. When galactorrhea occurs in a high school student, a diagnostic associated finding


would be
A. Gonadal atrophy
B. Bitemporal hemianopia
C. Exophthalmos and lid lag
E. “Buffalo hump”

3. The diagnosis of primary hyperparathyroidism is most strongly suggested by


A. Serum acid phosphatase above 120 IU/L
B. Serum alkaline phosphatase above 120 IU/L
C. Serum calcium above 11 mg/dL
D. Urinary calcium below 100 mg/day

4. Somatostatin contributes to which of the following processes?


B. Inhibition of pancreatic cells

5. A 35-year-old woman undergoes her first screening mammogram. Which of the


following mammographic findings would require a breast biopsy?
A. Breast calcifications larger than 2 mm in diameter
B. Five or more clustered breast microcalcifications per square centimeter
C. A density that effaces with compression
E. Multiple round well-circumscribed breast densities

6. Which of the following statements concerning Cushing syndrome secondary to


adrenal adenoma is true?
A. Adrenal adenomas cause 40–60% of all cases of Cushing syndrome
B. Biochemical and x-ray procedures are generally unsuccessful in lateralizing the
tumors preoperatively
C. Exploration of both adrenal glands is indicated
D. For uncomplicated tumors, an open transperitoneal surgical approach is usually
employed

7. Which statement concerning radiation-induced thyroid cancer is true?


E. The treatment of choice is a neartotal (or total) thyroidectomy

8. Fibrocystic disease of the breast has been associated with elevated blood levels of
A. Testosterone
B. Progesterone
C. Estrogen
D. Luteinizing hormone
9. As an incidental finding during an upper abdominal CT scan, a 3-cm mass in the
adrenal gland is noted. The appropriate next step in analysis and management of
this finding would be
A. Observation
B. CT-guided needle biopsy
C. Excision of the mass
D. Measurement of urine catecholamine excretion
E. Cortisol provocation test

10. The most likely diagnosis in a patient with hypertension, hypokalemia, and a 7-
cm suprarenal mass is
A. Hypernephroma
B. Cushing’s disease
C. Adrenocortical carcinoma
D. Pheochromocytoma

11. Appropriate treatment of this condition would include which of the following?
A. Embolization of the arterial blood supply,
B. Metronidazole
C. Mitotane
E. Phenoxybenzamine
12. For pregnant women who are found to have breast cancer
E. Administration of adjuvant chemotherapy is safe for the fetus during the
second and third trimesters

13. True statements regarding Paget’s disease of the breast include that it
A. Usually precedes development of Paget’s disease of bone
B. Presents with nipple-areolar eczematous changes
C. Does not involve axillary lymph nodes because it is a manifestation of intraductal
carcinoma only
D. Accounts for 10–15% of all newly diagnosed breast cancers
E. Is adequately treated with wide excision when it presents as a mass

14. Of the common complicationsof thyroidectomy, the one that may be avoided
through prophylaxis is
A. Injury to the recurrent laryngeal nerve
B. Injury to the superior laryngeal nerve
D. Thyroid storm
E. Postoperative hemorrhage and wound hematoma

15. Following correction of the patient’s hypercalcemia with hydration and gentle
diuresis with furosemide, the most likely therapeutic approach would be
D. Neck exploration and resection of a parathyroid adenoma

16. This 30-year-old woman presented with weakness, bone pain, an elevated
parathormone level, and a serum calcium level of 15.2 mg/dL. Skeletal survey films
were taken, including the hand films and chest x-ray shown. The most likely cause of
these findings is
E. Primary hyperparathyroidism

17. A 36-year-old woman, 20 wk pregnant, presents with a 1.5-cm right thyroid


mass. Fine-needle aspiration is consistent with a papillary neoplasm. The mass is
“cold” by scan and solid by ultrasound. Which method of treatment would be
contraindicated?
A. Right thyroid lobectomy
B. Subtotal thyroidectomy
C. Total thyroidectomy
D. Total thyroidectomy with lymph node dissection
E. 131I radioactive ablation of the thyroid gland
18. Incisional biopsy of a breast mass in a 35-year-old woman demonstrates a
hypercellular fibroadenoma (cystosarcoma phylloides) at the time of frozen section.
Appropriate management of this lesion could include
A. Wide local excision with a rim of normal tissue

19. True statements about discharge from the nipple include


A. Intermittent thin or milky discharge can be physiologic
B. Expressible nipple discharge is an indication for open biopsy
C. Bloody discharge is indicative of an underlying malignancy
D. Galactorrhea is indicative of an underlying malignancy

20. The incidence of breast cancer


A. Increases with increasing age
B. Has declined since the 1940s
C. Is related to dietary fat intake
D. Is related to coffee intake
E. Is related to vitamin C intake

21. A cross-match is performed by incubating


B. Donor lymphocytes with recipient serum and complement

22. The primary mechanism of action of cyclosporine A is inhibition of


D. Interleukin 2 production

23. After the first postoperative year of cardiac transplantation, the most common
cause of death is
C. Accelerated graft arteriosclerosis

24. Which of the following precludes cadaveric renal transplantation?


A. Positive cross-match

25. In centers with experienced personnel, 1-year liver transplant survival is now
approximately
B. 80%

26. Graft-versus-host disease has occurred with the transplantation of which of the
following?
D. Bone marrow
27. The most useful serum marker for detecting recurrent disease after treatment of
nonseminomatous testicular cancer is
B. alpha-fetoprotein (AFP)

28. For which of the following malignancies does histologic grade best correlate with
prognosis?
E. Soft tissue sarcoma

29. Interferons are correctly characterized by which of the following statements?


B. They are produced by virus-infected cells

30. Which of the following potentially operable complications is a common


occurrence among patients receiving systemic chemotherapy?
B. Perirectal abscess

31. Human immunodeficiency virus (HIV) has been isolated from many body fluids.
Which of the following is a major source of transmission?
A. Tears
B. Sweat
C. Semen
D. Urine
E. Breast milk

32. Which of the following agents causes hemorrhagic cystitis?


E. Cyclophosphamide

33. What is the most common serious complication of an end colostomy?


A. Bleeding
B. Skin breakdown
C. Parastomal hernia
D. Colonic perforation during irrigation
E. Stomal prolapse

34. Which of the following colonic pathologies is thought to have no malignant


potential?
A. Ulcerative colitis
B. Villous adenomas
C. Familial polyposis
D. Peutz-Jeghers syndrome
E. Crohn’s colitis

35. Which of the following hernias follows the path of the spermatic cord within the
cremaster muscle?
A. Femoral
B. Direct inguinal
C. Indirect inguinal
D. Spigelian

36. Spontaneous closure of which of the following congenital abnormalities of the


abdominal wall generally occurs by the age of 4?
A. Umbilical hernia
B. Patent urachus
C. Patent omphalomesenteric duct
D. Omphalocele

37. Laparoscopic cholecystectomy is indicated for symptomatic gallstones in which


of the following conditions?
A. Cirrhosis
B. Prior upper abdominal surgery
C. Suspected carcinoma of the gallbladder
D. Morbid obesity
E. Coagulopathy

38. In determining the proper treatment for a sliding hiatal hernia, the most useful
step would be
A. Barium swallow with cinefluoroscopy during Valsalva maneuver
B. Flexible endoscopy
C. 24-h monitoring of esophageal pH
D. Measuring the size of the hernia

39. A previously healthy 9-year old child comes to the emergency room because of
fulminant upper gastrointestinal bleeding. The hemorrhage is most likely to be the
result of
A. Esophageal varices
B. Mallory-Weiss syndrome
C. Gastritis
D. A gastric ulcer

40. Intragastric pressure remains steady near 2–5 mm Hg during slow gastric filling,
but rises rapidly to high levels after reaching a volume of
A. 400–600 mL
B. 700–900 mL
C. 1000–1200 mL
D. 1300–1500 mL

41. Local stimuli that inhibit the release of gastrin from the gastric mucosa include
which of the following?
A. Small proteins
B. 20-proof alcohol
C. Caffeine
D. Acidic antral contents

42. For a symptomatic partial duodenal obstruction secondary to an annular


pancreas, the operative treatment of choice is
A. A Whipple procedure
B. Gastrojejunostomy
D. Partial resection of the annular pancreas
E. Duodenojejunostomy

43. Which of the following would be expected to stimulate intestinal motility?


A. Fear
B. Gastrin
C. Secretin
D. Acetylcholine
E. Cholecystokinin

44. Which of the following statements concerning carcinoma of the esophagus is


true?
B. Squamous carcinoma is the most common type at the cardioesophageal junction
C. It has a higher incidence in males
D. It occurs more commonly in patients with corrosive esophagitis
E. Surgical excision is the only effective treatment

45. The most common clinical presentation of idiopathic retroperitoneal fibrosis is


A. Ureteral obstruction
B. Leg edema
D. Jaundice
E. Intestinal obstruction

46. In planning the management of a 2.8-cm epidermoid carcinoma of the anus, the
first therapeutic approach should be
E. Combined radiation therapy and chemotherapy
47. Indications for operation in Crohn’s disease include which of the following?
A. Intestinal obstruction
B. Enterovesical fistula
C. Ileum–ascending colon fistula
E. Free perforation

48. Which of the following is most likely to require surgical correction?


A. Large sliding esophageal hiatal hernia
B. Paraesophageal hiatal hernia
C. Traction diverticulum of esophagus
D. Schatzki’s ring of distal esophagus

49. Which statement regarding adenocarcinoma of the pancreas is true?


A. It occurs most frequently in the body of the gland
B. It carries a 1–2% 5-year survival rate
C. It is nonresectable if it presents as painless jaundice
D. It can usually be resected if it presents in the body or tail of the pancreas and does
not involve the common bile duct

50. Which of the following statements regarding direct inguinal hernias is true?
A. They are the most common inguinal hernias in women
B. They protrude medially to the inferior epigastric vessels
C. They should be opened and ligated at the internal ring
D. They commonly protrude into the scrotal sac in men

51. Which of the following statements regarding stress ulceration is true?


A. It is true ulceration, extending into and through the muscularis mucosa
B. It classically involves the antrum
C. Increased secretion of gastric acid has been shown to play a causative role
D. It frequently involves multiple sites

52. Which statement concerning cholangitis is correct?


A. The most common infecting organism is Staphylococcus aureus
B. The diagnosis is suggested by the Charcot triad (fever, jaundice, pain)
C. The disease occurs primarily in young, immunocompromised patients
D. Cholecystectomy is the procedure of choice in affected patients

53. Indications for surgical removal of polypoid lesions of the gallbladder include
A. Size greater than 0.5 cm
B. Presence of clinical symptoms
C. Patient age of over 25 years
D. Presence of multiple small lesions
54. A patient who has a total pancreatectomy might be expected to develop which of
the following complications?
A. Diabetes mellitus
B. Hypercalcemia
C. Hyperphosphatemia
D. Constipation
E. Weight gain

55. True statements regarding cavernous hemangiomata of the liver in adults include
A. The majority become symptomatic
B. They may undergo malignant transformation
C. They enlarge under hormonal stimulation
D. They should be resected to avoid spontaneous rupture and lifethreatening
hemorrhage

1. The superior vena cava syndrome is most frequently seen in association with
A. Histoplasmosis (sclerosing mediastinitis)
C. Thoracic aortic aneurysm
D. Constrictive pericarditis
E. Bronchogenic carcinoma

2. A 3-year-old child with congenital cyanosis is most probably suffering from


A. Tetralogy of Fallot
B. Ventricular septal defect
C. Tricuspid atresia
D. Transposition of the great vessels

3. Superior pulmonary sulcus carcinomas (Pancoast tumors) are bronchogenic


carcinomas that typically produce which of the following clinical features?
A. Atelectasis of the involved apical segment
B. Horner syndrome (miosis, ptosis, anhidrosis)
C. Pain in the T4 and T5 dermatomes
D. Nonproductive cough

4. Which of the following statements is true concerning aortocoronary bypass


grafting?
A. It is indicated for crescendo (preinfarction) angina

8. Which of the following statements is true regarding the thoracic outlet syndrome?
C. If conservative measures fail, it is best treated by surgical decompression of the
brachial plexus
6. A 2-year-old asymptomatic child is noted to have a systolic murmur, hypertension,
and diminished femoral pulses. Which of the following is true about this child’s
disorder?
C. Rib notching is often seen on x-ray (Coarctation of the aorta)

7. A correct statement concerning bronchial carcinoid tumors is that


C. They rarely produce the carcinoid syndrome

8. Patients with phlebographically confirmed deep vein thrombosis of the calf


E. Are at risk for significant pulmonary embolism

9. Symptoms or signs of atherosclerotic occlusive disease of the bifurcation of the


abdominal aorta (Leriche syndrome) include
A. Claudication of the buttock and thigh

10. Initial management of a patient who has a flaccid neurogenic bladder may
include which of the following measures?
A. Surgical bladder augmentation
B. Self-catheterization
C. Supravesical urinary diversion
D. Limiting fluid intake to less than 300 mL/day
11. The recommended treatment for stage A (superficial and sub
mucosal)transitional cell carcinoma of the bladder is
A. Local excision
B. Radical cystectomy
D. Topical (intravesicular) chemotherapy
E. Systemic chemotherapy

12. Seminoma is accurately described by which of the following statements?


A. It is the most common type of testicular cancer
B. Metastases to liver and bone are frequently found
C. It does not respond to radiation
E. Common presentation is that of a painful lump that trans illuminates

13. Meniscal tears usually result from which of the following circumstances?
A. Hyperextension
B. Flexion and rotation
C. Simple hyperflexion
D. Compression
14. In an uncomplicated dislocation of the glenohumeral joint, the humeral head
usually dislocates primarily in which of the following directions?
A. Anteriorly
B. Superiorly
C. Posteriorly
D. Laterally

15. Which of the following fractures is most commonly seen in healthy bones
subjected to violent falls?
A. Colles fracture
B. Femoral neck fracture
C. Intertrochanteric fracture
D. Clavicular fracture

16. Which of the following statements regarding compartment syndromes following


orthopedic injuries is true?
A. The first sign is usually loss of pulse in the extremity
B. Passive flexion of the extremity proximal to the involved compartment will
aggravate the pain
C. Surgical decompression (fasciectomy) is necessary only as a last resort
D. These syndromes are most commonly associated with supracondylar fractures of
the humerus and tibial shaft

17. In contrast to closed reduction, open reduction of a fracture


A. Produces a shorter healing time
B. Decreases trauma to the fracture site
C. Produces a higher incidence of nonunion
D. Reduces the risk of infection

18. In a failed suicide gesture, a depressed student severs her Radial nerve at the
wrist. The expected disability is
A. Loss of ability to extend the wrist
B. Loss of ability to flex the wrist
C. Wasting of the intrinsic muscles of the hand
D. Sensory loss over the thenar pad and the thumb web

19. The most severe epiphyseal growth disturbance is likely to result from which of
the following types of fracture?
E. Crushing injury compressing the growth plate

20. Which of the following statements regarding the Glasgow coma scale is true?
B. A high score correlates with a high mortality
C. It includes measurement of intracranial pressure
D. It includes measurement of pupillary reflexes
E. It includes measurement of verbal response

21. Which of the following statements regarding glioblastoma multiforme is true?


B. It arises from the malignant degeneration of an astrocytoma

22. Which of the following statements regarding skull fractures is true?


C. Any bone fragment displaced more than 1 cm inwardly should be elevated
surgically

23. An acute increase in intracranial pressure is characterized by which of the


following clinical findings?
A. Respiratory irregularities
B. Decreased blood pressure
C. Tachycardia
D. Papilledema

24. Which of the following statements about schwannomas is true?


B. Treatment is via excision

25. Which of the following statements regarding cerebral contusions is true?


B. They may occur opposite the point of skull impact

26. True statements regarding meningiomas include that they


C. Are treated primarily by surgical excision

27. Which of the following statements about craniopharyngiomas is true?


D. The tumors may cause compression of the optic tracts and visual symptoms

28. Which of the following statements regarding symptomatic thyroglossal duct cysts
is true?
A. Over 90% manifest themselves before age 12
B. Treatment includes resection of the hyoid bone
C. They usually present as a painful swelling in the lateral neck
29. Pleomorphic adenomas (mixed tumors) of the salivary glands are characterized
by which of the following?
A. They occur most commonly on the lips, tongue, and palate

30. Verrucous carcinoma of the buccal mucosa is identified with which of the
following characteristics?
C. It has a predilection for the gingivobuccal gutter

1-The diagnosis of an inguinal hernia:


 In infants often depends on the history given by its mother.
 In the adult is most easily made with the patient in the sitting position.
 Depends on the hernial sac or cough impulse being felt below the inguinal
ligament.
 Is supported by the presence of a transilluminable scrotal swelling.
 B&C only.
2- Inguinal herniae:
 In children are usually of the direct type.
 Of the indirect type are congenital in origin.
 Will regress spontaneously in children.
 In young adults are most commonly of the direct type.
 C&D only.

3- Strangulated contents of hernial sacs:


 Are always accompanied by intestinal obstruction.
 Are more common in direct than indirect inguinal herniae.
 Are usually reducible.
 Produce local pain and tenderness.
 All of the above

4- Incisional herniae are related to:)All correct except one)


 Wound infections.
 Anaemia and malnutrition.
 Obesity.
 The use of absorbable suture materials.
 To the technique of wound closure.

5- herniae in the umbilical region:


 Are always congenital in origin.
 Usually occur in males.
 Usually require surgical repair in infants.
 Rarely strangulate.
 All of the above.

6- A discharge from the umbilicus: (all correct except one)


 May indicate a patent vitello-intestinal duct.
 May indicate an anomaly of the urachus.
 At the time of menstration may indicate endometriosis.
 In the neonate is of no immediate clinical significance.
 May indicate infection of the umbilicus (omphalitis)
7- An exomphalos:
 Is a congenital defect of the urethra.
 Is acquired defect of the anterior abdominal wall.
 Is otherwise known as gastroschisis.
 Needs urgent surgical treatment.
 A&B only.

8- The cremaster muscle is derived from:


 External oblique aponeurosis.
 Internal oblique muscle.
 Transversus abdominis muscle
 Transversalis fascia
 Parietal peritoneum.

9- structures lying within the spermatic cord include: ( all correct except
one)
 Direct hernia sac
 Testicular artery.
 Indirect hernia sac.
 Properitoneal fat.
 Vas deferens.

10- True statements concerning a femoral hernia include which of the following?
 It is common in male.
 It is usually results from a defect in lateral part of transversalis fascia.
 It is common in children.
 It is less common in female.
 It may be confused with inguinal lymphadenopathy.

1- Acute abdominal pain which is


(all correct except on)
A. Colicky in nature indicates obstruction of hollow viscus.
B. In right upper quadrant increased by inspiration is typical of cholecystitis
C. Continuous is typical of inflammation
D. Maximal in the right loin is typical with duodenal ulcer.
E. Maximal in epigastrium and related to meal is typical of gastric problem.

2- Vomiting in acute abdomen


A. Occurring soon after the oncet of colicky pain indicates pathology outside the
gastrointestinal tract.
B. Of fluid containing no bile is characteristic of small bowl obstruction
C. Of faeculent fluid usually indicates gastrointestinal fistula.
D. Of bile stain fluid usually indicates stomach outlet obstruction
E. B&C only

3- Faeculent vomiting:
A. Is commonly seen after upper gastrointestinal tract bleeding.
B. Indicates large bowel obstruction.
C. Indicates bacterial proliteration in the upper intestinal.
D. Suggests a gastro-colic fistula.
E. Suggests small bowel fistula

4- A patient with generalized peritonitis:


(all correct except one)
A. Usually has an elevated temperature and pulse rate.
B. Characteristically complains of spasmodic severe pain which causes him to be
restless.
C. Is characteristically vomites.
D. Will usually have a rapid and deep respiratory pattern.
E. Is Usually has an elevated WBCs.

5- A perforated duodenal ulcer:


A. Usually lies on the anterior or superior surface of the duodenum.
B. Usually presents with the acute onset of severe back pain.
C. Produces radiological evidence of free gas in the peritoneum in over 90 percent of
the patients.
D. Is usually treated by vagotomy and pyloroplasty
E. Is usually treated conservatively.

6- Congential pyloric stenosis:


(all correct except one)
A. Occurs more commonly in male children.
B. Usually presents in the first few days of life.
C. Presents with non bile stained vomiting.
D. Is usually diagnosed on clinical examination.
E. May cause right upper quadrant tenderness.

7- Appendicitis is:
A. More common in females
B. Distributed evenly thoughout the world's population
C. More likely to occur if the appendix is in the retrocaecal position.
D. Commonly the result of appendicular obstruction.
E. B&C only.
8- Likely differential diagnoses in a young woman with appendicitis
include:
A. Ovarian carcinoma
B. Ruptured ectopic pregnancy.
C. Colonic diverticulitis.
D. Caecal carcinoma.
E. C&D only.

9- Obstruction of the lumen of the appendix may lead to:


(all correct except one)
A. Mucosal ulceration
B. Gangrenous appendicitis.
C. A perforated appendix.
D. Intussusception of the appendix.
E. Acute appendicitis.

10- Acute non-specific mesenteric lymphadenitis: (all correct except one)


A. Is commonest between 5 and 12 years of age.
B. Is usually associated with an upper respiratory tract infection.
C. Is usually associated with cervical lymphadenopathy
D. Is characterized by enlarged mesenteric lymph nodes which are infected by gram-
negative organisms.
E. Is one of the differential diagnosis of acute appendicitis.

11- The level of intestinal obstruction can be determined by:


(all correct except one)
A. Questioning the patient.
B. Examining the patient.
C. Radiological examination of the patient.
D. Repeated measurements of the patient's girth.
E. Upper & lower contrast studies.

12- Acute small bowel obstruction:


(all correct except one)
A. Is commonly caused by postoperative adhesions.
B. Accompanied by the signs of peritonitis, suggests bowel strangulation.
C. Is often associated with a raised serum amylase.
D. Generally produces abdominal distension within 2 to 3 hours of onset.
E. Can be diagnosed by history and clinical examination.

13-In the treatment of intestinal obstruction:


A. Nasogastric suction should be instituted preoperatively.
B. Intravenous fluid replacement might be required.
C. Immediate surgery is essential.
D. Surgery should be restricted to those cases where strangulation is diagnosed.
E. C&D only.

14- Strangulation of the bowel:


(all correct except one)
A. commonly complicates closed loop obstruction.
B. Is difficult to distinguish from simple intestinal obstruction.
C. Is accompanied by bleeding into the affected bowel.
D. Frequently causes peritonitis.
E. Is characterized by severe abdominal pain.

15- Large bowel obstruction:


A. Is most commonly caused by colonic cancer
B. Has its maximum incidence before the age of 50.
C. Frequently presents with nausea and vomiting.
D. Usually heralds its onset with constant suprapubic pain.
E. Frequently treated conservatively.

16- Patients with acute colonic deveticulitis:


A. Often give a history of recent lower abdominal colic.
B. Often present with pyrexia.
C. Can be frequently diagnosed on sigmoidoscopic appearances.
D. Frequently develop faecal peritonitis.
E. All the above.

17- Acute pancreatitis typically:


(all correct except one)
A. Is accompanied by hypercalcaemia.
B. Produces paralytic ileus.
C. Is associated with a pleural effusion.
D. Produces pyloric stenosis.
E. Upper abdominal pain and vomiting.

18- Acute pancreatitis:


(all correct except one)
A. Often simulates a perforated peptic ulcer in its presentation.
B. Often presents with the signs of hypovolaemia.
C. Can readily be distinguished from other causes of acute abdominal pain by the
presence of a raised serum amylase.
D. Frequently has a raised concentration of urinary amylase.
E. Most commonly caused as a complication of GB stones.

19- The treatment of acute pancreatitis:


A. Is largely nonspecific and supportive.
B. Should include a laparotomy in the majority of cases.
C. Should routinely include the administration of calcium.
D. Should routinely include the administration of antibiotics.
E. All the above.

20- Childhood intussusception:


(all correct except one)
A. Usually presents during the first year of life.
B. Is frequently ileocolic.
C. Can usually be diagnosed without x-ray examination of the abdomen.
D. Rarely requires surgical treatment.
E. Can be diagnosed by abdominal US.

21- Meconium ileus:


A. Is the presenting feature in the majority of patients with cystic fibrosis.
B. Is associated with achlorhydria.
C. Presents with a distended abdomen and bilious vomiting.
D. Often can be effectively treated with acetyl cysteine.
E. All the above.

22- Neonatal duodenal obstruction:


A. May be associated with down's syndrome.
B. Is more frequently found in premature infants.
C. Typically presents with gross abdominal distension.
D. Usually presents with vomiting of non-bile stained fluid
E. B&C only.

23- acute superior mesenteric artery occlusion: (all correct except one)
A. Characteristically presents with sudden pain and tenderness of increasing intensity.
B. Is frequently accompanied by overt or occult blood loss in the stools.
C. Frequently produces peritonitis.
D. Can usually be diagnosed on plain abdominal x-rays.
E. Can be diagnosed by mesenteric artery ongiography.

24- A ruptured ectopic pregnancy:


(all correct except one)
A. Usually occurs in the first month of pregnancy.
B. Usually presents with severe lower abdominal pain.
C. Frequently presents with hypovolaemic shock.
D. Can usually be diagnosed by pelvic examination.
E. Can usually be diagnosed by transvaginal US.

25- Biliary colic typically:


A. Occurs 3 to 4 hours after meals.
B. Lasts 5 to 20 minutes.
C. Radiates from the upper abdomen to the right subscapular region.
D. Is made better by deep inspiration.
E. B&C only.

1- When determining the depth of a burn:


A. A knowledge of the type of injury is important.
B. The presence of blisters is of no clinical significance.
C. Impairment of sensibility of the burned area denotes full thickness burn.
D. The presence of severe pain denotes a full thickness skin loss.
E. A&C only.

2- Estimation of the area of a burn: ( all correct except one)


A. Is of very little clinical significance.
B. Provides important prognostic information.
C. Is an important factor in the estimation of the fluid required.
D. Can be based on a formula which states that the adult trunk is 36 per cent of the
whole body surface area.
E. Should be recorded in each chart of burned patient.

3- Patients with major burns:


A. Are in a negative nitrogen balance.
B. Have normal calorie requirements.
C. Do not generally become anaemic.
D. Are resistant to septicaemia.
E. All of the above.

4- The catabolic response to trauma and infection is characterized by:


A. An increase in lean body mass.
B. A positive nitrogen balance.
C. Gluconeogenesis.
D. A falling haemoglobin level.
E. A&B only.

5- The catabolic response to trauma:


A. Is related to the severity of the trauma.
B. Is accompanied by decreased urinary losses of potassium and nitrogen.
C. Can be prevented by parenteral nutrition.
D. Does not occur in the adrenalectomised patient.
E. C&D only.

6- Scalds:
A. Are more frequent in children.
B. Commonly cause full thickness skin loss.
C. Should be skin grafted within 48 hours of the injury.
D. Need routine antibiotic treatment.
E. All of the above.

7- The dressing of a small burn should be:


A. Occlusive.
B. Non-absorbtive.
C. Non-compressive.
D. Changed daily as a routine.
E. All of the above

8- A partial thickness burn:


A. Usually requires grafting.
B. May deteriorate into full thickness skin loss.
C. Rarely causes severe physiological derangement of the patient.
D. Heals within 7 days in the absence of infection.
E. C&D only.

9-Fluid losses in a major burn:


A. Are maximal between 12 and 24 hours after the injury.
B. Are related to the age of the patient.
C. Are not related to the weight of the patient.
D. Are related to the area burnt.
E. Are not related to the burnt duration.

10- The increased fluid requirements of a patient with a full thickness


burn are due to: (all correct except one)
A. Increased evaporative water loss.
B. Sequestration of fluid in the injured tissues.
C. Serum exuding from the burned area.
D. Destruction of blood in the skin vessels.
E. Vomiting & paralytic ileus occasionally

11- 48 hours after a major burn and with satisfactory fluid therapy a
patient:
A. Has very few abnormal fluid losses.
B. May need a blood transfusion.
C. Is often hypornatraemic.
D. Usually needs skin grafting.
E. C&D only.

12- Major burns are sometimes complicated by: (all correct except one)
A. Acute gastric and duodenal ulcers.
B. Paralytic ileus.
C. Cerebral oedema.
D. Mesenteric thrombosis.
E. Septicemia.

13- If burned patient, associated pulmonary injury: (all correct except


one)
A. Should be suspected in head and neck burns.
B. Should be suspected when the nasal hairs are burnt.
C. Does not appear clinically in the first 24 hours.
D. Should be suspected if burns occurred in closed space.
E. May require ICU care.

14- Secondary infection of burns:


A. Is less common in partial than in full thickness skin loss.
B. Is relatively more common in burns of more than 20 per cent body area.
C. Is avoided by leaving the burn eschar intact.
D. Is avoided by the immediate application of a sterile occlusive dressing.
E. All of the above.

15- . The early management of a burn wound may include:


A. Early excision.
B. Occlusive dressings.
C. Exposure treatment.
D. Dressings with local antibiotics.
E. All of the above.

16- Skin grafting of a burn wound:


A. Should usually be with full thickness skin grafts.
B. Is more likely to be successful if undertaken in the first week after injury.
C. Will be unsuccessful unless the wound surface is sterile.
D. Minimizes scar contracture.
E. A&C only.

17- The prognosis of a burned patient is:


A. Not related to the patient's age.
B. Related to the area burnt.
C. Generally better below the age of 10 years.
D. Very poor in the patient with burns of over 40 per cent surface area.
E. C&D only.
1- Diverticular disease of the colon:
• Is usually asymptomatic.
• Often presents with lower abdominal pain.
• May present with severe rectal haemorrhage.
• May present with peritonitis.
• All of the above are correct.
2- Ulcerative colitis:
• Is more common in female than males.
• Appears most commonly between the ages of 20 and 30.
• Usually presents with abdominal discomfort and diarrhoea.
• Can usually be diagnosed on sigmoidoscopic examination.
• All of the above correct.
3- Crohn’s disease: ( all correct except one)
• Is associated with an anal fissure or fistula in the majority of cases.
• May produce intestinal obstruction
• May involve the colon and the ileum.
• Is characterised by long periods of remission.
• May produce pritonitis.
4-Regarding ischaemic colitis:
(all correct except one)
• Often presents with diarrhoea.
• Often presents with rectal bleeding
• Is commonly setuated around the splenic flexure.
• Is often effectively managed by non-surgical means.
• The baruim enema may reveal localised bowel oedema or stricture.
5- A volvulus of the colon:
(all correct except one)
• Most frequently occurs in eastern Europe and Africa.
• Most frequently occurs on the right side and includes the caecum.
• Usually produces abdominal distension which is most marked on the left side.
• May be effectively treated without resorting to laparotomy.
• May produce bowel strangulation.
6- Hirschsprung’s disease:
• Is the result of acquired aganglionosis of the large bowel.
• Usually becomes evident in early adult life.
• Can usually be diagnosed on a barium enema.
• Can usually be managed by dietary means.
• A&D only.
7- Colonic polyps: (all correct except one)
• Are associated with colonic cancer.
• May be hereditary.
• Should not be removed if they are asymptomatic.
• May be hyperplastic.
• Are commonly adenomatous.
8- A right sided colonic cancer frequently presents with:
• Anaemia.
• Intestinal obstruction.
• Rectal bleeding
• An abdominal mass.
• A&D only
9- A left sided colonic cancer frequently presents with: ( all correct except one)
• Anaemia.
• Intestinal obstruction.
• Rectal bleeding.
• Abdominal pain.
• Pencil like stools.
1- Acute cholecystitis: (all correct except one)
• Is almost invariably related to the presence of gallstones.
• Usually presents with biliary colic.
• Is often associated with jaundice.
• Is characterised by a pyrexia in the early hours of the disease..
• May be complicated by peritonitis.
2- Acute cholecystitis should usually be treated by: (all correct except one)
• Nasogastric suction and intravenous fluids.
• Antibiotic therapy.
• Urgent cholecystectomy.
• Cholecystostomy.
• Analgesics and antispasmotics.
3- Gallstones: (all correct except one)
• Have an incidence which increases with age.
• Are more frequent in females.
• Usually contain a predominance of cholesterol.
• Are formed in bile which is supersaturated with bile acids.
• Are formed in bile which is supersaturated with cholesterol.
4- The presence of stones in the common bile duct: ( all correct except one)
• Is commonly associated with a long history of dyspepsia.
• Is usually associated with jaundice.
• Must be considered during every cholecystectomy.
• May requeres treatment by choledochoduodenostomy.
• May cause liver abscess.
5- stones in the common bile duct:
• Are present in nearly 50 per cent of cases of cholecystitis.
• Often give rise to jaundice, fever and biliary colic.
• Are usually accompanied by progressive jaundice.
• Are usually associated with a distended gallbladder.
• A&D only.
6- In severe jaundice diagnostic evidence of an extrahepatic obstrction of the biliary
tract may be gained by: (all correct except one)
• Intravenous cholangiography.
• A barium meal.
• Endoscopic retrograde cholangiography.
• Percutaneous transhepatic cholangiography.
• CT scan of the abdomen.
7- Internal biliary fistulae:
(all correct except one)
• Most commonly arise as a consequence of cholecystitis.
• Most commonly occur between the gallbladder and the duodenum.
• Can result in intestinal obstruction.
• Are in most cases fatal complications.
• May cause cholongitis.
8- Carcinoma of the gallbladder:
• Is usually a squamous cell neoplasm.
• Is more common in men.
• Is rarely associated with gallstones.
• Has a relatively poor prognosis.
• Non of the above is correct.
9- hypersplenism:
• Results in anaemia, leucopenia and thrombocytopenia.
• Only occurs in the presence of a large spleen.
• Frequently follows liver cirrhosis.
• May be diagnosed by bone marrow biopsy.
• All of the above are correct.
10- Acute pancreatitis: (all correct except one)
• Has a higher incidence in alcoholics.
• Is commonly associated with the presence of gallstones.
• Occurs most commonly in diabetics.
• Becomes less severe with each recurring episode.
• May occurs as a complication of abdomenal trauma.
11- Acute pancratitis is characterised by:
(all correct except one)
• A history of gallstones
• Diffuse epigastric pain.
• Exaggerated bowel sounds.
• An elevated urinary amylase.
• An elevated serum amylase.

12-chronic pancreatitis:
• Is commonly associated with alcoholism.
• Is associated with diabetes.
• May be diagnosed by the analysis of pancreatic secretions.
• May be treated by surgical procedures which decompress the pancreatic duct.
• All of the above are correct.
13-Pancreatic pseudocysts:
(all correct except one)
• Are developmental in origin.
• Usually arise in the lesser peritoneal sac
• Produce a smooth epigastric mass which does not moves on respiration.
• May be effectively treated by internal drainage.
• May be associated by elevation of serum amylase.
The effective osmotic pressure between the plasma and -1 •
:interstitial fluid compartments is primarily controlled by
• A. Bicarbonate.
• B. Chloride ion.
• C. Potassium ion.
• D. Protein.
• E. Sodium ion.

• 2- Symptoms and signs of extracellular fluid volume deficit
include all of the following except:
• A. Anorexia.
• B. Apathy.
• C. Decreased body temperature.
• D. High pulse pressure.
• E. Orthostatic hypotension.

• 3- The osmolarity of the extracellular fluid space is determined
primarily by the concentration of:
• A. Bicarbonate
• B. Chloride ion
• C. Phosphate radicals
• D. Sodium ion
• E. Sulfate radicals

• 4- When lactic acid is produced in response to injury, the body
minimizes pH change by:
• A. Decreasing production of sodium bicarbonate in tissues.
• B. Excreting carbon dioxide through the lungs.
• C. Excreting lactic acid through the kidneys
• D. Lowering renal output of chloride ions
• E. Metabolizing the lactic acid in the liver

• 5- The simplest effective method of estimating the degree of
acidosis in a patient in shock is the measurement of:
• A. Arterial pH
• B. End tidal CO concentration
• C. pH of mixed venous blood
• D. serum CO level
• E. urinary pH


• 6- A decrease in intracellular water can be precipitated by:
• A. A decrease in sodium in extracellular fluid
• B. An increase in sodium in extracellular fluid
• C. An increase in sodium in intracellular fluid
• D. An isotonic decrease in extracellular fluid
• E. An isotonic increase in extracellular fluid

• 7- The first step in the management of acute hypercalcemia
should be:
• A. Correction of deficit of extracellular fluid volume
• B. Hemodialysis
• C. Administration of furosemide
• D. Administration of mithramycin
• E. Parathyroidectomy.

• 8- Postoperative third-space accumulation should be managed by
intravenous:
• A. Albumin
• B. Dextrose in water
• C. Fluid restriction
• D. 1/2 normal saline with potassium supplements
• E. Normal saline

• 9- The normal adult value for:
• A. urine output is 1.5 litre/day.
• B. Insensible water loss is 200 ml/day.
• C. Potassium requirement is 150 mEq (150mmol)/day.
• D. Protein requirement is 120 g/day.
• E. B&C only

• 10- Potassium deficiency should be suspected: ( All Correct Except
one)
• A. In cases of paralytic ileus.
• B. When the patient's reflexes are exaggerated.
• C. If there is a decrease in height and peaking of the T waves of an ECG.
• D. In alkalotic states.
• E. In intestinal obstruction.

• 11- The sodium ion:
• A. Is the principal regulator of the intracellular volume.
• B. Is the major ionic component of the intracellular fluid volume.
• C. Is present in greater concentration in intracellular fluid than extracellular fluid.
• D. Is excreted in larger amounts than normal in the early postoperative period.
• E. C&D only


• 12-Acute post traumatic renal failure:
(All Correct Except One)
• A. May be due to hypovolaemia and poor tissue perfusion.
• B. Is particularly associated with crush injuries.
• C. May be due to kidney damage following tubular obstruction.
• D. Should initially be treated by fluid restriction.
• E. Should be treated initially by increase the amount of intravenous fluid.

• 13- The anuric patient:
• A. Should have a fluid in take of 1 to 1.5 litres per day.
• B. Should have no potassium administered.
• C. Is at risk from metabolic alkalosis.
• D. Should be on continuous urinary catheter drainage.
• E. A&D only.

• 14- Intravenous parenteral feeding:
(All Correct Except One)
• A. Should deliver at least 2500 calories/day to an adult.
• B. Should deliver at least 10g of nitrogen (i.e. 66g of protein)/day to an adult.
• C. Can be effectively achieved with isotonic solutions.
• D. Is with complications with present day solutions and methods of
administration.
• E. Can be given by central intravenous infusion line (CVL)

1- A fracture is said to be:
• Closed if an overlying skin laceration has been sutured.
• Comminuted if there has been associated damage to adjacent nerves or vessels.
• A fatigue fracture if it occurs through a diseased bone.
• Pathological if it occurs through a bony metastasis.
• Non of the above correct.

2- In a healing fracture:
(All correct except one)
• The haematoma is initially invaded by osteoblasts.
• The tissue formed by the invading osteoblasts is termed osteoid.
• Calcium salts are laid down in the osteoid tissue.
• The final stage of repair is the remodelling of the callus.
• The callus formation is related to the amount of stress at fracture side.

3- Non-union is often seen in:


• Fractures of the 4th metatarsal.
• Fractures of the neck of the femur.
• Fractures of the condyle of the mandible.
• Colles’ fractures.
• Oblique fracture line of femur.

4- Fractures of the clavicle:


• Are usually of the greenstick variety in children under the age of 10 years.
• Are usually the result of direct violence.
• Can be recognised by the abnormal elevation of the distal fragment.
• Are usually treated by internal fixation.
• C&D only.

5- in fractures of the surgical neck of the humerus: (All correct except one)
• The lesion is usually due to indirect violence.
• The fragments are usually impacted.
• The proximal fragment is usually internally rotated.
• The distal fragment is usually adducted.
• Early mobilisation is encouraged.

6- In a fracture of the distal third of the shaft of the humerus:


• The distal fragment is usually posteriorly angulated by the action of biceps.
• The radial nerve is rarely damaged.
• Delayed radial nerve palsy is usually due to oedema.
• Late onset of radial nerve palsy is usually due the involvement of the nerve
with callus.
• Non of the above is correct.

7- A supracondylar fracture of the humerus:


( All correct except one)
• Is a fracture commonly seen in childhood.
• Is particularly subject to the complication of ischaemic muscle contracture.
• Is held in the position of reduction by the tendon of brachioradialis.
• When properly reduced has the index finger pointing approximately to the tip
of the shoulder of the same side.
• When reduced care must be taken to ensure that the radial pulse is present.

8- A transverse fracture of the scaphoid is:


• Prone to infection.
• Usually seen in old men.
• Prone to avascular necrosis.
• Usually seen on an early scaphoid x-ray.
• All of the above is correct.

9- In a colles’ fracture the distal radial fragment:


• Is dorsally angulated on the proximal radius.
• Is usually torn from the intra-articular triangular disc.
• Is deviated to the ulnar side.
• Is rarely impacted.
• Is ventrally displaced.

10- dislocations of the shoulder joint:


• Most commonly occur in middle age.
• Usually occur when the arm is in the abducted position.
• Usually have the head of the humerus situated behind the glenoid fossa.
• Are often recurrent in the young.
• B&D only.

11- In pelvic fractures:


• Avulsion injuries are usually treated by early mobilisation.
• Undisplaced lesions of the ischial or pubic rami are usually treated by early
dmbilisation.
• Extraperitoneal urinary extravasation may be due to damage either to the
membraneous urethra or to the base of the bladder.
• Which are unstable are accompanied by extensive hemorrhage.
• All of the above are correct.
12- Intracapsular fractures of the upper end of the femur are usually: (All correct
except one)
• Accompanied by shortening of the leg.
• Accompanied by external rotation of the leg.
• Accompanied by adduction of the leg.
• Treated by internal fixation.
• Accompanied with a vascular necrosis of the head of femur.

13- Extracapsular fractures of the upper end of the femur are usually:
• Subtrochanteric in position.
• Subject t o avascular necrosis of the head of the femur.
• Accompanied by internal rotation of the leg.
• Treated by external fixation.
• Non of the above are correct.

14- In fractures of the mid shaft of the femur:


( all correct except one)
• The proximal fragment is usually flexed.
• The proximal fragment is usually abducted.
• The distal fragment is usually adducted.
• The common femoral vessels are usually damaged.
• Hamstring and quadriceps produce some shortening of the leg.

15- In fractures of the middle third of the tibia and fibula: (All correct except one)
• Delayed union is common.
• Indirect violence usually results in a spiral or oblique fracture line.
• Shortening and anterior angulation of the tibia are common.
• Comminuted fractures are usually treated external fixation.
• All of the above are correct.

16- In injuries of the ankle joint:


• Eversion injuries are the most commonly encoutered.
• Inversion injuries are usually accomapanied by a tear of the deltoid ligament.
• There is frequently associated posterior tibial nerve damage.
• The joint is rendered unstable by rupture of the inferior tibio-fibular ligament.
• A&B only.

17- Dislocation of the hip joint:


• Is most common when the hip is in a neutral position.
• Is usually associated with a fracture of the acetabular rim.
• Usually results in the femoral head coming to lie anteiorly over the pubis or
obturator externus.
• May be associated with injuries of the sciatic nerve.
• B&D only.

18- Acute osteomyelitis in childhood:


• Is usually the result of compound bony injuries.
• Is characterised by a constant bone pain.
• Characteristiclly produces necrosis of the periosteum overlying the infected
bone.
• Is not demonstrable radiologically for the first week of the disease.
• All of the above are correct.

19- In osteoarthritis of the hip joint:


• The articular cartilage undergoes initial hypertrophy and then becomes
hardened and eburnated.
• The joint capsule becomes stretched and lax.
• The leg is usually adducted and externally rotated when the patient lies supine.
• A femoral osteotomy usually helps halt the progress of the disease process.
• C&D only.

20- In a case of congenital dislocation of the hip:


• There is a defect of the posterior rim of the acetabulum.
• On bilateral hip abduction with the knees extended there is often limited
abduction on the diseased side.
• Reduction is sometimes hindered by a tight gluteus minimus muscle.
• Splinting of the limbs following reduction should be maintained until the
femoral epiphysis returns to its normal density on x-ray examination.
• Non of the above is correct.

21- Osteognic sarcoma:


• Are most frequent in the 10 to 25 year age group.
• Readily metastasise via the blood stream.
• Are frequently surrounded by non-malignant new bone formation.
• When treated by conventional methods have a 10 percent 5 years survival rate.
• All of the above are correct.

1- 'Shock' can be most comprehensively defined as:


A. A sudden large volume blood loss.
B. A diminished effective circulating fluid volume.
C. A hypotensive state with peripheral vasoconstriction.
D. An unexpected psychological insult.
E. A&C only.

2- In all forms of shock there is:


A. An impairment of cellular oxygenation.
B. A decreased cardiac output.
C. An increased effective circulating fluid volume.
D. A low central venous pressure (CVP).
E. An increased pulse rate.

3- The metabolic acidosis of shock can be effectively treated by:


A. Warming the patient.
B. Administering ammonia chloride.
C. Artificial ventilation.
D. Restoring normal tissue perfusion.
E. Nacl intravenous infusion.

4- In hypovolaemic shock:
A. The central venous pressure is high.
B. The difference in arteriovenous oxygen tension is unaffected.
C. The extremities are pale, cold and sweating.
D. Urine output is unaffected.
E. C&D only.

5- Hypovolaemic shock may result from:


(all correct except one)
A. A 25 per cent third degree burn.
B. Generalized peritonitis.
C. Massive pulmonary embolism.
D. Intestinal obstruction.
E. Massive blood loss.

6- Septicshock is particularly associated with: (all correct except one)


A. Thoracic surgical patients.
B. Hypovolaemia.
C. Indwelling urinary or intravenous catherers.
D. Gram-negative bacteraemia.
E. Gram-positive.

7- siptic shock is associated with a hypodynamic cardiovascular state:


( all correct except one)
A. if preceded by existing hypovolaemia.
B. In generalized peritonitis.
C. When there is a gram-positive bacteraemia.
D. In elderly patients.
E. In late gram negative septicemia.
8- The mortality from septic shock can be effectively reduced by:
(all correct except one)
A. Surgical drainage of abscesses.
B. The administration of appropriate antibiotics.
C. The restoration of a normal cardiovascular state.
D. Positive pressure respiration via an indotracheal intubation or tracheostomy.
E. The administration of corticosteroid.

9- In cardiogenic shock:
A. The central venous pressure is hig.
B. The difference in the arteriovenous oxygen tension is increased.
C. The haematocrit is raised.
D. The blood pressure is unaffected.
E. C&D only.

10- Prospective blood donors:


A. Should be asked about previous attacks of jaundice.
B. Should have serological tests for syphilis.
C. May transmit glandular fever to a recipient.
D. May transmit malaria to a recipient.
E. All the above.

11- A blood transfusion reaction:


(all correct except one)
A. May be due to incompatibility of the recipient serum and donor cells.
B. Is manifest by thrombophlebitis of the infusion site.
C. Occurs within the first 30 minutes of transfusion.
D. May produce renal damage.
E. May produce anaphylactic shock.

12- Pyrexial reactions to blood transfusions:


(all correct except one)
A. Have decreased since the introduction of sterile disposable infusion sets.
B. May be caused by allergic reactions.
C. May be caused by contaminated blood.
D. May be a response to a large transfusion of cooled blood.
E. Usually caused by pyrogen reaction.

13. Massive blood transfusions may be complicated by:


(all correct except one)
A. Hyperkalaemia.
B. Hypocalcaemia.
C. Coagulopathy.
D. Leucopenia.
E. DIC.

1- Malignant melanomata:
• Occur more commonly in the black races.
• Occur with equal frequency in all ages.
• Frequently arise from pre-existing benign naevi.
• Occur more frequently in non tropical regions.
• A&B only.
2- A malignant melanoma:
• Frequently arises from hair-bearing naevi.
• Frequently arises from junctional naevi.
• Has a worse prognosis when it arises on the leg.
• Should be suspected in any big pigmented lesion.
• Non of the above is correct.

3- The treatment of a malignant melanoma should include:


( All correct except one)
• A preliminary incision biopsy.
• Wide excision of the tumour.
• ‘En bloc’ removal of adjacent involved lymph nodes.
• Immediate excision of any enlarging lymph node in the postoperative period.
• Excision of 5cm margin of normal tissue.
4- Squamous cancer of the lip:
• Is most common in early adult life.
• Is more common in fair skinned subjects.
• Metastasises readily by the blood stream.
• Is preferably treated by radiotherapy once lymph node deposits are present.
• All of the above are correct.
5 - Basal cell carcinomas:
• Usually metastasise to regional lymph nodes.
• Are less common than squamous cell carcinomas.
• Are characterised histologically by epithelial pearls.
• Are particularly common in oriental races.
• Non of the above is correct.
6 – Capillary angiomas of childhood (strawberry naevi):
• Arise in the dermis.
• Are premalignant.
• Are most satisfactorily treated with superficial radiotherapy.
• Should be surgically excised.
• B&C only.
7- Fiboadenomata of the breast:
• Are commonest in early adult life.
• Are indiscrete and difficult to distinguish.
• Are usually painless.
• Resolve without treatment.
• A&C only.
8 –Fibrocystic disease of the breast:
• Is a variant of the normal cyclical changes that the breast undergoes during
menstruation.
• Is normally unilateral.
• Tends to progress in the post-menopausal years.
• Is precancerous.
• C&D only.
9 –The management of fibrocystic breast disease should:
• Usually be by surgical excision.
• Include mammography when available.
• Include therapy with oestrogens.
• Include therapy with progesterone.
• Corticosteroids.
10 – An intradut papilloma of the breast:
• May cause a colourless nipple descharge.
• May be diagnosed with the aid of contrast radiography.
• Should be treated by simple mastectomy.
• Is associated with fibrocystic disease of the breast.
• Is usually a premalignant lesion.

11 – Paget’s disease of the nipple:


• Usually presents as abilateral eczema of the nipple.
• Is always related to an underlying breast cancer.
• Indicates incurable breast cancer.
• Has non-specific histological characteristics.
• A&C only.
12- x-ray examijnation of the breast (mammography):
• Does not improve the clinician’s diagnosis rate of benign and malignant breast
disease.
• Is diagnostically most useful in young women.
• Is practical as a nationwide presymptomatic screening procedure.
• Contributes nothing to the management of the patients with clinically obvious
breast cancer.
• Non of the above is correct.
13- Breast cancer:
• Is one of the commonest female neoplasm.
• Has its highest incidence in social class v.
• Has a familial tendency.
• Is less common in nulliparous women.
• A&C only.
14 – Breast cancer: (all correct except one)
• May be presents with a history of breast pain.
• Is most common in the upper outer quadrant of the breast.
• Can be diagnosed preoperatively by the experienced clinician in 95 per cent
of cases.
• Must be considered on discovering any discrete mass in the breast.
• Usually presents as a painless lump.
15 –The signs and symptoms of breast cance include: (all correct except one)
• A bloody nipple discharge.
• Eczematous changes in the nipple and areola.
• Pre-menstrual breast pain.
• Skin tethering.
• A discrete breast lump.
16 – The histological study of breast cancers has shown that:
• The prognosis is not related to histological type.
• The commonest carcinoma is a squamous carcinoma.
• Most breast cancers arise from the epithelium of the breast duct.
• Satellite breast cancers are uncommon.
• B&D only.
17- Signs of incurable breast cancer include:
• Tumour fixity to the chest wall.
• Skin ulceration.
• Palpably enlarged mobile ipsilateral axillary lymph nodes.
• A bloody nipple discharge.
• A&B only.
18– The prognosis of treated breast cancer:
• Is related to the clinical staging of the cancer.
• Is not related to the number of axillary nodes found to be invaded by cancer at
operation.
• Is related to the use of postoperative radiotherapy to the regional nodes and
operative field.
• Is better after simple rather than radical mastectomy.
• Non of the above is correct.
19 – The prognosis of treated stage 1 breast cancer:
• Cannot be satisfactorily assessed until 15 years have elapsed.
• Is adversely affected by a subsequent pregnancy.
• Is worse in the male.
• Is worse if the cancer is discovered during late pregnancy or the puerperium.
• A&D only.

1. Which of the following is not true about neurogenic shock?


a. It may follow spinal fractures.
b. There is tachycardia.
c. The extremities are warm.
d. There is bradycardia and hypotension.
e. Vasopressors may be useful.

2. All the following are complications of massive blood transfusion except:


a. Hypothermia.
b. Hypocalcaemia.
c. Hypokalaemia.
d. Acidosis.
e. DIC.

3. In idiopathic thrombocytopenic purpura all the following are present


except:
a. Megakaryocytes are present in the bone marrow.
b. Cortisone therapy may improve the condition.
c. Marked enlargement of the spleen is present.
d. Haemarthrosis is not a complication.
e. The INR is normal.
f. The disease is due to the presence of immunoglobulins.

4. Regarding hyperkalemia all are true EXCEPT:


a. Is mostly the result of renal failure.
b. Is common with massive tissue destruction.
c. Muscle rigidity is a manifestation of severe hyperkalemia.
d. Causes a peaked T-wave on the electrocardiogram.
e. Urgent treatment is by Ca gluconate and Na bicarbonate.

5. One of the following is an anaerobic organism:


a. Staphylococcus aureus.
b. E. coli.
c. Klebsiella.
d. Pseudomonas aeroginosa.
e. Bacteriodes.

6. Which of the following tests will be affected after intake of non-steroidal


anti-inflammatory drugs:
a. Coagulation time.
b. PTT.
c. INR.
d. Bleeding time.
e. Thrombin time.

7. All of the following can be treated conservatively in a stable trauma


patient except:
a. Lung contusion.
b. Liver laceration.
c. Kidney laceration.
d. Splenic hematoma.
e. Perforation of the small intestine.

8. The initial maneuver to establish an airway in a patient with multiple


injuries is:
a. Oropharyngeal airway.
b. Uncuffed endo-tracheal tube.
c. Suctioning foreign debris and lifting up the mandible.
d. Cuffed endo-tracheal tube.
e. Tracheostomy.
9. Which is the most commonly injured intra-abdominal organ in blunt
trauma?
a. Pancreas.
b. Kidney.
c. Spleen.
d. Stomach.
e. Colon.
10. In malignant melanoma which of the following correlates best with the
prognosis?
a. Age of the patient.
b. Gender.
c. Clark’s level.
d. Site of the lesion.
e. Breslew thickness of the lesion.

11. Risk factors for breast cancer include the following except:
a. Prior breast cancer.
b. Mammary duct ectasia.
c. Breast cancer has affected a sister.
d. Early menarche and late menopause.
e. In situ lobular or duct carcinoma.

12. The commonest histological type of breast cancer is:


a. Duct carcinoma in situ.
b. Lobular carcinoma in situ.
c. Invasive duct carcinoma.
d. Invasive lobular carcinoma.
e. Paget’s disease of nipple.

13. What is the commonest presentation of a nephroblastoma?


a. Abdominal pain.
b. Haematuria.
c. Fever.
d. Abdominal mass.
e. Loss of weight.

14. All the following statements about follicular thyroid cancer are true
except:
a. It present at a later age than papillary carcinoma.
b. It disseminates via hematogenous route.
c. It is less common than papillary carcinoma.
d. It is frequently multicentric.
e. Bone is a favorite metastatic site.

15. To prevent foot infection in a diabetic patient the following


recommendations are all correct except:
a. Careful trimming of toe nails.
b. Monthly injection of the long-acting benzathene penicillin.
c. Regular washing and daily inspection of the feet by the patient.
d. Early treatment of taenia pedis infection.
e. Avoidance of walking bare-footed.

16. Which of the following tests is used to monitor the adequacy of


anticoagulation with warfarin?
a. Bleeding time.
b. Clotting time.
c. Activated partial thromboplastin time (APTT).
d. Fibrinogen level.
e. International normalized ratio (INR).
f. Marjolin’s ulcer.

17. Severe limb pain of sudden onset can be caused by all the following
conditions except:
a. Acute ischaemia.
b. Deep venous thrombosis.
c. Muscle tear.
d. Sciatica.
e. Bone fracture.

18. The following statements about gall bladder stones are all correct except:
a. Most of these stones are radio-opaque.
b. In many cases these stones produce no symptoms.
c. Gall stones are present in most cases of gall bladder cancer.
d. Ultrasonography is the preferred imaging study.
e. Removal of the gall balder is the standard treatment of symptomatic
cases.

19. The following statements about acute cholecystitis are all correct except:
a. The great majority of cases are caused by stones.
b. The condition is more serious in diabetics.
c. Differential diagnosis includes acute pancreatitis.
d. The main presentation is jaundice.
e. Initial treatment may be conservative.

20. Which of the following is a common cause for massive lower


gastrointestinal bleeding?
a. Crohn’s colitis.
b. Rectal polyp.
c. Angiodysplasia of the colon.
d. Carcinoma of the caecum.
e. Solitary rectal ulcer.

21. Which of the following is not true about achalasia of the cardia?
a. It usually occurs in middle age.
b. There are hyyperperistlatic waves in the body of the esophagus.
c. It predisposes to squamous cell carcinoma of the esophagus.
d. Manometric study is diagnostic.
e. Pneumatic dilatation is a line of treatment.

22. Organisms found in the large bowel are mostly:


a. Gram-positive cocci and anaerobes.
b. Gram-negative cocci and anaerobes.
c. Gram-negative bacilli and anaerobes.
d. Gram-positive bacilli and anaerobes.
e. Gram-negative cocci only.

23. Which of the following operations is considered as clean contaminated


operation?
a. Herniorrhophy.
b. Axillary block dissection.
c. Ureterolithotomy.
d. Urgent colectomy.
e. Appendicular abscess drainage.

24. During conservative treatment of a patient with adhesive intestinal


obstruction, which of the following is an indication to proceed to
surgery?
a. Nasogastric suction of more than 2 L/24 hours.
b. Exaggerated bowel sounds.
c. Abdominal rigidity.
d. Abdominal distension.
e. Serum K level below 2.5 mEq/L.
25. The most important prognostic index in breast cancer is:
a. Age of the patient.
b. Site of the lesion.
c. Presence of lymph node metastases.
d. Level of CEA.
e. Presence of micro-calcification.

11. Which of the following are not found in peritonitis?


a. Patient is lying still
b. Guarding
c. Rebound tenderness
d. Hyperactive bowel sounds
e. Rigid abdomen

The following is an indication for thoracotomy in chest .28


,injury

a. Cardiac tamponade
b. Uncontrolled pulmonary air leakage
c. Perforation of thoracic esophagus
d. Blood loss of 200ml/hr for 2-3 hrs via chest tube
e. All of the above

1. Which of the following do you consider to be the most important clinical sign in acute
appendicitis,

1. Abdominal tenderness around the umbilicus


2. Abdominal tenderness in the RIF
3. Tenderness over McBurney’s point
4. Rovsing’s sign positive
5. Suprapubic tenderness

2. All of the following are mechanisms of urinary calculi formation except,

1. Hypoparathyroidism
2. Prolonged recumbency
3. Infection with urea-splitting organisms
4. Foreign body
5. Urinary tract obstruction

3. Organisms most commonly isolated in UTIs,

1. Kleb
2. E. Coli
3. Proteus
4. Acinetobacter
5. All of the above

4-The ideal fluid therapy in a patient with pyloric stenosis and repeated vomiting is:
a. Isotonic crystaloid containing sodium chloride

b. Hypertonic crystaloid containing dextrose – saline

c. Isotonic solution containing dextrose

d. Large molecular weight colloid containing dextran

5. Excessive saliva in a newborn is due to

1. Esophageal atresia
2. Salivary gland tumour
3. Primary Hypertrophic Pyloric Stenosis
4. Midgut volvulus
5. Hirschprung’s disease

6. The evaluation of a comatose patient with a head injury begins with:

a.The cardiovascular system.

b. Pupillary reflexes.

c. Establishment of an airway.

d. Computed tomography (CT) of the brain

e .insertion of Intravenous canula

7. The following is an indication for thoracotomy in chest injury,

f. Cardiac tamponade
g. Uncontrolled pulmonary air leakage
h. Perforation of thoracic esophagus
i. Blood loss of 200ml/hr for 2-3 hrs via chest tube
j. All of the above

8. In an upper gastrointestinal bleed, in some cases a Sengstaken-Blakemore tube is


incerted. What is the primary purpose of the tube?

a. Aspirate blood from the stomach


b. Tube feeding
c. Tamponade for varices
d. To decompress bowel
e. All of the above
9. Which of the following is not found in peritonitis?

a. Patient is lying still


b. Guarding
c. Rebound tenderness
d. Hyperactive bowel sounds
e. Rigid abdomen

10. Commonest site for Cancer breast

a. Upper outer quadrant


b. Upper inner quadrant
c. Lower outer quadrant
d. Lower inner quadrant
e. None of the above

11. The back of the medial epicondyle is related to the:

a. Radial nerve.

b. Axillary nerve.

c. Ulnar nerve.

d. Median nerve.

e. None of the above.

12. Regarding Gallstones all of the following are true except:-

a. Prevalence increases with advancing age


b. 30% of gallstones are radio-opaque
c. Cholesterol stones result from a change in solubility of bile constituents
d. Biliary infection, stasis and changes in gallbladder function can
precipitate stone formation
e. Gram-negative organisms are the most common isolated

13.Clinical features of limb ischaemia includes

a. Paraesthesia
b. Pallor
c. Pulselessness
d. Paralysis
e. All of the above

14. Regarding Hydatid disease all of the following are true except:-
a. Due to infection with the helminth Ecchinococcus granulosa
b. Man is an accidental intermediate host
c. Lunges is the commonest organ involved
d. Diagnosis can be confirmed by indirect haemagglutinin assay
e. Aspiration should not be performed if hydatid disease is suspected

15.Which hypersensitivity reaction is associated with serum sickness?


a. Type I: immediate
b. Type II: cytotoxic
c. Type III: immune complex
d. Type IV: cell mediated
e. None of the above

16. The most common cause of mass in the posterior mediastinum is


a. Vascular
b. Esophageal diverticula
c. Neurogenic tumors
d. Lymphomas
e. Bronchogenic cysts

17.In obstructive jaundice:


a. Urinary conjugated bilirubin is increased
b. Serum unconjugated bilirubin is increased
c. Urinary urobilinogen increased
d. Serum conjugated bilirubin is reduced
e. Faecal stercobilinogen is increased

18-Which of the following statements about extracellular fluid is true?

a. The total extracellular fluid volume represents 40% of the body weight.
b. The plasma volume constitutes 20%of the total extracellular fluid
volume.
c. Potassium is the principal cation in extracellular fluid.
d. The protein content of the plasma produces a lower concentration of
cations than in the interstitial fluid.
e. The interstitial fluid equilibrates slowly with the other body
compartments.

19. The following cause hypercalcaemia except :


a. Sarcoidosis
b. Primary hyperparathyroidism
c. Acute pancreatitis
d. Metastatic bronchial carcinoma
e. Milk-Alkali syndrome

20.For a 22-kg baby the maintenance daily fluid requirement is approximately


which of the following?
a. 1100 ml
b. 1250 ml
c. 1550 ml
d. 1700 ml
e. 1850 ml

21.The transfusion of fresh frozen plasma (FFP) is indicated for which of the
following reasons?
a. Volume replacement.
b. As a nutritional supplement.
c. Treatment of prolonged bleeding time.
d. For the correction of abnormal PT secondary to warfarin therapy,
vitamin K deficiency, or liver disease.
e. All are correct

22.A 9 month old boy presents with an acute scrotal swelling. The following
diagnoses are likely:
a. Epididymitis
b. Orchitis
c. Torsion of the testicular appendage
d. Irreducible inguinal hernia
e. Acute idiopathic scrotal oedema

23- A 24-year-old man has sustained significant brain injury following an assault. He
is showing signs of a raised intra-cranial pressure. Which of the cranial nerves is
usally the first to be affected by a raised intracranial pressure?

a. The abducent nerve

b. The facial nerve

c. The hypoglossal nerve

d. The vagus nerve

e. The vestibulocochlear nerve


24. Anaphylactic shock:

a. Is an immune-mediated reaction.

b. Results in mast cell activation and increased circulating histamine


concentrations.

c. Produces microcirculatory changes similar to hypovolaemic shock.

d. Requires prompt treatment with parenteral adrenaline and


hydrocortisone.

e. May occur after ingestion of drugs

25. A 33-year-old woman develops a reducible mass of the groin that is inferolateral
to the pubic tubercle and medial to the femoral vein. Which of the following is the
most likely diagnosis?
a. Direct inguinal hernia

b. Femoral hernia

c. Incisional hernia

d. Indirect inguinal hernia

e. Umbilical hernia

26- Regarding papillary thyroid cancer

a. Account for 40% of thyroid cancer

b. Early blood spread

c. Commonest thyroid cancer in children

d. Common in endemic goiter areas

e. Multifocality is rare

27- The most commonly used imaging method for diagnosis of acute cholecystitis is:

a. CT of the abdomen.

b. Ultrasonography of the gallbladder.


c. Oral cholecystogram.

d. Radionuclide (HIDA) scan of the gallbladder

e. MRI

28. Fibro adenoma

a. Is common in females between 15-25 years

b. Usually soft indiscreet lump

c. Malignant changes in 5 %

d. Arise from duct of breast

e. none of the above

29-In tension pneumothorax the following signs are present except:

a. Hypoxia
b. Hyperresonance to percussion on the affected side
c. Tracheal deviation to the ipsilateral side
d. Distended neck veins
e. Tachycardia

30-Foot droop is usually associated with paralysis of the:

a. Tibial nerve
b. Common peroneal
c. Obturater
d. Medial planter
e. Lateral planter

1.The ilio-inguinal nerve:

A. supplies the rectus abdominis muscle

B. supplies skin on inner side of thigh

C. supplies the cremasteric muscle

D. supplies the urethra


E. does none of the above

2. The skin of the tip of the index finger is supplied by


the:

A. Radial nerve only

B. Radial & median nerves

C. Median & ulnar nerves

D. Ulnar nerve only

E. Median nerve only

3. Hypovolaemic shock is characterized by:

A. A low central venous pressure , high cardiac output,


low peripheral resistance

B. A high central venous pressure, high cardiac output,


high peripheral resistance

C. A low central venous pressure , low cardiac output,


high peripheral resistance

D. A low central venous pressure , high cardiac output,


high peripheral resistance

E. A high central venous pressure, low cardiac output,


low peripheral resistance

4. Which of the following in NOT true of Hodgkin's


disease?

A. Usually starts from several groups of nodes simultaneously


B. Usually involves liver & spleen

C. Sometimes manifests itself as pyrexia of unknown origin

D. Severe pain follows ingestion of alcohol

E. Shows increased susceptibility to opportunistic infection

5. Tetanus toxoid:

A. Is produced by injecting animals with antititanic serum

B. Is administered to previously immunized subjects with potentially


infected wounds

C. Frequently gives rise to anaphylactic reaction

D. Is used to induce active immunity against tetanus

6. The most probable cause of shock in a patient with multiple injuries &
craniocerebral trauma is:

A. Depression of vital medullary centres

B. Hypoperfuion control over subcortical areas

C. Loss of cortical control over subcortical areas

D. Hypovolaemia

E. Inadequate ADH response

7. The most sensitive guide to acute changes in fluid balance in a surgical


patient is:

A. Accurate daily weight

B. Serial serum Na concentration

C. Fluid balance sheets recording inputs & outputs

D. Daily urine output


E. Serial anion gap measurements

8. cellullitis is:

A. Inflammation of the bone marrow

B. Inflammation of the mastoid cells

C. Inflammation of the subcutaneous tissues

D. Infiltration of the skin by gaint cells

E. A malignant condition

9. secondary haemorrhage occurs:

A. Within 6 hours of operation

B. 7-14 days after operation

C. As a result of violent coughing on recovery from anaesthesia

D. Due to a blood transfusion line disconnected

E. When a ligature slips

10. the minimum urine output for 24 hours required to excrete end
products of protein metabolism is:

A. 200 ml

B. 300 ml

C. 400 ml

D. 500 ml

E. 600 ml

11. Potassium deficiency is present if the plasma-potassium level is:


A. 6.0 mmol/l

B. 5.0 mmol/l

C. 4.5 mmol/l

D. 4.0 mmol/l

E. 3.0 mmol/l

12.in health the pH of the blood lies between the range:

A. pH 7.05-7.19

B. 7.20-7.35

C. 7.36-744

D. 7.45-7.59

E. 7.60-7.80

13. TNM classification of a malignant tumour was designed as:

A. An histological staging

B. A clinical staging

C. A staging carried out at operation

D. A staging dependent upon radio scanning & skeletal survey

E. A staging dependent upon ultrasound

14. a blue-green discharge from an ulcer will be seen to contain:

A. Pseudomonas pyocyaneus

B. Streptococcus viridians

C. Candida albicans
D. Staphylococcus aureus

E. Haemophilius influenzae

15. a rodent ulcer is:

A. A squamous cell carcinoma

B. A basal cell carcinoma

C. Only occur on the face

D. Contains epithelial pearls

E. A venous ulcer

16. the space of Parona is:

A. In the wrist between the deep flexor tendons & the pronator quadratus

B. Above the patella between the quadriceps muscle & the femur

C. Benath the tendon of the iliopsos

D. Between the achills tendon & the posterior aspect of the tibia

E. The web space of the palm

17. 'rest pain' occurs:

A. Anywhere in the body at rest

B. In the thigh of a patient with Buerger's disease

C. In the calf of a patient with intermittent claudicating

D. In the foot of a patient with severe vascular disease

E. In the back

18. ischaemia means:


A. Pain in the ischial tuberosities

B. Anaemia due to malignant seconderies in the ischial part of the pelvis

C. Lack of blood flow

D. Increased blood flow

E. Polycythaemia

19. Colles' fracture is:

A. A common in adolescence

B. A fracture about the ankle joint

C. Common in elderly women

D. A fracture of the head of the radius

E. A fracture of scaphoid

20. Pott's disease is;

A. A fracture dislocation about the ankle

B. A neuropathic joint

C. Traumatic ostechondritis of the spine

D. Tuberculosis of the spine

E. A secondary tumour in the skull

21. Vincent's angina is a form of angina associated with:

A. Spasm of the oesophagus

B. Diphtheria

C. An infection of the mouth


D. Coronary artery spasm

E. Carcinoma of the bronchus

22. Ludwig's angina is due to :

A. A type of coronary artery spasm

B. Oesophageal spasm

C. Retropharyngeal infection

D. A virulent infection of the cellular tissue around the submandibular


salivary gland

E. Infection with candida

23. in simple nodular goiter:

A. Carcinoma occurs in 30% of cases

B. The nodular stage is irreversible

C. Operation is contraindicated

D. The patient does not develop hyperthyroidism

E. Cretinism is the presenting feature

24. Hashimoto's disease is:

A. A granulomatous thyroiditis

B. An auto-immune thyroiditis

C. An infiltrating fibrosis of the thyrois & the adjacent muscles

D. Focal thyroiditis

E. A parathyroid tumour
25. A thyroglossal fistula:

A. Is never congenital

B. Follows inadequate removal of a thyroglossal cyst

C. Has a hood of skin with its concavity upwards

D. Is lined throughout by squamous epithelium

E. Occurs in carcinoma of the tongue

26. The following are clinical signs supporting an early diagnosis of


carcinoma of the breast:

A. A prickling sensation in a breast lump

B. Peau d'ornge

C. Brawny arm

D. Cancer en cuirasse

E. A krukenderg tumour

27.The gastroduodenal artery is a branch of the:

A. Celiac axis

B. Hepatic artery

C. Superior mesenteric artery

D. Gastroepiploic artery

E. Splenic artery

28.Chronic gastric ulcers most often occur in patients with:

A. Blood groub A

B. Tend to occur in alkaline mucosa


C. Muscularis mucosae is separated from the muscularis at the edge of the
ulcer

D. Are malignant when there is epithelial proliferation & downgrowths

E. Are never large

29. Meckel's diverticulum:

A. Is present in 20% of the human race

B. Arises from the mesenteric border of the jejunum

C. May contain heterotopic pancreas

D. Is only present in the male sex

E. Is a diverticulum of the bladder

30. Intussusception is related to:

A. Mucoviscidosis

B. Swollen Peyer's patches

C. Volvulus

D. A littre's hernia

E. A patent vitello intestinal duct

31. The site of the neck of a femoral hernia is the:

A. Transversalia fascia

B. Iliopectineal ligament

C. Femoral ring

D. Cribriform fascia

E. Obturator foramen
32. Regarding operation for an indirect inguinal hernia:

A. It should not be performed on patients who have chronic bronchitis

B. General anaesthesia has to be used

C. In infants the posterior inguinal wall should be repaired

D. In adults the internal inguinal ring usually needs to be strengthened

E. Mesh implants are mandatory

2. Which hypersensitivity reaction is associated with a tuberculin reaction?


a. Type I: immediate
b. Type II: cytotoxic
c. Type III: immune complex
d. Type IV: cell mediated

3. The most common location for a gastric ulcer is


a. Fundus
b. Greater curvature
c. Cardia
d. Body
e. Antrum

4. Regarding the management of major trauma


a. Deaths follow a trimodal distribution
b. Cardiac tamponade is characterised by raised BP, low JVP and muffled heart
sounds
c. Assessment of uncomplicated limb fractures should occur during the primary
survey
d. Deterioration of the casualty during the primary survey should lead to the
secondary survey
e. All are false

5. All of the following are true about neurogenic shock except:


a. There is a decrease in systemic vascular resistance and an increase in venous
capacitance.
b. Tachycardia or bradycardia may be observed, along with hypotension.
c. The use of an alpha agonist such as phenylephrine is the mainstay of treatment.
d. Severe head injury, spinal cord injury, and high spinal anesthesia may all cause
neurogenic shock.

6. The following cause hypercalcaemia except :


a. Sarcoidosis
b. Primary hyperparathyroidism
c. Acute pancreatitis
d. Metastatic bronchial carcinoma
e. Milk-Alkali syndrome

7. For a 40-kg baby the maintenance daily fluid requirement is approximately which of the
following?
a. 1100 ml
b. 1250 ml
c. 1550 ml
d. 1700 ml
e. 2000 ml

8. . Infantile hypertrophic pyloric stenosis


a. Occurs with a male: female ratio of 4:1.
b. Presents between six and eight months of age
c. Typically presents with bile stained projectile vomiting
d. Surgical treatment is by Heller's Cardiomyotomy
e. all are false

8. Which of the following do you consider to be the most important clinical sign in acute
appendicitis,

1. Abdominal tenderness around the umbilicus


2. Abdominal tenderness in the RIF
3. Tenderness over McBurney’s point
4. Rovsing’s sign positive
5. Suprapubic tenderness

9.Heparin

1. Acts as an anti-platelet
2. Acts as an anti-thromboplastin
3. Acts as an antithrombin
4. All of the above
5. None of the above
10. All of the following are mechanisms of urinary calculi formation except,

1. Hypoparathyroidism
2. Prolonged recumbency
3. Infection with urea-splitting organisms
4. Foreign body
5. Urinary tract obstruction

11. Which of the following are not found in peritonitis?

1. Patient is lying still


2. Guarding
3. Rebound tenderness
4. Hyperactive bowel sounds
5. Rigid abdomen

12-Which one of the following suggest a diagnosis of Hirschsprung's disease?

a. A contrast-study showing dilatation of the aganglionic bowel segment.


b. Early presentation with vomiting.
c. Neonatal large bowel obstruction.
d. Presentation after 1 year of age.
e. Red current jelly stools.

13-Which of the following regarding the anatomy of the heart is true?

a. The aortic valve is tricuspid.


b. The ascending aorta is entirely outside the pericardial sac.

c. The left atrial appendage is identified readily by transthoracic echocardiography.


d. The pulmonary trunk lies anterior to the ascending aorta.
e. The right atrium is posterior to the left atrium.

14-Which of the following is true concerning Scaphoid fractures?

a. Rarely occur in young adults


b. when complicated by avascular necrosis the proximal pole is usually affected

c. should be treated by bone grafting and internal fixation even if undisplaced

d. wrist fractures are uncommon


e. anteriorposterior and lateral radiographs reveal most fractures
15-Which of the following statements is true of upper limb nerve injuries?

a. Injury to the median nerve results in a wrist drop


b. Injury to the radial nerve results in loss of sensation over the palmar aspect of the
index finger
c. Injury to the median nerve results in loss of sensation in the anatomical snuffbox
d. Injury to the ulnar nerve results in a claw hand
e. Injury to the ulnar nerve results in loss of sensation over the thumb

16- Regarding intravenous solutions:

a. Normal saline contains 180mmol/l of sodium


b. Ringer's lactate solutions is designed for intracellular fluid replacement
c. Sodium bicarbonate 8.4% is a hyperosmolar solution
d. Normal saline with added potassium is appropriate therapy to correct a
respiratory alkalosis
e. All are True

17-Which of the following concerning the Femoral sheath is false:

a. Contains the femoral artery


b. Contains lymph nodes
c. Contains the femoral canal
d. Contains femoral nerve
e. Contains the femoral vein

18-The following is true of the spleen:

a. Is the largest lymphoid organ in the body


b. Lies obliquely between the seventh and tenth rib
c. The lower pole extends beyond the mid-axillary line
d. Is usually palpable when normal
e. Usually measures 16cm in maximum length when healthy

19-Breast cancer risk is increased in association with the following factors except:

a. Nulliparity
b. Immediately after pregnancy
c. Early menarche
d. Early age at first pregnancy
e. Late menopause

20-In tension pneumothorax the following signs are present except:

f. Hypoxia
g. Hyperresonance to percussion on the affected side
h. Tracheal deviation to the ipsilateral side
i. Distended neck veins
j. Tachycardia

21- The most common hernia in females is:

a. Femoral hernia.
b. Direct inguinal hernia.
c. Indirect inguinal hernia.
d. Obturator hernia.
e. Umbilical hernia.

22-. The most helpful diagnostic radiographic procedure in small bowel obstruction is:

a. CT of the abdomen.
b. Contrast study of the intestine.
c. Supine and erect x-rays of the abdomen.
d. Ultrasonography of the abdomen.
e. MRI Abdomen

23- In role of nine extent of burn if entire trunk is burned it will be equal to:

a. 9% body surface area.


b. 18% body surface area.
c. 36% body surface area.
d. 27% body surface area.
e. 45% body surface area.

24-. If torsion of the testicle is suspected, surgical exploration:

a. Can be delayed 24 hours and limited to the affected side.


b. Can be delayed but should include the asymptomatic side.
c. Should be immediate and limited to the affected side.
d. Should be immediate and include the asymptomatic side.

25- Hyperthyroidism can be caused by all of the following except:

a. Graves' disease.
b. Plummer's disease.
c. Struma ovarii.
d. Hashimoto's disease.
e. Medullary carcinoma of the thyroid.

26- A 9 month old boy presents with an acute scrotal swelling. The following diagnoses are likely:

a. Epididymitis
b. Orchitis
c. Torsion of the testicular appendage
d. Irreducible inguinal hernia
e. Acute idiopathic scrotal oedema

27. The evaluation of a comatose patient with a head injury begins with:

a.The cardiovascular system.

b. Pupillary reflexes.

c. Establishment of an airway.

d. Computed tomography (CT) of the brain

e .insertion of Intravenous canula

28. The following is an indication for thoracotomy in chest injury,

k. Cardiac tamponade
l. Uncontrolled pulmonary air leakage
m. Perforation of thoracic esophagus
n. Blood loss of 200ml/hr for 2-3 hrs via chest tube
o. All of the above

29. Regarding Gallstones all of the following are true except:-

f. Prevalence increases with advancing age


g. 30% of gallstones are radio-opaque
h. Cholesterol stones result from a change in solubility of bile constituents
i. Biliary infection, stasis and changes in gallbladder function can precipitate stone
formation
j. Gram-negative organisms are the most common isolated

30.In obstructive jaundice:


a. Urinary conjugated bilirubin is increased
b. Serum unconjugated bilirubin is increased
c. Urinary urobilinogen increased
d. Serum conjugated bilirubin is reduced
e. Faecal stercobilinogen is increased

1) Abduction of the vocal cords results from contraction of the:


A) crico-thyroid muscles
B) posterior crico-arytenoid muscles
C) vocalis muscles
D) thyro-ary-epiglottic muscles
E) lateral crico-arytenoid and transverse arytenoids muscles
2) The rectum:
A) is devoid of peritoneum
B) is surrounded by peritoneum
C) has peritoneum on its lateral surfaces for its upper two- thirds, and
on its anterior surface for its upper one-third
D) has pritoneum on its anterior surface for its upper two-thirds, and
on its lateral surfaces for its upper one-third
E) has peritoneum on its anterior surface only
3) The umbilicus:
A) lies near the to the xiphoid than to the pubis
B) derives its cutaneous innervation from the eleventh thoracic nerve
C) transmits, during development, the umbilical cord two arteries and
two veins
D) usually lies at about the level between the third and fourth lumbar
vertebra
E) emberiologicall, may transmit urine but never bowel contents
4) The superficial perineal pouch:
A) is limited inferiorly by the urogenital diaphragm
B) is not continuous with the space in the scrotum occupied by the
testes
C) has a membranous covering which provides a fascial sheath around
the penis
D) is traversedby the urethera in the male but not the urethera and
vagina in the female
E) in the female, the greater vestibular glands are situated outside this
pouch
5) The tongue:
A) has a foramen caecum at the base of the frenulum
B) is separated from the epiglottis by the valleculae on each side of the
midline
C) has 7-12 circumvallate papillae situated behind the sulcus terminalis
D) is attached to the hyoid bone by the genioglossus muscle
E) is supplied only by hypoglossal nerve
6) Hypovolaemic shock is characterized by:
A) a low central venous pressure , low cardiac output , low peripheral
resistance
B) a high central venous pressure , high cardiac output , low peripheral
resistance
C) a low central venoys pressure , low cardiac output , high periphera
resistance
D) a low central venous pressure , high cardiac output , high peripheral
resistance
E) a high central venous pressure , low cardiac output , low peripheral
resistance
7) An oxygen debt is:
A) the amount of oxygen in excess of the resting metabolic needs that
must be consumed after completion of exercise
B) build up because the pulmonary capillaries limit the uptake of
Oxygen at high rates of oxygen consumption
C) related to the fact that skeletal muscle cannot function temporarily in
the absence of oxygen
D) associated with a decrease in blood lactate
E) associated with alkalosis
8) Pulmonary embolism may be a complication of:
A) prolonged bed rest
B) a surgical operation
C) vitamine K deficiency
D) oral contraceptive therapy
E) Antithrombin III deficiency
9) Which of the following statements regarding potassium metabolism is NOT
True?:
A) potassium deficiency commonly results from thiazide diuretic theraoy
B) the normal compensation for potassium deficiency is a metabolic
extracellular acidosis
C) aldosterone increases urinary potassium loss
D) hyperkalaemia causes bradycardia and loss of P waves on the ECG
E) hypokalaemia aggrevates the cardiac effects of digitalis toxicity
10) Cutaneous pain:
A) is due to overstimulation of receptors serving other sensory modalities
B) cannot be elicited more readily if the tissue has recently been injured
C) is due to exitation of receptors by pain-producing chemical substances in
the injured tissue
D) shows marked adaptation, i. e. decrease in severity in response to a
constant stimulus
E) is conducted through the medial spinothalamic tract
11) Which of the following is NOT associated with hyperthyroidism?:
A) increase size of the thyroid gland
B) increased amount of colloid in thyroid follicle
C) increased height of epithelium of the thyroid follicle
D) increased vascularity of the thyroid gland
E) increased uptake of iodine by the thyroid gland

12) Sarcomata may show all of the following EXCEPT:


A) production of myxomatous tissue
B) production of collagen
C) spindle shaped cells
D) signet ring cells
E) blood stream metastasis
13) Anaphylaxis is characterized by all of the following EXCEPT:
A) is a reaction either local or general , frequently occurs within five
minutes
B) causes an urticarial eruption
C) is produced by IgA antibody
D) causes eosinophilia
E) causes degranulation of basophils and mast cells
14) Autoimmunity is characterized by the following EXCEPT:
A) occurs because of a breakdown in the ability of the body to distinguish
between self and non self
B) is involved in some forms of orchitis
C) is involved in formation of cryo globulin
D) is important in the pathogenesis of lupus erytheromatosus
E) does not result in immune complex disease
15) Pseudomembranous enterocolitis is caused by the following organisms:
A) Clostridium sporogenes
B) Clostridium defficile
C) Streptococcus faecalis
D) Penicillin sensitive staphylocci
E) Pseudomonas aeruginos

B) CLINICAL SUGERY
16) The “ white clot syndrome”:
A) is usually characterized with antithrombin III deficiency
B) most often present with arterial complicatios of heparin induced
throbocytopenin
C) is best managed by loe molecular weight dextran
D) is best managed by halving the therapeutic dose of heparin sodium
E) results from nitric oxide deficiency of endothelial cells
17) A 21-year-old man who was the driver in a head-on collision has a pulse of
140/min , respiratory rate of 36 and blood pressure of 75 palpable. His trachea
is deviated to the left, with palpable subcutaneous emphysema and poor breath
sounds in the right hemithorax, The most appropriate initial treatment must
be
A) immediate thoracotomy
B) catheter insertion in the subclavian vein for fluid resuscitation
C) intubation and ventilation
D) tube thoracostomy
E) immediate tracheostomy
18) The best test to monitor the adequacy of levothyroxin therapy is:
F) radioactive iodine uptake
G) thyroglobulin
H) free thyroxine index (T4)
I) triiodothyronine resin uptake (T3)
J) thyroid stimulating hormone (TSH)
19) Which of the following statements about fungal infection is NOT true ?:
A) Prior or synchronous culture positive for Candida at another site
occurs in few patients with candidimia
B) For critically ill patients nonhaematogenous sites of candida are
appropriately treated with systemic antifungal therapy
C) Mortality rates are similar regardlss of whether C. albicans fungmia
is treated with amphotericin B or fluconazole
D) Intravenous catheters and the gastrointestinal tract are common
portals for Candida to gain blood stream access
E) Septic emboli are more common with fungal endocarditis than with
bacterial endocarditis
20) The maximum safe dose of local anaesthetic administered
subcutaneously in a 70-kg man is:
A) 10 to 20 ml of 1% lidocaine
B) 40 to 50ml oh 2% lidocaine with epinephrine
C) 40 to 50 ml of 1% lidcaine with epinephrine
D) 40 to 50 ml of 1% bupivacaine (marcaine)
E) 40 to 50 ml of 1%lidocaine without epinephrine
21) Two days after right hemicolectomy for a Dukes B caecal carcinoma ,
the
Patient complains of sharp right-sided chest pain and dyspnea.
HisPaO2
Is 64mmHg ,his PaCo2 is 32mmHg. CVP is 26 cm water, and the blood
pressure is 102/78mmHg. A pulmonary embolus is suspected,
The next step in management should be:
A) A ventilation- perfusion lung scan
B) A pulmonary arteriogram
C) Postrioanterior and lateral chest x-rays
D) Heparin sodium ,100 units/kg intravenously
E) Immediate duplex scanning of both lower extremities
22) The major cause of graft loss in heart and kidney allograft is:
A) acute rejection
B) hyperacute rejection
C) vascular thrombosis
D) chronic rejection
E) graft infection
23) All of the following are indicators of tumor aggressiveness and poor
outcome for papillary carcinoma of the thyroid gland EXCEPT:
A) age over 50 years
B) microscopic lymph node metstasis
C) tumor larger than 4 cm
D) poorly differentiated histological grade
E) invasion through capsule to adjacent tissues
24) A 40-year-old woman has extensive microcalcifications involving the
entire upper aspect of the right breast. Biopsy shows a commedo
pattern of intraductal carcinoma.
The most appropriate treatment is :
A) wide local excision
B) radiation therapy
C) wide local excision plus radiation therapy
D) right total mastectomy
E) right modified radical mastectomy
25) In the conventional ventilator management of acute adult respiratory
distress syndrome (ARDS) , arterial O2 saturation is maintained above
90% by all the following EXCEPT :
F) increasing the ventilatory rate
G) the use of positive end-expiratory pressure (PEEP)
H) increasing mean airway pressure
I) increasing tidal volumes
J) increasing FiO2
26) Which of the following statements about patients with abdominal
compartment syndrome is NOT true ?
A) Abdominal pressure is usually measured indirectly through inferior
vena cava
B) Multiple contributing factors are commonly responsible
C) The chief manifestations are reflected in central venous pressure ,
ventilatory function, and oliguria
D) Decopression of the abdomen is required to resverse the syndrome
E) Aggressive hemodynamic monitoring and management is required
when the abdomen is opened
27) The most appropriate treatment for histologically malignant
cystadenoma phylloides is :
A) total mastectomy without axillary node dissection
B) total mastectomy with axillary node dissection
C) wide margin (3) cm excision of the lesion
D) post operative hormonal manipulation
E) postoperative adjuvant chemotherapy
28) Deep venous thrombosis resulting from upper extremity central
venous lines:
A) should be treated with catheter removal, heparin therapy, and long
term anticoagulants
B) is best with urokinase through the catheter
C) is innocuous and self limiting, and best treated with catheter
removal only
D) is best treated with low-dose warfarin (coumadin, 1 mg / day) ,
without catheter removal
E) is best managed by single systemic dose of low molecular weight
heparin daily and continued catheter use
29) Emergency surgery is indicated for all of the following complications
of ulcerative colitis EXCEPT:
A) colonic dilatation greater than 12 cm (toxic mega colon )
B) free perforation
C) complete intestinal obstruction
D) intractable haemorrhage
E) abscess formation
30) All the following statements concerning carcinoma of the oesophagus
are true EXCEPT that:
A) it has a higher incidence in males than females
B) alcohol has been implicated as a precipitating factor
C) adenocarcinoma is the most common type at the cardio esophageal
junction
D) it occurs more commonly in patients with corrosive oesophagitis
E) surgical excision is the only effective treatment
31-. Which of the following statements about epiphrenic diverticula of the esophagus
is/are correct?

f) They are traction diverticula that arise close to the tracheobronchial tree.
g) They characteristically arise proximal to an esophageal reflux stricture.
h) The degree of dysphagia correlates with the size of the pouch.
i) They are best approached surgically through a right thoracotomy.
j) The operation of choice is a stapled diverticulectomy, long esophagomyotomy,
and partial fundoplication.

32- Which of the following statements about Schatzki's ring is correct?

f) The ring represents a panmural fibrotic stricture resulting from gastroesophageal


reflux.
g) Dysphagia occurs when the ring diameter is 13 mm. or less.
h) The ring occurs within 1 to 2 cm. of the squamocolumnar epithelial junction.
i) Schatzki's ring indicates reflux esophagitis.
j) Schatzki's ring signifies the need for an antireflux operation.

33. Which of the following statements about pathology encountered at


esophagoscopy is/are correct?

f) Reflux esophagitis should be graded as mild, moderate, or severe, to promote


consistency among different observers.
g) An esophageal reflux stricture with a 2-mm. lumen is not dilatable and is best
treated with resection.
h) A newly diagnosed radiographic distal esophageal stricture warrants dilation and
antireflux medical therapy.
i) In patients with Barrett's mucosa, the squamocolumnar epithelial junction occurs 3
cm. or more proximal to the anatomic esophagogastric junction.
j) After fasting at least 12 hours, a patient with megaesophagus of achalasia can
safely undergo flexible fiberoptic esophagoscopy.

34. Infantile hypertrophic pyloric stenosis


f) Occurs with a female : male ratio of 4:1.
g) Sons of affected mothers have a 20% risk of developing the lesion
h) Invariably presents between six and eights months of age
i) Typically presents with bile stained projectile vomiting
j) Surgical treatment is by Heller's Cardiomyotomy

35-. Which of the following statements regarding the pathology of esophageal


carcinoma is/are correct?

f) Worldwide, adenocarcinoma is the most common esophageal malignancy.


g) Squamous cell carcinoma is most common in the distal esophagus, whereas
adenocarcinoma predominates in the middle third.
h) Patients with Barrett's metaplasia are 40 times more likely than the general
population to develop adenocarcinoma.
i) Metastases from esophageal carcinoma are characteristically localized to regional
mediastinal lymph nodes adjacent to the tumor.
j) Achalasia, radiation esophagitis, caustic esophageal stricture, Barrett's mucosa,
and Plummer-Vinson syndrome are all premalignant esophageal lesions that
predispose to the development of squamous cell carcinoma.

36-45-year-old with isolated 6-cm colorectal metastasis in the liver 2 years after
colectomy, otherwise healthy pest treatment would be:

f) Radiofrequency ablation
g) Systemic chemotherapy
h) Hepatic lobectomy
i) Liver transplantation
j) Cryosurgical ablation

37- Oesophageal atresia all true except:


f) Is often associated with a distal trachea-oesophageal fistula
g) Polyhydramnios is often present late in pregnancy
h) 50% have other associated congenital abnormalities
i) Contrast X-ray studies are necessary to confirm the diagnosis
j) Post-operatively over 30% develop oesophageal strictures

38-All are true about the dumping syndrome except:


f) Symptoms can be controlled with a somatostatin analog.
g) Diarrhea is always part of the dumping syndrome.
h) Flushing and tachycardia are common features of the syndrome.
i) Separating solids and liquids in the patient's oral intake alleviates some of the
symptoms of the syndrome.
j) Early postoperative dumping after vagotomy often resolves spontaneously.

39-Which of the following statements about gastric polyps is/are true?

f) Like their colonic counterparts, gastric epithelial polyps are common tumors.
g) They are analogous to colorectal polyps in natural history.
h) Endoscopy can uniformly predict the histology of a polyp based on location and
appearance.
i) In a given patient, multiple polyps are generally of a multiple histologic type.
j) Gastric adenomatous polyps greater than 2 cm. in diameter should be excised
because of the risk of malignant transformation.

40-All of the following statements about surgical management of gastric lymphomas


are true except:

f) Stage I gastric lymphomas (small lesions confined to the stomach wall) can be
cured completely with surgical therapy alone.
g) Extensive gastric lymphomas that initially are treated with radiation and/or
chemotherapy occasionally perforate during treatment and require secondary
resection.
h) Patients explored with a presumptive diagnosis of gastric lymphoma should
undergo an attempt at curative resection when this is safe and feasible.
i) Without a preoperative diagnosis resection for gastric mass should not be
attempted unless lymphoma can be excluded.
j) Appropriate staging for primary gastric lymphoma includes bone marrow biopsy.

41-the most accurate test to confirm diagnosis of infected necrotizing pancreases is:

f) Abdominal ultrasound study


g) Indium-labeled leeukocte scan
h) Cimputed tomographic scan
i) Elevated serum level of interleukain 6 and 8
j) Percutaneous needle aspiration

42- Which of the following variables best predicts prognosis for patients with a
recent diagnosis of cutaneous melanoma and no clinical evidence of metastatic
disease?

f) Breslow thickness.
g) Clark's level.
h) Ulceration.
i) Gender.
j) Celtic complexion.
43-the following are true about intracranial tumors except:

f) The most common location of brain tumors of childhood is the posterior cranial
fossa.
g) With few exceptions, examination of the CSF is of no value in the diagnosis of an
intracranial tumor.
h) Even the most malignant of primary brain tumors seldom spread outside the
confines of the central nervous system (CNS).
i) The majority of astrocytomas can be cured surgically.
j) Primary neoplasms of astrocytic, oligodendroglial, or ependymal origin represent
gradations of a spectrum from slowly growing to rapidly growing neoplasms.

44 A right-sided disc herniation at the L5–S1 level typically may cause:

f) Low back pain and left sciatica.


g) Weakness of dorsiflexion of the right foot.
h) A diminished or absent right ankle jerk.
i) Diminution of sensation over the medial aspect of the right foot, including the
great toe.
j) Weakness of dorsiflexion of the left foot.

45-. The preferred operation for initial management of a thyroid nodule that is
considered suspicious for malignancy by FNAB is:

f) Excision.
g) Partial lobectomy.
h) Total lobectomy and isthmusectomy.
i) Total thyroidectomy.
j) All methods are correct

46-the most common presentation of Meckel,s diverticulum in an adult is:

f) GIT bleeding
g) GIT obstruction
h) Intussuception
i) Litter,s hernia
j) Diverticulitis

47-Optimal front-line treatment of squamous cell carcinoma of the rectum includes:

f) Abdominal perineal resection.


g) Low anterior resection when technically feasible.
h) Radiation therapy.
i) Chemotherapy.
j) Combined radiation and chemotherapy.

48 -65-year-old man presents with complaints of mucous discharge and perianal


discomfort. Physical examination reveals a fistulous opening lateral to the anus.
Anoscopic examination permits passage of a probe through the fistula tract. The
fistula traverses the internal anal sphincter, the intersphincteric plane, and a portion
of the external anal sphincter. The fistula is categorized as which type?

a) Intersphincteric
b) Transsphincteric
c) Suprasphincteric
d) Extrasphincteric
e) Non of the above

49-Warthin's tumor:

f) Is a pleomorphic adenoma of salivary gland


g) Should be treated by total paritidectomy
h) Is considered a benign salivary gland neoplasia
i) Respond well to preoperative radiotherapy
j) Often present with facial nerve compression

50-A 38 year old woman presents with right upper quadrant pain and
bouts of vomiting. She is known to have gallstones and has had similar
episodes in the past. Which of the following might support a diagnosis
of acute cholecystitis rather than biliary colic

a) duration of symptoms
b) Severity of vomiting
c) Presence of Murphy's sign
d) Presence of gas under right hemidiaphragm on erect CXR.

1. The radial nerve.


a) Is a branch of the anterior cord of the brachial plexus.
b) Is derived from the posterior primary rami of C5 to C7.
c) Supplies the flexors of the arm.
d) Gives rise to the anterior interosseous nerve.
e) Injury above elbow produces a classical wrist drop.
2. Regarding femoral canal all are true except:
a) Lies lateral to the femoral vein.
b) Has the inguinal ligament as its anterior border.
c) Has the lacunar ligament as its medial border.
d) Has the pectineal ligament as its posterior border.
e) Contains the lymph node of Cloquet.
3. The following causes hypercalcaemia except:
a) Sarcoidosis.
b) Primary hyperparathyroidism.
c) Acute pancreatitis.
d) Metastatic bronchial carcinoma.
e) Milk-Alkali syndrome.
4. Number of human chromosomes in human female are:
a) 23 pairs +XX.
b) 21 pairs +XY.
c) 22 pairs +XY.
d) 22 pairs +XX.
e) 23 pairs +XY.
5. Potassium deficiency should be suspected in all the following
except:
a) In cases of paralytic ileus.
b) When the patient's reflexes are exaggerated.
c) If there is a decrease in height and peaking of the T waves of an
ECG.
d) In alkalotic states.
6. Active immunization in case of tetanus:
a) Antitetanus human serum.
b) Gives short period of protection.
c) Given in case proved tetanus.
d) Use of immunoglobulin.
e) None of the above.
7. All of the following are signs of rised intracranial pressure
except:
a) Headache.
b) Vomiting.
c) Papilledema.
d) Aphasia.
e) Bradycardia.
8. All of the following are extra cranial hematomas except:
a) Subcutaneos haematoma.
b) Extra dural haematoma.
c) Cephalohaematoma.
d) Subgaleal hematoma.
e) Subperiostial haematoma.
9. Glasgow coma scale all the following are true except:
a) Used for evaluation of comatose patient.
b) It ranges from ( 3 to 15).
c) Useful for neurological follow up.
d) Useful for pupils evaluation.
e) Best motor response given 6 point.
10. All the followings are Indications for central line insertion
EXCPET:
a) Massive fluid replacement
b) Massive blood replacement
c) Measurement of central venous pressure
d) Prolonged Intervenes fluid therapy
11. Most common early complication of central venous line is:
a) Sepsis
b) Pneumothorax
c) Thoracic duct injury
d) Thrombosis
12. The following are Complications of shock:
a) Acute Respiratory failure
b) Acute myocardial infarction
c) Acute renal failure
d) A&C only
e) All the above
13. Causes of delayed union of fractures includes all the following
EXCPET:
a) Compound fracture
b) Infection
c) Adequate immobilization
d) Poor blood supply
14. In Head injury C.T. scan is indicated in the following
a) Aphasia after the injury
b) Deterioration of level of consciousness
c) Skull fracture with persistent headache
d) A&B only
e) All the above
15. Most important steps in management of head injury include:
a) Prevent hypoxia
b) Prevent Dehydration
c) Assure Brain Metabolism
d) Prevent secondary brain injury
e) All the above
16. Tension pneumothorax
a) is the commonest type of chest injuries
b) Needs urgent X-Ray chest
c) Is a clinical Diagnosis
d) Causes flat neek viens
e) Treated by thoracotomy tube after chest X-ray.
17. Calcitonin hormon is secreted to the blood circulations from:
a) Parathyroid gland.
b) Parafollicular cells of thyroid gland.
c) Supra renal gland.
d) Pituitary gland.
e) Gonads.
18. Regarding tension pneumothorax, the first step in the management
is:
a) Obtaining a stat chest x-ray.
b) Cricothyroidectomy.
c) Pass an endotracheal tube.
d) Starting oxygen by a valve-mask device.
e) Chest decompression needle.
19. The following are features of thyrotoxicosis except:
a) Weight gain.
b) Palpitations.
c) Proximal myopathy.
d) Increased skin pigmentation.
e) Pretibial myxoedema.
20. The following is a clinical feature of Horner's syndrome:
a) Miosis
b) Failure of abduction of the orbit
c) Increased sweating on the contralateral side of the forehead
d) Exophthalmos.
e) All are true
21. In role of nine extent of burn if entire trunk is burned it will be
equal to:
a) 9% body surface area.
b) 18% body surface area.
c) 36% body surface area.
d) 27% body surface area.
e) 45% body surface area.
22. Trachlea (4th) cranial nerve supply :
a) Lateral rectus muscle of th eye.
b) Medial rectus muscle of the eye.
c) Superior obligue rectus muscle of the eye.
d) Superior oblique muscle of the eye.
e) Muscles of the upper eye lid.
23. Regarding pathological terms :
a) Hypertrophy is an increase in tissue size due to increased cell
number.
b) Hyperplasia is an increase in tissue size due to an increase in cell
size.
c) Atrophy is an increase in tissue size due to disuse.
d) Metaplasia is a change form one abnormal tissue type to another.
e) A hamartoma is a developmental abnormality.
24. Regarding nephroblastomas:
a) They are otherwise known as a Wilm's tumour.
b) Account for 10% of childhood tumours.
c) The commonest presentation is with an abdominal mass.
d) Most commonly present between 2nd and 4th year of life.
e) All are true.
25. Regarding fluid losses in a major burn all are true except:
a) Are maximal between 12 and 24 hours after the injury.
b) Are related to the age of the patient.
c) Are not related to the weight of the patient.
d) Are related to the area burnt.
e) Are not related to the burn duration.
26. In obstructive jaundice :
a) Urinary conjugated bilirubin is increased.
b) Serum unconjugated bilirubin is increased.
c) Urinary urobilinogen is increased.
d) Serum conjugated bilirubin is reduced.
e) Faecal stercobilinogen is increased.
27. Regarding Hydatid disease:
a) Is due to Ecchinococcus granulosa.
b) Man is an accidental intermediate host.
c) The liver is the commonest site of infection.
d) Can be diagnosed by the Casoni test.
e) All are true.
28. The first aid of treatment in fracture of cervical spine should
be:
a) Cervical spine x-ray.
b) Analgesia.
c) Neck immoblization.
d) Cervical traction.
e) Non of teh above.
30. Regarding local anaesthesia:
a) Local anaesthetics act on small before large nerve fibres
b) Adrenaline reduces absorption and prolongs the local effects
c) Preparations containing adrenaline are safe to use on digits and
appendages
d) Lignocaine has a longer duration of action than bupivicaine.
e) All are false.
31. Small bowel obstruction often results in: (all correct except one)
a) Hyperkalaemia.
b) Metabolic alkalosis.
c) Oliguria.
d) Hypovolaemia.
e) Severe dehydration.
32. A serious intra-abdominal injury in a comatose patient may be
diagnosed by: (all are correct except one)
a) Abdominal paracentesis.
b) The observation of bruising pattern on the abdominal wall.
c) Falling of heamoglobin values.
d) The presence of marked abdominal distetion.
e) The presence of diarrhea.
33. A perforated duodenal ulcer, all are true except:
a) Usually lies on the anterior or superior surface of the duodenum.
b) Usually presents with the acute onset of severe back pain.
c) Produces radiological evidence of free gas in the peritoneum in
over 90 percent of the patients.
d) Is usually treated by vagotomy and pyloroplasty.
e) Is usually treated conservatively.
34. Acute pancreatitis typically: (all correct except one)
a) Is accompanied by hypocalcaemia.
b) Produces paralytic ileus.
c) Is associated with a pleural effusion.
d) Produces pyloric stenosis.
e) Upper abdominal pain and vomiting.
35. Biliary colic typically:
a) Occurs 3 to 4 hours after meals.
b) Lasts 5 to 20 minutes.
c) Radiates from the upper abdomen to the right subscapular region.
d) Is made better by deep inspiration.
e) B&C only.
36. In post operative DVT, the following are true except:
a) Clinical DVT occures in the 4th post operative day.
b) If complicated by pulmonary embolism, it occures usually after the
7th post operative day.
c) The process of DVT starts preoperatively with the induction of
anaesthesia .
d) When discovered we should start the patient on coumadin
"Warfarin anticoagulation".
e) It may lead to chronic venous in suffering as a complication of
DVT.
37. In acute appendicitis all of the following are true except:
a) Anorexia.
b) Abdominal pain usually preceedes vomiting.
c) Pain after begins in the paraumbilical region.
d) Constipation diarrhea may occur.
e) Dysuria excludes the diagnosis.
38. The most common cause of massive haemorrhage in the lower
gastroinfestinal tract is :
a) Carcinoma.
b) Diverticulosis
c) Diverticulitis
d) Polyp.
e) Ulcerative colitis.
39. Painless haematuria is the leading presentation of :
a) Renal cell carcinoma.
b) Transitional cell carcinoma of the bladder .
c) Ureteric stone.
d) Pelvi-ureteric obstruction.
e) Ureterocele.
40. All of the following are complications of massive blood transfusion
except:
a) Acute congestive heart failure.
b) Transmission of infection.
c) Hypercalcaemia.
d) Hyperkalaemia.
e) Transfusion reactions.
41. Complication of undescended testis include all of the following
except :
a) Malignant degeneration.
b) Increased susceptability to trauma.
c) Increased spermatogenesis.
d) More liable to testiculer torsion.
e) Psychological complication.
42. The recurrent laryngoeal nerve is branch of :
a) Facial nerve.
b) Glosso-pharyngeal nerve.
c) Cervical plexus.
d) Vagus nerve.
e) Brachial plexus.
43. The thyroid tumor which is may be associated with
pheochromocytoma
is :
a) Papillary carcinoma.
b) Medullary carcinoma.
c) Follicular carcinoma.
d) Anaplastic carcinoma.
e) Malignant lymphoma.
44. The most common pancreatic cyst is :
a) Dermoid cyst of the pancreas.
b) Hydatid cyst of the pancreas.
c) Pancreas pseudocyst.
d) Pancreatic cystadenoma.
e) Congenital cystic disease of the pancreas.
45. The anatomical division between the anus and rectum :
a) Lateral haemorrhoidal groove.
b) Inter haemorrhoidal groove.
c) Dentate line.
d) Arcuate line
e) Ano-rectal ring.
46. The comments type of Anorectal abscess is:
a) Ischio rectal
b) Perianal
c) Submucons
d) Pelvirectal
47. Anal Fissure:
a) Usually anterior
b) May be caused by previous anal surgery
c) Can cause dark bleeding PR.
d) Sometimes is painful
e) Treated by steroids
48. Neonatal duodenal obstruction:
a) May be associated with down's syndrome.
b) Is more frequently found in premature infants.
c) Typically presents with gross abdominal distension.
d) Usually presents with vomiting of non-bile stained fluid
e) B&C only.
49. acute superior mesenteric artery occlusion: (all correct except one)
a) Characteristically presents with sudden pain and tenderness of
increasing intensity.
b) Is frequently accompanied by overt or occult blood loss in the stools.
c) Frequently produces peritonitis.
d) Can usually be diagnosed on plain abdominal x-rays.
e) Can be diagnosed by mesenteric artery ongiography.
50. Regarding the management of polytrauma:
a) Death follow a trimodal distribution.
b) X-ray after primary survey should be AP cervical spine, chest
and pelvis.
c) Cardiac tamponade is characterized by raised B.p, a low JUP.
d) Assessment of uncomplicated limb fractures should occur
during the primary survey.
e) A and B only.
51. Injuries to the urethra (all are correct except one)
a) Are more common in male.
b) Are often caused by road traffic accidents.
c) Are easily diagnosed on intra venous pyelography.
d) Require urgent surgical treatment.
e) Diagnosed by retrograde urethragraphy.
52 .Car seat belts when properly adjusted
a) Prevent injuries to abdominal organs.
b) May cause small bowel injuries.
c) Do not reduce the incidence of head injuries of passengers involving
in RTA.
d) Protect the cervical spine during sudden acceleration .
e) A & D only.
53. Patients with major burns:
a) Are in a negative nitrogen balance.
b) Have normal calorie requirements.
c) Do not generally become anaemic.
d) Are resistant to septicaemia.
e) All of the above.
54 . In a healing fracture: (All correct except one)
a) The haematoma is initially invaded by osteoblasts.
b) The tissue formed by the invading osteoblasts is termed osteoid.
c) Calcium salts are laid down in the osteoid tissue.
d) The final stage of repair is the remodelling of the callus.
e) The callus formation is related to the amount of stress at fracture side.
55. In a colles’ fracture the distal radial fragment:
a) Is dorsally angulated on the proximal radius.
b) Is usually torn from the intra-articular triangular disc.
c) Is deviated to the ulnar side.
d) Is rarely impacted.
e) Is ventrally displaced.
56. A malignant melanoma:

a) Frequently arises from hair-bearing naevi.


b) Frequently arises from junctional naevi. *
c) Has a worse prognosis when it areses on the leg.
d) Should be suspected in any big pigmented lesion.
e) Non of the above is correct.
57. Squamous cancer of the lip:
a) Is most common in early adult life.
b) Is more common in fair skinned subjects.
c) Metastasises readily by the blood stream.
d) Is preferably treated by radiotherapy once lymph node deposits are
present.
e) All of the above are correct.
58- Basal cell carcinomas:
a) Usually metastasise to regional lymph nodes.
b) Are less common than squamous cell carcinomas.
c) Are characterised histologically by epithelial pearls.
d) Are particularly common in oriental races.
e) Non of the above is correct.
59- Fiboadenomata of the breast:
a) Are commonest in early adult life.
b) Are indiscrete and difficult to distinguish.
c) Are usually painless.
d) Resolve without treatment.
e) A&C only.
60. Paget’s disease of the nipple:
a) Usually presents as abilateral eczema of the nipple.
b) Is always related to an underlying breast cancer.
c) Indicates incurable breast cancer.
d) Has non-specific histological characteristics.
e) A&C only.
61- stones in the common bile duct:
a) Are present in nearly 50 per cent of cases of cholecystitis.
b) Often give rise to jaundice, fever and biliary colic.
c) Are usually accompanied by progressive jaundice.
d) Are usually associated with a distended gallbladder.
e) A&D only.
62- Colonic polyps: (all correct except one)
a) Are associated with colonic cancer.
b) May be hereditary.
c) Should not be removed if they are asymptomatic.
d) May be hyperplastic.
e) Are commonly adenomatous.

A 40-year-old female has a 4-cm hemangioma in the right lobe


of the liver on computed tomography scan. She is
asymptomatic. Appropriate action should be:

(a) fine-needle biopsy

(b) arrangement for elective resection

(c) no further action

(d) angiographic embolization

Regarding an amebic liver abscess:

(a) surgical drainage is usually required

(b) negative stool testing for amebiasis rules out the disease

(c) it should be drained percutaneously under com- puted


tomography guidance
(d) it is treated with metronidazole

A single organism is usually the causative agent in:


(a) pelvic inflammatory disease
(b) perforated diverticulitis
(c) acute cholecystitis
(d) primary peritonitis
(e) diabetic foot infections

Signs and symptoms of hemolytic transfusion reactions include


a. Hypothermia
b. Hypertension
c. Polyuria
d. Abnormal bleeding
e. Hypesthesia at the transfusion site

In a hemolytic reaction caused by an incompatible blood


transfusion, the treatment that is most likely to be helpful is:
a. Promoting a diuresis with 250 ml of 50% mannitol
b. Treating anuria with fluid and potassium replacement
c. Acidifying the urine to prevent hemoglobin precipitation in the
renal tubules
d. Removing foreign bodies, such as Foley catheters, which may
cause hemorrhagic complications
e. Stopping the transfusion immediately

Which statements about extrahepatic bile duct cancer are


correct?

A. Cholangiography is not essential in evaluating patients for


resectability.

B. The prognosis is excellent when appropriate surgical and


adjuvant therapy are given.
C. The location of the tumor determines the type of surgical
procedure.

D. The disease usually becomes manifest by moderate to severe


right-side upper quadrant pain.

Which of the following statements about the diagnosis of acute


calculous cholecystitis is true?

A. Pain is so frequent that its absence almost precludes the


diagnosis.

B. Jaundice is present in a majority of patients.

C. Ultrasonography is the definitive diagnostic test.

D. Cholescintigraphy is not definitive diagnostic test.

Which statement about acute acalculous cholecystitis is


correct?

A. The disease is often accompanied by or associated with other


conditions.

B. The diagnosis is often difficult.

C. The mortality rate is higher than that for acute calculous


cholecystitis.

D. The disease has been treated successfully by percutaneous


cholecystostomy

E- all are correct


Which of the following statements about laparoscopic
cholecystectomy are correct?

A. The procedure is associated with less postoperative pain and


earlier return to normal activity.

B. The incidence of bile duct injury is lower than for open


cholecystectomy.

C. Laparoscopic cholecystectomy should be used in asymptomatic


patients because it is safer than open cholecystectomy.

D. Pregnancy is a contraindication.

A 15-year-old female presents with RUQ abdominal pain.


Workup reveals a choledochal cyst. Which of the following
statements is TRUE?

(A) Choledochal cysts are more common in men.


(B) Laparoscopic cholecystectomy is the recommended treatment.
(C) Patients with a choledochal cyst have an increased risk of
cholangiocarcinoma.
(D) All patients with a choledochal cyst have abdominal pain, a
RUQ mass, and jaundice.
(E) The etiology is infectious.

An 85-year-old man is brought to the hospital with a 2-day


history of nausea and vomiting. He has not passed gas or
moved his bowels for the last 5 days. Abdominal films show
dilated small bowel, no air in the rectum and air in the biliary
tree. Which of the following statements is TRUE?

(A) Air in the biliary tree associated with small-bowel


obstruction suggests a diagnosis of gallstone ileus.
(B) An enterotomy should be distal to the site of obstruction and
the stone should be removed.
(C) Gallstone ileus is more common in the young adults.
(D) Cholecystectomy is contraindicated.
(E) Small-bowel obstruction usually occurs in the distal jejunum.

A45-year-old patient with chronic pancreatitis is suffering from


malnutrition and weight loss secondary to inadequate
pancreatic exocrine secretions. Which is TRUE regarding
pancreatic secretions?
(A) Secretin releases fluid rich in enzymes.
(B) Secretin releases fluid rich mainly in electrolytes and
bicarbonate.
(C) Cholecystokinin releases fluid,predominantly rich in
electrolytes, and bicarbonate.
(D) All pancreatic enzymes are secreted in an inactive form.
(E) The pancreas produces proteolytic enzymes only.

A 43-year-old woman has gallstone pancreatitis that resolves in


2 days with conservative treatment. She has no abdominal
complaints and her liver and pancreatic laboratory values have
returned to normal. She is scheduled for laparoscopic
cholecystectomy. Which of the following statements is TRUE?
(A) Intraoperative cholangiography is associated with a
decreased risk of biliary tract injury .
(B) The procedure should be scheduled for 6 weeks after
resolution of symptoms .
(C) Intraoperative cholangiography in this patient will identify
choledocholithiasis in 50% of cases .
(D) Preoperative endoscopic retrograde
cholangiopancreatography (ERCP) should be performed.
(E) The sensitivity of magnetic resonance
cholangiopancreatography (MRCP) for choledocholithiasis in this
patient is less than 50%.

A 39-year-old woman is admitted with gallstone pancreatitis


and epigastric pain. Pertinent data include amylase, 2000 U/L;
bilirubin, 1.2 mg/dL; and WBC count, 15,000/mm3 . After 2
days of medical management, her epigastric pain resolves. Her
amylase is 340 U/L and her bilirubin and WBC count have
returned to normal. Laparoscopic cholecystectomy should be
attempted:

(A) after endoscopic retrograde cholangiopancreatography


(ERCP) and sphincterotomy
(B) prior to discharge
(C) once her amylase is normal
(D) 4 to 6 weeks later
(E) only if the patient develops recurrent pancreatitis

Following a motor vehicle accident a truck driver complains of


severe abdominal pain. Serum amylase level is markedly
increased to 800 U. Grey Turner’s sign is seen in the flanks.
Pancreatic trauma is suspected. Which statement is true of
pancreatic trauma?
(A) It is mainly caused by blunt injuries.
(B) It is usually an isolated single-organ injury.
(C) It often requires a total pancreatectomy.
(D) It may easily be overlooked at operation.
(E) It is proved by the elevated amylase level.

In obstructive jaundice, which of the following statements is


true regarding alkaline phosphatase?
(A) Its level increases before that of bilirubin.
(B) Its level is unlikely to be increased in pancreatic malignancy.
(C) Its elevation indicates bone metastasis.
(D) Its elevation excludes hepatic metastasis.
(E) Its level falls after that of the bilirubin,following surgical
intervention.

A48-year-old female travel agent presents with jaundice.


Radiological findings confirm the presence of sclerosing
cholangitis. She gives a long history of diarrhea for which she
has received steroids on several occasions. She is likely to suffer
from which of the following?
(A) Pernicious anemia
(B) Ulcerative colitis
(C) Celiac disease
(D) Liver cirrhosis
(E) Crohn’s disease

A38-year-old male lawyer develops abdominal pain after


having a fatty meal. Examination reveals tenderness in the
right hypochondrium and a positive Murphy’s sign. Which test
is most likely to reveal acute cholecystitis?
(A) trans abdominal ultrasound
(B) Oral cholecystogram
(C) Intravenous cholangiogram
(D) CT scan of the abdomen
(E) ERCP

A 65-year-old woman is admitted with RUQ pain radiating to


the right shoulder, accompanied by nausea and vomiting.
Examination reveals tenderness in the RUQ and a positive
Murphy’s sign. A diagnosis of acute cholecystitis is made. What
is the most likely finding?
(A) Serum bilirubin levels may be elevated.
(B) Cholelithiasis is present in 40–60%.
(C) Bacteria are rarely found at operation.
(D) An elevated amylase level excludes this diagnosis.
(E) A contracted gallbladder is noted on ultrasound.

A 32-year-old diabetic woman who has taken contraceptive


pills for 12 years develops RUQ pain. CT scan of the abdomen
reveals a 5-cm hypodense lesion in the right lobe of the liver
consistent with a hepatic adenoma. What should the patient be
advised to do?
(A) Undergo excision of the adenoma
(B) Stop oral contraceptives only
(C) Stop oral hypoglycemic medication
(D) Undergo right hepatectomy
(E) Have serial CT scans every 6 months

A 9 month old boy presents with an acute scrotal swelling. The


mos likely diagnosis is:
A. Epididymitis
B. Orchitis
C. Torsion of the testicular appendage
D. Irreducible inguinal hernia
E. Acute idiopathic scrotal oedema

A 76-year-old man presents with weight loss, dark urine,


and pale stools which are difficult to flush away. An excess
of which of the following would account for this history?
a. Conjugated bilirubin
b. Hyperbilirubinaemia
c. Stercobilinogen
d. Unconjugated bilirubin
e. Urobilinogen

An otherwise well 13-year-old boy is admitted complaining


of sudden onset severe left sided testicular pain 2 hours prior to
admission. He gives no history of trauma, dysuria or frequency.
On examination he is found to have a tender, high-riding
testicle.What is the most appropriate next step in this young
?man’s management
a. Herniography
b. Scrotal Doppler ultrasound on the next available list
c. FBC and U&E
d. Scrotal Doppler ultrasound as an emergency
e. Surgical exploration of his scrotum

Acute scrotum
a. Torsion testis should be operated within 12 hour of
presentation
b. Epidedimoorchitis pain increase by testicular elevation
c. If in doubt scrotum should be explored
d.Doppler ultrasound has no role in diagnosis
e.None of the above

stones in the common bile duct:


a. Are present in nearly 50 per cent of cases of
cholecystitis.
b. Often give rise to jaundice, fever and biliary colic.
c. Are usually accompanied by progressive jaundice.
d. Are usually associated with a distended gallbladder.
e. A&D only.

Which of the following statements regarding whole blood


transfusion is correct?
a. Whole blood is the most commonly used red cell
preparation for
transfusion in the
b. Whole blood is effective in the replacement of acute
blood loss.
c. Most blood banks have large supplies of whole blood
available.
d. The use of whole blood produces higher rates of disease
transmission than the use of individual component therapies.
e. Old Whole blood is effective in the replacement of
platelets.

Acute cholecystitis all are true except


a. Commonest bacteria is E .coli
b. Wall thickness more than 3mm by ultrasound
c. WCC is between 10-15 000 cell/mm3
d. Mild elevated bilirubin may accompany it
e. HIDA scan has no role in diagnosis of acute
cholecystitis

A 51-year-old male experiences the sudden onset of massive


emesis of bright red blood. There have been no prior episodes
of hematemesis. He is known to be hepatitis B surface antigen
positive. His hematemesis is most likely a consequence of which
of the following abnormalities of the esophagus?
a. Varices
b. Barrett esophagus
c. Candidiasis
d. Reflux esophagitis
e. Squamous cell carcinoma

A 61-year-old male has had ascites for the past year. After a
paracentesis with removal of 1 L of slightly cloudy,
serosanguinous fluid, physical examination reveals a firm,
nodular liver.Laboratory findings include positive serum
HBsAg and presence of hepatitis B core antibody. He has a
markedly elevated serum alpha-fetoprotein (AFP) level. Which
of the following hepatic lesions is he most likely to have?
a. Hepatocellular carcinoma
b. Massive hepatocyte necrosis
c. Marked steatosis
d. Wilson disease
e. Autoimmune hepatitis

Acute pancreatitis
a.Serum calcium start to rise after 48 hours
b.Hypoglycaemia is bad prognostic factor
c.Age is an important prognostic factor
d.Serum amylase is more specific than serum lipase
e.Severe pancreatitis compromise around 40% of cases
The most commonly used imaging method for diagnosis of
acute cholecystitis is:
f. CT of the abdomen.
g. Ultrasonography of the gallbladder.
h. Oral cholecystogram.
i. Radionuclide (HIDA) scan of the gallbladder
j. MRI
A 23-year-old male presents to the emergency department after
being involved in a motor vehicle accident. On physical
examination, he opens his eyes spontanously, he occasionally
mumbles incomprehensible sounds, he localizes to painful
stimulation with his right upper extremity, His pupils are 4 mm
bilaterally and reactive. This patient’s Glasgow Coma Scale
(GCS) score:
a. 7
b. 9
c. 8
d. 11
e. 12

Complication of undescended testis include all of the following


except :
a. Malignant degeneration.
b. Increased susceptability to trauma.
c. Increased spermatogenesis.
d. More liable to testiculer torsion.
e. Psychological complication
Neonatal duodenal obstruction:
a. May be associated with down's syndrome.
b. Is more frequently found in premature infants.
c. Typically presents with gross abdominal distension.
d. Usually presents with vomiting of non-bile stained fluid
e. B&C only.

Markedly elevated alpha-fetoprotein is diagnostic


(A) Hepatic hemangioma
(B) Angiosarcoma in the liver
(C) Hepatic adenoma
(D) Focal nodular hyperplasia
(E) Hepatocellular carcinoma
on clinical examination of tortion testis all are true except:
a. testis is tender and swollen.
b.testis is elevated and raised.
c.loss of cremasteric reflex.
d.redness with possible reactive hydrocele.
e. pain decrease with elevation of the testis .

Regarding cryptorchidism(undesended testis) all are true except


a. refers to the interruption of the normal descent of the testis into
the scrotum.
b. The testicle may reside in the retroperitoneum, in the —
internal inguinal ring, in the inguinal canal, or even at the
.external ring
C. At birth, approximately 95% of infants have the testicles —
.normally positioned in the scrotum
D. it’s a common disorder and incidence increased up to 30 % —
. in premature
E. undescended testis is always regarded as an ectopic —
.testis

.:Regarding GI(gastrointestinal bleeding ) all are true except


a.Lower GI hemorrhage is defined as an abnormal intra luminal
.blood loss from a source distal to the Treitz ligament
b.The most cause of massive lower GI bleeding in adults are
.diverticulosis and angiodysplasia
c.Cancer colon is usually associated with massive lower GI
.hemorrhage
d.Hemorrhage from diverticular disease stops spontaneously —
.in 80% of patients
E. patients with massive upper GI bleeding may present with —
.maroon stools or bright red blood from the rectum
—
:Signs of severe bood loss include the following except —
a.Pallor —
b.Clammy skin —
c.bradycardia —
d.Tachycardia —
e.Hypotension —
: regarding stigmata of bleeding in peptic ulcer
. a.its associated with increase risks for rebleeding
.b.adherent clot is the most significant stigmata
c.arterial spurting hemorrhage associated with low risk of
. rebleeding
d.risk of rebleeding in ulcer with clean base is above 50%
e.nonbleeding visible vessel is not associated with risk of
. rebleeding

Features of inflammatory response syndrome (SIRS) include


the following except:
a. Temperature> 38.4C
b. Temperature <36.C
c. WCC<4.ooo cells per ml
d. Respiratory rate >20 per minute
e. PCO2> 32 mmHg

The development of thrombocytopenia and arterial thrombosis


with heparin requires:
a. Continuation of heparin and platelet transfusion
b. Continuation of heparin and thrombolysis
c. Doubling the heparin dosage
d. Changing the route of heparin administration
e. Discontinuation of heparin

Regarding Heparin-induced thrombocytopenia (HIT) all are


true except
a. Is a special case of drug-induced immune
thrombocytopenia.
b. The platelet count typically begins to fall 5 to 14
days after heparin has been started.
c. Thrombocytopenia is usually severe.
d. HIT should be suspected if the platelet count
falls to less than 100,000 or if it drops by 50%
from baseline in a patient receiving heparin
e. HIT is more common with full-dose
unfractionated heparin (1 to 3%)

Regarding gall bladder and bile secretion all are true except :
— a.The gallbladder is a pear-shaped, about 7 to 10 cm long
with an average capacity of 30 to 50ml.
— b.When obstructed, the gallbladder can distend markedly and
contain up to 300 mL
— c. Anomalies of the hepatic artery and the cystic artery are
quite common, occurring in as many as 50% of cases.
— d.liver produces 500 to 1000 mL of bile a day
— e.Vagal stimulation decreases secretion of bile
—
— Regarding gall bladder stones all are true except:
a. Prevalence increases with advancing age —
b. Over 10% of those with stones in the gallbladder —
.have stones in the common bile duct
c.10-20% become symptomatic
d.cholesterol stones are the most common type.
e.pigment stones are associated with secondary common
bile duct stones .

In acute cholecystitis all true except :


a.Most common organisms are E. coli
b. 90% cases result from obstruction to the cystic duct by a
stone.
C .patient present with constant pain usually greater than 6
hours duration in right upper quadrant .
d.presence of gall stones and percholecystic fluid on US is
diagnostic
e.Cholecystectomy is contraindicated in acute stage .

complication of acute cholecystitis include all the following


except:
a.Gangrenous cholecystitis
b.Gallbladder perforation
c.Cholecystoenteric fistula
d.mesnteric ischemia
e.Gallstone ileus
Regarding choledocholithisis all are true except:
a. may be silent and or may cause obstruction, complete or
incomplete
b. may manifest with cholangitis or gallstone pancreatitis.
c.present with severe jaundice and cholangitis in case of stone
impaction
d.Rt upper quadrant pain ,fever,and jaundice are called charcots
triad in cholangitis.
e.impaction of small stones has no relation with acute
pancreatitis .

Regarding acalculous cholecystistis all are true except:


a. Acute inflammation of the gallbladder can occur without
gallstones
b. Acalculous cholecystitis typically develops in critically ill
patients in the intensive care unit.
c. Patients on parenteral nutrition with extensive burns, sepsis,
major operations are at risk for developing acalculous
cholecystitis.
d.US is not a good diagnostic tool.
E.can be managed by cholecystectomy or percutanous
cholecyststomy .

Regardind gall bladder cancer all are true except:


a. Larger stones (>3 cm) are associated with a 10-fold increased
risk of cancer.
b. up to 95% of patients with carcinoma of the gallbladder have
gallstones.
c. Polypoid lesions of the gallbladder are not associated
with increased risk of cancer
d. Patients with choledochal cysts have an increased risk of
developing cancer
e. Sclerosing cholangitis is risk factor for developing gall
bladder cancer .

Regarding tumors of the liver all are true except :


a.Hemangioma is the most common solid benign lesion .
b.Spontaneous rupture in hemangioma (bleeding) is rare.
c.Hepatic adenomas carry a significant risk of spontaneous
rupture with intraperitoneal bleeding.
d. Hepatic adenomas have a risk of malignant transformation to
a well-differentiated HCC(hepatocellular carcinoma).
e.focal nodular hyperplasia lesions(FNH) lesions usually
rupture spontaneously and have significant risk of
malignant transformation .

Regarding pyogenic liver abscess all are true except :


a. arise as a result of biliary sepsis.
b.associated with high mortality.
c .appendicitis is unlikely the cause pyogenic liver abscess.
d.30% of patient have pleural effusion on presentation .
e.lab.investiation show elevated WBC and abnormal liver
function .
Regarding acute pancreatitis all are true except:
a.Gallstones less than 5mm diameter are more likely to cause
pancreatitis than larger ones
b.. The mortality associated with infected necrosis is about 40%
c.Cullen's sign is a sign of retroperitoneal hemorrhage in severe
hemorrhagic pancreatitis .
d.elevated serum amylase is a significant predictor of
severity .
e.. 50% of deaths occur within first week due to multi-organ
failure .

All occur as a complication of acute pancreatitis except:


a.panreatic fluid collection
b.colonic necrosis
c.coagulopathy
d.hypercalcemia
e.respiratory failure

Regarding head trauma all are true except:


a.basal skull fracture regarded when one of the orbital roof
,sphenoidal bone or petromastoid portion are involved .
b.epidural hematoma is an Lens shape hematoma between dura
and the skull.
c.subdural hematoma is crescent shaped hematoma ,between
brain and dura
d.secondary brain injury is preventable.
e.GCS glascow Coma scale 3/15 indicate uncomprehensive
sounds .

Regarding chronic lower limb ischemia all are true except:


a. Claudication distance is distance after which the pain is felt.
b. Rest pain is continous severe burning pain in the foot which
indicate critical ischemia.
c. trophic changes include tapering digits ( loss of S.C fat ) and
muscle wasting .
d. usuall presentations of patients with lower limb ischemia are
pain,trophic changes and gangrene.
e. venous filling time more than 2 minutes indicates mild
lower limb ischemia.

Regarding 4 weeks 4 Kg bodywt. old full term neonate


presented with rapidly progressive projectile non bilious
vomiting and palpable upper abdominal mass all are true
except :
a. dehydration and alkalosis are prominent features.
b.maintenance fluid therapy is about 4ml /Kg /hour.
c.administration of IV fluids with 5% dextrose, 0.5% normal
saline, and KCl usually corrects the alkalosis .
d.Estimated total blood volume is about 320 cc.
E. the most likely diagnosis is high jejunal atresia
All are true regarding jejunoileal atresia except:
a. present with bile stained vomiting .
b. Failure to pass meconium or small amounts of mucus or
meconium maybe passed per rectum.
c. present with abdominal distention.
d. X-ray show double bubble appearance .
e. The x-rays usually show multiple air-fluid levels.

regarding malrotation of the gut all are true except:


a.The patient might be asymptomatic and then develop the
symptoms when he is older.
b.Commonest abnormality results in caecum lying close to DJ
flexure.
c.Fibrous bands may be present between caecum and DJ flexure
(Ladd's bands).
d. the patient is unlikely to have clinical picture of duodenal
obstruction .
e. In malrotation midgut mesentery is abnormally narrow and
liable to volvulus.

26. The initial maneuver to establish an airway in a patient with multiple


injuries is:
a. Oropharyngeal airway.
b. Uncuffed endo-tracheal tube.
c. Suctioning foreign debris and lifting up the mandible.
d. Cuffed endo-tracheal tube.
e. Tracheostomy.

27. Which is the most commonly injured intra-abdominal organ in blunt


trauma?
a. Pancreas.
b. Kidney.
c. Spleen.
d. Stomach.
e. Colon.

28. The most important principle in the management of severe


hemorrhagic shock is to:
a) Obtain blood for possible type-specific transfusion.
b) Place CVP lines early for fluid resuscitation and monitoring.
c) Rapidly infuse colloid fluids.
d) Apply MAST garment.
e) Secure the airway and adequate ventilation.

29. A 6-year-old child sustained the following injuries in an accident.


Which one of the followings should be managed first?
a. Extradural hematoma.
b. Pneumothorax.
c. Hollow viscus injury.
d. Renal injury.
e. Liver laceration.

30. All the following are complications of massive blood transfusion


except:
a. Hypothermia.
b. Hypocalcaemia.
c. Hypokalaemia.
d. Acidosis.
e. DIC.
6.Severe limb pain of sudden onset can be caused by all the
following conditions except:
a. Acute ischaemia.
b. Deep venous thrombosis.
c. Muscle tear.
d. Sciatica.
e. Bone fracture.

?Which of the following is not a classic sign of a basal skull fracture .7


a. Battle sign
b. racoon eyes
c. hemotympanum
d. Gray –Turner sign
e. CSF rhinnorhea/ottorrhea
.e. thiamine

:Tension pneumothorax is best diagnosed with.8


A, stat CT scan
b. chest x-ray
c. watch and wait
d. clinical exam
.e. none of the above

:While doing thoracocentesis, it is advisable to introduce needle along .9


.a. Upper border of the rib
.b. lower border of the rib
.c. In the center of the inter-costal space
.d. In anterior part of inter-costal space
.e. any where

10. The most significant immediate complication associated with pelvic fracture is:
a. Hemorrhage.
b. Rectal or vaginal lacerations.
c. Sciatic nerve injury.
d. Infection.
e. Myositis ossificans.

11 - Which is not contributory to Glasgow Coma Scale?

a. obey commands
b. localizes painful stimuli
c. open eyes to calling
d. incomprehensible sounds
e. 5mm pupils

12. In role of nine extent of burn if entire trunk is burned it will be equal to:
a. 9% body surface area.
b. 18% body surface area.
c. 36% body surface area.
d. 27% body surface area.
e. 45% body surface area.

13. A 54-year-old man presents with two episodes of hematemesis since


yesterday. The most likely cause of this patient’s upper gastrointestinal
bleeding is:
a. Gastritis.
b. Esophagitis.
c. Esophageal varices.
d. Peptic ulcer disease.
e. Mallory-Weiss tear.

14. Adequate minimum urine output in a 70kg man during resuscitation is

a) 35 ml/hr
b) 20 ml/hr
c) 50 ml/hr
d) 45 ml /hr
e) 60 ml/hr

15. A 28 year old lady complains of painful defecation associated with fresh per-
rectal bleed. Possible diagnosis to consider:

a) hiradenitis suppurativa
b) dermoid cyst
c) pilonidal sinus
d) anal fissure
e) pruritis ani

16. Which is not a cause of pancreatitis?

a) hypercalcaemia
b) hypokalaemia
c) hyperlipidaemia
d) obstruction at ampulla of Vater
e) thiazide

17-All of the following can be treated conservatively in a stable trauma


patient except:
a. Lung contusion.
b. Liver laceration.
c. Kidney laceration.
d. Splenic hematoma.
e. Perforation of the small intestine.

18.What is the commonest presentation of a nephroblastoma?


a) Abdominal pain.
b) Haematuria.
c) Fever.
d) Abdominal mass.
e) Loss of weight.

19. Which of the following are not found in peritonitis?


f. Patient is lying still
g. Guarding
h. Rebound tenderness
i. Hyperactive bowel sounds
j. Rigid abdomen

The following is an indication for thoracotomy in chest .20


,injury

p. Cardiac tamponade
q. Uncontrolled pulmonary air leakage
r. Perforation of thoracic esophagus
s. Blood loss of 200ml/hr for 2-3 hrs via chest tube
t. All of the above

1. In the preoperative assessment,which of the following is not suggestive


of a platelet disorder?

(A) Epistaxis

(B) Gingival bleeding

(C) Hemarthrosis

(D) Easy bruising

(E) Petechia

2. Which of the following is a physical finding specific for vascular


structures?

(A) Reducibility

(B) Fluctuation

(C) Transillumination

(D) Compressibility
(E) Crepitancy

3. Which of the following medications must be stopped preoperatively:

(A) Atenolol

(B) Thyroxine

(C) Albuterol

(D) Aspirin

(E) Enalapril

4. A patient has undergone an ileal resection. Which of the following


conditions would be the least likely to develop?

(A) Alopecia

(B) Megaloblastic anemia

(C) Neurological symptoms

(D) Steatorrhea

(E) Cholelithiasis

5. Which of the following is the best method to assess the perioperative


pulmonary risk?

(A) Arterial blood gas

(B) Pulmonary function test

(C) CXR

(D) Detailed history & physical exam

(E) Diffusion capacity study

6. Which of the following is the least important data/test to obtain for the
preoperative risk assessment of coagulopathy?

(A) Bleeding time


(B) History& physical examination

(C) Prothrombin time

(D) liver function test

(E) Fibrinogen levels

7. The most consistent physical sign of hyperthyroidism is:

(A) Tachycardia

(B) Exophthalmus

(C) Excessive sweating

(D) Confusion

(E) Hyperperistalsis

8. Which of the following would be the most difficult to manage


preoperatively?

(A) Chronic renal failure

(B) Hemophilia

(C) COPD

(D) Heart failure

(E) Advanced cirrhosis

9. Which of the following is the least likely to develop in hospitalized


surgical patients?

(A) DVT

(B) Pneumonia

(C) Surgical wound infection

(D) Systemic candidiasis


(E) Urinary tract infection

10. When should parenteral antibiotics be given before mastectomy?

(A) the night before

(B) 6 h prior to surgery

(C) Not indicated in such a case

(D) at the time of incision

(E) 30 min after incision

11. A patient with a postoperative wound cellulitis should be treated with:

(A) Antibiotics and warm soaks to the wound

(B) Incision & drainage

(C) Antibiotics and opening the wound

(D) Antibiotics alone

(E) observation

12. Which of the following is the most likely cause of fever in the first 2
days postoperatively?

(A) Malignant hyperthermia

(B) wound abscess

(C) Urinary tract infection

(D) Pulmonary complications

(E) drug fever

13. Neurogenic shock is characterized by all of the following except:


(A) Warm skin

(B) Bradycardia

(C) Vasoconstriction

(D) Normal blood volume

(E) Good response to vasopressors

14. A 24-year-old man is brought into the emergency department after a fall
from 4 meters height. His breathing is distressed, and he is cyanotic. No
breath sounds can be heard in the right lung field, which is resonant to
percussion. Chest tube was inserted. The next step in his management
should be:

(A) Mechanical ventilation

(B) Surgical intervention

(C) IV fluid replacement

(D) Passing an oral endotracheal tube

(E) Obtaining an urgent chest X-ray

15. Which of the following is not a cause of cardiogenic/cardiac


compressive shock in a trauma patient?

(A) Air embolism

(B) Tension pneumothorax

(C) Flail chest

(D) Cardiac tamponade

(E) Myocardial contusion

16. The severity of hypovolemic shock has been found to correlate with the

(A) hematocrit

(B) pulmonary capillary wedge pressure (PCWP)

(C) The percentage of volume lost

(D) PaO2
(E) white blood cell count

17. A patient is transported to the ER following a fall from height . The Glasgow Com
Scale (GCS) can categorize the patient's neurologic status by assessing all of the
following except:

(A) Withdrawal to pain

(B) Inappropriate words

(C) Eye opening response

(D) pupil response to light

(E) Obeying commands

18. Neurogenic shock can be caused by all of the following except:

(A) Spinal anaesthesia

(B) Isolated head ingury

(C) Extremely severe pain

(D) Accidental back trauma

(E) Unpleasent scenes

19. Which of the following types of shock is characterized by a higher


incidence of delayed mortality?

(A) hypovolemic

(B) cardiogenic

(C) neurogenic

(D) cardiac compressive

(E) septic

20.What is the first priority in the management of trauma patients ?


(A) continued intravenous hydration

(B) Oxygen mask

(C) Chest tube

(D) Airway & C-spine control

(E) Blood transfusion

21. In physical examination if an inguinal hernia is completely irreducible it


is called

(A) Strangulated

(B) obstructed

(C) Recurrent

(D) Fixed

(E) Incarcerated

22. Which of the following is not a characteristic of uncomplicated inguinal


hernia

(A) Reducible

(B) Expansile

(C) May be congenital or acquired

(D) Pulsatile

(E) May be caused by chronic cuogh

23. The extension of the existing pain to a site other than the original one
is called:

(A) Radiation

(B) Shifting

(C) migration

(D) Exacerbation

(E) All of the above


24. All of the following symptoms and signs are suggestive of acute
pancreatitis EXCEPT

(A) Anorexia

(B) Severe epigastric pain

(C) Pain is colicky in nature

(D) Vomiting

(E) Ileus

25. A 42-year-old female comes to the ER complaining of right


upper quadrant abdominal pain for the last 36 h, associated with
fever up to 39°C, bilious vomiting, and jaundice. Direct bilirubin 2.2,
alkaline phosphatase 450, WBC 19,000.

What is the best cost- effective imaging study to define the


etiology?

(A) HIDA scan

(B) Ultrasound

(C) CT

(D) MRI

(E) Plain abdominal x-ray

26. The diagnosis of appendicitis depends mainly on :

(A) CBC

(B) Ultrasound

(C) Clinical assessment

(D) CT scan

(E) None of the above

27. The risk of which of the following infections is markedly increased after
splenectomy?
(A) Candidiasis

(B) Pneumococcal pneumonia

(C) Cytomegalovirus

(D) E.coli

(E) All of them

28. In breast examination dimpling of the overlying skin is most likely due
to:

(A) Breast abscess

(B) Mastitis

(C) Cancer

(D) Fibroadenoma

(E) None of the above

29. Melena may be caused by which of the following?

(A) Diverticular disease

(B) Hemorrhiods

(C) Peptic ulcer

(D) Rectal cancer

(E) Anal fissure

30. All of the following increase the risk of breast cancer except?

(A) Estrogen containing oral contraceptives

(B) Early menarche

(C) Prolonged and multiple episodes of lactation

(D) Late menopause


(E) The presence of family history of breast cancer

Nd 1

A 44-year-old man presents with painless rectal bleeding of 1 month's duration. He reports a history
of constipation. He works in heavy labor.
For this patient, which of the following statements regarding internal hemorrhoids is true?

Choose one answer


a. Stapled hemorrhoidectomy should be done for grade 1 and 2 hemorrhoids
b. All of the above
c. Internal hemorrhoids are located proximal to the dentate line and therefore are usually painless
d. A grade 1 internal hemorrhoid represents bleeding with prolapse

39. A 34-year-old woman presents for evaluation of severe and frequent bloody bowel movements, as
well as abdominal pain, dehydration, and anemia. She has had these symptoms for 2 days. She has
not had any similar symptoms in the past, and she has been in relatively good health.
If the patient has toxic megacolon, under what circumstances emergency surgical management is
indicated?

Choose one answer.


a. There is a perforation
b. Any of the above
c. The patient's clinical or radiographic status worsens
d. There is no improvement in 24 to 36 hours after aggressive medical therapy

Complications of untreated pancreatic pseudocysts include all of the following EXCEPT:

Choose one answer.


a. intracystic hemorrhage
b. abscess
c. pancreatic necrosis
d. free rupture
e. gastrointestinal obstruction

Platelets in the wound form a hemostatic clot and release clotting factors to produce:

Choose one answer.


a. thrombin
b. Fibrin
c. Fibrinogen
d. Fibroblasts
e. thromboplastin

A 43-year-old man presents to the office for evaluation of recent weight loss and frequent loose stools.
He is concerned because his father was diagnosed with colon cancer at the age of 50.

Besides family history, what are some other risk factors for colorectal cancer?
Choose one answer.
a. Hypertension
b. Diabetes
c. All of the above
d. Inflammatory bowel disease

A 40-year-old woman presents to the office for evaluation of yellowish skin. She states that over the
past few weeks, she has noticed that her eyes and skin have developed a yellow tint. She also reports
that she has dark urine and pale-colored stools. Further history elicits periodic bouts of right upper
quadrant pain after eating. She is otherwise healthy. She denies using any medications. On physical
examination, a yellowish tint is observed on the patient's skin, sclera, and mucous membranes.

On the basis of this patient's history and clinical examination, which type of bilirubin would you
expect to predominate?

Choose one answer.


a. Mixed
b. Conjugated
c. Unconjugated
d. Indirect

39. A 38-year-old man presents with a complaint of a slow-growing mass over his right parotid gland.
The lesion is fixed to the underlying structures and has recently become painful.
Which of the following features strongly suggests that this patient's lesion is a malignancy?

Choose one answer.


a. Ipsilateral numbness of the tongue
b. .All of the above
c. Overlying skin involvement
d. Facial nerve paralysis

Axillary lymph nodes are classified according to the relationship with the

Choose one answer.


a. pectoralis minor muscle
b. pectoralis major muscle
c. axillary vein
d. serratus anterior muscle
e. latissimus dorsi muscle

39. A 78-year-old man is recovering from abdomino-perineal (A-P) resection for Ca rectum, which
was performed 3 days ago. The patient is now complaining of mild shortness of breath and chest pain.
On physical examination, the patient's right leg is slightly more swollen than his left. The pulse
oximetry reading is 90%.
What is the principal method of diagnosing acute pulmonary embolism?

Choose one answer.


a. Magnetic resonance imaging
b. Chest x-ray
c. Ultrasound
d. Spiral computed tomography scanning

10

Compartment syndrome

Choose one answer.


a. Passive stretch decrease muscle pain
b. Due to decrease pressure in muscle compartments
c. Pulse is the first thing to disappear
d. Cause severe pain in the limb
e. Treatment is by delayed fasciotomy

11

The most significant risk factor for the development of adenocarcinoma of the esophagus is:

Choose one answer.


a. lye stricture
b. alcohol abuse
c. Barrett's esophagus
d. long-standing achalasia
e. smoking

12

All of the following statements are true about patients with carcinoid tumors EXCEPT:

Choose one answer.


a. the combination of streptozotocin and 5-fluorouracil (chemotherapy) can often result in objective
response.
b. tumor growth is often slow
c. they often have evidence of serotonin production
d. the majority have carcinoid syndrome
e. they have a much better prognosis if the tumors are less than 2 cm.

13

39. A 67-year-old man presents with left-lower-quadrant pain and low-grade fever. He has had these
symptoms for 1 day. The patient denies experiencing any rectal bleeding, but for the past week, his
bowel movements have been irregular.
For this patient, which of the following statements is true regarding diverticular disease?

Choose one answer.


a. The sigmoid colon is the most common site of diverticula
b. All of the above
c. Most diverticula of the colon involve the muscular layer
d. Smoking does not seem to be related to the development of diverticular disease

14

All of the following are components of the MEN type 2B syndrome except:

Choose one answer.


a. Multiple neuromas on the lips, tongue, and oral mucosa
b. Medullary thyroid carcinoma
c. Pheochromocytoma
d. Hyperparathyroidism.

15

The risk of bilateral breast cancer is HIGHEST if the first breast shows:

Choose one answer.


a. inflammatory carcinoma
b. medullary carcinoma
c. infiltrating ductal carcinoma
d. paget’s disease
e. lobular carcinoma

16

The best initial therapy for deep venous thrombosis of the common femoral vein is:

Choose one answer.


a. warfarin
b. streptokinase
c. Heparin
d. venous thrombectomy
e. placement of a vena caval filter

17

Complications after thyroidectomy include all the following EXCEPT:

Choose one answer.


a. recurrent laryngeal nerve paralysis
b. parathyroid insufficiency
c. thyrotoxic crisis(storm) on operating on inadequitly prepared thyrotoxic patient
d. tracheomalacia
e. hypercalcemia

18

A 55-year-old man presents with hematemesis that began 2 hours ago. He is hypotensive and has
altered mental status. No medical history is available.
For this patient, which of the following statements regarding nasogastric aspiration is true?

Choose one answer.


a. None of the above
b. A clear, nonbilious aspirate rules out the need for EGD
c. A clear, bilious aspirate rules out the need for EGD
d. A bloody aspirate is an indication for esophagogastroduodenoscopy (EGD)

19

The major cause of impaired wound healing is:

Choose one answer.


a. steroid use
b. malnutrition
c. diabetes mellitus
d. local tissue infection
e. anemia

20

Common presenting conditions in patients with pancreatic carcinoma include all of the following
EXCEPT:

Choose one answer.


a. esophageal varices.
b. palpable gallbladder
c. weight loss.
d. abdominal pain

21

All of the following statements about keloids are true EXCEPT:

Choose one answer.


a. Keloid tissue contains an abnormally large amount of collagen
b. A keloid does not regress spontaneously
c. Keloid tissue contains an unusually large amount of soluble collagen
d. A keloid extends beyond the boundaries of the original wound
e. Keloids or hypertrophic scars are best managed by excision and careful reapproximation of the
wound

22

The treatment of choice for a 40-year-old man who is found on endoscopy and biopsy to have a
gastric lymphoma would be:

Choose one answer.


a. wide local excision
b. subtotal gastrectomy
c. chemotherapy
d. subtotal gastrectomy and radiotherapy
e. Radiotherapy

23

The treatment of an esophageal burn with a caustic agent may include all of the following EXCEPT:

Choose one answer.


a. boogieing. (dilatation)
b. induction of vomiting
c. gastrectomy
d. expeditious administration of an antidote
e. steroids and antibiotics.

24

A 32-year-old man with a family history of familial adenomatous polyposis (FAP) presents with
hematochezia. He denies having any diarrhea, abdominal pain, or fever.

For this patient, which of the following statements regarding FAP is true?

Choose one answer.


a. CRC does not occur in patients with FAP if they are given adequate medical treatment
b. Total proctocolectomy (TPC) is considered the only option for the surgical management of FAP
c. For patients with FAP, there is a 10% risk of CRC by age 40 if prophylactic colectomy is not performed
d. In the setting of FAP, colorectal cancer (CRC) is more commonly located on the left side

25

When stage I breast cancer is treated by partial mastectomy and axillary dissection, further therapy
should include:

Choose one answer


a. antiestrogen agents.
b. radiation of the affected breast.
c. oophorectomy if premenopausal.
d. nothing
e. chemotherapy

26

For the patient in Question 68, which of the following statements is true regarding an esophageal
varix as the site of bleeding?

Choose one answer.


a. I.V. propranolol should be administered first
b. Balloon tamponade should be performed first
c. Rubber banding or intravariceal sclerotherapy should be performed first
d. I.V. somatostatin should be administered first

27

Drugs which may produce gynecomastia include all of the following EXCEPT:

Choose one answer.


a. furosemide
b. cimetidine
c. Verapamil
d. Diazepam
e. Tamoxifen

28

Splenectomy is commonly indicated for the following EXCEPT:

Choose one answer.


a. hypersplenism associated with cirrhosis
b. hereditary spherocytosis
c. splenic tumor
d. immune thrombocytopenic purpura
e. grade four splenic injury in trauma

29

An ischiorectal abscess is characterized by all of the following EXCEPT:

Choose one answer.


a. Requires deroofing
b. Should be treated entirely by antibiotics
c. May be tuberculous in origin
d. Can be followed by anal fistula
e. Is an infective necrosis of the fat of the ischiorectal fossa

30
Marks: 1

The most common presentation of Meckel’s diverticulum in an adult is:

Choose one answer.


a. intussuception
b. Littre’s hernia
c. Gastrointestinal bleeding
d. diverticulitis

31

For the patient in Question 65, which of the following is an indication for immediate surgery?

Choose one answer.


a. Closed-loop obstruction
b. All of the above
c. Complete bowel obstruction
d. Incarcerated hernia

32

Choledocholithiasis in a patient who previously had cholecystectomy is BEST treated with:

Choose one answer.


a. endoscopic sphincterotomy
b. choledochoduodenostomy
c. choledochojejunostomy.
d. dissolution with mono-octanoin
e. open common bile duct exploration with stone removal

33

39. A 77-year-old man undergoes endoscopic ultrasonography as part of a workup for jaundice. He is
found to have a tumor in the head of the pancreas.
For this patient, which of the following findings would indicate that the tumor is unresectable?

Choose one answer.


a. All of the above
b. Peritoneal metastases
c. Invasion of the superior mesenteric artery
d. Metastases to celiac lymph nodes

34

Regarding polyps of the colon

Choose one answer.


a. Villous polyps are usually pedunculated
b. Villous polyps occur more proximal in colon
c. Adenomatous polyps are usually solitary
d. Cancer risk is not related to size of polyp
e. Metaplastic polyps are not precancerous

35

The most likely diagnosis in elderly patient with abdominal pain and colonoscopy finding of patchy
mucosal ulceration at the splenic flexure of the colon is :

Choose one answer.


a. ulcerative colitis
b. crohns disease
c. ischemic colitis
d. diverticulitis
e. lymphogranuloma venerum

36

39. A 54-year-old man presents with a neck mass of 2 weeks' duration. He has no significant medical
history. He smokes two packs of cigarettes a day and has been doing so since he was 21 years of age.
For this patient, which of the following statements is true?

Choose one answer.


a. Low cervical nodes are more likely to contain metastases from a primary source other than the
head and neck, whereas upper cervical nodes are more likely to contain metastases from the head
and neck
b. Soft or tender nodes are more likely to derive from an inflammatory or infectious condition,
whereas hard, fixed, painless nodes are more likely to represent metastatic cancer
c. All of the above
d. Enlarged lymph nodes are by far the most common neck masses encountered

37

The first-choice diagnostic study for suspected deep venous thrombosis of the lower extremity is:

Choose one answer.


a. real-time Doppler imaging
b. contrast sonography
c. radioactive labeled fibrinogen uptake
d. impedance plethysmography
e. isotope injection with gamma scintillation scanning

38

Decreased PaCO2 levels should be attained in a patient at serious risk for cerebral edema secondary
to a head injury in order to :

Choose one answer.


a. prevent neurogenic pulmonary edema
b. prevent increased capillary permeability
c. prevent metabolic acidosis
d. allow reciprocally high levels of PaO2 in the brain
e. prevent cerebral vasodilation

39

Causes of third space loss include all except:

Choose one answer.


a. Acute pancreatitis
b. Pancreatic fistula
c. Necrotizing fasciitis
d. Site of major surgery
e. Crush syndrome

40

Causes of metabolic acidosis include all except:

Choose one answer.


a. Small bowel fistula
b. Shock
c. CO poisoning
d. severe anemia
e. All of the above

41

A 39-year-old man comes in for evaluation of intermittent anal pain and bleeding after bowel
movements. He also has hard stools. He has had these symptoms for over 1 year .
For this patient, which of the following is included in the classic triad of signs of chronic anal
fissures?

Choose one answer.


a. An anal fissure
b. All of the above
c. A sentinel skin tag
d. Hypertrophy of the anal papilla

42

All the following statements about achalasia are true EXCEPT:

Choose one answer.


a. it occurs most commonly in persons between the ages of 30 and 50 years
b. in most affected persons, ganglion cells in the body of the esophagus either are absent or have
degenerated
c. esophageal cancer is seven times as common in affected persons as in the general population
d. pressure in the body of the esophagus is lower than normal
e. affected persons usually experience more difficulty swallowing cold foods than warm foods ??

43

A patient with the Zollinger-Ellison syndrome is found to have the multiple endocrine neoplasia type
I (MEN-I) syndrome. Appropriate management for the ulcer symptoms should be:

Choose one answer.


a. Omeprazole
b. pancreatic resection
c. streptozocin
d. cimetidine
e. total gastrectomy

44

Fat absorption occurs primarily in the:

Choose one answer.


a. Ileum
b. third portion of the duodenum
c. Stomach
d. Jejunum
e. first portion of the duodenum

45

39. A 56-year-old woman has been experiencing abdominal pain for 4 hours. The pain is in right
upper quadrant and radiates into the scapular region. She has had multiple episodes of vomiting.
For this patient, which of the following signs on physical examination is associated with acute
cholecystitis?

Choose one answer.


a. Carnett sign
b. Kehr sign
c. Murphy sign
d. Rovsing sign

46

Mammary duct ectasia is characterized by the following EXCEPT :

Choose one answer.


a. Is treated usually by simple mastectomy
b. May present with nipple retraction and Peau d'orange picture
c. Anaerobic superinfection cmmmonly occurs in this recurrent periductal plasma cell mastitis
d. Is defined as primary dilatation of major ducts of breast in middle aged women
e. Iscommonly pre-malignant

47

Regarding veins of lower limbs all are true except:

Choose one answer.


a. Valves allow flow from deep to superficial system Venous return from lower limbs is aided by
respiratory movements
b. Superficial veins lie in subcutaneous tissue
c. The pressure in veins of the foot while standing is 100mmHg
d. a) Stasis of blood is important factor in developing varicose veins

48

Most common complication of central venous access is:

Choose one answer.


a. major artery damage.
b. Catheter problems.
c. Thrombosis of central vein
d. Catheter related sepsis
e. Pleural space damage, pneumothorax
49

Factors associated with increase risk of death in acute pancreatitis include all except:

Choose one answer.


a. Ranson score more than five
b. Obesity?
c. High APACHE_II score
d. Age more than 70 years
e. sterile necrosis

50

For the patient in Question 56, which of the following chronic conditions can cause a neck mass?

Choose one answer.


a. Tuberculosis
b. Sarcoidosis
c. AIDS
d. All of the above

51

For the patient in Question 56, if metastatic cancer is suspected initially, which of the following would
be the most appropriate step to take next in the workup?

Choose one answer.


a. CT scan
b. Empirical therapy with antibiotics
c. Fine-needle aspiration (FNA)
d. Observation only

52

The largest component of intestinal gas is:

Choose one answer.


a. Nitrogen
b. carbon dioxide
c. Oxygen
d. ammonia
e. hydrogen

53

A 66-year-old woman presents to the office complaining of a sharp, constant pain in her lower
abdomen. She has had this pain for the past 2 weeks. Examination of her abdomen is normal.
However, subsequent ultrasound reveals a 4.5 cm Aortic abdominal aneurysm (AAA). Spiral computed
tomography confirms the ultrasound findings.

Aside from rupture, which of the following is a complication associated with (AAA) ?

Choose one answer.


a. Aortoenteric fistula
b. Lower-extremity atheroemboli
c. All of the above
d. Thrombosis

54

Diagnosis of esophageal perforation is best established by:

Choose one answer.


a. transesophageal ultrasound study
b. esophagoscopy with a flexible esophagoscope
c. contrast esophagograms
d. esophagoscopy with a rigid esophagoscope
e. upright X-rays of the chest including lateral and oblique films

55

For the patient in Question 68, which of the following is an indication for surgery?

Choose one answer.


a. Ongoing hemorrhage occurs from a gastric ulcer in a hemodynamically unstable patient
b. All of the above
c. Bleeding continues from either a duodenal ulcer or a gastric ulcer despite medical and endoscopic
therapy
d. Substantial bleeding occurs from a duodenal ulcer that is not controlled by EGD

56

Yesterday, a 38-year-old woman underwent a laparoscopic cholecystectomy for cholelithiasis and was
discharged home 8 hours after surgery. She returns this morning complaining of worsening
abdominal pain. The oral narcotics that the patient was prescribed are ineffective in controlling the
pain. The patient's temperature is (38.3 C). Laboratory studies reveal an elevated white blood cell
count. Abdominal ultrasonography shows a large subhepatic fluid collection. The fluid is
percutaneously aspirated and reveals enteric contents.

What step should be taken next in the management of this patient?

Choose one answer.


a. Immediate laparotomy
b. I.V. antibiotics and close observation
c. None of the above
d. Observation in the hospital until pain improves

57

The most common symptom after major pulmonary embolism is:

Choose one answer.


a. cough
b. Dyspnea?
c. Hemoptysis
d. pleural pain
e. fear of death

58

Treatment of paralytic ileus includes all of the following


EXCEPT :

Choose one answer.


a. intravenous fluids
b. cessation of oral intake
c. correction of electrolyte imbalance
d. early operation
e. nasogastric suction

59
Lymphedema :
Choose one answer
a. e) None of the above
b. b) should be bilateral
c. c) may be pitting in early stage
d. d) A & C only
e. a) may be Congenital

60

The level of consciousness for a head injury patient is BEST evaluated by :

Choose one answer.


a. CT scan
b. visual evoked potentials
c. Glasgow coma scale
d. papillary responses
e. response to pain

61

A 52-year-old male weighing 70 kg, sustained a 65% total body surface area (TBSA) burn. What are
his fluid requirements?

Choose one answer.


a. 18,200 cc in 16 h
b. 9100 cc in the first 8 h
c. 8000 cc in 24 h
d. 12,800 cc in 24 h

62

Management of cholangitis may include all of the following EXCEPT:

Choose one answer.


a. decomperession of the common bile duct.
b. cholecystostomy
c. IV antibiotics.
d. correct underlying cause.
e. percutaneous transhepatic cholangiography.

63

A 48-year-old woman presents to the emergency department complaining of right upper quadrant
pain, which began 4 hours ago. She reports the pain as being spasmodic and sharp and that it
radiates to her right shoulder blade. She says that she has had similar episodes over the past few
months, especially after eating large meals. Associated with the pain is nausea and vomiting. Her
blood pressure is 120/85 mm Hg, and her pulse is 100 beats/min. On physical examination, the patient
is found to have a nontender abdomen with no palpable masses. Her chest and cardiovascular
examinations are normal. The nurse notices that her sclerae are slightly icteric. On subsequent
laboratory studies, her serum bilirubin level is found to be 10 mg/dl.

What imaging study should be performed next for this patient with presumed posthepatic jaundice?
Choose one answer.
a. Percutaneous transhepatic cholangiography (PTC)
b. Ultrasonography
c. Endoscopic retrograde cholangiopancreatography (ERCP)
d. Magnetic resonance imaging

64

39. An HIV-positive man presents for evaluation of new oral cavity lesions he discovered last month.
Physical examination reveals purple exophytic masses involving the palate mucosa and gingiva.
What is the most likely diagnosis of these oral cavity lesions?

Choose one answer.


a. Oral hairy leukoplakia
b. Non-Hodgkin lymphoma
c. Syphilis
d. Kaposi sarcoma

65

Brain injury alone

Choose one answer.


a. causes shock only if the skull is intact
b. rarely causes shock
c. causes shock that is reversed by very simple measures
d. frequently causes shock
e. causes shock if hypoxia is superimposed

66

All of the following substances are irritating to the peritoneum EXCEPT:

Choose one answer.


a. bile.
b. blood
c. gastric content.
d. meconium
e. pus

67

Regarding volvulus of the sigmoid colon, each of the following is true except :

Choose one answer.


a. likely results from redundant sigmoid colon with an elongated narrow mesocolon
b. there appears to be a congenital predisposition sigmoid volvulus.
c. diagnostic barium enema for sigmoid colon is essential
d. diagnostic x-ray for sigmoid volvulus shows a dilated loop of colon which points toward the right upper
quadrant.

68

Gallstones are characterized by all the following EXCEPT:

Choose one answer.


a. cause mucocoele of the gall bladder
b. are present in the common bile duct in 40% of patients with stones in the gall bladder
c. are becoming common in post-partum primipara who were pre pregnancy ‘Pill’ takers
d. are frequently the cause of flatulent dyspepsia
e. may be present in the newborn

69

Following a burn, the agent responsible for early increased capillary permeability is

Choose one answer.


a. thromboxane A2
b. serotonin
c. histamine?
d. prostacyclin PGI2
e. bradykinin

70

Incisonal hernias all are true except:

Choose one answer.


a. 10 % of all hernias
b. Usually easy to reduce
c. Multiloculated sac
d. Operative technique is most important cause
e. Complication is common

71

In the treatment of gastric cancer, all of the following are true EXCEPT:

Choose one answer.


a. five-year survival rates in the continue to be between 10% and 25%.
b. Palliative resection is frequently helpful with advanced disease.
c. Lymph node involvement is associated with a poorer prognosis.
d. Finding early disease at the time of operation is associated with a better prognosis.
e. total gastrectomy is mandated in most patient. ***

72

of the involved area and : Severe cases of hidradenitis suppurativa in the groin area are best managed
by excision

Choose one answer.


a. transfer of a rectus abdominus muscle flap
b. split thickness skin grafting
c. primary closure
d. delayed primary closure
e. closure by secondary intention

73

Patients at increased risk for gastric carcinoma include all the following EXCEPT:

Choose one answer.


a. those who have undergone gastric bypass for morbid obesity
b. those who have undergone gastric resection for duodenal ulcer
c. those with pernicious anemia
d. those with a high consumption of smoked fish
e. those with blood group A

74

A 56-year-old woman presents with symptoms of abdominal pain, weight loss, and rectal bleeding.
She is anemic and hypotensive, but she is stable.
For this patient, which of the following should be done first if the gastric lavage yields copious
amounts of bile?

Choose one answer.


a. Arteriography
b. Emergency laparotomy
c. Colonoscopy
d. Esophagogastroduodenoscopy

75

A 39-year-old man presents with lower GI bleeding. He has no abdominal discomfort and has
experienced no loss of weight.
For this patient, which of the following statements regarding the etiology of lower GI bleeding is
true?

Choose one answer.


a. Lower GI bleeding from diverticulosis often requires surgery
b. Arteriovenous malformations (AVMs) are the most common cause of lower GI bleeding
c. 6% to 10% of patients with ulcerative colitis have lower GI bleeding severe enough to necessitate
emergency surgical resection
d. None of the above

76

For the patient in Question 65, which of the following statements regarding abdominal radiographic
findings is true?

Choose one answer.


a. In gastric outlet obstruction, no gastric air will be seen, but large amounts of air will be seen in the small
bowel and colon
b. Mechanical small bowel obstruction usually shows no air-fluid levels, nor will distended bowel loops of
similar sizes be seen
c. All of the above
d. High-grade obstruction of the colon in association with an incompetent ileocecal valve may mimic small
bowel obstruction on x-ray

77

A 41-year-old female presents to the emergency department after sustaining a gunshot wound to the
abdomen, with injuries to the liver and large bowel. Despite successful resuscitation and operative
intervention, the patient dies 2 weeks later of multisystem organ failure in the intensive care unit.
Which organ most likely first experienced dysfunction?

Choose one answer.


a. lung
b. heart
c. liver
d. gastrointestinal tract
e. kidney

78

39. A 35-year-old man presents with severe acute abdominal pain of sudden onset. The pain initially
began in the upper abdomen and has now settled in the whole abdomen. On examination, the
abdomen was rigid. Chest X-Ray showed free gas under diaphragm.
Which of the following is the most appropriate course of management for this patient?

Choose one answer.


a. Observation
b. Urgent laparotomy
c. Nonurgent laparotomy
d. Laparoscopy

79

All are true about the dumping syndrome except:

Choose one answer.


a. Symptoms can be controlled with a somatostatin analogue.
b. Early postoperative dumping after vagotomy often resolves spontaneously.
c. Flushing and tachycardia are common features of the syndrome.
d. Diarrhea is always part of the dumping syndrome.
e. Separating solids and liquids in the patient's oral intake alleviates some of the symptoms of the syndrome.

80

An anal fissure is characterized by the following EXCEPT:

Choose one answer.


a. Is more common in women.
b. Is an ulcer of the anal mucosa.
c. Can be treated coservatively.
d. Usually lies anteriorly.

81

All of the following statements are true of esophageal carcinoma


EXCEPT

Choose one answer.


a. squamous cell tumor is the most frequent histology.
b. operation is frequently curative.
c. patient often require a pyloroplasty with operation.
d. patients usually die within one year of diagnosis
e. patient can have intestinal continuity reestablished using the stomach after esophageal resection.

82

A serum tumor marker correlated with recurrence after management of colon cancer is:

Choose one answer.


a. carcinoembryonic antigen (CEA)
b. Calcitonin
c. 5-hydroxyindoleacetic acid
d. CA 15-3
e. Alpha-fetoprotein
83

Conditions associated with gastric cancer include all of the following EXCEPT:

Choose one answer.


a. adenomatous polyps.
b. a high intake of dietary nitrates.
c. pernicious anemia.
d. higher socioeconomic groups.
e. chronic atrophic gastritis.

84

Which of the following is required for addressing any pressure sore?

Choose one answer.


a. pressure reduction
b. hyperbaric oxygen
c. antibiotics
d. skin flap coverage

Treatment of pressure sores requires relief of pressure with special cushions and beds and
nutritional support to promote healing.
85

The most effective treatment of achalasia is:

Choose one answer.


a. antireflux surgical procedures
b. Esophagomyotomy
c. dilation of the lower esophageal sphincter
d. antispasmodic medication
e. resection of the cardioesophageal junction

86

Regarding desmoid tumors all are true except:

Choose one answer.


a. Treatment is wide local excision with safety margins
b. Has high rate of local recurrence
c. Is a low grade fibro sarcoma
d. More common in females
e. Is radio resistant

87

Which of the following variables best predicts prognosis for patients with a recent diagnosis of
cutaneous melanoma and no clinical evidence of metastatic disease?

Choose one answer.


a. Ulceration.
b. Clark's level.
c. Gender
d. Bleeding
e. Breslow thickness.

88
In the treatment of acute cholecystitis, most patient are BEST served with:
Choose one answer.
a. endoscopic sphincterotomy.
b. cholecystostomy.
c. percutaneous drainage of the gallbladder.
d. early cholecystectomy (within 3 days of onset of symptoms).
e. IV antibiotics and cholecystectomy in 6 to 8 weeks.

89

Regarding crohn's disease all are true except

Choose one answer.


a. Cause non caseating granuloma
b. commonly affect small bowel
c. Causes sub mucosal fibrosis
d. It is full thickness inflammation
e. Cause depletion of goblet cell mucin

90

Advantages of laparoscopic versus open cholecystectomy include all of the following EXCEPT:

Choose one answer.


a. decreased pain.
b. less risk of bile duct injury.
c. reduced hospitalization.
d. improved cosmetic.
e. reduced ileus.

91

The most common cause of massive hemorrhage in the lower gastrointestinal tract is:

Choose one answer.


a. carcinoma
b. Diverticulosis
c. Diverticulitis
d. Polyp
e. ulcerative colitis

92

Carcinoma of the gallbladder is

Choose one answer.


a. rarely associated with jaundice.
b. associated with a good prognosis.
c. usually not diagnosed preoperatively.
d. most commonly metastatic to the lung.
e. best treated with radiation and chemotherapy

93

All the following statements concerning nipple discharges are true EXCEPT:
Choose one answer.
a. benign duct papillomas are the most common cause of bloody discharges.
b. when bloody , the discharge is due to a malignancy 70% of the time
c. a) excision of involved duct may be necessary to determine the etiology
d. they may be caused by multiple lesions.
e. a milky discharge may be due to a pituitary adenoma .

94

39. A 28-year-old woman presents with a complaint of a growing, painless mass in her neck. Physical
examination reveals a firm, fixed nodule measuring 2 cm on the right lobe of her thyroid. The
surgeon recommends fine-needle aspiration (FNA) of the lesion instead of excisional biopsy.
What are the advantages of FNA over excisional biopsy?

Choose one answer.


a. FNA is associated with a decreased risk of tumor seeding
b. FNA requires only an office visit
c. Anesthesia is not necessary
d. All of the above

95

For the patient in Question 74, which of the following is a sign of a colovesical fistula associated with
diverticulitis, as seen on computed tomography with contrast?

Choose one answer.


a. Thickening of the bladder and the colon
b. Sigmoid diverticula
c. All of the above
d. Air in the bladder

96

All of the following are true statements concerning paget ’ s disease of the nipple EXCEPT:

Choose one answer.


a. it is an eczematoid lesion.
b. it is very uncommon, accounting for only 2% of all breast cancers.
c. it is an in situ squamous cell malignancy of the nipple.
d. it can be confused with malignant melanoma histologically
e. it has a better prognosis than the majority of other breast cancers.

97

The classical picture of Acute arterial embolism include all the following except:

Choose one answer.


a. peripheral pulses
b. Parasthesia
c. All the above
d. Pain
e. Pallor

98

Extra colonic manifestation of inflammatory bowel disease include all except:

Choose one answer.


a. Erythema nodosum
b. Mixed connective tissue disease
c. Primary Sclerosing cholangitis
d. Polyarthopathy
e. Episcleritis

99

A 45-year-old woman presents with abdominal pain and vomiting of 1 day's duration. The patient
underwent an exploratory laparotomy after a motor vehicle accident 8 years ago.
Which of the following statements is true for this patient?

Choose one answer.


a. Pain from mechanical obstruction is usually more severe than pain from ileus
b. The severity of pain from mechanical obstruction may decrease over time
c. Pain from mechanical obstruction is usually localized in the middle of the abdomen, whereas pain from
ileus or pseudo-obstruction is diffuse
d. All of the above

100

A 4-year-old girl has been experiencing dyspnea on exertion. Chest x-ray shows a Morgagni hernia.
For this patient, which of the following statements is true?

Choose one answer.


a. Morgagni hernias can be repaired with a subcostal, a paramedian, or a midline incision
b. Morgagni hernias are most commonly seen on the right side
c. All of the above
d. The average age at diagnosis is typically greater for patients with Morgagni hernia than for patients with
Bochdalek hernia

Abduction of the vocal cords results from contraction of the:


K) crico-thyroid muscles
L) posterior crico-arytenoid muscles
M) vocalis muscles
N) thyro-ary-epiglottic muscles
O) lateral crico-arytenoid and transverse arytenoids muscles

17) The rectum:


A) is devoid of peritoneum
B) is surrounded by peritoneum
C) has peritoneum on its lateral surfaces for its upper two-
thirds, and on its anterior surface for its upper one-third
D) has pritoneum on its anterior surface for its upper two-thirds,
and on its lateral surfaces for its upper one-third
E) has peritoneum on its anterior surface only

18) The umbilicus:


A) lies near the to the xiphoid than to the pubis
B) derives its cutaneous innervation from the eleventh thoracic
nerve
C) transmits, during development, the umbilical cord two arteries
and two veins
D) usually lies at about the level between the third and fourth
lumbar vertebra
E) emberiologicall, may transmit urine but never bowel contents

19) The superficial perineal pouch:


A) is limited inferiorly by the urogenital diaphragm
B) is not continuous with the space in the scrotum occupied by
the testes
C) has a membranous covering which provides a fascial sheath
around the penis
D) is traversedby the urethera in the male but not the urethera
and vagina in the female
E) in the female, the greater vestibular glands are situated
outside this pouch
20) The tongue:
A) has a foramen caecum at the base of the frenulum
B) is separated from the epiglottis by the valleculae on each side
of the midline
C) has 7-12 circumvallate papillae situated behind the sulcus
terminalis
D) is attached to the hyoid bone by the genioglossus muscle
E) is supplied only by hypoglossal nerve
21) Hypovolaemic shock is characterized by:
A) a low central venous pressure , low cardiac output , low
peripheral resistance
B) a high central venous pressure , high cardiac output , low
peripheral resistance
C) a low central venoys pressure , low cardiac output , high
periphera resistance
D) a low central venous pressure , high cardiac output , high
peripheral resistance
E) a high central venous pressure , low cardiac output , low
peripheral resistance

22) An oxygen debt is:


A) the amount of oxygen in excess of the resting metabolic needs
that
must be consumed after completion of exercise
B) build up because the pulmonary capillaries limit the uptake
of
Oxygen at high rates of oxygen consumption
C) related to the fact that skeletal muscle cannot function temporarily in the
absence of oxygen

F) associated with a decrease in blood lactate


G) associated with alkalosis

23) Pulmonary embolism may be a complication of:


A) prolonged bed rest
B) a surgical operation
C) vitamine K deficiency
D) oral contraceptive therapy
E) Antithrombin III deficiency
24) Which of the following statements regarding potassium
metabolism is NOT
True?:
A) potassium deficiency commonly results from thiazide diuretic
theraoy
B) the normal compensation for potassium deficiency is a
metabolic extracellular acidosis
C) aldosterone increases urinary potassium loss
D) hyperkalaemia causes bradycardia and loss of P waves on the
ECG
E) hypokalaemia aggrevates the cardiac effects of digitalis
toxicity

25) Cutaneous pain:


A) is due to overstimulation of receptors serving other sensory
modalities
B) cannot be elicited more readily if the tissue has recently been
injured
C) is due to exitation of receptors by pain-producing chemical
substances in the injured tissue
D) shows marked adaptation, i. e. decrease in severity in
response to a constant stimulus
E) is conducted through the medial spinothalamic tract

26) Which of the following is NOT associated with hyperthyroidism?:


A) increase size of the thyroid gland
B) increased amount of colloid in thyroid follicle
C) increased height of epithelium of the thyroid follicle
D) increased vascularity of the thyroid gland
E) increased uptake of iodine by the thyroid gland

27) Sarcomata may show all of the following EXCEPT:


A) production of myxomatous tissue
B) production of collagen
C) spindle shaped cells
D) signet ring cells
E) blood stream metastasis

28) Anaphylaxis is characterized by all of the following EXCEPT:


A) is a reaction either local or general , frequently occurs within
five minutes
B) causes an urticarial eruption
C) is produced by IgA antibody
D) causes eosinophilia
E) causes degranulation of basophils and mast cells

29) Autoimmunity is characterized by the following EXCEPT:


A) occurs because of a breakdown in the ability of the body to
distinguish between self and non self
B) is involved in some forms of orchitis
C) is involved in formation of cryo globulin
D) is important in the pathogenesis of lupus erytheromatosus
E) does not result in immune complex disease

30) Pseudomembranous enterocolitis is caused by the following


organisms:
A) Clostridium sporogenes
B) Clostridium defficile
C) Streptococcus faecalis
D) Penicillin sensitive staphylocci
E) Pseudomonas aeruginos

B) CLINICAL SUGERY

31) The “ white clot syndrome”:


A) is usually characterized with antithrombin III deficiency
B) most often present with arterial complicatios of heparin
induced throbocytopenin
C) is best managed by loe molecular weight dextran
D) is best managed by halving the therapeutic dose of heparin
sodium
E) results from nitric oxide deficiency of endothelial cells

17) A 21-year-old man who was the driver in a head-on collision has
a pulse of 140/min , respiratory rate of 36 and blood pressure of 75
palpable. His trachea is deviated to the left, with palpable subcutaneous
emphysema and poor breath sounds in the right hemithorax, The most
appropriate initial treatment must be
A) immediate thoracotomy
B) catheter insertion in the subclavian vein for fluid
resuscitation
C) intubation and ventilation
D) tube thoracostomy
E) immediate tracheostomy

18) The best test to monitor the adequacy of levothyroxin


therapy is:
F) radioactive iodine uptake
G) thyroglobulin
H) free thyroxine index (T4)
I) triiodothyronine resin uptake (T3)
J) thyroid stimulating hormone (TSH)

24) Which of the following statements about fungal infection is NOT


true ?:
A) Prior or synchronous culture positive for Candida at
another site occurs in few patients with candidimia
B) For critically ill patients nonhaematogenous sites of
candida are appropriately treated with systemic
antifungal therapy
C) Mortality rates are similar regardlss of whether C.
albicans fungmia is treated with amphotericin B or
fluconazole
D) Intravenous catheters and the gastrointestinal tract are
common portals for Candida to gain blood stream access
E) Septic emboli are more common with fungal endocarditis
than with bacterial endocarditis

25) The maximum safe dose of local anaesthetic administered


subcutaneously in a 70-kg man is:
A) 10 to 20 ml of 1% lidocaine
B) 40 to 50ml oh 2% lidocaine with epinephrine
C) 40 to 50 ml of 1% lidcaine with epinephrine
D) 40 to 50 ml of 1% bupivacaine (marcaine)
E) 40 to 50 ml of 1%lidocaine without epinephrine

26) Two days after right hemicolectomy for a Dukes B caecal


carcinoma , the
Patient complains of sharp right-sided chest pain and dyspnea. HisPaO2
Is 64mmHg ,his PaCo2 is 32mmHg. CVP is 26 cm water, and the blood
pressure is 102/78mmHg. A pulmonary embolus is suspected,
The next step in management should be:
F) A ventilation- perfusion lung scan
G) A pulmonary arteriogram
H) Postrioanterior and lateral chest x-rays
I) Heparin sodium ,100 units/kg intravenously
J) Immediate duplex scanning of both lower extremities

27) The major cause of graft loss in heart and kidney allograft is:
A) acute rejection
B) hyperacute rejection
C) vascular thrombosis
D) chronic rejection
E) graft infection
28) All of the following are indicators of tumor aggressiveness and
poor outcome for papillary carcinoma of the thyroid gland
EXCEPT:
A) age over 50 years
B) microscopic lymph node metstasis
C) tumor larger than 4 cm
D) poorly differentiated histological grade
E) invasion through capsule to adjacent tissues

24) A 40-year-old woman has extensive microcalcifications


involving the
entire upper aspect of the right breast. Biopsy shows a commedo pattern of
intraductal carcinoma.
The most appropriate treatment is :
F) wide local excision
G) radiation therapy
H) wide local excision plus radiation therapy
I) right total mastectomy
J) right modified radical mastectomy

25) In the conventional ventilator management of acute adult respiratory


distress syndrome (ARDS) , arterial O2 saturation is maintained above 90%
by all the following EXCEPT :
F) increasing the ventilatory rate
G) the use of positive end-expiratory pressure (PEEP)
H) increasing mean airway pressure
I) increasing tidal volumes
J) increasing FiO2

31) Which of the following statements about patients with


abdominal compartment syndrome is NOT true ?
A) Abdominal pressure is usually measured indirectly
through inferior vena cava
B) Multiple contributing factors are commonly responsible
C) The chief manifestations are reflected in central venous
pressure , ventilatory function, and oliguria
D) Decopression of the abdomen is required to resverse the
syndrome
E) Aggressive hemodynamic monitoring and management is
required when the abdomen is opened

32) The most appropriate treatment for histologically malignant


cystadenoma phylloides is :
A) total mastectomy without axillary node dissection
B) total mastectomy with axillary node dissection
C) wide margin (3) cm excision of the lesion
D) post operative hormonal manipulation
E) postoperative adjuvant chemotherapy

33) Deep venous thrombosis resulting from upper extremity central


venous lines:
A) should be treated with catheter removal, heparin therapy,
and long term anticoagulants
B) is best with urokinase through the catheter
C) is innocuous and self limiting, and best treated with
catheter removal only
D) is best treated with low-dose warfarin (coumadin, 1 mg /
day) , without catheter removal
E) is best managed by single systemic dose of low molecular
weight heparin daily and continued catheter use

34) Emergency surgery is indicated for all of the following


complications of ulcerative colitis EXCEPT:
A) colonic dilatation greater than 12 cm (toxic mega colon )
B) free perforation
C) complete intestinal obstruction
D) intractable haemorrhage
E) abscess formation

35) All the following statements concerning carcinoma of the


oesophagus are true EXCEPT that:
A) it has a higher incidence in males than females
B) alcohol has been implicated as a precipitating factor
C) adenocarcinoma is the most common type at the cardio
esophageal junction
D) it occurs more commonly in patients with corrosive
oesophagitis
E) surgical excision is the only effective treatment

31-. Which of the following statements about epiphrenic diverticula of the


esophagus is/are correct?

k) They are traction diverticula that arise close to the tracheobronchial


tree.
l) They characteristically arise proximal to an esophageal reflux
stricture.
m) The degree of dysphagia correlates with the size of the pouch.
n) They are best approached surgically through a right thoracotomy.
o) The operation of choice is a stapled diverticulectomy, long
esophagomyotomy, and partial fundoplication.

32- Which of the following statements about Schatzki's ring is correct?

k) The ring represents a panmural fibrotic stricture resulting from


gastroesophageal reflux.
l) Dysphagia occurs when the ring diameter is 13 mm. or less.
m) The ring occurs within 1 to 2 cm. of the squamocolumnar epithelial
junction.
n) Schatzki's ring indicates reflux esophagitis.
o) Schatzki's ring signifies the need for an antireflux operation.

33. Which of the following statements about pathology encountered at


esophagoscopy is/are correct?

k) Reflux esophagitis should be graded as mild, moderate, or severe, to


promote consistency among different observers.
l) An esophageal reflux stricture with a 2-mm. lumen is not dilatable and
is best treated with resection.
m) A newly diagnosed radiographic distal esophageal stricture warrants
dilation and antireflux medical therapy.
n) In patients with Barrett's mucosa, the squamocolumnar epithelial
junction occurs 3 cm. or more proximal to the anatomic
esophagogastric junction.
o) After fasting at least 12 hours, a patient with megaesophagus of
achalasia can safely undergo flexible fiberoptic esophagoscopy.

34. Infantile hypertrophic pyloric stenosis


k) Occurs with a female : male ratio of 1:4.
l) Sons of affected mothers have a 20% risk of developing the lesion
m) Invariably presents between 3 and 4 WK of age
n) Typically presents with out bile stained projectile vomiting
o) Surgical treatment is by Heller's( Ramsted myotomy) Cardiomyotomy

35-. Which of the following statements regarding the pathology of


esophageal carcinoma is/are correct?

k) Worldwide, adenocarcinoma is the most common esophageal


malignancy.
l) Squamous cell carcinoma is most common in the distal esophagus,
whereas adenocarcinoma predominates in the middle third.
m) Patients with Barrett's metaplasia are 40 times more likely than the
general population to develop adenocarcinoma.
n) Metastases from esophageal carcinoma are characteristically
localized to regional mediastinal lymph nodes adjacent to the tumor.
o) Achalasia, radiation esophagitis, caustic esophageal stricture,
Barrett's mucosa, and Plummer-Vinson syndrome are all premalignant
esophageal lesions that predispose to the development of squamous
cell carcinoma.

36-45-year-old with isolated 6-cm colorectal metastasis in the liver 2 years


after colectomy, otherwise healthy pest treatment would be:

k) Radiofrequency ablation
l) Systemic chemotherapy
m) Hepatic lobectomy
n) Liver transplantation
o) Cryosurgical ablation

37- Oesophageal atresia all true except:


k) Is often associated with a distal trachea-oesophageal fistula
l) Polyhydramnios is often present late in pregnancy( Early)
m) 50% have other associated congenital abnormalities
n) Contrast X-ray studies are necessary to confirm the diagnosis
o) Post-operatively over 30% develop oesophageal strictures
38-All are true about the dumping syndrome except:

k) Symptoms can be controlled with a somatostatin analog.


l) Diarrhea is always part of the dumping syndrome.
m) Flushing and tachycardia are common features of the syndrome.
n) Separating solids and liquids in the patient's oral intake alleviates
some of the symptoms of the syndrome.
o) Early postoperative dumping after vagotomy often resolves
spontaneously.

39-Which of the following statements about gastric polyps is/are true?

k) Like their colonic counterparts, gastric epithelial polyps are common


tumors.
l) They are analogous to colorectal polyps in natural history.
m) Endoscopy can uniformly predict the histology of a polyp based on
location and appearance.
n) In a given patient, multiple polyps are generally of a multiple
histologic type.
o) Gastric adenomatous polyps greater than 2 cm. in diameter should be
excised because of the risk of malignant transformation.

40-All of the following statements about surgical management of gastric


lymphomas are true except:

k) Stage I gastric lymphomas (small lesions confined to the stomach


wall) can be cured completely with surgical therapy alone.
l) Extensive gastric lymphomas that initially are treated with radiation
and/or chemotherapy occasionally perforate during treatment and
require secondary resection.
m) Patients explored with a presumptive diagnosis of gastric lymphoma
should undergo an attempt at curative resection when this is safe and
feasible.
n) Without a preoperative diagnosis resection for gastric mass should
not be attempted unless lymphoma can be excluded.
o) Appropriate staging for primary gastric lymphoma includes bone
marrow biopsy.

41-the most accurate test to confirm diagnosis of infected necrotizing


pancreases is:
k) Abdominal ultrasound study
l) Indium-labeled leeukocte scan
m) Cimputed tomographic scan
n) Elevated serum level of interleukain 6 and 8
o) Percutaneous needle aspiration

42- Which of the following variables best predicts prognosis for patients
with a recent diagnosis of cutaneous melanoma and no clinical evidence of
metastatic disease?

k) Breslow thickness.
l) Clark's level.
m) Ulceration.
n) Gender.
o) Celtic complexion.

43-the following are true about intracranial tumors except:

k) The most common location of brain tumors of childhood is the


posterior cranial fossa.
l) With few exceptions, examination of the CSF is of no value in the
diagnosis of an intracranial tumor.
m) Even the most malignant of primary brain tumors seldom spread
outside the confines of the central nervous system (CNS).
n) The majority of astrocytomas can be cured surgically.
o) Primary neoplasms of astrocytic, oligodendroglial, or ependymal
origin represent gradations of a spectrum from slowly growing to
rapidly growing neoplasms.

44 A right-sided disc herniation at the L5–S1 level typically may cause:

k) Low back pain and left sciatica.


l) Weakness of dorsiflexion of the right foot.
m) A diminished or absent right ankle jerk.
n) Diminution of sensation over the medial aspect of the right foot,
including the great toe.
o) Weakness of dorsiflexion of the left foot.
45-. The preferred operation for initial management of a thyroid nodule that
is considered suspicious for malignancy by FNAB is:

k) Excision.
l) Partial lobectomy.
m) Total lobectomy and isthmusectomy.
n) Total thyroidectomy.
o) All methods are correct

46-the most common presentation of Meckel,s diverticulum in an adult is:

k) GIT bleeding
l) GIT obstruction
m) Intussuception
n) Litter,s hernia
o) Diverticulitis

47-Optimal front-line treatment of squamous cell carcinoma of the rectum


includes:

k) Abdominal perineal resection.


l) Low anterior resection when technically feasible.
m) Radiation therapy.
n) Chemotherapy.
o) Combined radiation and chemotherapy.

48 -65-year-old man presents with complaints of mucous discharge and


perianal discomfort. Physical examination reveals a fistulous opening
lateral to the anus. Anoscopic examination permits passage of a probe
through the fistula tract. The fistula traverses the internal anal sphincter, the
intersphincteric plane, and a portion of the external anal sphincter. The
fistula is categorized as which type?

a) Intersphincteric
b) Transsphincteric
c) Suprasphincteric
d) Extrasphincteric
e) Non of the above

49-Warthin's tumor:

k) Is a pleomorphic adenoma of salivary gland


l) Should be treated by total paritidectomy
m) Is considered a benign salivary gland neoplasia
n) Respond well to preoperative radiotherapy
o) Often present with facial nerve compression

50-A 38 year old woman presents with right upper quadrant pain and bouts
of vomiting. She is known to have gallstones and has had similar episodes in
the past. Which of the following might support a diagnosis of acute
cholecystitis rather than biliary colic

a) duration of symptoms
b) Severity of vomiting
c) Presence of Murphy's sign
d) Presence of gas under right hemidiaphragm on erect CXR.

1. The radial nerve.


f) Is a branch of the anterior cord of the brachial plexus.
g) Is derived from the posterior primary rami of C5 to C7.
h) Supplies the flexors of the arm.
i) Gives rise to the anterior interosseous nerve.
j) Injury above elbow produces a classical wrist drop.

2. Regarding femoral canal all are true except:


f) Lies lateral to the femoral vein.
g) Has the inguinal ligament as its anterior border.
h) Has the lacunar ligament as its medial border.
i) Has the pectineal ligament as its posterior border.
j) Contains the lymph node of Cloquet.

3. The following causes hypercalcaemia except:


a) Sarcoidosis.
b) Primary hyperparathyroidism.
c) Acute pancreatitis.( hypocalcemia)
d) Metastatic bronchial carcinoma.
e) Milk-Alkali syndrome.

4. Number of human chromosomes in human female are:


f) 23 pairs +XX.
g) 21 pairs +XY.
h) 22 pairs +XY.
i) 22 pairs +XX.
j) 23 pairs +XY.

6. Potassium deficiency should be suspected in all the following except:


e) In cases of paralytic ileus.
f) When the patient's reflexes are exaggerated.
g) If there is a decrease in height and peaking of the T waves of an
ECG.
h) In alkalotic states.

6. Active immunization in case of tetanus:


f) Antitetanus human serum.
g) Gives short period of protection.
h) Given in case proved tetanus.
i) Use of immunoglobulin.
j) None of the above.

7. All of the following are signs of rised intracranial pressure


except:
a) Headache.
b) Vomiting.
c) Papilledema.
d) Aphasia.
e) Bradycardia.

8. All of the following are extra cranial hematomas except:


a) Subcutaneos haematoma.
b) Extra dural haematoma.
c) Cephalohaematoma.
d) Subgaleal hematoma.
e) Subperiostial haematoma.

9. Glasgow coma scale all the following are true except:


f) Used for evaluation of comatose patient.
g) It ranges from ( 3 to 15).
h) Useful for neurological follow up.
i) Useful for pupils evaluation.
j) Best motor response given 6 point.

10. All the followings are Indications for central line insertion EXCPET:
e) Massive fluid replacement
f) Massive blood replacement
g) Measurement of central venous pressure
h) Prolonged Intervenes fluid therapy

11. Most common early complication of central venous line is:


a) Sepsis
b) Pneumothorax( But if late answer is A or D )
c) Thoracic duct injury
d) Thrombosis

12. The following are Complications of shock:


a) Acute Respiratory failure
b) Acute myocardial infarction
c) Acute renal failure
d) A&C only
e) All the above

13. Causes of delayed union of fractures includes all the following


EXCPET:
a) Compound fracture
b) Infection
c) Adequate immobilization
d) Poor blood supply

14. In Head injury C.T. scan is indicated in the following


f) Aphasia after the injury
g) Deterioration of level of consciousness
h) Skull fracture with persistent headache
i) A&B only
j) All the above

15. Most important steps in management of head injury include:


f) Prevent hypoxia
g) Prevent Dehydration
h) Assure Brain Metabolism
i) Prevent secondary brain injury
j) All the above

16. Tension pneumothorax


f) is the commonest type of chest injuries
g) Needs urgent X-Ray chest
h) Is a clinical Diagnosis
i) Causes flat neek viens
j) Treated by thoracotomy tube after chest X-ray.
17. Calcitonin hormon is secreted to the blood circulations from:
f) Parathyroid gland.
g) Parafollicular cells of thyroid gland.( Calcitonin increase in
medullary carcinoma)
h) Supra renal gland.
i) Pituitary gland.
j) Gonads.

18. Regarding tension pneumothorax, the first step in the management is:
f) Obtaining a stat chest x-ray.
g) Cricothyroidectomy.
h) Pass an endotracheal tube.
i) Starting oxygen by a valve-mask device.
j) Chest decompression needle.
19. The following are features of thyrotoxicosis except:
f) Weight gain.
g) Palpitations.
h) Proximal myopathy.
i) Increased skin pigmentation.
j) Pretibial myxoedema.

20. The following is a clinical feature of Horner's syndrome:


e) Miosis
f) Failure of abduction of the orbit
g) Increased sweating on the contralateral side of the forehead
h) Exophthalmos.
e) All are true

21. In role of nine extent of burn if entire trunk is burned it will be equal to:
f) 9% body surface area.
g) 18% body surface area.
h) 36% body surface area.
i) 27% body surface area.
j) 45% body surface area.

22. Trachlea (4th) cranial nerve supply :


f) Lateral rectus muscle of th eye.
g) Medial rectus muscle of the eye.
h) Superior obligue rectus muscle of the eye.
i) Superior oblique muscle of the eye.
j) Muscles of the upper eye lid.

23. Regarding pathological terms :


f) Hypertrophy is an increase in tissue size due to increased cell
number.
g) Hyperplasia is an increase in tissue size due to an increase in
cell size.
h) Atrophy is an increase in tissue size due to disuse.
i) Metaplasia is a change form one abnormal tissue type to
another.
j) A hamartoma is a developmental abnormality.

24. Regarding nephroblastomas:


f) They are otherwise known as a Wilm's tumour.
g) Account for 10% of childhood tumours.
h) The commonest presentation is with an abdominal mass.
i) Most commonly present between 2nd and 4th year of life.
j) All are true.

25. Regarding fluid losses in a major burn all are true except:
f)Are maximal between 12 and 24 hours after the injury.
g) Are related to the age of the patient.
h) Are not related to the weight of the patient.
i)Are related to the area burnt.
j)Are not related to the burn duration.

26. In obstructive jaundice :


f)Urinary conjugated bilirubin is increased.
g) Serum unconjugated bilirubin is increased.
h) Urinary urobilinogen is increased.
i)Serum conjugated bilirubin is reduced.
j)Faecal stercobilinogen is increased.

27. Regarding Hydatid disease:


f) Is due to Ecchinococcus granulosa.
g) Man is an accidental intermediate host.
h) The liver is the commonest site of infection.
i) Can be diagnosed by the Casoni test.
j) All are true.

28. The first aid of treatment in fracture of cervical spine should be:
f) Cervical spine x-ray.
g) Analgesia.
h) Neck immoblization.
i) Cervical traction.
j) Non of teh above.

29. The first aid of treatment in fracture of cervical spine should be:
a) Cervical spine x-ray.
b) Analgesia.
c) Neck immoblization.
d ) Cervical traction.

30. Regarding local anaesthesia:


a) Local anaesthetics act on small before large nerve
fibres
b) Adrenaline reduces absorption and prolongs the local
effects
c) Preparations containing adrenaline are safe to use on
digits and appendages
d) Lignocaine has a longer duration of action than
bupivicaine.
e) All are false.

31. Small bowel obstruction often results in: (all correct except one)
f)Hyperkalaemia.
g) Metabolic alkalosis.
h) Oliguria.
i)Hypovolaemia.
j)Severe dehydration.
32. A serious intra-abdominal injury in a comatose patient may be
diagnosed by: (all are correct except one)
f)Abdominal paracentesis.
g) The observation of bruising pattern on the abdominal wall.
h) Falling of heamoglobin values.
i)The presence of marked abdominal distetion.
j)The presence of diarrhea.

33. A perforated duodenal ulcer, all are true except:


f) Usually lies on the anterior or superior surface of the
duodenum.
g) Usually presents with the acute onset of severe back pain.
h) Produces radiological evidence of free gas in the peritoneum
in over 90 percent of the patients.
i) Is usually treated by vagotomy and pyloroplasty.
j) Is usually treated conservatively.

34. Acute pancreatitis typically: (all correct except one)


f)Is accompanied by hypocalcaemia.
g) Produces paralytic ileus.
h) Is associated with a pleural effusion.
i)Produces pyloric stenosis.
j)Upper abdominal pain and vomiting.
35. Biliary colic typically:
f)Occurs 3 to 4 hours after meals.
g) Lasts 5 to 20 minutes.
h) Radiates from the upper abdomen to the right subscapular
region.
i)Is made better by deep inspiration.
j)B&C only.
36. In post operative DVT, the following are true except:
f) Clinical DVT occures in the 4th post operative day.( any time)
g) If complicated by pulmonary embolism, it occures usually after the
7th post operative day.
h) The process of DVT starts preoperatively with the induction of
anaesthesia .
i)When discovered we should start the patient on coumadin "Warfarin
anticoagulation".
j)It may lead to chronic venous in suffering as a complication of DVT.

37. In acute appendicitis all of the following are true except:


f)Anorexia.
g) Abdominal pain usually preceedes vomiting.
h) Pain after begins in the paraumbilical region.
i)Constipation diarrhea may occur.
j)Dysuria excludes the diagnosis.

38. The most common cause of massive haemorrhage in the lower


gastroinfestinal tract is :
f)Carcinoma.
g) Diverticulosis
h) Diverticulitis
i)Polyp.
j)Ulcerative colitis.

39. Painless haematuria is the leading presentation of :


f)Renal cell carcinoma.
g) Transitional cell carcinoma of the bladder .
h) Ureteric stone.
i)Pelvi-ureteric obstruction.
j)Ureterocele.

40. All of the following are complications of massive blood transfusion


except:
f)Acute congestive heart failure.
g) Transmission of infection.
h) Hypercalcaemia.
i)Hyperkalaemia.
j)Transfusion reactions.

41. Complication of undescended testis include all of the following except :


f) Malignant degeneration.
g) Increased susceptability to trauma.
h) Increased spermatogenesis.
i) More liable to testiculer torsion.
j) Psychological complication.

42. The recurrent laryngoeal nerve is branch of :


f) Facial nerve.
g) Glosso-pharyngeal nerve.
h) Cervical plexus.
i) Vagus nerve.
j) Brachial plexus.

43. The thyroid tumor which is may be associated with pheochromocytoma


is :
f)Papillary carcinoma.
g) Medullary carcinoma.
h) Follicular carcinoma.
i)Anaplastic carcinoma.
j)Malignant lymphoma.

44. The most common pancreatic cyst is :


f)Dermoid cyst of the pancreas.
g) Hydatid cyst of the pancreas.
h) Pancreas pseudocyst.
i)Pancreatic cystadenoma.
j)Congenital cystic disease of the pancreas.
45. The anatomical division between the anus and rectum :
f)Lateral haemorrhoidal groove.
g) Inter haemorrhoidal groove.
h) Dentate line.
i)Arcuate line
j)Ano-rectal ring.

46. The comments type of Anorectal abscess is:


e) Ischio rectal
f) Perianal
g) Submucons
h) Pelvirectal

47. Anal Fissure:

f) Usually anterior
g) May be caused by previous anal surgery
h) Can cause dark bleeding PR.
i) Sometimes is painful
j) Treated by steroids

48. Neonatal duodenal obstruction:

f) May be associated with down's syndrome.


g) Is more frequently found in premature infants.
h) Typically presents with gross abdominal distension.
i) Usually presents with vomiting of non-bile stained fluid
j) B&C only.

49. acute superior mesenteric artery occlusion: (all correct except one)

f) Characteristically presents with sudden pain and tenderness of


increasing intensity.
g) Is frequently accompanied by overt or occult blood loss in the stools.
h) Frequently produces peritonitis.
i) Can usually be diagnosed on plain abdominal x-rays.
j) Can be diagnosed by mesenteric artery ongiography.

50. Regarding the management of polytrauma:

f) Death follow a trimodal distribution.


g) X-ray after primary survey should be AP cervical spine, chest
and pelvis.
h) Cardiac tamponade is characterized by raised B.p, a low JUP.
i) Assessment of uncomplicated limb fractures should occur
during the primary survey.
j) A and B only.

51. Injuries to the urethra (all are correct except one)

a) Are more common in male.


b) Are often caused by road traffic accidents.
c) Are easily diagnosed on intra venous pyelography.
d) Require urgent surgical treatment.
e) Diagnosed by retrograde urethragraphy.

52 .Car seat belts when properly adjusted


a) Prevent injuries to abdominal organs.
b) May cause small bowel injuries.
c) Do not reduce the incidence of head injuries of passengers involving
in RTA.
d) Protect the cervical spine during sudden acceleration .
e) A & D only.

53. Patients with major burns:


f) Are in a negative nitrogen balance.
g) Have normal calorie requirements.
h) Do not generally become anaemic.
i) Are resistant to septicaemia.
j) All of the above.

54 . In a healing fracture: (All correct except one)


a) The haematoma is initially invaded by osteoblasts.
b) The tissue formed by the invading osteoblasts is termed osteoid.
c) Calcium salts are laid down in the osteoid tissue.
d) The final stage of repair is the remodelling of the callus.
e) The callus formation is related to the amount of stress at fracture side.

55. In a colles’ fracture the distal radial fragment:


f) Is dorsally angulated on the proximal radius.
g) Is usually torn from the intra-articular triangular disc.
h) Is deviated to the ulnar side.
i) Is rarely impacted.
j) Is ventrally displaced.

56. A malignant melanoma:

a) Frequently arises from hair-bearing naevi.


b) Frequently arises from junctional naevi. *
c) Has a worse prognosis when it areses on the leg.
d) Should be suspected in any big pigmented lesion.
e) Non of the above is correct.

57. Squamous cancer of the lip:

a) Is most common in early adult life.


b) Is more common in fair skinned subjects.
c) Metastasises readily by the blood stream.
d) Is preferably treated by radiotherapy once lymph node deposits are
present.
e) All of the above are correct.

58- Basal cell carcinomas:


a) Usually metastasise to regional lymph nodes.
b) Are less common than squamous cell carcinomas.
c) Are characterised histologically by epithelial pearls.
d) Are particularly common in oriental races.
e) Non of the above is correct.

59- Fiboadenomata of the breast:


a) Are commonest in early adult life.
b) Are indiscrete and difficult to distinguish.
c) Are usually painless.
d) Resolve without treatment.
e) A&C only.

60. Paget’s disease of the nipple:


a) Usually presents as abilateral eczema of the nipple.
b) Is always related to an underlying breast cancer.
c) Indicates incurable breast cancer.
d) Has non-specific histological characteristics.
e) A&C only.

61- stones in the common bile duct:


a) Are present in nearly 50 per cent of cases of cholecystitis.
b) Often give rise to jaundice, fever and biliary colic.
c) Are usually accompanied by progressive jaundice.
d) Are usually associated with a distended gallbladder.
e) A&D only.

62- Colonic polyps: (all correct except one)


f) Are associated with colonic cancer.
g) May be hereditary.
h) Should not be removed if they are asymptomatic.
i) May be hyperplastic.
j) Are commonly adenomatous.

1-Which is the shortest phase of the normal cell cycle?


f) G1 phase
g) S phase
h) G2 phase
i) M phase
j) All phases approximately equal in length

2- A 67 year-old women with rectal cancer is admitted to gereral surgical


floor which of the following laboratory studies should be included in the
surgeon’s initial nutritional assessment :
f) Transferrin
g) Prealbumin
h) Albumin
i) Glutamine
j) All of above

3- In which of the following conditions is the entral route appropriate for


nutrition :
f) Upper gastrointestinal obstruction
g) Complete small bowel obstruction
h) Acute flare-up of Crohn’s disease
i) Low out put colonic fistula
j) Non of the above

4-5-Which is the most commonly cultured hospital acquired organism in


critical care with aspiration pneumonia:
f) Staphylococcus aureus
g) Streptococcus pneumonia
h) Anaerobic species
i) Pseudomonas aeroginosa
j) Haemophlus influenzae

5-Which is the most appropriate single agent for empiric coverage of the
above patient :
f) Metranidazole
g) Clindamycin
h) Pipracillin_tazobactam
i) Vancomycin
j) First generation penicillin

6-All of the following are true about neurogenic shock except:

f) There is a decrease in systemic vascular resistance and an increase in


venous capacitance.
g) Tachycardia or bradycardia may be observed, along with hypotension.
h) The use of an alpha agonist such as phenylephrine is the mainstay of
treatment.
i) Severe head injury, spinal cord injury, and high spinal anesthesia may
all cause neurogenic shock.
j) A and B

7-Which of the following statements about head injury and concomitant


hyponatremia are true?

f) There are no primary alterations in cardiovascular signs.


g) Signs of increased intracranial pressure may be masked by the
hyponatremia.
h) Oliguric renal failure is an unlikely complication.
i) Rapid correction of the hyponatremia may prevent central pontine
injury.
j) This patient is best treated by restriction of water intake

8 Regarding Cushing's Syndrome


f) 20% of cases are due to pituitary adenomas (Cushing's Disease)
g) Most ACTH secreting pituitary adenomas are more than 2 cm in
diameter
h) Is characterised by loss of the diurnal rhythm of cortisol secretion
i) Cortisol production is suppressed by low-dose dexamethasone
j) Adrenal carcinomas are more common than adrenal adenomas

9-Which of the following statements about extracellular fluid are true?

f) The total extracellular fluid volume represents 40% of the body


weight.
g) The plasma volume constitutes one fourth of the total extracellular
fluid volume.
h) Potassium is the principal cation in extracellular fluid.
i) The protein content of the plasma produces a lower concentration of
cations than in the interstitial fluid.
j) The interstitial fluid equilibrates slowly with the other body
compartments.

10-In patients receiving massive blood transfusion for acute blood loss,
which of the following is/are correct?

f) Packed red blood cells and crystalloid solution should be infused to


restore oxygen-carrying capacity and intravascular volume.
g) Two units of FFP should be given with every 5 units of packed red
blood cells in most cases.
h) A “six pack” of platelets should be administered with every 10 units of
packed red blood cells in most cases.
i) One to two ampules of sodium bicarbonate should be administered
with every 5 units of packed red blood cells to avoid acidosis.
j) One ampule of calcium chloride should be administered with every 5
units of packed red blood cells to avoid hypocalcemia.

11- Which of the following statements about the presence of gallstones in


diabetes patients is/are correct?

f) Gallstones occur with the same frequency in diabetes patients as in


the healthy population.
g) The presence of gallstones, regardless of the presence of symptoms, is
an indication for cholecystectomy in a diabetes patient.
h) Diabetes patients with gallstones and chronic biliary pain should be
managed nonoperatively with chemical dissolution and/or lithotripsy
because of severe complicating medical conditions and a high
operative risk.
i) The presence of diabetes and gallstones places the patient at high risk
for pancreatic cancer.
j) Diabetes patients with symptomatic gallstones should have prompt
elective cholecystectomy, to avoid the complications of acute
cholecystitis and gallbladder necrosis.

12. concerning Tetanus all true except:


a. Is due to an infection with a gram-negative spore forming rod
b. The organism produces a powerful exdotoxin
c. The toxin prevents the release of inhibitory neurotransmitter
d. Clostridium tetani is sensitive to penicillin
e. Risus sardonicus is the typical facial spasm

13-when should parentral antibiotics be given perioperatively?

f) The night before


g) 6 hr prior to surgery
h) 30 minutes prior to incision .
i) at the time of incision
j) 30 minutes after incision

14. Which of the following statements about esophageal anatomy is correct?

f) The esophagus has a poor blood supply, which is segmental in


distribution and accounts for the high incidence of anastomotic
leakage.
g) The esophageal serosa consists of a thin layer of fibroareolar tissue.
h) The esophagus has two distinct muscle layers, an outer, longitudinal
one and an inner, circular one, which are striated in the upper third
and smooth in the distal two thirds.
i) Injury to the recurrent laryngeal nerve results in vocal cord
dysfunction but does not affect swallowing.
j) The lymphatic drainage of the esophagus is relatively sparse,
localized primarily to adjacent paraesophageal lymph nodes.
15 Wich of the following medication should be given in preparation of a
pation with pheochromocytoma?

f) Phnoxybenzamine
g) Nifedipine
h) Linsinopril
i) Hydrochlorothiazide
j) Propranolol

16-Which of the following statement(s) is true concerning excessive scarring


processes?

f) Keloids occur randomly regardless of gender or race


g) Hypertrophic scars and keloid are histologically different
h) Keloids tend to develop early and hypertrophic scars late after the
surgical injury
i) Simple reexcision and closure of a hypertrophic scar can be useful in
certain situations such as a wound closed by secondary intention
j) Non of the above

17-A 22-year-old man sustains a single stab wound to the left chest and
presents to the emergency room with hypotension. Which of the following
statement(s) is true concerning his diagnosis and management?

e) The patient likely is suffering from hypovolemic shock and should


respond quickly to fluid resuscitation
f) Beck’s triad will likely be an obvious indication of compressive
cardiogenic shock due to pericardial tamponade
g) Echocardiography is the most sensitive noninvasive approach for
diagnosis of pericardial tamponade
h) The placement of bilateral chest tubes will likely resolve the problem

18-Which of the following statement(s) is/are true concerning septic shock?

e) The clinical picture of gram negative septic shock is specifically


different than shock associated with other infectious agents
f) The circulatory derangements of septic shock precede the
development of metabolic abnormalities
g) Splanchnic vascular resistance falls in similar fashion to overall
systemic vascular resistance
h) Despite normal mechanisms of intrinsic expansion of the circulating
blood volume, exogenous volume resuscitation is necessary

19- During surgery on the submandibular gland


f) An incision on the lower border of the mandible is safe
g) The submandibular gland is seen to wrap around the posterior border
of mylohyoid
h) The facial artery and vein are divided as they course through the deep
part of the gland
i) The hypoglossal nerve is seen to loop under the submandibular duct
j) Damage to the lingual nerve will cause loss of sensation to the
posterior third of the tongue

20- Regarding benign breast disease except


f) Cyclical mastalgia is the commonest reason for referral to the breast
clinic
g) Fibroadenomas are derived from the breast lobule
h) Lactational breast abscesses are usually due to Staph aureus
i) Duct ectasia is more common in smokers
j) Atypical lobular hyperplasia is associated with an decreased risk of
breast cancer

21-. Which of the following statements regarding unusual hernias is


incorrect?

f) An obturator hernia may produce nerve compression diagnosed by a


positive Howship-Romberg sign.
g) Grynfeltt's hernia appears through the superior lumbar triangle,
whereas Petit's hernia occurs through the inferior lumbar triangle.
h) Sciatic hernias usually present with a painful groin mass below the
inguinal ligament.
i) Littre's hernia is defined by a Meckel's diverticulum presenting as the
sole component of the hernia sac.
j) Richter's hernia involves the antimesenteric surface of the intestine
within the hernia sac and may present with partial intestinal
obstruction

22- Staples may safely be placed during laparoscopic hernia repair in each
of the following structures except:
f) Cooper's ligament.
g) Tissues superior to the lateral iliopubic tract.
h) The transversus abdominis aponeurotic arch.
i) Tissues inferior to the lateral iliopubic tract.
j) The iliopubic tract at its insertion onto Cooper's ligament.

23-The following statements about the repair of inguinal hernias are true
except:

f) The conjoined tendon is sutured to Cooper's ligament in the Bassini


hernia repair.
g) The McVay repair is a suitable option for the repair of femoral
hernias.
h) The Shouldice repair involves a multilayer, imbricated repair of the
floor of the inguinal canal.
i) The Lichtenstein repair is accomplished by prosthetic mesh repair of
the inguinal canal floor in a tension-free manner.
j) The laparoscopic transabdominal preperitoneal (TAPP) and totally
extraperitoneal approach (TEPA) repairs are based on the
preperitoneal repairs of Cheattle, Henry, Nyhus, and Stoppa.

24-A number of special circumstances exist in the repair of inguinal hernias.


The following statement(s) is correct.

f) Simultaneous repair of bilateral direct inguinal hernias can be


performed with no significant increased risk of recurrence
g) The preperitoneal approach may be appropriate for repair of a
multiple recurrent hernia
h) A femoral hernia repair can best be accomplished using a Bassini or
Shouldice repair
i) Management of an incarcerated inguinal hernia with obstruction is
best approached via laparotomy incision
j) All are correct.

25- Which of the following statement(s) is true about benign lesions of the
liver?

e) Adenomas are true neoplasms with a predisposition for complications


and should usually be resected.
f)Focal nodular hyperplasia (FNH) is a neoplasm related to birth control
pills (BCPs) and usually requires resection.
g) Hemangiomas are the most common benign lesions of the liver that
come to the surgeon's attention.
h) Nodular regenerative hyperplasia does not usually accompany
cirrhosis

26. Ligation of all of the following arteries usually causes significant hepatic
enzyme abnormalities except:

e) Ligation of the right hepatic artery.


f) Ligation of the left hepatic artery.
g) Ligation of the hepatic artery distal to the gastro-duodenal branch.
h) Ligation of the hepatic artery proximal to the gastroduodenal artery.

27- Which of the following is the most effective definitive therapy for both
prevention of recurrent variceal hemorrhage and control of ascites?

f) Endoscopic sclerotherapy.
g) Distal splenorenal shunt.
h) Esophagogastric devascularization (Sugiura procedure).
i) Side-to-side portacaval shunt.
j) End-to-side portacaval shunt.

28-which of the following is associated with best prognosis for patient with
breast cancer?

f) Male sex
g) Estrogen receptor positive
h) Patient age <35 years
i) Pregnant patient
j) Tumor with overexpression of HER/ner.

29-A 49-year-old women has a palpable breast mass in the upper outer
quadrant. The size of the mass has increased over the last month .
exicisional biopsy reveals cystic carcinoma with invasion .appropriate
management now would be :

f) Re-excision with wide margins


g) Axillary node dissection and hormonal therapy
h) Simple mastectomy
i) Modified radical mastectomy
j) Bilateral mastectomies
30-A contraindication to stereotactic core biopsy of the breast is the
mammographic presence of:

f) Microcacification
g) A radial scar
h) A nonpalpable mass lesion
i) Lesions<8 mm in diameter
j) Mutifocal lesions.

31-. Which of the following statements about epiphrenic diverticula of the


esophagus is/are correct?

p) They are traction diverticula that arise close to the tracheobronchial


tree.
q) They characteristically arise proximal to an esophageal reflux
stricture.
r) The degree of dysphagia correlates with the size of the pouch.
s) They are best approached surgically through a right thoracotomy.
t) The operation of choice is a stapled diverticulectomy, long
esophagomyotomy, and partial fundoplication.

32- Which of the following statements about Schatzki's ring is correct?

p) The ring represents a panmural fibrotic stricture resulting from


gastroesophageal reflux.
q) Dysphagia occurs when the ring diameter is 13 mm. or less.
r) The ring occurs within 1 to 2 cm. of the squamocolumnar epithelial
junction.
s) Schatzki's ring indicates reflux esophagitis.
t) Schatzki's ring signifies the need for an antireflux operation.

33. Which of the following statements about pathology encountered at


esophagoscopy is/are correct?

p) Reflux esophagitis should be graded as mild, moderate, or severe, to


promote consistency among different observers.
q) An esophageal reflux stricture with a 2-mm. lumen is not dilatable and
is best treated with resection.
r) A newly diagnosed radiographic distal esophageal stricture warrants
dilation and antireflux medical therapy.
s) In patients with Barrett's mucosa, the squamocolumnar epithelial
junction occurs 3 cm. or more proximal to the anatomic
esophagogastric junction.
t) After fasting at least 12 hours, a patient with megaesophagus of
achalasia can safely undergo flexible fiberoptic esophagoscopy.

34. Infantile hypertrophic pyloric stenosis


p) Occurs with a female : male ratio of 4:1.
q) Sons of affected mothers have a 20% risk of developing the lesion
r) Invariably presents between six and eights months of age
s) Typically presents with bile stained projectile vomiting
t) Surgical treatment is by Heller's Cardiomyotomy

35-. Which of the following statements regarding the pathology of


esophageal carcinoma is/are correct?

p) Worldwide, adenocarcinoma is the most common esophageal


malignancy.
q) Squamous cell carcinoma is most common in the distal esophagus,
whereas adenocarcinoma predominates in the middle third.
r) Patients with Barrett's metaplasia are 40 times more likely than the
general population to develop adenocarcinoma.
s) Metastases from esophageal carcinoma are characteristically
localized to regional mediastinal lymph nodes adjacent to the tumor.
t) Achalasia, radiation esophagitis, caustic esophageal stricture,
Barrett's mucosa, and Plummer-Vinson syndrome are all premalignant
esophageal lesions that predispose to the development of squamous
cell carcinoma.

36-45-year-old with isolated 6-cm colorectal metastasis in the liver 2 years


after colectomy, otherwise healthy pest treatment would be:

p) Radiofrequency ablation
q) Systemic chemotherapy
r) Hepatic lobectomy
s) Liver transplantation
t) Cryosurgical ablation

37- Oesophageal atresia all true except:


p) Is often associated with a distal trachea-oesophageal fistula
q) Polyhydramnios is often present late in pregnancy
r) 50% have other associated congenital abnormalities
s) Contrast X-ray studies are necessary to confirm the diagnosis
t) Post-operatively over 30% develop oesophageal strictures
38-All are true about the dumping syndrome except:

p) Symptoms can be controlled with a somatostatin analog.


q) Diarrhea is always part of the dumping syndrome.
r) Flushing and tachycardia are common features of the syndrome.
s) Separating solids and liquids in the patient's oral intake alleviates
some of the symptoms of the syndrome.
t) Early postoperative dumping after vagotomy often resolves
spontaneously.

39-Which of the following statements about gastric polyps is/are true?

p) Like their colonic counterparts, gastric epithelial polyps are common


tumors.
q) They are analogous to colorectal polyps in natural history.
r) Endoscopy can uniformly predict the histology of a polyp based on
location and appearance.
s) In a given patient, multiple polyps are generally of a multiple
histologic type.
t) Gastric adenomatous polyps greater than 2 cm. in diameter should be
excised because of the risk of malignant transformation.

40-All of the following statements about surgical management of gastric


lymphomas are true except:

p) Stage I gastric lymphomas (small lesions confined to the stomach


wall) can be cured completely with surgical therapy alone.
q) Extensive gastric lymphomas that initially are treated with radiation
and/or chemotherapy occasionally perforate during treatment and
require secondary resection.
r) Patients explored with a presumptive diagnosis of gastric lymphoma
should undergo an attempt at curative resection when this is safe and
feasible.
s) Without a preoperative diagnosis resection for gastric mass should
not be attempted unless lymphoma can be excluded.
t) Appropriate staging for primary gastric lymphoma includes bone
marrow biopsy.
41-the most accurate test to confirm diagnosis of infected necrotizing
pancreases is:

p) Abdominal ultrasound study


q) Indium-labeled leeukocte scan
r) Cimputed tomographic scan
s) Elevated serum level of interleukain 6 and 8
t) Percutaneous needle aspiration

42- Which of the following variables best predicts prognosis for patients
with a recent diagnosis of cutaneous melanoma and no clinical evidence of
metastatic disease?

p) Breslow thickness.
q) Clark's level.
r) Ulceration.
s) Gender.
t) Celtic complexion.

43-the following are true about intracranial tumors except:

p) The most common location of brain tumors of childhood is the


posterior cranial fossa.
q) With few exceptions, examination of the CSF is of no value in the
diagnosis of an intracranial tumor.
r) Even the most malignant of primary brain tumors seldom spread
outside the confines of the central nervous system (CNS).
s) The majority of astrocytomas can be cured surgically.
t) Primary neoplasms of astrocytic, oligodendroglial, or ependymal
origin represent gradations of a spectrum from slowly growing to
rapidly growing neoplasms.

44 A right-sided disc herniation at the L5–S1 level typically may cause:

p) Low back pain and left sciatica.


q) Weakness of dorsiflexion of the right foot.
r) A diminished or absent right ankle jerk.
s) Diminution of sensation over the medial aspect of the right foot,
including the great toe.
t) Weakness of dorsiflexion of the left foot.
45-. The preferred operation for initial management of a thyroid nodule that
is considered suspicious for malignancy by FNAB is:

p) Excision.
q) Partial lobectomy.
r) Total lobectomy and isthmusectomy.
s) Total thyroidectomy.
t) All methods are correct

46-the most common presentation of Meckel,s diverticulum in an adult is:

p) GIT bleeding
q) GIT obstruction
r) Intussuception
s) Litter,s hernia
t) Diverticulitis

47-Optimal front-line treatment of squamous cell carcinoma of the rectum


includes:

p) Abdominal perineal resection.


q) Low anterior resection when technically feasible.
r) Radiation therapy.
s) Chemotherapy.
t) Combined radiation and chemotherapy.

48 -65-year-old man presents with complaints of mucous discharge and


perianal discomfort. Physical examination reveals a fistulous opening
lateral to the anus. Anoscopic examination permits passage of a probe
through the fistula tract. The fistula traverses the internal anal sphincter, the
intersphincteric plane, and a portion of the external anal sphincter. The
fistula is categorized as which type?

a) Intersphincteric
b) Transsphincteric
c) Suprasphincteric
d) Extrasphincteric
e) Non of the above
49-Warthin's tumor:

p) Is a pleomorphic adenoma of salivary gland


q) Should be treated by total paritidectomy
r) Is considered a benign salivary gland neoplasia
s) Respond well to preoperative radiotherapy
t) Often present with facial nerve compression

50-A 38 year old woman presents with right upper quadrant pain and bouts
of vomiting. She is known to have gallstones and has had similar episodes in
the past. Which of the following might support a diagnosis of acute
cholecystitis rather than biliary colic

a) duration of symptoms
b) Severity of vomiting
c) Presence of Murphy's sign
d) Presence of gas under right hemidiaphragm on erect CXR.

32) Abduction of the vocal cords results from contraction of


the:
A) crico-thyroid muscles
B) posterior crico-arytenoid muscles
C) vocalis muscles
D) thyro-ary-epiglottic muscles
E) lateral crico-arytenoid and transverse arytenoids
muscles

33) The rectum:


A) is devoid of peritoneum
B) is surrounded by peritoneum
C) has peritoneum on its lateral surfaces for its upper
two- thirds, and on its anterior surface for its upper
one-third
D) has pritoneum on its anterior surface for its upper
two-thirds, and on its lateral surfaces for its upper
one-third
E) has peritoneum on its anterior surface only
34) The umbilicus:
A) lies near the to the xiphoid than to the pubis
B) derives its cutaneous innervation from the eleventh
thoracic nerve
C) transmits, during development, the umbilical cord
two arteries and two veins
D) usually lies at about the level between the third and
fourth lumbar vertebra
E) emberiologicall, may transmit urine but never bowel
contents

35) The superficial perineal pouch:


A) is limited inferiorly by the urogenital diaphragm
B) is not continuous with the space in the scrotum
occupied by the testes
C) has a membranous covering which provides a fascial
sheath around the penis
D) is traversedby the urethera in the male but not the
urethera and vagina in the female
E) in the female, the greater vestibular glands are
situated outside this pouch

36) The tongue:


A) has a foramen caecum at the base of the frenulum
B) is separated from the epiglottis by the valleculae on
each side of the midline
C) has 7-12 circumvallate papillae situated behind the
sulcus terminalis
D) is attached to the hyoid bone by the genioglossus
muscle
E) is supplied only by hypoglossal nerve

37) Hypovolaemic shock is characterized by:


A) a low central venous pressure , low cardiac output ,
low peripheral resistance
B) a high central venous pressure , high cardiac output ,
low peripheral resistance
C) a low central venoys pressure , low cardiac output ,
high periphera resistance
D) a low central venous pressure , high cardiac output ,
high peripheral resistance
E) a high central venous pressure , low cardiac output ,
low peripheral resistance

38) An oxygen debt is:


A) the amount of oxygen in excess of the resting
metabolic needs that
must be consumed after completion of exercise
B) build up because the pulmonary capillaries limit
the uptake of
Oxygen at high rates of oxygen consumption
C) related to the fact that skeletal muscle cannot function
temporarily in the absence of oxygen

H) associated with a decrease in blood lactate


I) associated with alkalosis

39) Pulmonary embolism may be a complication of:


A) prolonged bed rest
B) a surgical operation
C) vitamine K deficiency
D) oral contraceptive therapy
E) Antithrombin III deficiency

40) Which of the following statements regarding potassium


metabolism is NOT
True?:
A) potassium deficiency commonly results from thiazide
diuretic theraoy
B) the normal compensation for potassium deficiency is
a metabolic extracellular acidosis
C) aldosterone increases urinary potassium loss
D) hyperkalaemia causes bradycardia and loss of P
waves on the ECG
E) hypokalaemia aggrevates the cardiac effects of
digitalis toxicity

41) Cutaneous pain:


A) is due to overstimulation of receptors serving other
sensory modalities
B) cannot be elicited more readily if the tissue has
recently been injured
C) is due to exitation of receptors by pain-producing
chemical substances in the injured tissue
D) shows marked adaptation, i. e. decrease in severity
in response to a constant stimulus
E) is conducted through the medial spinothalamic tract

42) Which of the following is NOT associated with


hyperthyroidism?:
A) increase size of the thyroid gland
B) increased amount of colloid in thyroid follicle
C) increased height of epithelium of the thyroid follicle
D) increased vascularity of the thyroid gland
E) increased uptake of iodine by the thyroid gland

43) Sarcomata may show all of the following EXCEPT:


A) production of myxomatous tissue
B) production of collagen
C) spindle shaped cells
D) signet ring cells
E) blood stream metastasis

44) Anaphylaxis is characterized by all of the following


EXCEPT:
A) is a reaction either local or general , frequently
occurs within five minutes
B) causes an urticarial eruption
C) is produced by IgA antibody
D) causes eosinophilia
E) causes degranulation of basophils and mast cells

45) Autoimmunity is characterized by the following


EXCEPT:
A) occurs because of a breakdown in the ability of the
body to distinguish between self and non self
B) is involved in some forms of orchitis
C) is involved in formation of cryo globulin
D) is important in the pathogenesis of lupus
erytheromatosus
E) does not result in immune complex disease

46) Pseudomembranous enterocolitis is caused by the


following organisms:
A) Clostridium sporogenes
B) Clostridium defficile
C) Streptococcus faecalis
D) Penicillin sensitive staphylocci
E) Pseudomonas aeruginos

B) CLINICAL SUGERY

47) The “ white clot syndrome”:


A) is usually characterized with antithrombin III
deficiency
B) most often present with arterial complicatios of
heparin induced throbocytopenin
C) is best managed by loe molecular weight dextran
D) is best managed by halving the therapeutic dose of
heparin sodium
E) results from nitric oxide deficiency of endothelial
cells

17) A 21-year-old man who was the driver in a head-on


collision has a pulse of 140/min , respiratory rate of 36 and
blood pressure of 75 palpable. His trachea is deviated to the
left, with palpable subcutaneous emphysema and poor breath
sounds in the right hemithorax, The most appropriate initial
treatment must be
A) immediate thoracotomy
B) catheter insertion in the subclavian vein for fluid
resuscitation
C) intubation and ventilation
D) tube thoracostomy
E) immediate tracheostomy

18) The best test to monitor the adequacy of


levothyroxin therapy is:
F) radioactive iodine uptake
G) thyroglobulin
H) free thyroxine index (T4)
I) triiodothyronine resin uptake (T3)
J) thyroid stimulating hormone (TSH)

29) Which of the following statements about fungal infection


is NOT true ?:
A) Prior or synchronous culture positive for Candida
at another site occurs in few patients with
candidimia
B) For critically ill patients nonhaematogenous sites
of candida are appropriately treated with systemic
antifungal therapy
C) Mortality rates are similar regardlss of whether C.
albicans fungmia is treated with amphotericin B
or flconazole
D) Intravenous catheters and the gastrointestinal
tract are common portals for Candida to gain
blood stream access
E) Septic emboli are more common with fungal
endocarditis than with bacterial endocarditis

30) The maximum safe dose of local anaesthetic administered


subcutaneously in a 70-kg man is:
A) 10 to 20 ml of 1% lidocaine
B) 40 to 50ml oh 2% lidocaine with epinephrine
C) 40 to 50 ml of 1% lidcaine with epinephrine
D) 40 to 50 ml of 1% bupivacaine (marcaine)
E) 40 to 50 ml of 1%lidocaine without epinephrine

31) Two days after right hemicolectomy for a Dukes B caecal


carcinoma , the
Patient complains of sharp right-sided chest pain and
dyspnea. HisPaO2
Is 64mmHg ,his PaCo2 is 32mmHg. CVP is 26 cm
water, and the blood pressure is 102/78mmHg. A
pulmonary embolus is suspected,
The next step in management should be:
K) A ventilation- perfusion lung scan
L) A pulmonary arteriogram
M) Postrioanterior and lateral chest x-rays
N) Heparin sodium ,100 units/kg intravenously
O)Immediate duplex scanning of both lower
extremities

32) The major cause of graft loss in heart and kidney


allograft is:
A) acute rejection
B) hyperacute rejection
C) vascular thrombosis
D) chronic rejection
E) graft infection
33) All of the following are indicators of tumor
aggressiveness and poor outcome for papillary
carcinoma of the thyroid gland EXCEPT:
A) age over 50 years
B) microscopic lymph node metstasis
C) tumor larger than 4 cm
D) poorly differentiated histological grade
E) invasion through capsule to adjacent tissues

24) A 40-year-old woman has extensive


microcalcifications involving the
entire upper aspect of the right breast. Biopsy shows a
commedo pattern of intraductal carcinoma.
The most appropriate treatment is :
K) wide local excision
L) radiation therapy
M) wide local excision plus radiation therapy
N) right total mastectomy
O)right modified radical mastectomy

25) In the conventional ventilator management of acute


adult respiratory distress syndrome (ARDS) , arterial O2
saturation is maintained above 90% by all the following
EXCEPT :
F) increasing the ventilatory rate
G) the use of positive end-expiratory pressure (PEEP)
H) increasing mean airway pressure
I) increasing tidal volumes
J) increasing FiO2
36) Which of the following statements about patients with
abdominal compartment syndrome is NOT true ?
A) Abdominal pressure is usually measured indirectly
through inferior vena cava
B) Multiple contributing factors are commonly
responsible
C) The chief manifestations are reflected in central
venous pressure , ventilatory function, and
oliguria
D) Decopression of the abdomen is required to
resverse the syndrome
E) Aggressive hemodynamic monitoring and
management is required when the abdomen is
opened

37) The most appropriate treatment for histologically


malignant cystadenoma phylloides is :
A) total mastectomy without axillary node dissection
B) total mastectomy with axillary node dissection
C) wide margin (3) cm excision of the lesion
D) post operative hormonal manipulation
E) postoperative adjuvant chemotherapy

38) Deep venous thrombosis resulting from upper extremity


central venous lines:
A) should be treated with catheter removal, heparin
therapy, and long term anticoagulants
B) is best with urokinase through the catheter
C) is innocuous and self limiting, and best treated
with catheter removal only
D) is best treated with low-dose warfarin (coumadin,
1 mg / day) , without catheter removal
E) is best managed by single systemic dose of low
molecular weight heparin daily and continued
catheter use

39) Emergency surgery is indicated for all of the following


complications of ulcerative colitis EXCEPT:
A) colonic dilatation greater than 12 cm (toxic mega
colon )
B) free perforation
C) complete intestinal obstruction
D) intractable haemorrhage
E) abscess formation

40) All the following statements concerning carcinoma of


the oesophagus are true EXCEPT that:
A) it has a higher incidence in males than females
B) alcohol has been implicated as a precipitating
factor
C) adenocarcinoma is the most common type at the
cardio esophageal junction
D) it occurs more commonly in patients with
corrosive oesophagitis
E) surgical excision is the only effective treatment
1

A 44-year-old man presents with painless rectal bleeding of 1 month's


duration. He reports a history of constipation. He works in heavy labor.
For this patient, which of the following statements regarding internal
hemorrhoids is true?

Choose one answer


e. Stapled hemorrhoidectomy should be done for grade 1 and 2
hemorrhoids
f. All of the above
g. Internal hemorrhoids are located proximal to the dentate line and
therefore are usually painless
h. A grade 1 internal hemorrhoid represents bleeding with prolapse

39. A 34-year-old woman presents for evaluation of severe and frequent


bloody bowel movements, as well as abdominal pain, dehydration, and
anemia. She has had these symptoms for 2 days. She has not had any
similar symptoms in the past, and she has been in relatively good health.
If the patient has toxic megacolon, under what circumstances emergency
surgical management is indicated?

Choose one answer.


e. There is a perforation
f. Any of the above
g. The patient's clinical or radiographic status worsens
h. There is no improvement in 24 to 36 hours after aggressive medical
therapy

Complications of untreated pancreatic pseudocysts include all of the


following EXCEPT:

Choose one answer.


f. intracystic hemorrhage
g. abscess
h. pancreatic necrosis
i. free rupture
j. gastrointestinal obstruction
4

Platelets in the wound form a hemostatic clot and release clotting factors
to produce:

Choose one answer.


f. thrombin
g. Fibrin
h. Fibrinogen
i. Fibroblasts
j. thromboplastin

A 43-year-old man presents to the office for evaluation of recent weight


loss and frequent loose stools. He is concerned because his father was
diagnosed with colon cancer at the age of 50.

Besides family history, what are some other risk factors for colorectal
cancer?

Choose one answer.


e. Hypertension
f. Diabetes
g. All of the above
h. Inflammatory bowel disease

A 40-year-old woman presents to the office for evaluation of yellowish


skin. She states that over the past few weeks, she has noticed that her eyes
and skin have developed a yellow tint. She also reports that she has dark
urine and pale-colored stools. Further history elicits periodic bouts of
right upper quadrant pain after eating. She is otherwise healthy. She
denies using any medications. On physical examination, a yellowish tint is
observed on the patient's skin, sclera, and mucous membranes.

On the basis of this patient's history and clinical examination, which type
of bilirubin would you expect to predominate?

Choose one answer.


e. Mixed
f. Conjugated
g. Unconjugated
h. Indirect

39. A 38-year-old man presents with a complaint of a slow-growing mass


over his right parotid gland. The lesion is fixed to the underlying
structures and has recently become painful.
Which of the following features strongly suggests that this patient's lesion
is a malignancy?

Choose one answer.


e. Ipsilateral numbness of the tongue
f. .All of the above
g. Overlying skin involvement
h. Facial nerve paralysis

Axillary lymph nodes are classified according to the relationship with the

Choose one answer.


f. pectoralis minor muscle
g. pectoralis major muscle
h. axillary vein
i. serratus anterior muscle
j. latissimus dorsi muscle

39. A 78-year-old man is recovering from abdomino-perineal (A-P)


resection for Ca rectum, which was performed 3 days ago. The patient is
now complaining of mild shortness of breath and chest pain. On physical
examination, the patient's right leg is slightly more swollen than his left.
The pulse oximetry reading is 90%.
What is the principal method of diagnosing acute pulmonary embolism?

Choose one answer.


e. Magnetic resonance imaging
f. Chest x-ray
g. Ultrasound
h. Spiral computed tomography scanning
10

Compartment syndrome

Choose one answer.


f. Passive stretch decrease muscle pain
g. Due to decrease pressure in muscle compartments
h. Pulse is the first thing to disappear
i. Cause severe pain in the limb
j. Treatment is by delayed fasciotomy

11

The most significant risk factor for the development of adenocarcinoma of


the esophagus is:

Choose one answer.


f. lye stricture
g. alcohol abuse
h. Barrett's esophagus
i. long-standing achalasia
j. smoking

12

All of the following statements are true about patients with carcinoid
tumors EXCEPT:

Choose one answer.


f. the combination of streptozotocin and 5-fluorouracil (chemotherapy)
can often result in objective response.
g. tumor growth is often slow
h. they often have evidence of serotonin production
i. the majority have carcinoid syndrome
j. they have a much better prognosis if the tumors are less than 2 cm.

13

39. A 67-year-old man presents with left-lower-quadrant pain and low-


grade fever. He has had these symptoms for 1 day. The patient denies
experiencing any rectal bleeding, but for the past week, his bowel
movements have been irregular.
For this patient, which of the following statements is true regarding
diverticular disease?
Choose one answer.
e. The sigmoid colon is the most common site of diverticula
f. All of the above
g. Most diverticula of the colon involve the muscular layer
h. Smoking does not seem to be related to the development of
diverticular disease

14

All of the following are components of the MEN type 2B syndrome except:

Choose one answer.


e. Multiple neuromas on the lips, tongue, and oral mucosa
f. Medullary thyroid carcinoma
g. Pheochromocytoma
h. Hyperparathyroidism.

15

The risk of bilateral breast cancer is HIGHEST if the first breast shows:

Choose one answer.


f. inflammatory carcinoma
g. medullary carcinoma
h. infiltrating ductal carcinoma
i. paget’s disease
j. lobular carcinoma

16

The best initial therapy for deep venous thrombosis of the common
femoral vein is:

Choose one answer.


f. warfarin
g. streptokinase
h. Heparin
i. venous thrombectomy
j. placement of a vena caval filter

17

Complications after thyroidectomy include all the following EXCEPT:


Choose one answer.
f. recurrent laryngeal nerve paralysis
g. parathyroid insufficiency
h. thyrotoxic crisis(storm) on operating on inadequitly prepared
thyrotoxic patient
i. tracheomalacia
j. hypercalcemia

18

A 55-year-old man presents with hematemesis that began 2 hours ago. He


is hypotensive and has altered mental status. No medical history is
available.
For this patient, which of the following statements regarding nasogastric
aspiration is true?

Choose one answer.


e. None of the above
f. A clear, nonbilious aspirate rules out the need for EGD
g. A clear, bilious aspirate rules out the need for EGD
h. A bloody aspirate is an indication for esophagogastroduodenoscopy
(EGD)

19

The major cause of impaired wound healing is:

Choose one answer.


f. steroid use
g. malnutrition
h. diabetes mellitus
i. local tissue infection
j. anemia

20

Common presenting conditions in patients with pancreatic carcinoma


include all of the following EXCEPT:

Choose one answer.


e. esophageal varices.
f. palpable gallbladder
g. weight loss.
h. abdominal pain

21

All of the following statements about keloids are true EXCEPT:

Choose one answer.


f. Keloid tissue contains an abnormally large amount of collagen
g. A keloid does not regress spontaneously
h. Keloid tissue contains an unusually large amount of soluble collagen
i. A keloid extends beyond the boundaries of the original wound
j. Keloids or hypertrophic scars are best managed by excision and
careful reapproximation of the wound

22

The treatment of choice for a 40-year-old man who is found on endoscopy


and biopsy to have a gastric lymphoma would be:

Choose one answer.


f. wide local excision
g. subtotal gastrectomy
h. chemotherapy
i. subtotal gastrectomy and radiotherapy
j. Radiotherapy

23

The treatment of an esophageal burn with a caustic agent may include all
of the following EXCEPT:

Choose one answer.


f. boogieing. (dilatation)
g. induction of vomiting
h. gastrectomy
i. expeditious administration of an antidote
j. steroids and antibiotics.

24

A 32-year-old man with a family history of familial adenomatous polyposis


(FAP) presents with hematochezia. He denies having any diarrhea,
abdominal pain, or fever.
For this patient, which of the following statements regarding FAP is true?

Choose one answer.


a. CRC does not occur in patients with FAP if they are given adequate
medical treatment
b. Total proctocolectomy (TPC) is considered the only option for the surgical
management of FAP
c. For patients with FAP, there is a 10% risk of CRC by age 40 if
prophylactic colectomy is not performed
d. In the setting of FAP, colorectal cancer (CRC) is more commonly located
on the left side

25

When stage I breast cancer is treated by partial mastectomy and axillary


dissection, further therapy should include:

Choose one answer


a. antiestrogen agents.
b. radiation of the affected breast.
c. oophorectomy if premenopausal.
d. nothing
e. chemotherapy

26

For the patient in Question 68, which of the following statements is true
regarding an esophageal varix as the site of bleeding?

Choose one answer.


e. I.V. propranolol should be administered first
f. Balloon tamponade should be performed first
g. Rubber banding or intravariceal sclerotherapy should be performed
first
h. I.V. somatostatin should be administered first

27

Drugs which may produce gynecomastia include all of the following


EXCEPT:

Choose one answer.


f. furosemide
g. cimetidine
h. Verapamil
i. Diazepam
j. Tamoxifen

28

Splenectomy is commonly indicated for the following EXCEPT:

Choose one answer.


f. hypersplenism associated with cirrhosis
g. hereditary spherocytosis
h. splenic tumor
i. immune thrombocytopenic purpura
j. grade four splenic injury in trauma

29

An ischiorectal abscess is characterized by all of the following EXCEPT:

Choose one answer.


f. Requires deroofing
g. Should be treated entirely by antibiotics
h. May be tuberculous in origin
i. Can be followed by anal fistula
j. Is an infective necrosis of the fat of the ischiorectal fossa

30
Marks: 1

The most common presentation of Meckel’s diverticulum in an adult is:

Choose one answer.


e. intussuception
f. Littre’s hernia
g. Gastrointestinal bleeding
h. diverticulitis

31

For the patient in Question 65, which of the following is an indication for
immediate surgery?

Choose one answer.


e. Closed-loop obstruction
f. All of the above
g. Complete bowel obstruction
h. Incarcerated hernia

32

Choledocholithiasis in a patient who previously had cholecystectomy is


BEST treated with:

Choose one answer.


f. endoscopic sphincterotomy
g. choledochoduodenostomy
h. choledochojejunostomy.
i. dissolution with mono-octanoin
j. open common bile duct exploration with stone removal

33

39. A 77-year-old man undergoes endoscopic ultrasonography as part of a


workup for jaundice. He is found to have a tumor in the head of the
pancreas.
For this patient, which of the following findings would indicate that the
tumor is unresectable?

Choose one answer.


e. All of the above
f. Peritoneal metastases
g. Invasion of the superior mesenteric artery
h. Metastases to celiac lymph nodes

34

Regarding polyps of the colon

Choose one answer.


f. Villous polyps are usually pedunculated
g. Villous polyps occur more proximal in colon
h. Adenomatous polyps are usually solitary
i. Cancer risk is not related to size of polyp
j. Metaplastic polyps are not precancerous
35

The most likely diagnosis in elderly patient with abdominal pain and
colonoscopy finding of patchy mucosal ulceration at the splenic flexure of
the colon is :

Choose one answer.


f. ulcerative colitis
g. crohns disease
h. ischemic colitis
i. diverticulitis
j. lymphogranuloma venerum

36

39. A 54-year-old man presents with a neck mass of 2 weeks' duration. He


has no significant medical history. He smokes two packs of cigarettes a
day and has been doing so since he was 21 years of age.
For this patient, which of the following statements is true?

Choose one answer.


e. Low cervical nodes are more likely to contain metastases from a
primary source other than the head and neck, whereas upper cervical
nodes are more likely to contain metastases from the head and neck
f. Soft or tender nodes are more likely to derive from an inflammatory or
infectious condition, whereas hard, fixed, painless nodes are more
likely to represent metastatic cancer
g. All of the above
h. Enlarged lymph nodes are by far the most common neck masses
encountered

37

The first-choice diagnostic study for suspected deep venous thrombosis of


the lower extremity is:

Choose one answer.


f. real-time Doppler imaging
g. contrast sonography
h. radioactive labeled fibrinogen uptake
i. impedance plethysmography
j. isotope injection with gamma scintillation scanning
38

Decreased PaCO2 levels should be attained in a patient at serious risk for


cerebral edema secondary to a head injury in order to :

Choose one answer.


f. prevent neurogenic pulmonary edema
g. prevent increased capillary permeability
h. prevent metabolic acidosis
i. allow reciprocally high levels of PaO2 in the brain
j. prevent cerebral vasodilation

39

Causes of third space loss include all except:

Choose one answer.


f. Acute pancreatitis
g. Pancreatic fistula
h. Necrotizing fasciitis
i. Site of major surgery
j. Crush syndrome

40

Causes of metabolic acidosis include all except:

Choose one answer.


f. Small bowel fistula
g. Shock
h. CO poisoning
i. severe anemia
j. All of the above

41

A 39-year-old man comes in for evaluation of intermittent anal pain and


bleeding after bowel movements. He also has hard stools. He has had
these symptoms for over 1 year .
For this patient, which of the following is included in the classic triad of
signs of chronic anal fissures?

Choose one answer.


e. An anal fissure
f. All of the above
g. A sentinel skin tag
h. Hypertrophy of the anal papilla

42

All the following statements about achalasia are true EXCEPT:

Choose one answer.


f. it occurs most commonly in persons between the ages of 30 and 50
years
g. in most affected persons, ganglion cells in the body of the esophagus
either are absent or have degenerated
h. esophageal cancer is seven times as common in affected persons as in
the general population
i. pressure in the body of the esophagus is lower than normal
j. affected persons usually experience more difficulty swallowing cold
foods than warm foods

43

A patient with the Zollinger-Ellison syndrome is found to have the


multiple endocrine neoplasia type I (MEN-I) syndrome. Appropriate
management for the ulcer symptoms should be:

Choose one answer.


f. Omeprazole
g. pancreatic resection
h. streptozocin
i. cimetidine
j. total gastrectomy

44

Fat absorption occurs primarily in the:

Choose one answer.


f. Ileum
g. third portion of the duodenum
h. Stomach
i. Jejunum
j. first portion of the duodenum
45

39. A 56-year-old woman has been experiencing abdominal pain for 4


hours. The pain is in right upper quadrant and radiates into the scapular
region. She has had multiple episodes of vomiting.
For this patient, which of the following signs on physical examination is
associated with acute cholecystitis?

Choose one answer.


e. Carnett sign
f. Kehr sign
g. Murphy sign
h. Rovsing sign

46

Mammary duct ectasia is characterized by the following EXCEPT :

Choose one answer.


f. Is treated usually by simple mastectomy
g. May present with nipple retraction and Peau d'orange picture
h. Anaerobic superinfection cmmmonly occurs in this recurrent
periductal plasma cell mastitis
i. Is defined as primary dilatation of major ducts of breast in middle
aged women
j. Iscommonly pre-malignant

47

Regarding veins of lower limbs all are true except:

Choose one answer.


e. Valves allow flow from deep to superficial system Venous return from
lower limbs is aided by respiratory movements
f. Superficial veins lie in subcutaneous tissue
g. The pressure in veins of the foot while standing is 100mmHg
h. a) Stasis of blood is important factor in developing varicose veins

48

Most common complication of central venous access is:

Choose one answer.


f. major artery damage.
g. Catheter problems.
h. Thrombosis of central vein
i. Catheter related sepsis
j. Pleural space damage, pneumothorax
49

Factors associated with increase risk of death in acute pancreatitis include


all except:

Choose one answer.


f. Ranson score more than five
g. Obesity
h. High APACHE_II score
i. Age more than 70 years
j. sterile necrosis

50

For the patient in Question 56, which of the following chronic conditions
can cause a neck mass?

Choose one answer.


e. Tuberculosis
f. Sarcoidosis
g. AIDS
h. All of the above

51

For the patient in Question 56, if metastatic cancer is suspected initially,


which of the following would be the most appropriate step to take next in
the workup?

Choose one answer.


e. CT scan
f. Empirical therapy with antibiotics
g. Fine-needle aspiration (FNA)
h. Observation only

52

The largest component of intestinal gas is:


Choose one answer.
f. Nitrogen
g. carbon dioxide
h. Oxygen
i. ammonia
j. hydrogen

53

A 66-year-old woman presents to the office complaining of a sharp,


constant pain in her lower abdomen. She has had this pain for the past 2
weeks. Examination of her abdomen is normal. However, subsequent
ultrasound reveals a 4.5 cm Aortic abdominal aneurysm (AAA). Spiral
computed tomography confirms the ultrasound findings.

Aside from rupture, which of the following is a complication associated


with (AAA) ?

Choose one answer.


e. Aortoenteric fistula
f. Lower-extremity atheroemboli
g. All of the above
h. Thrombosis

54

Diagnosis of esophageal perforation is best established by:

Choose one answer.


f. transesophageal ultrasound study
g. esophagoscopy with a flexible esophagoscope
h. contrast esophagograms
i. esophagoscopy with a rigid esophagoscope
j. upright X-rays of the chest including lateral and oblique films

55

For the patient in Question 68, which of the following is an indication for
surgery?

Choose one answer.


e. Ongoing hemorrhage occurs from a gastric ulcer in a
hemodynamically unstable patient
f. All of the above
g. Bleeding continues from either a duodenal ulcer or a gastric ulcer
despite medical and endoscopic therapy
h. Substantial bleeding occurs from a duodenal ulcer that is not
controlled by EGD

56

Yesterday, a 38-year-old woman underwent a laparoscopic cholecystectomy


for cholelithiasis and was discharged home 8 hours after surgery. She
returns this morning complaining of worsening abdominal pain. The oral
narcotics that the patient was prescribed are ineffective in controlling the
pain. The patient's temperature is (38.3 C). Laboratory studies reveal an
elevated white blood cell count. Abdominal ultrasonography shows a large
subhepatic fluid collection. The fluid is percutaneously aspirated and
reveals enteric contents.

What step should be taken next in the management of this patient?

Choose one answer.


e. Immediate laparotomy
f. I.V. antibiotics and close observation
g. None of the above
h. Observation in the hospital until pain improves

57

The most common symptom after major pulmonary embolism is:

Choose one answer.


f. cough
g. Dyspnea
h. Hemoptysis
i. pleural pain
j. fear of death

58

Treatment of paralytic ileus includes all of the following


EXCEPT :

Choose one answer.


f. intravenous fluids
g. cessation of oral intake
h. correction of electrolyte imbalance
i. early operation
j. nasogastric suction

59
Lymphedema :
Choose one answer
f. e) None of the above
g. b) should be bilateral
h. c) may be pitting in early stage
i. d) A & C only
j. a) may be Congenital

60

The level of consciousness for a head injury patient is BEST evaluated


by :

Choose one answer.


f. CT scan
g. visual evoked potentials
h. Glasgow coma scale
i. papillary responses
j. response to pain

61

A 52-year-old male weighing 70 kg, sustained a 65% total body surface


area (TBSA) burn. What are his fluid requirements?

Choose one answer.


a. 18,200 cc in 16 h
b. 9100 cc in the first 8 h
c. 8000 cc in 24 h
d. 12,800 cc in 24 h

62

Management of cholangitis may include all of the following EXCEPT:


Choose one answer.
a. decomperession of the common bile duct.
b. cholecystostomy
c. IV antibiotics.
d. correct underlying cause.
e. percutaneous transhepatic cholangiography.

63

A 48-year-old woman presents to the emergency department complaining


of right upper quadrant pain, which began 4 hours ago. She reports the
pain as being spasmodic and sharp and that it radiates to her right
shoulder blade. She says that she has had similar episodes over the past
few months, especially after eating large meals. Associated with the pain is
nausea and vomiting. Her blood pressure is 120/85 mm Hg, and her pulse
is 100 beats/min. On physical examination, the patient is found to have a
nontender abdomen with no palpable masses. Her chest and
cardiovascular examinations are normal. The nurse notices that her
sclerae are slightly icteric. On subsequent laboratory studies, her serum
bilirubin level is found to be 10 mg/dl.

What imaging study should be performed next for this patient with
presumed posthepatic jaundice?

Choose one answer.


a. Percutaneous transhepatic cholangiography (PTC)
b. Ultrasonography
c. Endoscopic retrograde cholangiopancreatography (ERCP)
d. Magnetic resonance imaging

64

39. An HIV-positive man presents for evaluation of new oral cavity lesions
he discovered last month. Physical examination reveals purple exophytic
masses involving the palate mucosa and gingiva.
What is the most likely diagnosis of these oral cavity lesions?

Choose one answer.


a. Oral hairy leukoplakia
b. Non-Hodgkin lymphoma
c. Syphilis
d. Kaposi sarcoma
65

Brain injury alone

Choose one answer.


a. causes shock only if the skull is intact
b. rarely causes shock
c. causes shock that is reversed by very simple measures
d. frequently causes shock
e. causes shock if hypoxia is superimposed

66

All of the following substances are irritating to the peritoneum EXCEPT:

Choose one answer.


a. bile.
b. blood
c. gastric content.
d. meconium
e. pus

67

Regarding volvulus of the sigmoid colon, each of the following is true


except :

Choose one answer.


a. likely results from redundant sigmoid colon with an elongated narrow
mesocolon
b. there appears to be a congenital predisposition sigmoid volvulus.
c. diagnostic barium enema for sigmoid colon is essential
d. diagnostic x-ray for sigmoid volvulus shows a dilated loop of colon which
points toward the right upper quadrant.

68

Gallstones are characterized by all the following EXCEPT:

Choose one answer.


a. cause mucocoele of the gall bladder
b. are present in the common bile duct in 40% of patients with stones in the
gall bladder
c. are becoming common in post-partum primipara who were pre pregnancy
‘Pill’ takers
d. are frequently the cause of flatulent dyspepsia
e. may be present in the newborn

69

Following a burn, the agent responsible for early increased capillary


permeability is

Choose one answer.


a. thromboxane A2
b. serotonin
c. histamine
d. prostacyclin PGI2
e. bradykinin

70

Incisonal hernias all are true except:

Choose one answer.


a. 10 % of all hernias
b. Usually easy to reduce
c. Multiloculated sac
d. Operative technique is most important cause
e. Complication is common

71

In the treatment of gastric cancer, all of the following are true EXCEPT:

Choose one answer.


a. five-year survival rates in the continue to be between 10% and 25%.
b. Palliative resection is frequently helpful with advanced disease.
c. Lymph node involvement is associated with a poorer prognosis.
d. Finding early disease at the time of operation is associated with a better
prognosis.
e. total gastrectomy is mandated in most patient. ***

72
of the involved area and : Severe cases of hidradenitis suppurativa in the
groin area are best managed by excision

Choose one answer.


a. transfer of a rectus abdominus muscle flap
b. split thickness skin grafting
c. primary closure
d. delayed primary closure
e. closure by secondary intention

73

Patients at increased risk for gastric carcinoma include all the following
EXCEPT:

Choose one answer.


a. those who have undergone gastric bypass for morbid obesity
b. those who have undergone gastric resection for duodenal ulcer
c. those with pernicious anemia
d. those with a high consumption of smoked fish
e. those with blood group A

74

A 56-year-old woman presents with symptoms of abdominal pain, weight


loss, and rectal bleeding. She is anemic and hypotensive, but she is stable.
For this patient, which of the following should be done first if the gastric
lavage yields copious amounts of bile?

Choose one answer.


a. Arteriography
b. Emergency laparotomy
c. Colonoscopy
d. Esophagogastroduodenoscopy

75

A 39-year-old man presents with lower GI bleeding. He has no abdominal


discomfort and has experienced no loss of weight.
For this patient, which of the following statements regarding the etiology
of lower GI bleeding is true?

Choose one answer.


a. Lower GI bleeding from diverticulosis often requires surgery
b. Arteriovenous malformations (AVMs) are the most common cause of
lower GI bleeding
c. 6% to 10% of patients with ulcerative colitis have lower GI bleeding
severe enough to necessitate emergency surgical resection
d. None of the above

76

For the patient in Question 65, which of the following statements


regarding abdominal radiographic findings is true?

Choose one answer.


a. In gastric outlet obstruction, no gastric air will be seen, but large amounts
of air will be seen in the small bowel and colon
b. Mechanical small bowel obstruction usually shows no air-fluid levels, nor
will distended bowel loops of similar sizes be seen
c. All of the above
d. High-grade obstruction of the colon in association with an incompetent
ileocecal valve may mimic small bowel obstruction on x-ray

77

A 41-year-old female presents to the emergency department after


sustaining a gunshot wound to the abdomen, with injuries to the liver and
large bowel. Despite successful resuscitation and operative intervention,
the patient dies 2 weeks later of multisystem organ failure in the intensive
care unit. Which organ most likely first experienced dysfunction?

Choose one answer.


a. lung
b. heart
c. liver
d. gastrointestinal tract
e. kidney

78

39. A 35-year-old man presents with severe acute abdominal pain of


sudden onset. The pain initially began in the upper abdomen and has now
settled in the whole abdomen. On examination, the abdomen was rigid.
Chest X-Ray showed free gas under diaphragm.
Which of the following is the most appropriate course of management for
this patient?

Choose one answer.


a. Observation
b. Urgent laparotomy
c. Nonurgent laparotomy
d. Laparoscopy

79

All are true about the dumping syndrome except:

Choose one answer.


a. Symptoms can be controlled with a somatostatin analogue.
b. Early postoperative dumping after vagotomy often resolves spontaneously.
c. Flushing and tachycardia are common features of the syndrome.
d. Diarrhea is always part of the dumping syndrome.
e. Separating solids and liquids in the patient's oral intake alleviates some of
the symptoms of the syndrome.

80

An anal fissure is characterized by the following EXCEPT:

Choose one answer.


a. Is more common in women.
b. Is an ulcer of the anal mucosa.
c. Can be treated coservatively.
d. Usually lies anteriorly.

81

All of the following statements are true of esophageal carcinoma


EXCEPT

Choose one answer.


a. squamous cell tumor is the most frequent histology.
b. operation is frequently curative.
c. patient often require a pyloroplasty with operation.
d. patients usually die within one year of diagnosis
e. patient can have intestinal continuity reestablished using the stomach
after esophageal resection.
82

A serum tumor marker correlated with recurrence after management of


colon cancer is:

Choose one answer.


a. carcinoembryonic antigen (CEA)
b. Calcitonin
c. 5-hydroxyindoleacetic acid
d. CA 15-3
e. Alpha-fetoprotein

83

Conditions associated with gastric cancer include all of the following


EXCEPT:

Choose one answer.


a. adenomatous polyps.
b. a high intake of dietary nitrates.
c. pernicious anemia.
d. higher socioeconomic groups.
e. chronic atrophic gastritis.

84

Which of the following is required for addressing any pressure sore?

Choose one answer.


a. pressure reduction
b. hyperbaric oxygen
c. antibiotics
d. skin flap coverage

85

The most effective treatment of achalasia is:

Choose one answer.


a. antireflux surgical procedures
b. Esophagomyotomy
c. dilation of the lower esophageal sphincter
d. antispasmodic medication
e. resection of the cardioesophageal junction
86

Regarding desmoid tumors all are true except:

Choose one answer.


a. Treatment is wide local excision with safety margins
b. Has high rate of local recurrence
c. Is a low grade fibro sarcoma
d. More common in females
e. Is radio resistant

87

Which of the following variables best predicts prognosis for patients with a
recent diagnosis of cutaneous melanoma and no clinical evidence of
metastatic disease?

Choose one answer.


a. Ulceration.
b. Clark's level.
c. Gender
d. Bleeding
e. Breslow thickness.

88
In the treatment of acute cholecystitis, most patient are BEST served with:
Choose one answer.
a. endoscopic sphincterotomy.
b. cholecystostomy.
c. percutaneous drainage of the gallbladder.
d. early cholecystectomy (within 3 days of onset of symptoms).
e. IV antibiotics and cholecystectomy in 6 to 8 weeks.

89

Regarding crohn's disease all are true except

Choose one answer.


a. Cause non caseating granuloma
b. commonly affect small bowel
c. Causes sub mucosal fibrosis
d. It is full thickness inflammation
e. Cause depletion of goblet cell mucin

90

Advantages of laparoscopic versus open cholecystectomy include all of the


following EXCEPT:

Choose one answer.


a. decreased pain.
b. less risk of bile duct injury.
c. reduced hospitalization.
d. improved cosmetic.
e. reduced ileus.

91

The most common cause of massive hemorrhage in the lower


gastrointestinal tract is:

Choose one answer.


a. carcinoma
b. Diverticulosis
c. Diverticulitis
d. Polyp
e. ulcerative colitis

92

Carcinoma of the gallbladder is

Choose one answer.


a. rarely associated with jaundice.
b. associated with a good prognosis.
c. usually not diagnosed preoperatively.
d. most commonly metastatic to the lung.
e. best treated with radiation and chemotherapy

93

All the following statements concerning nipple discharges are true


EXCEPT:

Choose one answer.


a. benign duct papillomas are the most common cause of bloody discharges.
b. when bloody , the discharge is due to a malignancy 70% of the time
c. a) excision of involved duct may be necessary to determine the etiology
d. they may be caused by multiple lesions.
e. a milky discharge may be due to a pituitary adenoma .

94

39. A 28-year-old woman presents with a complaint of a growing, painless


mass in her neck. Physical examination reveals a firm, fixed nodule
measuring 2 cm on the right lobe of her thyroid. The surgeon recommends
fine-needle aspiration (FNA) of the lesion instead of excisional biopsy.
What are the advantages of FNA over excisional biopsy?

Choose one answer.


a. FNA is associated with a decreased risk of tumor seeding
b. FNA requires only an office visit
c. Anesthesia is not necessary
d. All of the above

95

For the patient in Question 74, which of the following is a sign of a


colovesical fistula associated with diverticulitis, as seen on computed
tomography with contrast?

Choose one answer.


a. Thickening of the bladder and the colon
b. Sigmoid diverticula
c. All of the above
d. Air in the bladder

96

All of the following are true statements concerning paget ’ s disease of the
nipple EXCEPT:

Choose one answer.


a. it is an eczematoid lesion.
b. it is very uncommon, accounting for only 2% of all breast cancers.
c. it is an in situ squamous cell malignancy of the nipple.
d. it can be confused with malignant melanoma histologically
e. it has a better prognosis than the majority of other breast cancers.
97

The classical picture of Acute arterial embolism include all the following
except:

Choose one answer.


a. peripheral pulses
b. Parasthesia
c. All the above
d. Pain
e. Pallor

98

Extra colonic manifestation of inflammatory bowel disease include all


except:

Choose one answer.


a. Erythema nodosum
b. Mixed connective tissue disease
c. Primary Sclerosing cholangitis
d. Polyarthopathy
e. Episcleritis

99

A 45-year-old woman presents with abdominal pain and vomiting of 1


day's duration. The patient underwent an exploratory laparotomy after a
motor vehicle accident 8 years ago.
Which of the following statements is true for this patient?

Choose one answer.


a. Pain from mechanical obstruction is usually more severe than pain from
ileus
b. The severity of pain from mechanical obstruction may decrease over time
c. Pain from mechanical obstruction is usually localized in the middle of the
abdomen, whereas pain from ileus or pseudo-obstruction is diffuse
d. All of the above

100
A 4-year-old girl has been experiencing dyspnea on exertion. Chest x-ray
shows a Morgagni hernia.
For this patient, which of the following statements is true?

Choose one answer.


a. Morgagni hernias can be repaired with a subcostal, a paramedian, or a
midline incision
b. Morgagni hernias are most commonly seen on the right side
c. All of the above
d. The average age at diagnosis is typically greater for patients with
Morgagni hernia than for patients with Bochdalek hernia

1. A 40-year-old female has a 4-cm hemangioma in the right lobe of the liver on
computed tomography scan. She is asymptomatic. Appropriate action should be:
A. fine-needle biopsy
B. arrangement for elective resection
C. no further action
D. angiographic embolization

2. Regarding an amebic liver abscess:


A. surgical drainage is usually required
B. negative stool testing for amebiasis rules out the disease
C. it should be drained percutaneously under com- puted tomography guidance
D. it is treated with metronidazole

3. A single organism is usually the causative agent in:


A. pelvic in&#64258;ammatory disease
B. perforated diverticulitis
C. acute cholecystitis
D. primary peritonitis
E. diabetic foot infections

4. Signs and symptoms of hemolytic transfusion reactions include


A. Hypothermia
B. Hypertension
C. Polyuria
D. Abnormal bleeding
E. Hypesthesia at the transfusion site

5. In a hemolytic reaction caused by an incompatible blood transfusion, the


treatment that is most likely to be helpful is:
A. Promoting a diuresis with 250 ml of 50% mannitol
B. Treating anuria with fluid and potassium replacement
C. Acidifying the urine to prevent hemoglobin precipitation in the renal tubules
D. Removing foreign bodies, such as Foley catheters, which may cause hemorrhagic
complications
E. Stopping the transfusion immediately

6. Which statements about extrahepatic bile duct cancer are correct?


A. Cholangiography is not essential in evaluating patients for resectability.
B. The prognosis is excellent when appropriate surgical and adjuvant therapy are
given.
C. The location of the tumor determines the type of surgical procedure.
D. The disease usually becomes manifest by moderate to severe right-side upper
quadrant pain.

7. Which of the following statements about the diagnosis of acute calculous


cholecystitis is true?
A. Pain is so frequent that its absence almost precludes the diagnosis.
B. Jaundice is present in a majority of patients.
C. Ultrasonography is the definitive diagnostic test.
D. Cholescintigraphy is not definitive diagnostic test.

8. Which statement about acute acalculous cholecystitis is correct?


A. The disease is often accompanied by or associated with other conditions.
B. The diagnosis is often difficult.
C. The mortality rate is higher than that for acute calculous cholecystitis.
D. The disease has been treated successfully by percutaneous cholecystostomy
E. all are correct

9. Which of the following statements about laparoscopic cholecystectomy are


correct?
A. The procedure is associated with less postoperative pain and earlier return to
normal activity.
B. The incidence of bile duct injury is lower than for open cholecystectomy.
C. Laparoscopic cholecystectomy should be used in asymptomatic patients because it
is safer than open cholecystectomy.
D. Pregnancy is a contraindication.

10. A 15-year-old female presents with RUQ abdominal pain. Workup reveals a
choledochal cyst. Which of the following statements is TRUE?
A. Choledochal cysts are more common in men.
B. Laparoscopic cholecystectomy is the recommended treatment.
C. Patients with a choledochal cyst have an increased risk of cholangiocarcinoma.
D. All patients with a choledochal cyst have abdominal pain, a RUQ mass, and
jaundice.
E. The etiology is infectious.

11. An 85-year-old man is brought to the hospital with a 2-day history of nausea and
vomiting. He has not passed gas or moved his bowels for the last 5 days. Abdominal
films show dilated small bowel, no air in the rectum and air in the biliary tree.
Which of the following statements is TRUE?
A. Air in the biliary tree associated with small-bowel obstruction suggests a diagnosis
of gallstone ileus.
B. An enterotomy should be distal to the site of obstruction and the stone should be
removed.
C. Gallstone ileus is more common in the young adults.
D. Cholecystectomy is contraindicated.
E. Small-bowel obstruction usually occurs in the distal jejunum.

12. A 45-year-old patient with chronic pancreatitis is suffering from malnutrition


and weight loss secondary to inadequate pancreatic exocrine secretions. Which is
TRUE regarding pancreatic secretions?
A. Secretin releases fluid rich in enzymes.
B. Secretin releases fluid rich mainly in electrolytes and bicarbonate.
C. Cholecystokinin releases fluid,predominantly rich in electrolytes, and bicarbonate.
D. All pancreatic enzymes are secreted in an inactive form.
E. The pancreas produces proteolytic enzymes only.

13. A 43-year-old woman has gallstone pancreatitis that resolves in 2 days with
conservative treatment. She has no abdominal complaints and her liver and
pancreatic laboratory values have returned to normal. She is scheduled for
laparoscopic cholecystectomy. Which of the following statements is TRUE?
A. Intraoperative cholangiography is associated with a decreased risk of biliary tract
injury .
B. The procedure should be scheduled for 6 weeks after resolution of symptoms .
C. Intraoperative cholangiography in this patient will identify choledocholithiasis in
50% of cases .
D. Preoperative endoscopic retrograde cholangiopancreatography (ERCP) should be
performed.
E. The sensitivity of magnetic resonance cholangiopancreatography (MRCP) for
choledocholithiasis in this patient is less than 50%.

14. A 39-year-old woman is admitted with gallstone pancreatitis and epigastric pain.
Pertinent data include amylase, 2000 U/L; bilirubin, 1.2 mg/dL; and WBC count,
15,000/mm3 . After 2 days of medical management, her epigastric pain resolves. Her
amylase is 340 U/L and her bilirubin and WBC count have returned to normal.
Laparoscopic cholecystectomy should be attempted:
A. after endoscopic retrograde cholangiopancreatography (ERCP) and sphincterotomy
B. prior to discharge
C. once her amylase is normal
D. 4 to 6 weeks later
E. only if the patient develops recurrent pancreatitis

15. A 48-year-old woman is admitted with acute cholecystitis. The bilirubin level is
elevated, as are the serum and urinary amylase levels. Which radiologic sign
indicates biliary obstruction in pancreatitis?
A. Pancreatic intraductal calcification
B. Smooth narrowing of the distal CBD
C. Stomach displaced anteriorly
D. Calcified gallstone
E. Air in the biliary tree)

16. Following a motor vehicle accident a truck driver complains of severe abdominal
pain. Serum amylase level is markedly increased to 800 U. Grey Turner’s sign is seen
in the flanks. Pancreatic trauma is suspected. Which statement is true of pancreatic
trauma?
A. It is mainly caused by blunt injuries.
B. It is usually an isolated single-organ injury.
C. It often requires a total pancreatectomy.
D. It may easily be overlooked at operation.
E. It is proved by the elevated amylase level.

17. A 73-year-old woman is evaluated for obstructive jaundice after an injury to the
CBD, 7 months previously at laparoscopic cholecystectomy. The alkaline
phosphatase is elevated. In obstructive jaundice, which of the following statements is
true regarding alkaline phosphatase?
A. Its level increases before that of bilirubin.
B. Its level is unlikely to be increased in pancreatic malignancy.
C. Its elevation indicates bone metastasis.
D. Its elevation excludes hepatic metastasis.
E. Its level falls after that of the bilirubin,following surgical intervention.

18. A48-year-old female travel agent presents with jaundice. Radiological findings
confirm the presence of sclerosing cholangitis. She gives a long history of diarrhea
for which she has received steroids on several occasions. She is likely to suffer from
which of the following?
A. Pernicious anemia
B. Ulcerative colitis
C. Celiac disease
D. Liver cirrhosis
E. Crohn’s disease

19. A38-year-old male lawyer develops abdominal pain after having a fatty meal.
Examination reveals tenderness in the right hypochondrium and a positive
Murphy’s sign. Which test is most likely to reveal acute cholecystitis?
A. HIDA scan
B. Oral cholecystogram
C. Intravenous cholangiogram
D. CT scan of the abdomen
E. ERCP

20. A 65-year-old woman is admitted with RUQ pain radiating to the right shoulder,
accompanied by nausea and vomiting. Examination reveals tenderness in the RUQ
and a positive Murphy’s sign. A diagnosis of acute cholecystitis is made. What is the
most likely finding?
A. Serum bilirubin levels may be elevated.
B. Cholelithiasis is present in 40–60%.
C. Bacteria are rarely found at operation.
D. An elevated amylase level excludes this diagnosis.
E. A contracted gallbladder is noted on ultrasound.

21. A 32-year-old diabetic woman who has taken contraceptive pills for 12 years
develops RUQ pain. CT scan of the abdomen reveals a 5-cm hypodense lesion in the
right lobe of the liver consistent with a hepatic adenoma. What should the patient be
advised to do?
A. Undergo excision of the adenoma
B. Stop oral contraceptives only
C. Stop oral hypoglycemic medication
D. Undergo right hepatectomy
E. Have serial CT scans every 6 months

22. A 9 month old boy presents with an acute scrotal swelling. The following
diagnoses are likely:
A. Epididymitis
B. Orchitis
C. Torsion of the testicular appendage
D. Irreducible inguinal hernia
E. Acute idiopathic scrotal oedema

23. A 76-year-old man presents with weight loss, dark urine, and pale stools which
are difficult to flush away. An excess of which of the following would account for this
history?
A. Conjugated bilirubin
B. Hyperbilirubinaemia
C. Stercobilinogen
D. Unconjugated bilirubin
E. Urobilinogen
24. An otherwise well 13-year-old boy is admitted complaining of sudden onset
severe left sided testicular pain 2 hours prior to admission. He gives no history of
trauma, dysuria or frequency. On examination he is found to have a tender, high-
riding testicle.What is the most appropriate next step in this young man’s
management?
A. Herniography
B. Scrotal Doppler ultrasound on the next available list
C. FBC and U&E
D. Scrotal Doppler ultrasound as an emergency
E. Surgical exploration of his scrotum

25. Acute scrotum


A. Torsion testis should be operated within 12 hour of presentation
B. Epidedimoorchitis pain increase by testicular elevation
C. If in doubt scrotum should be explored
D. Doppler ultrasound has no role in diagnosis
E. None of the above

26. stones in the common bile duct:


A. Are present in nearly 50 per cent of cases of cholecystitis.
B. Often give rise to jaundice, fever and biliary colic.
C. Are usually accompanied by progressive jaundice.
D. Are usually associated with a distended gallbladder.
E. A&D only.

27. Which of the following statements regarding whole blood transfusion is correct?
A. Whole blood is the most commonly used red cell preparation for transfusion in the
B. Whole blood is effective in the replacement of acute blood loss.
C. Most blood banks have large supplies of whole blood available.
D. The use of whole blood produces higher rates of disease transmission than the use
of individual component therapies.
E. Old Whole blood is effective in the replacement of platelets.

28. Acute cholecystitis all are true except


A. Commonest bacteria is E .coli
B. Wall thickness more than 3mm by ultrasound
C. WCC is between 10-15 000 cell/mm3
D. Mild elevated bilirubin may accompany it
E. HIDA scan has no role in diagnosis of acute cholecystitis
29. A 51-year-old male experiences the sudden onset of massive emesis of bright red
blood. There have been no prior episodes of hematemesis. He is known to be
hepatitis B surface antigen positive. His hematemesis is most likely a consequence of
which of the following abnormalities of the esophagus?
A. Varices
B. Barrett esophagus
C. Candidiasis
D. Reflux esophagitis
E. Squamous cell carcinoma

30. A 61-year-old male has had ascites for the past year. After a paracentesis with
removal of 1 L of slightly cloudy, serosanguinous fluid, physical examination reveals
a firm, nodular liver.Laboratory findings include positive serum HBsAg and
presence of hepatitis B core antibody. He has a markedly elevated serum alpha-
fetoprotein (AFP) level. Which of the following hepatic lesions is he most likely to
have?
A. Hepatocellular carcinoma
B. Massive hepatocyte necrosis
C. Marked steatosis
D. Wilson disease
E. Autoimmune hepatitis

31. A 76-year-old man presents with weight loss, dark urine, and pale stools which
are difficult to flush away. An excess of which of the following would account for this
history?
A. Conjugated bilirubin
B. Hyperbilirubinaemia
C. Stercobilinogen
D. Unconjugated bilirubin
E. Urobilinogen

32. Acute pancreatitis


A. Serum calcium start to rise after 48 hours
B. Hypoglycaemia is bad prognostic factor
C. Age is an important prognostic factor
D. Serum amylase is more specific than serum lipase
E. Severe pancreatitis compromise around 40% of cases

33. The most commonly used imaging method for diagnosis of acute cholecystitis is:
A. CT of the abdomen.
B. Ultrasonography of the gallbladder.
C. Oral cholecystogram.
D. Radionuclide (HIDA) scan of the gallbladder
E. MRI
34. A 23-year-old male presents to the emergency department after being involved in
a motor vehicle accident. On physical examination, he opens his eyes spontanously,
he occasionally mumbles incomprehensible sounds, he localizes to painful
stimulation with his right upper extremity, His pupils are 4 mm bilaterally and
reactive. This patient’s Glasgow Coma Scale (GCS) score:
A. 7
B. 9
C. 8
D. 11
E. 12

35. Complication of undescended testis include all of the following except :


A. Malignant degeneration.
B. Increased susceptability to trauma.
C. Increased spermatogenesis.
D. More liable to testiculer torsion.
E. Psychological complication
36. Neonatal duodenal obstruction:
A. May be associated with down's syndrome.
B. Is more frequently found in premature infants.
C. Typically presents with gross abdominal distension.
D. Usually presents with vomiting of non-bile stained fluid
E. B&C only.

37. Markedly elevated alpha-fetoprotein is diagnostic


A. Hepatic hemangioma
B. Angiosarcoma in the liver
C. Hepatic adenoma
D. Focal nodular hyperplasia
E. Hepatocellular carcinoma

38. on clinical examination of tortion testis all are true except:


A. testis is tender and swollen.
B. testis is elevated and raised.
C. loss of cremasteric reflex.
D. redness with possible reactive hydrocele.
E. pain decrease with elevation of the testis .

39. Regarding cryptorchidism(undesended testis) all are true except :


A. refers to the interruption of the normal descent of the testis into the scrotum.
B. The testicle may reside in the retroperitoneum, in the internal inguinal ring, in the
inguinal canal, or even at the external ring.
C. At birth, approximately 95% of infants have the testicles normally positioned in the
scrotum.
D. it’s a common disorder and incidence increased up to 30 % in premature .
E. undescended testis is always regarded as an ectopic testis.
40. Regarding GI(gastrointestinal bleeding ) all are true except:
A. Lower GI hemorrhage is defined as an abnormal intra luminal blood loss from a
source distal to the Treitz ligament.
B. The most cause of massive lower GI bleeding in adults are diverticulosis and
angiodysplasia.
C. Cancer colon is usually associated with massive lower GI hemorrhage.
D. Hemorrhage from diverticular disease stops spontaneously in 80% of patients.
E. patients with massive upper GI bleeding may present with maroon stools or bright
red blood from the rectum.

41. Signs of severe bood loss include the following except:


A. Pallor
B. Clammy skin
C. bradycardia
D. Tachycardia
E. Hypotension

42. regarding stigmata of bleeding in peptic ulcer :


A. its associated with increase risks for rebleeding .
B. adherent clot is the most significant stigmata.
C. arterial spurting hemorrhage associated with low risk of rebleeding .
D. risk of rebleeding in ulcer with clean base is above 50%
E. nonbleeding visible vessel is not associated with risk of rebleeding .

43. Features of inflammatory response syndrome (SIRS) include the following


except:
A. Temperature> 38.4C
B. Temperature <36.C
C. WCC<4.ooo cells per ml
D. Respiratory rate >20 per minute
E. PCO2> 32 mmHg

44. The development of thrombocytopenia and arterial thrombosis with heparin


requires:
A. Continuation of heparin and platelet transfusion
B. Continuation of heparin and thrombolysis
C. Doubling the heparin dosage
D. Changing the route of heparin administration
E. Discontinuation of heparin
45. Regarding Heparin-induced thrombocytopenia (HIT) all are true except
A. Is a special case of drug-induced immune thrombocytopenia.
B. The platelet count typically begins to fall 5 to 14 days after heparin has been
started.
C. Thrombocytopenia is usually severe.
D. HIT should be suspected if the platelet count falls to less than 100,000 or if it drops
by 50% from baseline in a patient receiving heparin
E. HIT is more common with full-dose unfractionated heparin (1 to 3%)

46. Regarding gall bladder and bile secretion all are true except :
A. The gallbladder is a pear-shaped, about 7 to 10 cm long with an average capacity
of 30 to 50ml.
B. When obstructed, the gallbladder can distend markedly and contain up to 300 mL
C. Anomalies of the hepatic artery and the cystic artery are quite common, occurring
in as many as 50% of cases.
D. liver produces 500 to 1000 mL of bile a day
E. Vagal stimulation decreases secretion of bile

47. Regarding gall bladder stones all are true except:


A. Prevalence increases with advancing age
B. Over 10% of those with stones in the gallbladder have stones in the common bile
duct.
C. 10-20% become symptomatic
D. cholesterol stones are the most common type.
E. pigment stones are associated with secondary common bile duct stones .

48. In acute cholecystitis all true except :


A. Most common organisms are E. coli
B. 90% cases result from obstruction to the cystic duct by a stone.
C. patient present with constant pain usually greater than 6 hours duration in right
upper quadrant .
D. presence of gall stones and percholecystic fluid on US is diagnostic
E. Cholecystectomy is contraindicated in acute stage .

49. complication of acute cholecystitis include all the following except:


A. Gangrenous cholecystitis
B. Gallbladder perforation
C. Cholecystoenteric fistula
D. mesnteric ischemia
E. Gallstone ileus
50. Regarding choledocholithisis all are true except:
A. may be silent and or may cause obstruction, complete or incomplete
B. may manifest with cholangitis or gallstone pancreatitis.
C. present with severe jaundice and cholangitis in case of stone impaction
D. Rt upper quadrant pain ,fever,and jaundice are called charcots triad in cholangitis.
E. impaction of small stones has no relation with acute pancreatitis .

51. Regarding acalculous cholecystistis all are true except:


A. Acute inflammation of the gallbladder can occur without gallstones
B. Acalculous cholecystitis typically develops in critically ill patients in the intensive
care unit.
C. Patients on parenteral nutrition with extensive burns, sepsis, major operations are
at risk for developing acalculous cholecystitis.
D. US is not a good diagnostic tool.
E. can be managed by cholecystectomy or percutanous cholecyststomy .

52. Regardind gall bladder cancer all are true except:


A. Larger stones (>3 cm) are associated with a 10-fold increased risk of cancer.
B. up to 95% of patients with carcinoma of the gallbladder have gallstones.
C. Polypoid lesions of the gallbladder are not associated with increased risk of cancer
D. Patients with choledochal cysts have an increased risk of developing cancer
E. Sclerosing cholangitis is risk factor for developing gall bladder cancer .

53. Regarding tumors of the liver all are true except :


A. Hemangioma is the most common solid benign lesion .
B. Spontaneous rupture in hemangioma (bleeding) is rare.
C. Hepatic adenomas carry a significant risk of spontaneous rupture with
intraperitoneal bleeding.
D. Hepatic adenomas have a risk of malignant transformation to a well-differentiated
HCC(hepatocellular carcinoma).
E. focal nodular hyperplasia lesions(FNH) lesions usually rupture spontaneously and
have significant risk of malignant transformation .

54. Regarding pyogenic liver abscess all are true except :


A. arise as a result of biliary sepsis.
B. associated with high mortality.
C. appendicitis is unlikely the cause pyogenic liver abscess.
D. 30% of patient have pleural effusion on presentation .
E. lab.investiation show elevated WBC and abnormal liver function .

55. Regarding acute pancreatitis all are true except:


A. Gallstones less than 5mm diameter are more likely to cause pancreatitis than larger
ones
B. The mortality associated with infected necrosis is about 40%
C. Cullen's sign is a sign of retroperitoneal hemorrhage in severe hemorrhagic
pancreatitis .
D. elevated serum amylase is a significant predictor of severity .
E. 50% of deaths occur within first week due to multi-organ failure .

56. All occur as a complication of acute pancreatitis except:


A. panreatic fluid collection
B. colonic necrosis
C. coagulopathy
D. hypercalcemia
E. respiratory failure

57. Regarding head trauma all are true except:


A. basal skull fracture regarded when one of the orbital roof ,sphenoidal bone or
petromastoid portion are involved .
B. epidural hematoma is an Lens shape hematoma between dura and the skull.
C. subdural hematoma is crescent shaped hematoma ,between brain and dura
D. secondary brain injury is preventable.
E. GCS glascow Coma scale 3/15 indicate uncomprehensive sounds .

58. Regarding chronic lower limb ischemia all are true except:
A. Claudication distance is distance after which the pain is felt.
B. Rest pain is continous severe burning pain in the foot which indicate critical
ischemia.
C. trophic changes include tapering digits ( loss of S.C fat ) and muscle wasting .
D. usuall presentations of patients with lower limb ischemia are pain,trophic changes
and gangrene.
E. venous filling time more than 2 minutes indicates mild lower limb ischemia.

59. Regarding 4 weeks 4 Kg bodywt. old full term neonate presented with rapidly
progressive projectile non bilious vomiting and palpable upper abdominal mass all
are true except :
A. dehydration and alkalosis are prominent features.
B. maintenance fluid therapy is about 4ml /Kg /hour.
C. administration of IV fluids with 5% dextrose, 0.5% normal saline, and KCl usually
corrects the alkalosis .
D. Estimated total blood volume is about 320 cc.
E. the most likely diagnosis is high jejunal atresia
60. All are true regarding jejunoileal atresia except:
A. present with bile stained vomiting .
B. Failure to pass meconium or small amounts of mucus or meconium maybe passed
per rectum.
C. present with abdominal distention.
D. X-ray show double bubble appearance .
E. The x-rays usually show multiple air-fluid levels.

61. regarding malrotation of the gut all are true except:


A. The patient might be asymptomatic and then develop the symptoms when he is
older.
B. Commonest abnormality results in caecum lying close to DJ flexure.
C. Fibrous bands may be present between caecum and DJ flexure (Ladd's bands).
D. the patient is unlikely to have clinical picture of duodenal obstruction .
E. In malrotation midgut mesentery is abnormally narrow and liable to volvulus.

1 C 11 A 21 A 31 A 41 C 51 D 61 D
2 D 12 B 22 D 32 C 42 A 52 C
3 D 13 A 23 A 33 B 43 E 53 E
4 D 14 B 24 E 34 E 44 E 54 C
5 E 15 B 25 C 35 C 45 C 55 D
6 C 16 D 26 B 36 A 46 E 56 C
7 A 17 A 27 B 37 E 47 E 57 E
8 E 18 B 28 E 38 E 48 E 58 E
9 A 19 A 29 A 39 E 49 D 59 E
10 C 20 A 30 A 40 C 50 E 60 D

1. Which hypersensitivity reaction is associated with a tuberculin reaction?


A. Type I: immediate
B. Type II: cytotoxic
C. Type III: immune complex
D. Type IV: cell mediated

2. The most common location for a gastric ulcer is


A. Fundus
B. Greater curvature
C. Cardia
D. Body
E. Antrum

3. Regarding the management of major trauma


A. Deaths follow a trimodal distribution
B. Cardiac tamponade is characterised by raised BP, low JVP and muffled heart
sounds
C. Assessment of uncomplicated limb fractures should occur during the primary
survey
D. Deterioration of the casualty during the primary survey should lead to the
secondary survey
E. All are false

4. All of the following are true about neurogenic shock except:


A. There is a decrease in systemic vascular resistance and an increase in venous
capacitance.
B. Tachycardia or bradycardia may be observed, along with hypotension.
C. The use of an alpha agonist such as phenylephrine is the mainstay of treatment.
D. Severe head injury, spinal cord injury, and high spinal anesthesia may all cause
neurogenic shock.

5. The following cause hypercalcaemia except :


A. Sarcoidosis
B. Primary hyperparathyroidism
C. Acute pancreatitis
D. Metastatic bronchial carcinoma
E. Milk-Alkali syndrome

6. For a 40-kg baby the maintenance daily fluid requirement is approximately


which of the following?
A. 1100 ml
B. 1250 ml
C. 1550 ml
D. 1700 ml
E. 2000 ml

7. Infantile hypertrophic pyloric stenosis


A. Occurs with a male: female ratio of 4:1.
B. Presents between six and eight months of age
C. Typically presents with bile stained projectile vomiting
D. Surgical treatment is by Heller's Cardiomyotomy
E. all are false

8. Which of the following do you consider to be the most important clinical sign in
acute appendicitis,
A. Abdominal tenderness around the umbilicus
B. Abdominal tenderness in the RIF
C. Tenderness over McBurney’s point
D. Rovsing’s sign positive
E. Suprapubic tenderness

9. Heparin
A. Acts as an anti-platelet
B. Acts as an anti-thromboplastin
C. Acts as an antithrombin
D. All of the above
E. None of the above

10. All of the following are mechanisms of urinary calculi formation except,
A. Hypoparathyroidism
B. Prolonged recumbency
C. Infection with urea-splitting organisms
D. Foreign body
E. Urinary tract obstruction

11. Which of the following are not found in peritonitis?


A. Patient is lying still
B. Guarding
C. Rebound tenderness
D. Hyperactive bowel sounds
E. Rigid abdomen

12. Which one of the following suggest a diagnosis of Hirschsprung's disease?


A. A contrast-study showing dilatation of the aganglionic bowel segment.
B. Early presentation with vomiting.
C. Neonatal large bowel obstruction.
D. Presentation after 1 year of age.
E. Red current jelly stools.

13. Which of the following regarding the anatomy of the heart is true?
A. The aortic valve is tricuspid.
B. The ascending aorta is entirely outside the pericardial sac.
C. The left atrial appendage is identified readily by transthoracic echocardiography.
D. The pulmonary trunk lies anterior to the ascending aorta.
E. The right atrium is posterior to the left atrium.
14. Which of the following is true concerning Scaphoid fractures?
A. Rarely occur in young adults
B. when complicated by avascular necrosis the proximal pole is usually affected

C. should be treated by bone grafting and internal fixation even if undisplaced


D. wrist fractures are uncommon
E. anteriorposterior and lateral radiographs reveal most fractures

15. Which of the following statements is true of upper limb nerve injuries?
A. Injury to the median nerve results in a wrist drop
B. Injury to the radial nerve results in loss of sensation over the palmar aspect of the
index finger
C. Injury to the median nerve results in loss of sensation in the anatomical snuffbox
D. Injury to the ulnar nerve results in a claw hand
E. Injury to the ulnar nerve results in loss of sensation over the thumb

16. Regarding intravenous solutions:


A. Normal saline contains 180mmol/l of sodium
B. Ringer's lactate solutions is designed for intracellular fluid replacement
C. Sodium bicarbonate 8.4% is a hyperosmolar solution
D. Normal saline with added potassium is appropriate therapy to correct a respiratory
alkalosis
E. All are True

17. Which of the following concerning the Femoral sheath is false:


A. Contains the femoral artery
B. Contains lymph nodes
C. Contains the femoral canal
D. Contains femoral nerve
E. Contains the femoral vein

18. The following is true of the spleen:


A. Is the largest lymphoid organ in the body
B. Lies obliquely between the seventh and tenth rib
C. The lower pole extends beyond the mid-axillary line
D. Is usually palpable when normal
E. Usually measures 16cm in maximum length when healthy

19. Breast cancer risk is increased in association with the following factors except:
A. Nulliparity
B. Immediately after pregnancy
C. Early menarche
D. Early age at first pregnancy
E. Late menopause
20. In tension pneumothorax the following signs are present except:
A. Hypoxia
B. Hyperresonance to percussion on the affected side
C. Tracheal deviation to the ipsilateral side
D. Distended neck veins
E. Tachycardia
21. The most common hernia in females is:
A. Femoral hernia.
B. Direct inguinal hernia.
C. Indirect inguinal hernia.
D. Obturator hernia.
E. Umbilical hernia.

22. The most helpful diagnostic radiographic procedure in small bowel obstruction
is:
A. CT of the abdomen.
B. Contrast study of the intestine.
C. Supine and erect x-rays of the abdomen.
D. Ultrasonography of the abdomen.
E. MRI Abdomen

23. In role of nine extent of burn if entire trunk is burned it will be equal to:
A. 9% body surface area.
B. 18% body surface area.
C. 36% body surface area.
D. 27% body surface area.
E. 45% body surface area.

24. If torsion of the testicle is suspected, surgical exploration:


A. Can be delayed 24 hours and limited to the affected side.
B. Can be delayed but should include the asymptomatic side.
C. Should be immediate and limited to the affected side.
D. Should be immediate and include the asymptomatic side.

25. Hyperthyroidism can be caused by all of the following except:


A. Graves' disease.
B. Plummer's disease.
C. Struma ovarii.
D. Hashimoto's disease.
E. Medullary carcinoma of the thyroid.

26. A 9 month old boy presents with an acute scrotal swelling. The following
diagnoses are likely:
A. Epididymitis
B. Orchitis
C. Torsion of the testicular appendage
D. Irreducible inguinal hernia
E. Acute idiopathic scrotal oedema

27. The evaluation of a comatose patient with a head injury begins with:
A. The cardiovascular system.
B. Pupillary reflexes.
C. Establishment of an airway.
D. Computed tomography (CT) of the brain
E. insertion of Intravenous canula
28. The following is an indication for thoracotomy in chest injury,
A. Cardiac tamponade
B. Uncontrolled pulmonary air leakage
C. Perforation of thoracic esophagus
D. Blood loss of 200ml/hr for 2-3 hrs via chest tube
E. All of the above

29. Regarding Gallstones all of the following are true except:-


A. Prevalence increases with advancing age
B. 30% of gallstones are radio-opaque
C. Cholesterol stones result from a change in solubility of bile constituents
D. Biliary infection, stasis and changes in gallbladder function can precipitate stone
formation
E. Gram-negative organisms are the most common isolated

30. In obstructive jaundice:


A. Urinary conjugated bilirubin is increased
B. Serum unconjugated bilirubin is increased
C. Urinary urobilinogen increased
D. Serum conjugated bilirubin is reduced
E. Faecal stercobilinogen is increased

31. The ideal fluid therapy in a patient with pyloric stenosis and repeated vomiting
is:
A. Isotonic crystaloid containing sodium chloride
B. Hypertonic crystaloid containing dextrose – saline
C. Isotonic solution containing dextrose
D. Large molecular weight colloid containing dextran

32. Which of the following do you consider to be the most important clinical sign in
acute appendicitis,
A. Abdominal tenderness around the umbilicus
B. Abdominal tenderness in the RIF
C. Tenderness over McBurney’s point
D. Rovsing’s sign positive
E. Suprapubic tenderness

33. All of the following are mechanisms of urinary calculi formation except,
A. Hypoparathyroidism
B. Prolonged recumbency
C. Infection with urea-splitting organisms
D. Foreign body
E. Urinary tract obstruction

34. Which of the following is incorrect in Paget’s disease of nipple,


A. Presents with eczema and redness of the nipple and areola
B. Tumour arises from epidermal layer of nipple and not from epidermal layer of
underlying ducts
C. Underlying carcinoma may be present beneath the nipple
D. Mastectomy is considered the treatment of choice
E. It is considered carcinoma-in-situ

35. Organisms most commonly isolated in UTIs,


A. Kleb
B. E. Coli
C. Proteus
D. Acinetobacter
E. All of the above

36. Excessive saliva in a newborn is due to


A. Esophageal atresia
B. Salivary gland tumour
C. Primary Hypertrophic Pyloric Stenosis
D. Midgut volvulus
E. Hirschprung’s disease

37. The most frequent congenital diaphragmatic hernia seen in infants is,
A. Paraesophageal hernia
B. Sliding hernia
C. Congenitally short esophagus
D. Hernia through the foramen of Bochdalek
E. Hernia through the foramen of Morgagni

38. An undescended testis.


A. Can be associated with contralateral testicular dysfunction
B. Should be treated by orchidopexy by age 2
C. Is associated with inguinal hernia
D. Predisposes to malignant change
E. All of the above

39. The wound made to remove a perforated appendix is classified as,


A. Clean
B. Clean, contaminated
C. Contaminated
D. Dirty
E. Doesn’t apply as the wound isn’t a traumatic one

40. Concerning the treatment of haemorrhoids,


A. Can be by diet
B. By injection sclerotherapy
C. By banding
D. Is mainly by haemorrhoidectomy
E. All of the above

41. The following is an indication for thoracotomy in chest injury,


A. Cardiac tamponade
B. Uncontrolled pulmonary air leakage
C. Perforation of thoracic esophagus
D. Blood loss of 200ml/hr for 2-3 hrs via chest tube
E. All of the above
42. In an upper GI bleed, in some cases a Sengstaken-Blakemore tube is incerted.
What is the primary purpose of the tube?
A. Aspirate blood from the stomach
B. Tube feeding
C. Tamponade for varices
D. To decompress bowel
E. All of the above

43. Risk factors for pulmonary embolism do not include which of the following?
A. DVT
B. Recent surgery
C. Old age
D. Myocardial infarction
E. Chest infection

44. Which of the following is not found in peritonitis?


A. Patient is lying still
B. Guarding
C. Rebound tenderness
D. Hyperactive bowel sounds
E. Rigid abdomen
45. Commonest site for CA breast
A. Upper outer quadrant
B. Upper inner quadrant
C. Lower outer quadrant
D. Lower inner quadrant
E. None of the above

46. The most common site of fracture of the clavicle is:


A. Medial end.
B. Lateral end.
C. Midpoint of the clavicle.
D. Junction of the medial two-thirds and the lateral third.
E. Junction of the lateral two-thirds and the medial third.

47. The back of the medial epicondyle is related to the:


A. Radial nerve.
B. Axillary nerve.
C. Ulnar nerve.
D. Median nerve.
E. None of the above.

48. Regarding Gallstones all of the following are true except:-


A. Prevalence increases with advancing age
B. 30% of gallstones are radio-opaque
C. Cholesterol stones result from a change in solubility of bile constituents
D. Biliary infection, stasis and changes in gallbladder function can precipitate stone
formation
E. Gram-negative organisms are the most common isolated

49. Clinical features of limb ischaemia includes


A. Paraesthesia
B. Pallor
C. Pulselessness
D. Paralysis
E. All of the above

50. Regarding Hydatid disease all of the following are true except:-
A. Due to infection with the helminth Ecchinococcus granulosa
B. Man is an accidental intermediate host
C. Lunges is the commonest organ involved
D. Diagnosis can be confirmed by indirect haemagglutinin assay
E. Aspiration should not be performed if hydatid disease is suspected

1 D 11 D 21 C 31 A 41 E
2 E 12 C 22 C 32 C 42 C
3 A 13 A 23 C 33 A 43 E
4 C 14 B 24 D 34 A 44 D
5 C 15 D 25 D 35 B 45 A
6 E 16 C 26 D 36 A 46 D
7 A 17 D 27 C 37 D 47 C
8 C 18 A 28 E 38 E 48 B
9 C 19 D 29 B 39 B 49 E
10 A 20 C 30 A 40 E 50 C

28.A midline neck mass, located between the thyroid bone and suprasternal notch that
moves upward when the tongue is stuck out most likely is:
a. thyroglossal duct cyst-

29.A smooth cystic neck mass located along the anterior border of the
sternocleidomastoid muscle most likely is:
a. brachial cleft cyst -

30.A lymphatic neck cyst located in the posterior triangle just above the clavicle most
likely is:
a. cystic hygroma-

65.Battle's sign of basilar skull fracture refers to blue-gray discoloration:


a. behind the mastoid process =
118.The most common complication of thyroidectomy is:
a. hypoparathyroidism =

What is the most common tumor of the appendix?


a. carcinoid =

123.The classical triad for a ruptured abdominal aneurysm includes a pulsatile abdominal
aortic mass, back pain, and:
a. jaundice
b. vomiting
c. headache

126.Sigmoid volvulus should be initially treated with:


a. decompression=

133.Compression of the oculomotor (3rd) nerve by an aneurysm, most likely from:


a. posterior communicating artery=

210.The most common complication of a major hip fracture is:


a. hemorrhage =

211.The most common complication of dislocation of the knee is:


a. popliteal artery injury =

247.Which of the following is the most common indication for major surgery in females?
a. uterine leiomyomas (fibroids)=

300.Schatzki's ring is associated with:


a. hiatal hernia =

311.Benign bone tumors are not painful EXCEPT for:


a. endochondroma
b. osteoid osteoma -
c. giant cell tumors
d. unicameral bone cysts

567.Osteoid osteomas:
a. are malignant tumors
b. generally occur after 50 years of age
c. cause progressive localized ache -
d. occur in females more commonly than in males

312.Ewing's tumor is a:
a. malignant sarcoma-
329.A pseudocyst following an acute pancreatitis does NOT:
a. rupture
b. form an abscess
c. resolve spontaneously -
d. cause an internal hemorrhage

330.Which of the following suggests bad prognosis in a 58 year old man with acute
pancreatitis
a. total bilirubin of 1.0 mg/dL
b. blood glucose of 240 mg/Dl-
c. serum amylase of 120 U/L
d. white blood cell count of 14,000/mm3

568.The most common malignant tumor of the spine is:


a. multiple myeloma -

332.In trauma patients, which of the following should be treated first?


a. flail chest-
b. pleural effusion
c. laceration of the liver
d. fractured tibia and ulna

334.12 year old male complains of painful defecation, the most likely cause is:
a. anal cancer
b. anal abscess
c. anal fissure -
d. internal hemorrhoids

336.Following a major surgery, a 68 year old man develops massive colonic dilatation
and constipation, the most likely cause is:
a. paralytic ileus
b. toxic megacolon
c. fulminating colitis
d. Ogilvie's syndrome (pseudo-obstruction) -

353.The most common surgical procedure performed in patients with sickle cell anemia
is:
a. splenectomy
b. gastrectomy
c. appendectomy
d. cholecystectomy-

366.Which of the following is NOT a risk factor for colorectal cancer?


a. Turner's syndrome-
b. Turcot's syndrome
c. Gardner's syndrome
d. Oldfield's syndrome

507.Kohler disease is osteochondrosis of the:


a. navicular-

508.Sever disease is osteochondrosis of the:


a. apophysis of the os calis -

511.(Blade of grass) or (flame like) lesion on long bone X-ray and osteoporosis
circumscripta are features of:
a. Paget's disease of the bone -

524.Serum alkaline phosphatase is normal in:


a. osteoporosis -
b. osteomalacia
c. primary hyperparathyroidism

525.Bone X-ray shows pseudofractures (Looser's zones) in:


a. osteomalacia-

526.The characteristic dorsal kyphosis and cervical lordosis (dowager's hump) are
features of:

a. osteoporosis-

527.Following are secondary causes of osteoporosis, EXCEPT:


a. obesity -
b. anorexia nervosa
c. Marfan's syndrome
d. hyperprolactinemia

528.72 year old woman presents with back pain, X-ray shows compression fractures and
kyphosis in the spine, the most likely diagnosis is:
a. osteoporosis-

529.Back pain with increased urinary hydroxyproline and serum alkaline phosphatase in a
70 year old man is most likely due to:
a. Paget's disease of the bone -

530.Following are laboratory abnormalities seen patients with Paget's disease, EXCEPT
a. hypocalcemia -
b. hyperuricemia
c. increased urinary hydroxyproline
d. increased serum alkaline phosphatase
535.An acute upper GI bleeding from longitudinal esophageal tears after repeated
vomiting occurs in:
a. Mallory-Weiss syndrome -

548.Breast cancer my occur in women who:


a. are overweight -
b. had 5 or more children
c. went in menopause at the age of 38
d. had their first menstruation at the age of 20

549.Definitive diagnosis of breast cancer is made by:


a. X-ray
b. CT scan
c. mammogram
d. histological examination-

558.Bright red blood on the surface of the feces with pain during defecation in an 8 year
old boy is most likely due to:
a. anal fissure -

560.The best management for a ganglion tumor is:


a. surgery -

570.64 year old man complains of nocturia, dribbling and urinary hesitancy, the most
likely diagnosis is:
a. benign prostatic hypertrophy-

407.The most common site of bone metastasis from prostate cancer is:
a. pelvis-

408.The most common indication for surgery in regional enteritis is:


a. fistula
b. abscess
c. bleeding
d. obstruction-

413.Anal fissure is associated with:


a. ischemic colitis
b. Crohn's disease -
c. ulcerative colitis
d. irritable bowel syndrome

415.2 year old child has hematuria, hypertension and a palpable mass in the left flank, the
most likely diagnosis is:
a. Wilms' tumor-
416.Intussusception is characterized by:
a. projectile vomiting
b. currant-jelly stools -
c. olive-shaped mass in upper abdomen
d. all of the above

418.Which of the following types of kidney stones is radiotransparent?


a. uric acid -

423.The most common cause of hypercalcemia in a 32 year old woman is:


a. primary hyperparathyroidism-

424.What type of breast cancer has the worst prognosis?


a. Lobular-
b. medullary
c. inflammatory
d. infiltrating ductal

430.Which of the following is NOT a risk factor for osteoporosis?


a. Multiparity-
b. lack of exercise
c. low calcium intake
d. excessive alcohol intake

440.A transilluminated, soft mass in the posterior triangle just above the clavicle of an
infant is most likely due to:
a. cystic hygroma-

441.What percentage of congenital diaphragmatic hernias in neonates are located on the


left side?
a. 85% -

446.The most common bacteria found in wound infections is:


a. staphylococcus-

459.The most common cause of trochlear nerve palsy is:


a. trauma -

480.Diagnosis of Meckel's diverticulum is made by:


a. technetium scan -

493.What percentage of abdominal aortic aneurysms are diagnosed while they are still
asymptomatic?
a. 75% -
495.36 year old man presents with acute abdomen due to small bowel obstruction, the
most likely cause is:
a. adhesions -

496.The mortality rate for colonic obstruction is about:


a. 20%-

Ewing sarcoma most commonly occur in:


a. femur and tibia-

Tachypnea, cyanosis, dyspnea, and severe acidosis within few hours after birth is most
likely due to:
a. diaphragmatic hernia--

Renal biopsy is contraindicated in the presence of:


a. solitary or ectopic kidney --

(coffee-bean) shape of dilated bowel loop on plain X-ray and (ace of spades) shape on
barium enema are typical of:
a. volvulus --

957.Massive lower gastrointestinal bleeding in elderly patients is due to:


a. diverticulosis
b. angiodysplasia
c. both--

.A fragment of fractured bone pulled off by ligament or tendon at its attachment, this is
called:
a. avulsion --

964.A break with 3 or more fragments of a fractured bone is called:


a. comminuted--

968.Approximately what percentage of patients with cirrhosis have esophageal varices?


a. 50%--

933.Paralysis of abduction and adduction of all fingers is due to injured:


a. ulnar nerve --
b. radial nerve
c. median nerve

Pain, paresthesia, and numbness over the bottom of the foot at night is most likely due to:
a. tarsal tunnel syndrome --

862.The most common bone tumor in a 64 year old man is:


a. metastatic tumor--

873.The most common glenohumeral dislocation is:


a. anteroinferior--

32 year old woman presents with a solitary thyroid nodule, the best management is:
a. fine-needle aspiration--

816.What is the most common cause of primary hyperparathyroidism?


a. single adenoma --

820.Carpal tunnel syndrome is LEAST likely to be a feature of:


a. leukemia
b. myxedema
c. hypocalcemia
d. Parkinson's disease --

62 year old man is unable to fully extend his right 4th finger, the most likely diagnosis is:
a. Dupuytren's contracture --

32 year old man presents with a painful nodule on the dorsum of the hand, the
most likely diagnosis is:

a. ganglion-

703.A man presents with a painful encapsulated nodule near the nail, the most likely
diagnosis is:
a. glomus tumor-

Which of the following is NOT included in the four P's of compartment syndrome?

a. pallor
b. paralysis
c. penetration-
d. pulselessness

789.26 year old man presents with swelling around the margin of the nail plate of his
right index, the most likely diagnosis is:
a. paronychia -

754.62 year old man presents with sudden onset of severe pain in his left leg, the leg is
pale and the pulse is absent, the most likely diagnosis is:
a. arterial embolism -

One-third of individuals with inflammatory involvement of the terminal ileum


have:
a. gallstones -

736.The first and the most common sign/s of obstruction of extrahepatic system is/are:
a. progressive jaundice -

716.Osteoporosis is characterized by increased:


a. serum alkaline phosphatase
b. serum parathyroid hormone
c. serum calcium and phosphorus
d. the rate of the bone resorption -

717.Which of the following is NOT a risk factor for osteoporosis?


a. heparin therapy
b. copper deficiency
c. Marfan's syndrome
d. vitamin A deficiency -

718.Osteomalacia is characterized by:


a. bone pain
b. proximal muscle weakness
c. both -

719.Paget's disease of the bone is characterized by increased serum:


a. calcium
b. alkaline phosphatase-

What percentage of gallstones are symptomatic?

a. 30% -

1016. Following are predisposing factors for colorectal cancer, EXCEPT:


a. 50 years of age or older
b. consumption of vegetables
c. upper socioeconomic group

d. consumption of calories and meat protein

1019. Hepatocellular adenomas are benign tumors of the liver found


predominantly in females in their:
a. 3rd and 4th decades

b.
1020. Hepatocellular carcinoma is up to four times more common in men than
in women and with the peak incidence in the:
a. 5th and 6th decades
1021. Patients with severe abdominal pain often obtain relief by sitting with
the trunk flexed and knees drawn up in:
a. acute pancreatitis
1022. Paget's disease (osteitis deformans) is characterized by increased:
a. serum calcium
b. serum phosphate
c. both

d. neither
1023. Acute cholecystitis is characterized by the triad of sudden onset of right
upper quadrant tenderness, fever, and:
a. leukocytosis

b.
1024. Which of the following bone tumors is the most common one in a 16
year old boy?
a. osteosarcoma

b.
. Few days after coronary artery bypass surgery, a 74 year old man develops
melena, the most likely diagnosis is:
a. ischemic colitis

b.

a.

a.

1044. A patient is unable to oppose his thumb to the base of the little finger, which of
the following nerves is most likely to be injured?
a. median

b.
1069.The most accurate method for the diagnosis of thrombophlebitis of the deep veins
is:
a. ascending contrast venography

1070.The most common cause of septic thrombophlebitis is:


a. staphylococcus

1071.The most common initial presentation of chronic venous insufficiency is:


a. pain
b. edema
c. itching
d. hyperpigmentation

1152. Approximately what percentage of acute cholecystitis is NOT associated with


gallstones?
a. 10%

1153. 62 year old man has a metastatic renal cancer, the most likely source is the cancer
of:
a. lung
b. colon
c. liver

d. prostate
1159. Erysipelas is usually caused by:
a. group A beta-hemolytic streptococci
1173. 28 year old woman presents with breast pain, the most likely diagnosis is:
a. fibrocystic disease

46 year old woman presents with bullae and scarring on the dorsum of her
1174.
hands, the most likely diagnosis is:
a. porphyria cutanea tarda
1195. How long after surgery, wound infections classically become apparent?
a. 4 to 7 days

1032. Romberg How ship sign is associated with:


a. obturator hernia

1033. Double bubble sign on X-ray is associated with:


a. annular pancreas
b. duodenal atresia
c. intestinal malrotation

d. all of the above


A woman has phlebitis of the superficial veins in the outer quadrants of the
1219.
breast, the likeliest diagnosis is:
a. Paget's disease
b. Mondor's disease =
c. plasma cell mastitis

d. fibrocystic adenoma
1220. Patients with portal hypertension are LEAST likely to have:
a. hemorrhoids
b. facial edema =
c. caput medusa

d. Budd-Chiari syndrome
Caput medusae (abnormal wall collaterals increased in size, radiating from
1221.
the umbilicus) is a feature of:
a. portal hypertension=

Following are important diagnostic aids for peripheral arterial diseases, EXCEPT:
a. ultrasound
b. thermography =
c. plethysmography

d. plain X-ray studies


The (five P's) of acute arterial occlusion are pain, pallor, and all of the following, EXCEPT:
a. pulseless
b. paralysis
c. paresthesia

d. propagation ==
Buerger's disease is an inflammatory disease that involves:
a. veins
b. arteries
c. both =

The (gold standard) of the diagnosis of deep venous thrombosis (DVT) is:
a. plethysmograph
b. contrast venogram =
c. doppler ultrasound

d. radionuclide venogram
The most reason women bring their varicose veins to the attention of the physician is:
a. leg pain
b. leg swelling
c. purely cosmetic =

d. statsis dermatitis
What percentage of patients with proved pulmonary emboli will have demonstrable
deep venous thrombosis?
a. 95% =
Dilated veins on chest wall with swelling of neck and upper extremities in a 56 year old
man is most likely due to:
a. filariasis
b. lung cancer =
c. Buerger's disease

d. abdominal aortic aneurysm

The most decisive way to confirm the diagnosis of splenic rupture is:
a. plain X-ray
b. ultrasound
c. exploratory laparotomy=

d. CT scan of the abdomen


An abrupt cutoff of colnic gas in the region of the splenic flexure (colon cutoff sing) is
associated with:
a. diverticulosis
b. Crohn's disease
c. chronic pancreatitis

d. sever hemorrhagic pacreatitis =


The most common cause of a lower abdominal mass in males over 50 years of age is:
a. Hodgkin's disease
b. colon cancer
c. retroperitoneal cysts

d. bladder obstructed by benign prostatic hypertrophy =


An elderly male presents with urinary obstruction, the chance of having a prostatic
cancer is:
a. 20% =

Rupture of the posterior prostatic urethra is associated with:


a. scrotal swelling
b. fractures of the pelvis =
c. dripping of blood from urethra

d. he matoma under Buck's facia


What percentage of polyps and carcinomas of the large intestine arise within reach of
the sigmoidoscope?
a. 60% =
What is the chance for patients with diverticulosis of developing diverticulitis?
a. 1 in 6 =

The most common type of polyp of the large bowel is:


a. tubular =
b. villous
c. juvenile

d. Peutz-Jeghers

Injury to nerve root C6 (Intervertebral level C5-C6) causes motor deficit in:
a. Biceps=

Injury to nerve root C7 (Intervertebral level C6-C7) causes motor deficit in:
a. triceps =

Testicular tumors:

a. are benign in most cases


b. constitute 25% of all male malignant tumors
c. are choriocarcinomas in most cases

d. should be removed if malignancy suspected =


Complication of acute appendicitis
a. hemorrhage
b. intestinal obstruction
c. malignant transformation

d. rupture and periappendicual abscess =


Tylosis is associated with carcinoma of:

a. lung
b. colon
c. esophagus =

d. pancreas
749.Whipple's disease is diagnosed by:
a. jejunal biopsy -

Management of a cold thyroid nodule does NOT include:


a. ultrasound
b. needle biopsy
c. surgical excision
d. injection of thyroxine into the nodule -
314.Epulis is a swelling of:
a. gums-

49. Paralysis of the serratus anterior cause:


a. winging of the scapula

A femoral hernia with only a portion of bowel passed through hernia ring is called:
a. incarcerated hernia
b. Richter's hernia =
c. incisional hernia

d. strangulated hernia
Cryptorchidism is almost invariably associated with:
a. indirect inguinal hernia =
Which of the following pancreatic islet cells secret serotonin?
a. enterchromaffin cells=

Patients with necrotizing migratory erythema are most likely to have:


a. glucagonoma =

14 year old boy complains of aching in the front of the knee after exercise,
1242.
the most likely diagnosis is:
a. Osgood-Schlatter disease (tibial epiphysitis) =
1244. Regarding Ewing's tumor, which of the following in NOT true?
a. it is radiosensitive
b. usually occurs in ages 1 to 4 years =
c. usually affects diaphysis of the long bones

d. most commonly affects tibia, fibula, humerus and fermur

1. Which of the following statements about open fractures are correct?


A. Intravenous antibiotics should be administered as soon as possible.
B. They should be regarded as an emergency.
C. Wound closure is necessary within 8 hours.
D. Systematic wound débridement and irrigation should be performed.

Answer: C

2. The goals of proper fracture reduction include which of the following?


A. Providing patient comfort and analgesia.
B. Allowing for restoration of length of the extremity.
C. Correcting angular deformity and rotation.
D. Enabling immediate motion of all fractured extremities.

Answer: ABC
6. The neurovascular structure most commonly injured as a result of an anterior
dislocation of the shoulder is the:
A. Musculocutaneous nerve.
B. Axillary nerve.
C. Axillary artery.
D. Median nerve.

Answer: B

7. The classification of fractures of the proximal humerus is based on:


A. The number of fracture segments and amount of displacement.
B. The mechanism of injury.
C. Presence or absence of associated dislocations.

Answer: A
8. The radial nerve is at greatest risk for injury with which fracture?
A. Fracture of the surgical neck of the humerus.
B. Fracture of the shaft of the humerus.
C. Supracondylar fracture of the humerus.
D. Olecranon fractures.

Answer: B

9. The best method of treating a supracondylar fracture of the humerus in a child


that is unstable when the elbow is flexed to 90 degrees is:
A. Hyperflexion of the elbow to 130 degrees and casting.
B. Open reduction and internal fixation.
C. Percutaneous pinning.
Answer: C

10. Both-bone forearm fractures in adults are best managed by:


A. Closed reduction and casting.
B. Closed reduction and application of an external fixator.
C. Open reduction and placement of intramedullary rods.
D. Open reduction and internal fixation with compression plates.

Answer: D

11. The most consistent sign of a fracture of the carpal scaphoid is:
A. Wrist pain during attempted push-ups.
B. Diffuse swelling on the dorsum of the wrist.
C. Localized tenderness in the anatomic snuffbox.
D. Wrist popping on movement.
Answer: C

12. A patient describes a fall on the outstretched hand during sports activities.
Multiple radiographic views show no distinct fracture. He is tender to palpation in
the anatomic snuffbox. The most suitable method of management is:
A. Diagnose “sprained wrist” and apply an elastic bandage.
B. Diagnose suspected scaphoid fracture and apply a short-arm cast to include
the thumb.
C. Apply a canvas wrist splint for immobilization.
D. Prescribe salicylates and permit continued activity.

Answer: B

13. Median nerve compression syndrome in which the patient has motor
weakness of the flexor pollicis longus and the flexor digitorum profundus of the
index finger without alteration in sensibility is due to:
A. Compression of the median nerve at the elbow by the lacertus fibrosus.
B. Compression of the median nerve in the axilla.
C. Compression of the anterior interosseous nerve by the arcade of Frohse.
D. Compression of the anterior interosseous nerve by an aberrant accessory
forearm muscle.
Answer: D

15. The most common physical findings in a patient with median nerve
compression at the wrist (carpal tunnel syndrome) are:
C. A positive percussion test at the wrist and a positive wrist flexion test
producing paresthesias at the thumb, index, and
long fingers.

16. Which of the following describes the most desirable position in which to
immobilize the hand?
E. Wrist is extended, MCP joints are flexed, and IP joints are extended.

17. An early sign of compartment syndrome in the hand includes:


A. Pain with passive stretch of the digits.
B. Absent radial pulse.
C. Motor paralysis.
D. Swelling of the digits.
E. Stiffness of the digits.

Answer: A

18. Palmar dislocation of the PIP joint with fracture:


D. If not splinted properly, will cause a boutonniere deformity.

19. Fracture of the fifth metacarpal neck:


C. Is called a “boxer's fracture.”

20. A Bennett's fracture is:


B. Displaced by the pull of the abductor pollicis longus and adductor pollicis.

27. Prognosis of healing in tibial fractures correlates best with:


A. Energy absorption at the time of fracture.
B. Amount of soft tissue damage.
C. Location of the fracture (i.e., in the proximal, middle, or distal third).
D. Age of patient.

Answer: A

28. Management of a III-b tibia fracture is best treated initially by:


A. Plaster immobilization.
B. Immediate plating.
C. Reamed intramedullary nailing.
D. External fixation.

Answer: D

29. The most frequent forces acting on the foot that cause ankle fractures are:
A. External rotation.
B. Internal rotation.
C. Plantar flexion.
D. Dorsiflexion.

Answer: A

30. Patients who have abduction injuries to the foot are prone to injure the
following structures:
A. Medial malleolus and deltoid.
B. Lateral malleolus and deltoid ligament.
C. Interosseous ligament.
D. Posterior tibiofibular ligament.

Answer: A

31. Of the following bones in the foot, the tarsal bone that is most prone to
vascular compromise is the:
A. Calcaneus.
B. Navicular.
C. Talus.
D. Cuboid.

Answer: C

32. A Lisfranc fracture is a fracture-dislocation involving:


B. Tarsometarsal joint.

33. The most common reason for surgical amputation in the general population
is:
E. Ischemia.

36. Hematogenous osteomyelitis most frequently affects:


A. The diaphysis of long bones.
B. The epiphysis.
C. The metaphysis of long bones.
D. Flat bones.
Answer: C

47. The zone of flexor tendon injury that carries the poorest prognosis following
injury and repair is:
B. Zone II.

53. The most important structural component of connective tissue is collagen.


Which of the following statement(s) is/are
true concerning types of collagen? ex
a. All collagen is fiber forming
b. Type 1 collagen is the most abundant in the human body
c. Type 2 collagen is found in cartilage
d. The basement membrane collagens, type 4 and 5, do not form regular fibers

Answer: b, c, d
57. Which of the following statement(s) is/are true concerning bone remodeling?
ex
a. Remodeling can occur only on the surface of trabeculi
b. The remodeling process takes approximately 120 days in an adult
c. Trabecular bone remodeling occurs up to 10 times faster than cortical bone
remodeling
d. Bone modeling involves bone formation without resorption

Answer: b, c, d

121. Which of the following conditions is considered to increase the risk


of gastric cancer? ex
a. Pernicious anemia
b. Prior partial gastrectomy
c. Gastric hyperplastic polyps
d. Gastric adenomatous polyps

Answer: a, b, d

1. Which of the following statements about the anatomic course of the


esophagus is correct?
E. The esophagus deviates anteriorly and to the left as it enters the
.abdomen

2. Which of the following statements about esophageal anatomy is


correct?
C. The esophagus has two distinct muscle layers, an outer, longitudinal
one and an inner, circular one, which are striated in
.the upper third and smooth in the distal two thirds

6. Which of the following statements about achalasia is/are correct?


D. Manometry demonstrates failure of LES relaxation on swallowing and
absent or weak simultaneous contractions in the
esophageal body after swallowing.

8. Which of the following statements about epiphrenic diverticula of the


esophagus is/are correct?
E. The operation of choice is a stapled diverticulectomy, long
esophagomyotomy, and partial fundoplication

9. Which of the following statements about Schatzki's ring is correct?


.B. Dysphagia occurs when the ring diameter is 13 mm. or less

13. Which of the following statements regarding the pathology of


esophageal carcinoma is/are correct?
C. Patients with Barrett's metaplasia are 40 times more likely than the
.general population to develop adenocarcinoma
15. The best management for a 48-hour-old distal esophageal
perforation is:
A. Antibiotics and drainage.
B. Division of the esophagus and exclusion of the perforation.
C. Primary repair with buttressing.
E. T-tube fistula and drainage.

Answer: C

16. A 50-year-old patient develops sudden left lower chest pain and
epigastric pain after vomiting. The patient shows
diaphoresis, breath sounds are decreased on the left, and there is
abdominal guarding. The most appropriate diagnostic test is:
A. Aortography.
B. Esophagoscopy.
C. Electrocardiogram.
D. Film of the chest.
E. White blood count.

Answer: D
20. The presence of a nonmalignant mid- or upper esophageal stricture
always indicates the presence of:
A. Alkaline reflux esophagitis.
B. Barrett's esophagus.
C. Idiopathic reflux disease.
D. Mediastinal fibrosis.

Answer: B

21. Which of the following is most reliable for confirming the occurrence
of a significant esophageal caustic injury?
A. History of the event.
B. Physical examination of the patient.
C. Barium esophagraphy.
D. Endoscopy.

Answer: D
22. Indications for surgical reconstruction of the esophagus include
which of the following? ex
A. Continuing requirement for frequent dilation of an extensive
esophageal stricture for a minimum of 2 years.
B. Failure or refusal of the patient to comply with a treatment regimen of
regular dilation.
C. Development of a fistula between the esophagus and tracheobronchial
tree.
D. Iatrogenic perforation of the esophagus during attempted dilation.

Answer: BCD

23. First-line therapy for routine peptic duodenal ulcer disease includes:
A. Vagotomy and antrectomy.
B. Upper endoscopy and biopsy to rule out tumor.
C. Evaluation for Helicobacter pylori.
D. Serum gastrin determination.

Answer: C

24. Appropriate management of severe vomiting associated with gastric


outlet obstruction from peptic ulcer disease
includes all of the following except:
A. Nasogastric suction.
B. Intravenous hydration.
C. Nutritional assessment; upper endoscopy to rule out malignancy.
D. Intravenous H 2 antagonist.
E. Oral antacid therapy.

Answer: E

25. All of the following are complications of peptic ulcer surgery except:
A. Duodenal stump blowout.
B. Dumping.
C. Diarrhea.
D. Delayed gastric emptying.
E. Steatorrhea.

Answer: E

26. The presentation of Zollinger-Ellison syndrome includes all of the


following except:
A. Hyperparathyroidism in patients with multiple endocrine neoplasia
type 1 (MEN 1) syndrome.
B. Diarrhea.
C. Migratory rash.
D. Jejunal ulcers.
E. Duodenal ulcers.

Answer: C

27. All are true about the dumping syndrome except:


A. Symptoms can be controlled with a somatostatin analog.
B. Diarrhea is always part of the dumping syndrome.
C. Flushing and tachycardia are common features of the syndrome.
D. Separating solids and liquids in the patient's oral intake alleviates
some of the symptoms of the syndrome.
E. Early postoperative dumping after vagotomy often resolves
spontaneously.

Answer: B
28. In patients with bleeding duodenal ulcers, the endoscopic finding
associated with the highest incidence of
rebleeding is:
A. Visible vessel.
B. Cherry-red spot.
C. Clean ulcer bed.
D. Duodenitis.
E. Shallow, 3-mm. ulcer.

Answer: A

29. All of the following are contraindications for highly selective


vagotomy except:
A. Intractable duodenal ulcer disease.
B. Peptic ulcer disease causing gastric outlet obstruction.
C. Fundic peptic ulceration.
D. Cigarette chain smoking.
E. Perforated peptic ulcer disease with more than 24 hours' soilage.

Answer: A
30. All the following are true of omeprazole except:
A. It is the only drug available that has the potential to achieve
pharmacologically induced achlorhydria.
B. It works by blocking the hydrogen-potassium ATPase in the parietal
cell.
C. It is parietal cell specific.
D. It has a short half-life (about 90 minutes) when taken orally.
E. It has been associated with gastric neoplasm in a rat model.

Answer: D

31. All of the following statements about gastrin-releasing peptide (GRP)


are true except:
A. In species other than man and dog GRP is commonly referred to as
bombesin.
B. GRP serves as a neurotransmitter.
C. GRP inhibits pancreatic secretion when given intravenously.
D. GRP stimulates gastric acid secretion when given intravenously.
E. GRP is released in response to cholinergic stimulation of the
.parietal cells to stimulate release of gastrin
Answer: C

32. Cholecystokinin (CCK) is believed to function in all of the following


processes except:
A. It physiologically delays gastric emptying.
B. It appears to have a role in satiety regulation.
C. It contracts the gallbladder.
D. It stimulates pancreatic secretion.
E. It is important in the control of the anal sphincter.

Answer: E

33. All of the following measures have been recommended for control of
acid secretion in patients with Zollinger-
Ellison syndrome except:
A. Antrectomy.
B. Highly selective vagotomy.
C. Total gastrectomy.
D. Vagotomy and pyloroplasty.
E. Medical therapy with Prilosec (omeprazole).

Answer: A
34. All of the following contribute to peptic ulcer disease except:
A. Cigarette smoking.
B. Nonsteroidal anti-inflammatory drugs.
C. Helicobacter pylori.
D. Gastrinoma.
E. Spicy foods.

Answer: E

35. Which of the following statements about gastric polyps is/are true?
A. Like their colonic counterparts, gastric epithelial polyps are common
tumors.
B. They are analogous to colorectal polyps in natural history.
C. Endoscopy can uniformly predict the histology of a polyp based on
location and appearance.
D. In a given patient, multiple polyps are generally of a single histologic
type.
E. Gastric adenomatous polyps greater than 2 cm. in diameter should be
excised because of the risk of malignant
transformation.

Answer: DE
36. Which of the following statements about gastric leiomyomas is/are
true?
A. They are the most common type of gastric tumor of the stomach at
autopsy.
B. The leiomyoblastoma cell type reflects malignant transformation of
gastric leiomyomas.
C. A conservative surgical approach is indicated for their resection since
regional lymphadenectomy has not been proved
reliable even when they turn out to be malignant.
D. Severe hemorrhage may occur from deep ulcerations overlying the
intramural tumor.

Answer: ACD

39. Which of the following risk factors have been shown to increase
significantly the incidence of gastrointestinal
bleeding from stress gastritis in intensive care unit (ICU) patients?
A. Glucocorticoid administration.
B. Respiratory failure.
C. Coagulopathy.
D. Organ transplantation.
E. Jaundice.

Answer: BC
43. Numerous epidemiologic associations have been made between (1)
environmental and dietary factors and (2) the
incidence of gastric cancer, including all except:
A. Dietary nitrites.
B. Dietary salt.
C. Helicobacter pylori infection.
D. Dietary ascorbic acid.

Answer: D

44. All of the following benign conditions are associated with increased
rates of gastric cancer except:
A. Pernicious anemia.
B. Multiple endocrine neoplasia type I (MEN 1).
C. Adenomatous polyps.
D. Chronic atrophic gastritis.

Answer: B
45. Which of the following statements concerning the pathology of
gastric cancer is true?
A. Distal gastric cancers are becoming more common.
B. Intestinal-type gastric tumors resemble colon carcinomas and have a
better prognosis than diffuse type.
C. Early gastric cancers are confined to the mucosa and are lymph node
negative.
D. Broders' histologic grading system correlates well with survival:
patients with grade IV tumors have 5-year survival
rates around 65%.

Answer: B

46. An 80% distal gastrectomy is performed for a 6-cm. antral cancer


with extension to the muscularis propria and
three positive lymph nodes less than 3 cm. from the tumor. The stage of
this tumor was:
A. Stage I.
B. Stage II.
C. Stage III A.
D. Stage III B.

Answer: B
47. Which of the following statements about the surgical treatment of
gastric cancer is false?
A. Patients with tumors of the middle and proximal thirds should
undergo total gastrectomy.
B. Adenocarcinoma of the cardia-gastroesophageal junction may require
reconstruction in the abdomen, chest, or neck.
C. Palliative resection yields better results than palliative bypass.
D. Japanese patients who undergo gastric resection are, on average, 10
years younger and much leaner than their Western
counterparts.

Answer: A

51. Which of the following is/are contraindications to gastric bypass


surgery?
A. Diabetes mellitus.
B. Hypertension.
C. Pickwickian syndrome.
D. Failure to agree to long-term follow-up.
E. Sleep apnea.

Answer: D
53. Metabolic complications of subtotal gastrectomy with Billroth I or
Billroth II reconstruction include: ex
A. Hypothyroidism.
B. Anemia.
C. Reactive hypoglycemia.
D. Dumping syndrome.
E. Metabolic bone disease.

Answer: BCDE

62. Which of the following statement(s) concerning pharyngoesophageal


disorders is/are true? ex
a. In neuromuscular diseases, dysphagia is often worse for liquids than
for solids
b. Cricomyotomy may be indicated for a wide variety of neuromuscular
disorders involving the pharyngoesophageal
phase of swallowing
c. Excision of a Zenker’s diverticulum is indicated to prevent malignant
change in the sac
d. Complications of all operations on the cervical esophagus include
hematoma formation and recurrent nerve
paralysis

Answer: a, b, d
67. Which of the following statement(s) is/are true concerning the
surgical anatomy of the esophagus?
b. Spontaneous esophageal perforation tends to be associated with
leakage into the left chest

79. Which of the following statement(s) is/are true concerning caustic


injury to the esophagus?
a. Alkaline injury is more destructive than acid injury
b. Acid ingestion is not injurious to the stomach due to its normal acidic
pH
c. Ingested caustic agents rapidly pass through the esophagus and
stomach into the small intestine
d. Unless perforation occurs, clinical manifestations resolve quickly with
initial clinical improvement noted
e. Children are less likely to form a late esophageal stricture than adults

Answer: a, d, e
85. At a cellular level, the major stimulant(s) of acid secretion by the
gastric parietal cell is/are:
a. Histamine
b. Prostaglandin E2
c. Acetylcholine
d. Gastrin
e. Norepinephrine

Answer: a, c, d

86. Which of the following statement(s) regarding the vagus nerves


is/are true?
a. The right and left vagus nerves derive from a nerve plexus inferior to
the tracheal bifurcation
b. The posterior vagus nerve is closely applied to the intrathoracic
esophagus
c. The anterior vagus supplies a hepatic division which passes to the
right in the lesser omentum
d. Approximately 90% of vagal fibers are afferent, transmitting
information from the gastrointestinal tract to the
central nervous system
e. The vagus nerves transmit gastroduodenal pain sensations associated
with peptic ulceration

Answer: a, c, d
87. Important stimulants of gastrin release from endocrine cells in the
antrum include:
a. Acidification of the antral lumen
b. Small peptide fragments and amino acids from luminal proteolysis
c. Locally released somatostatin
d. Dietary fats

Answer: b

88. Which of the following statements regarding human gastric acid


secretion is/are true?
a. Fasting acid secretion, normally 2 to 5 mEq/h, is due to ambient vagal
tone and histamine secretion
b. Truncal vagotomy decreases basal secretion by 80%
c. Histamine2 receptor antagonist administration can decrease basal
acid secretion by 80%
d. Fasting acid secretion, normally 5 to 10 mEq/h, is due to circulating
levels of gastrin

Answer: a, b, c
93. Which of the following statements regarding intrinsic factor is/are
correct?
a. Intrinsic factor is produced in chief cells located in the gastric fundus
b. Total gastrectomy is followed by folate deficiency due to vitamin
malabsorption secondary to intrinsic factor
deficiency
c. Intrinsic factor secretion, like that of acid, is stimulated by gastrin,
histamine, and acetylcholine
d. Intrinsic factor deficiency accompanies H pylori-caused antral
gastritis

Answer: c

94. A 24-year-old woman develops epigastric pain and has a diagnosis of


duodenal ulcer confirmed by
esophagogastroduodenoscopy. The patient is in the third month of a
pregnancy. The most appropriate treatment would
be:
a. Proximal gastric vagotomy
b. Misoprostol 400 mg b.i.d.
c. Sucralfate 1 gm q.i.d.
d. Cimetidine 400 mg b.i.d.

Answer: c
104. Which of the following clinical circumstances have been identified
as predisposing factors for the development of
stress ulceration?
a. Intraperitoneal sepsis
b. Hemorrhagic shock
c. Isolated tibial fracture
d. 50% total surface area second degree burn
e. Adult respiratory distress syndrome

Answer: a, b, d, e

107. With regard to benign gastric ulceration, the most common location
of disease is which of the following?
a. Along the greater curvature
b. Immediately distal to the esophagogastric junction along the lesser
curvature
c. In the area of the incisura angularis along the lesser curvature
d. Within the gastric antrum

Answer: c

110. Agents demonstrated to have an efficacy of greater than 90% for


prophylactic treatment of stress ulceration include
which of the following?
a. Antacids
b. H2 receptor antagonists
c. Sucralfate
d. Misoprostil

Answer: a, b, c

1. You are called to see an elderly multi-trauma patient who has a known
history of significant coronary artery disease, atrial fibrillation, and mild heart failure.
He is hypotensive, his abdomen is distended, the FAST reveals a large amount of fluid
around the spleen, and there is an open fracture of the femur.
Which of the following will be most beneficial in his early treatment?
a. Norepinephrine to increase his blood pressure
b. Increasing the afterload to raise the blood pressure immediately
c. Cardioversion of the chronic atrial fibrillation to restore the “atrial kick”
d. Rapid infusion of 20 mL/kg of Ringer’s lactate
e. Beta-blockade to control excessive ventricular rates that can cause hypotension
2. You are treating a septic patient who is on high-dose Levophed for blood pressure
support. His PCWPis 20 mm Hg, with the most recent systemic vascular resistance
about 2600. Which of the following is the appropriate change to make?
a. Add a beta1-agonist
b. Add a beta2-blocker
c. Add an alpha1-agonist
d. Add an alpha2-blocker
e. Add epinephrine or vasopressin
3. You have just inserted a Swan-Ganz catheter and cannot be sure that, with the
balloon inflated, the tip is actually wedged. You recall that in the wedged position the
blood gas on the specimen obtained by drawing back on the line after discarding the
first 5 to 10 cc of blood should be in the range of:
a. pO2 >60
b. pO2 <60
c. pCO2 >60
d. pCO2 <30
e. pH >7.6
4. You indeed were in the wedged position and now obtain the following data. Your
patient’s systemic pressure is 90/40, the heart rate is 120, and the urine output is
scant. The RA pressure is 2, the RV pressure is 18/2, the PA pressure is 18/4, and the
wedged pressure is 4. The cardiac output is 1.8 L, and the systemic vascular
resistance is 2300. The likely cause of shock in this patient is:
a. Fluid overload
b. Pulmonary embolism
c. Cardiac tamponade
d. Hypovolemia
e. Neurogenic shock
5. Your patient with known coronary artery disease is in refractory shock in the
recovery room. He suffered from an acute myocardial infarction during a recent
colectomy. The cardiologist wants to do an emergency cardiac cath to determine
whether there is a blockage amenable to stenting. The cardiologist determined that a
balloon pump was needed for interim support and was inserting the percutaneous
balloon catheter when he had to leave for another emergency and asked that you
finish setting the time for balloon inflation. How can you tell when balloon inflation
is too early or too late?
a. Look at the ECG, time by the T wave
b. Look at the ECG, time by the QRS
c. Look at the arterial waveform proximal to the balloon site
d. Look at the arterial waveform distal to the balloon site
e. Check the cardiac output serially
6. How many of the following will increase venous return in patients on mechanical
ventilation?
 Small tidal volumes
 Brief inspiratory time
 Limiting PEEP to the minimal to maintain satisfactory oxygenation
a. All
b. 2
c. 1
d. None
7. All of the following are contraindications for the use of controlled hypercapnia
except:
a. CNS problems, trauma, tumor
b. Severe HTN
c. Severe metabolic alkalosis
d. Hypovolemia
e. Severe refractory hypoxia
8. Toxicity of tacrolimus includes all of the following EXCEPT:
a. Nephrotoxicity
b. Hypertension, headaches, and vasospasm
c. Alopecia
d. Hyperlipidemia
e. Poor wound healing
9. Which of the following cells are NOT active in innate immunity?
a. T cells and B cells
b. NK cells
c. Neutrophils
d. Mast cells
e. Macrophages
10. Macrophage mediators include all of the following EXCEPT:
a. Complement components
b. Proteolytic enzymes
c. Regulatory mediators – IL-1, TNF-, IL-6
d. Lysozyme and hydrolases
e. Antibodies to cells and organs
11. T cells recognize antigens through all but one of the following:
a. Direct binding to the antigen
b. Recognizing a peptide fragment of an antigen bound to an MHC molecule
c. Presentation by a professional dendritic cell
d. Presentation by a non-professional endothelial cell
e. Presentation by an endothelial cell
12. Immune cells communicate by all the following methods except for:
a. Direct cell-to-cell contact
b. Soluble factors chemoattractant cytokines
c. Combination of cell-to-cell contact and a cytokine
d. Contact with cell surface molecules, which can convert into soluble mediators
e. Use of an immunosuppressive drug
13. How often does identical HLA phenotype occur among siblings?
a. 25% of the time
b. 50% of the time
c. 75% of the time
d. 100% of the time
e. Occurs only in identical twins
14. A 28-year-old man is seen on a renal transplant consultation with his mother,
father, 3 brothers, and 1
sister. All are willing to donate a kidney, all are blood group compatible, and all are
very healthy.
Which one should be selected as a donor?
a. Mother who is a 1 haplotype match
b. Sister who is a 2 haplotype match
c. Brother who is a 1 haplotype match
d. Brother who is a 0 haplotype match
e. Father who is a 1 haplotype match
15. A 45-year-old man receives a deceased donor kidney and, 2 days following
operation, has a minimal urinary output of 10 cc/hr and still requires hemodialysis.
He has a normal duplex ultrasound of the kidney. The most likely diagnosis is:
a. Renal artery stenosis
b. Recurrent focal segmental glomerulosclerosis
c. Acute tubular necrosis
d. Hyperacute rejection
e. Tacrolimus nephrotoxicity
16. You are called to see a youngster who was playing baseball earlier that day and
was struck on the left side of his head by a line drive. He reportedly was knocked
down but got up shortly thereafter and continued to play saying he was “OK.” On the
way home from the game, his parents thought he should be checked when he said his
head was hurting. He lapsed into coma shortly thereafter. On examination, you find a
“fixed and dilated left pupil.” What is the most likely cause of this abnormality?
a. The youngster has a glass eye following prior enucleation for ocular melanoma
b. The youngster has an expanding contusion of the brain in the oculomotor cortex
c. The youngster has acute hydrocephaly
d. The youngster has an acute subdural hematoma on the side opposite the injury
e. The youngster has an acute epidural hematoma on the side of the injury
17. You are covering your surgical group for the weekend and are called to see your
partner’s
postoperative total thyroidectomy case, done for medullary carcinoma. The patient’s
wife is
concerned because the patient is “very hoarse.” He was scheduled for discharge by
your partner before he left for the weekend. On examination, he is breathing well but
is aphonic. What is the likely cause?
a. I do not know what “aphonic” means so guess “B” and go to the next question
b. The patient is having a conversion reaction
c. The patient has unilateral acute vocal cord paralysis, side to be determined
d. The patient has bilateral vocal cord paralysis
e. This scenario is corrupt. He likely has bilateral vocal cord paralysis but should
have been
stridorous, so the scenario is impossible.
18. You are evaluating your trauma patient who has undergone a stent placement for
an acute rupture of the aorta just distal to the subclavian artery. Pleased with the
“save,” you note that his legs are weak and that he has lost pinprick sensation
bilaterally yet has maintained proprioception. He likely has sustained an ischemic
insult and now has:
a. Brown-Sequard syndrome
b. Anterior cord syndrome
c. Central cord syndrome
d. Expected changes post surgery, open or closed
e. Acute reaction due to excessive administration of IV contrast material
19. Your patient with BRACA1-associated breast cancer has had a bilateral
mastectomy done
prophylactically. On in-office follow-up, she is concerned that her right shoulder
blade seems “weird” since the operation. You note that indeed she has a “winged
scapula” on the same side. The most likely cause is operative injury to the:
a. Vagus nerve, ipsilateral
b. Spinal accessory nerve
c. Ipsilateral long thoracic nerve
d. Ipsilateral thoracodorsal nerve
20. From what nerve does the “criminal nerve of Grassi" arise?
a. The greater splanchnic nerve
b. The lesser splanchnic nerve
c. The left vagus nerve
d. The right vagus nerve
21. You are evaluating a jaundiced man who was found to have a “replaced right
hepatic artery” on
angiography. What does that mean?
a. He has no arterial blood flow to the right lobe of the liver
b. He has double arteries perfusing the right lobe of the liver
c. He has no blood flow to the left lobe of the liver
d. The right hepatic artery arises from the SMA
e. The right hepatic artery arises from the IMA
22. The most common nonatherosclerotic cause of carotid stenosis is:
a. Fibromuscular dysplasia
b. Carotid dissection
c. Giant cell arteritis
d. Takayasu’s arteritis
23. The most appropriate management for a young, otherwise healthy patient with
claudication due to popliteal entrapment syndrome is:
a. Exercise regimen
b. Exercise regimen with cilostazol (Pletal®)
c. Transluminal angioplasty and stent placement
d. Interposition vein graft with division of medial head of the gastrocnemius muscle
24. The earliest form of atherosclerotic damage to peripheral arteries is:
a. Fatty streaks
b. Intermediate lesions
c. Prototypical plaque
d. Thrombosed plaque
25. The ankle-brachial index:
a. Is reliable in patients with severely calcified arteries
b. Should never be >1.0
c. Can be used to screen for coronary artery disease
d. Is an invasive test
e. Must be combined with treadmill exercise to be useful
26. Regarding swallowing:
a. Swallowing is not affected by the recurrent laryngeal nerve
b. A sleeve resection of the esophagus should not be performed as this disrupts the
propulsion of the esophagus
c. A large hiatal hernia can disrupt propulsion in the esophagus
d. Tertiary peristalsis is important in clearing residual material in the esophagus
e. All of the above are true
27. In regard to the anatomy of the stomach:
a. The anatomic beginning of the antrum is the angularis incisura
b. The criminal nerve of Grassi is near the pylorus
c. By finding the vagus nerve more distally on the esophagus, the surgeon is more
likely to
divide all the fibers in a vagotomy
d. The consistently largest artery to the stomach is the right gastric
e. The gastroepiploic arteries run along the lesser curvature of the stomach
28. Regarding acid secretion:
a. Acid secretion occurs when the H+/K+ ATPase enzyme is activated
b. Because PPIs irreversibly interfere with the enzyme associated with the proton
pump,
function doesn’t return when these medications are stopped
c. Somatostatin is a powerful stimulant of acid secretion
d. The intestinal phase accounts for the majority of acid secretion
e. All of the above
29. In terms of the small intestine:
a. The strength layer of the bowel is the muscularis mucosa
b. The strength layer of the small intestine is the serosa
c. The proximal intestine has a larger circumference
d. Peyer’s patches are most prominent in the duodenum
e. Short bowel syndrome is likely to occur if <300 cm of small bowel remains after
resection
30. Which is NOT true regarding anal continence?
a. Internal hemorrhoids contribute to anal continence
b. The external sphincter supplies the majority of the resting tone
c. The puborectalis muscle is important in anal continence
d. Decreased rectal compliance compromises continence
e. The pudendal nerve innervates the internal and external sphincters
31. CO2 pneumoperitoneum may require the anesthesiologist to alter the ventilator
settings by
a. Decreasing the rate
b. Increasing the rate
c. Increasing oxygen content
d. Increasing the isoflurane
e. All of the above
32. All of the following are a result of increasing the intra-abdominal pressure
except:
a. Increased SVR
b. Increased CI
c. Increased MAP
d. Increased CVP and PCWP
e. Increased PVR
33. Studies have shown that, after laparoscopy, there is LESS of a systemic rise for all
of the following
EXCEPT:
a. IL-6
b. TNF
c. ACTH
d. WBC
e. None of the above
34. A 35-year-old woman who is 22 weeks pregnant presents with a new palpable
mass in the UOQ of the right breast. The mass is 2.5 cm in size and feels benign in
nature. She has no palpable axillary nodes.
Ultrasound revealed the following:
Which of the following is the best next step in management?
a. Delay management until after delivery
b. Needle biopsy
c. Excisional biopsy
d. Bilateral mammogram
e. Breast MRI
35. In regard to the anus:
a. Has an anal transition zone that is composed of columnar epithelium
b. The hemorrhoidal plexus is synonymous with the inferior rectal vein
c. Patients with rectal varices have an increased risk for hemorrhoids
d. Melanomas of the anus may require an abdominoperineal resection
e. External hemorrhoids should not be treated with banding
36. Incidental appendectomy is recommended:
a. In individuals with Crohn disease when the cecum is free of disease
b. In children about to undergo chemotherapy
c. During Ladd’s procedure
d. In patients aged <25 years
e. All of the above
37. Laparoscopic appendectomy is NOT recommended in which of the following
situations?
a. Appendiceal mucocele
b. Perforated appendicitis
c. Obese patients
d. Patients with previous abdominal surgery
e. Elderly patients
38. In patients with significant rectal bleeding:
a. Anoscopy is not recommended, as significant bleeding rarely results from
hemorrhoidal
bleeding
b. Angiography is more sensitive than tagged red blood cell scan for detecting
bleeding
c. Tagged red blood cell scanning is more accurate in localizing rectal bleeding
d. Patients who complain of rectal bleeding should have a FOBT performed to
confirm
e. Angioembolization may be used for intestinal bleeding localized with angiography
39. In patients with diverticular disease:
a. Resection should include all obvious diverticula
b. When operating urgently, a sigmoid resection with end colostomy and Hartmann’s
pouch
should always be performed
c. Surgery should be avoided in immunosuppressed patients because of increased
complications with surgery
d. Diverticular abscesses do not respond to antibiotics
e. Most patients with uncomplicated diverticulitis will respond to outpatient therapy
with
broad-spectrum oral antibiotics
40. Advantages of CT colonography (virtual colonoscopy) include:
a. Less patient discomfort
b. Avoidance of a bowel preparation
c. Avoidance of insufflation and risk of perforation
d. Similar sensitivity to colonoscopy for lesions >1 cm
41. Which is true regarding PEGs?
a. Local wound infections can be decreased by administering pre-procedure
antibiotics
b. Gastrocolic fistulas do not occur, as the stomach is remote from the colon
c. Closure of the gastrostomy tube generally requires operative takedown
d. Local infections usually require removal of the gastrostomy tube for resolution
e. The crossbar should be tightened as much as possible to avoid separation of the
stomach
from the abdominal wall
42. Colonic polyps:
a. Should never be excised piecemeal, as this makes pathologic interpretation
difficult
b. Perforation following colonoscopy may not be evident for several days
c. Chromoendoscopy is routinely used for detection of ischemia
d. Argon plasma coagulation is not safe in the colon, as it has a significant depth of
penetration
e. Perforation following colonoscopy always mandates operative exploration
43. Regarding ERCP:
a. It is preferred in pregnancy to IOC as it doesn’t involve ionizing radiation
b. It includes evaluation of the esophagus and stomach for pathology
c. The minor papilla is proximal to the major papilla
d. Plastic stents are commonly used for malignant strictures
e. Metal stents are commonly used for benign strictures
44. The Lap-Band® procedure is associated with:
a. Excessive weight loss over time
b. Many metabolic problems
c. Low operative morbidity
d. Late stenoses
45. Comparing open to laparoscopic bariatric procedures, it is evident that:
a. The open operative approach is both more time consuming and more expensive to
perform
b. The laparoscopic approach is superior in patient comfort postoperatively
c. Ghrelin infusions shorten the length of stay
d. All of the above
e. None of the above
46. Following bariatric surgery, patients show:
a. No weight loss for the first 3 months
b. Rapid weight gain ONLY if the stomach staples fall out
c. Improvement in sensitivity to insulin
d. A frequent fatal “gas bloat” syndrome
e. None of the above
47. Bariatric surgery is now being performed on unemancipated minors age 13 to 16
years. The operative
consent should be signed, noting that:
a. All bariatric surgery is experimental and carries potentially lethal but unknown
risks
b. Parents or guardians lose their rights because they allowed their child to become
morbidly
obese, thus they are guilty of Munchausen syndrome by proxy
c. No signature of the teenager is advisable because the patient is underage
d. The teenager gave consent along with the responsible parent or guardian giving
consent
e. The teenager gave assent along with the responsible parent or guardian giving
consent
48. You have read an article on a study of a new drug treatment for Crohn disease. It
reports that the drug
was found to be effective, with a “P value” ≤0.05. This means that
a. The drug worked 95% of the time
b. If the drug did not really work, you still could end up with rejection of the null
hypothesis
5% of the time. In this case, the findings of the study would represent a “false
positive”
c. The study is underpowered 5% of the time
d. 95% of research is accurate
e. You have a beta error 5% of the time
49. A 66-year-old diabetic has been on a ventilator for 5 days following surgery for
peritonitis due to a perforated appendix. She remains nasotracheally intubated.
Initially febrile, she defervesced on POD 2 but now has spiked a fever to 39°C. The
chest x-ray remains clear, and the abdominal exam is unremarkable. A CT of the
sinuses revealed a left maxillary sinusitis. The most likely causative organism is:
a. Staphylococcus aureus
b. Enterococcus fecalis
c. Escherichia coli
d. Klebsiella pneumoniae
e. Pseudomonas aeruginosa
50. You have performed a total thyroidectomy for a large papillary carcinoma of the
thyroid, and the PGY1 resident who scrubbed on the case tells you that the patient
has a positive Chvostek sign in the recovery room. He is concerned that the patient is
hypocalcemic. Which of the following statements is most correct?
a. The Chvostek sign is rarely present in normocalcemic patients
b. The Chvostek sign is usually present in normocalcemic patients
c. The Chvostek sign is highly sensitive for hypocalcemia
d. The Chvostek sign is highly specific for hypocalcemia
e. The Chvostek sign is neither highly sensitive nor highly specific for hypocalcemia
51. A 72-year-old man presents with a parotid mass. Imaging studies confirm that
the mass is in the superficial lobe of the gland. Fine-needle aspiration of the mass is:
a. Useful in treatment planning
b. Likely to give a correct tissue diagnosis of tumor type in 95% of cases
c. Contraindicated
d. Likely to spread the disease to the lymph glands in 5% of cases
e. More dangerous than a core-needle biopsy
52. Following thyroid surgery, your patient is noted to be aphonic in the recovery
room. The most likely cause is:
a. Post-intubation edema of the vocal cords
b. Injury to the right recurrent laryngeal nerve
c. Injury to the left recurrent laryngeal nerve
d. Bilateral superior laryngeal nerve injury
e. Bilateral recurrent nerve injury
53. The blood supply to carotid body tumors arises from the:
a. Subclavian artery
b. Common carotid artery
c. Internal carotid artery
d. External carotid artery
e. Transverse cervical artery
54. Can a patient given a narcotic agent give informed consent?
a. Yes, but only in emergencies
b. Yes, but only if the intent of the medication was to get the patient to agree to
surgery
c. Yes, but only if the patient is a habitual user of narcotics
d. Never
e. Yes, but only if the patient is alert, oriented, and capable of making a decision
55. Concerning advance directives, all of the following are true except:
a. Availability of the forms is required by law at the time of hospitalization
b. It is a right of the patient to complete such forms
c. It is desirable that such forms be completed before major operative procedures
d. It is mandatory that the forms be completed before elective surgery
e. Patients may refuse to fill in such forms
56. Your elderly cancer patient has a DNR order. He has sustained an esophageal
perforation during an attempted PEG insertion. While in the operating room, he has
a cardiac arrest. Should resuscitation be attempted?
a. It is a simple answer, YES. DNR orders are automatically cancelled when a patient
enters
the OR
b. It is a simple answer, NO. DNR orders cannot be rescinded
c. Only if you believe it will be successful
d. Only if you believe the family wants it done
e. Only if there was preoperative discussion of suspension of the DNR order and the
patient
agreed to perioperative discontinuation of the DNR order
57. The typical patient believes that the principal cause of medical errors is:
a. Incompetence
b. Overtiredness
c. Insufficient attention by physician
d. Poor nursing care
58. Informed consent is best described as:
a. Following the doctor’s advice
b. A process of shared decision making based on mutual respect and participation
c. A process of communication that is essential only to treatment for a life-
threatening illness
d. Obtaining a second opinion prior to starting treatment
e. Providing the patient the consent form to study
59. Withholding or withdrawing treatment is best described as:
a. Passive euthanasia
b. Actively taking steps to cause a patient's death
c. Assisted suicide
d. Letting nature take its course
e. None of the above
60. The most important factor in determining if someone is a suitable candidate for
an organ transplant is:
a. Emotional stability
b. Someone for whom a transplant offers a reasonable likelihood of recovery
c. Ability to deal with stress
d. Age
e. Insurance coverage
61. The need for new methods of defining and determining death was created by
what medical
procedure?
a. Brain surgery
b. Open heart surgery
c. Artificial resuscitation
d. Organ transplantation
e. Cardiac pacemakers
62. You will receive ______ medical care if you do not have any advance directives.
a. little
b. full
c. no
d. reluctant
e. substandard
63. Esophageal manometry in patients with achalasia usually shows which one of
the following findings?
a. Frequent uncoordinated peristalsis
b. Primary motor waves
c. Secondary motor waves
d. Aperistalsis
e. Relaxation of the LES
64. Which of the following statements is characteristic of patients who have a Zenker
diverticulum?
a. Usually are in their 40s or 50s
b. Should have EGD to make the diagnosis
c. Medical therapy is warranted before surgery
d. GERD usually worsens after surgery on the Zenker diverticulum
e. Myotomy is indicated
65. Post-emetic esophageal perforation is characterized by the following:
a. Often occurs in patients with underlying esophageal disease
b. Barium esophagram should be done if Boerhaave syndrome is suspected
c. Subcutaneous emphysema is always present
d. Surgical repair is best done laparoscopically
e. 2-layer closure of the esophagus with some type of buttressing should be used for
delayed
repair
66. Risk factors for esophageal cancer include:
a. Achalasia
b. Barrett esophagus
c. GERD
d. Smoking, alcohol abuse
e. All of the above
67. Patients with GERD and short segments of Barrett epithelium should always have
the following:
a. 24-hour pH studies to evaluate the severity of reflux
b. Avoidance of aggressive medical management of GERD since it could mask
symptoms
c. EMR of Barrett mucosa
d. Laparoscopic Nissen fundoplication
e. EGD and 4-quadrant biopsies to detect HGD
68. A 33-year-old woman presents with worsening right lower quadrant abdominal
pain of 2 days' duration. She is carrying twins at 24 weeks’ gestation. She has a poor
appetite and mild nausea. Her abdominal exam reveals decreased bowel sounds,
moderate tenderness to palpation in the right mid and lower abdomen, questionable
rebound, and a gravid uterus that is nontender. Her cervix is closed, and there are no
uterine contractions on the monitor. Her WBC is 18,400 with a shift to the left. What
diagnosis is most likely, given the previous information?
a. Preterm labor
b. Placental abruption
c. Appendicitis
d. Duodenal ulcer
69. Which one of the following statements is true regarding appendicitis in
pregnancy?
a. Rovsing and psoas signs are as predictive in pregnancy as in non-pregnant women
b. Lack of fever excludes the diagnosis
c. CT scan cannot be used to aid in diagnosis
d. RLQ pain is seen in ~80% of patients
e. A WBC of >16K is predictive of appendicitis
70. A 33-year-old woman presents with acute upper abdominal pain of 1 day's
duration. She is carrying twins at 28 weeks’ gestation. She has a poor appetite,
nausea, and emesis. Her abdominal exam reveals decreased bowel sounds, moderate
tenderness to palpation in the RUQ and epigastrium, and a gravid uterus that is
nontender. Her cervix is closed, and there are no uterine contractions on the monitor.
Ultrasound reveals gallstones. Amylase lipase and total bilirubin are markedly
elevated. All the following are appropriate in preparation for surgery except:
a. Administering antibiotics
b. Considering tocolysis
c. Intraoperative fetal monitoring
d. Having cesarean section tray on standby
e. Positioning the patient flat
71. All the following principles are appropriate in surgery for this pregnant patient
except:
a. Providing left lateral displacement of the uterus
b. Accepting a pneumoperitoneum of 20 to 25 mm Hg, given the advanced
gestational age
c. Monitoring maternal end tidal CO2 to minimize maternal and fetal acidosis
d. Using an open Hasson technique to decrease the risk for uterine injury
72. Which of the following is an incorrect choice for the desired ultrasound imaging?
a. 3.5-MHz probe for a thyroid scan
b. 5.0-MHz transducer for a FAST exam
c. 7.5-MHz probe for ultrasound-guided aspiration of a breast cyst
d. 10.0-MHz laparoscopic probe for intraoperative localization of a pancreatic
endocrine
tumor
e. 12.0-MHz laparoscopic probe for ultrasonic cholangiography
73. Which of the following statements is INCORRECT?
a. A FAST examination will help localize the source of bleeding in a patient with
penetrating
abdominal trauma.
b. Ultrasound can be useful in differentiating between cholelithiasis, polyps, and
adenomyomatosis.
c. Ultrasound can be useful in differentiating between cystic masses and solid
masses.
d. Higher-frequency probes offer greater resolution at the expense of decreased
depth
penetration.
e. Anorectal ultrasound findings are predictive of tumor stage.
74.

Shown above is a sonographic examination of a breast in a 25-year-old nulliparous


woman with a history of a newly noted painful mass. The next step in management
should be:
a. NSAIDs
b. Ductal lavage
c. Percutaneous cyst drainage and follow-up prn
d. Excisional biopsy with sentinel lymph node sampling
e. Observation

75.

The structure identified above by the small arrowheads is most likely:


a. Lymph node
b. Parathyroid adenoma
c. Esophagus
d. Hyoid bone
e. Carotid body tumor
76. You are called in consult for a patient with hemoptysis who has been on a
ventilator in the ICU for the past 3 weeks. The most likely diagnosis is:
a. Tracheo-innominate fistula
b. Tuberculosis
c. Lung cancer
d. PA catheter perforation
e. Bleeding from the trauma of suctioning
77. You are called to see a young man with a recurrent spontaneous pneumothorax.
He was treated at another hospital 2 weeks ago, with chest tube drainage for 5 days.
Acceptable management includes all the following EXCEPT:
a. Smoking cessation counseling
b. Needle aspiration followed by observation
c. Initial chest tube placement
d. CT scanning of the chest
e. Thoracoscopic definitive treatment
78. Treatment options for chylothorax may include all the following EXCEPT:
a. Needle aspiration
b. Chest tube drainage
c. High-fat, high-protein, low-carbohydrate diet
d. Total parenteral nutrition
e. Thoracic duct ligation
79. Synthesis and breakdown of collagen occurs in which stage of wound healing?
a. Coagulation stage
b. Inflammatory stage
c. Proliferative stage
d. Remodeling stage
e. Wound contraction
80. Hydroxylation of proline and lysine to increase wound strength is dependent on:
a. Vitamin A
b. Vitamin D
c. Vitamin D3
d. Vitamin C
e. Vitamin E
81. Randomized prospective data exist to support the following benefits of VAC ®
therapy EXCEPT:
a. Decreasing the bacterial burden in wounds
b. Avoiding need for flap procedures when treating underlying osteomyelitis
c. Accelerating wound healing by secondary intention
d. Decreasing total wound care costs
e. Improving convenience for patient and wound caregiver
82. You are asked to assume care for a chronic medial ankle wound in a 60-year-old
diabetic with a 25 pack-year history of smoking. A comprehensive treatment plan
includes all the following EXCEPT:
a. Smoking cessation strategies
b. Application of PDGF (Regranex®)
c. Use of electromagnetic field therapy
d. Administration of pentoxifylline (Trental®) 400 mg TID
e. Aggressive hydrotherapy/Whirlpool
83. Currently available, topically applied agents, shown to be effective in modulating
scar appearance, include all EXCEPT:
a. Steroid preparations
b. Silicone gels and occlusive sheets
c. Hydroquinones
d. Antibiotic ointments
e. Pressure garments
84. A 55-year-old patient with history of sulfa allergy sustains a 20% TBSA second-
and third-degree burn to his chest and arms after falling asleep while smoking in
bed, igniting his pajamas. The most appropriate topical antimicrobial to use for his
burn care is:
a. Silver sulfadiazine (Silvadene®)
b. Mafenide acetate (Sulfamylon®)
c. Bacitracin antibiotic ointment
d. Mupirocin (Bactroban®)
e. Silver nitrate
85. The care of patients sustaining major electrical burns differs from that in thermal
burn patients in all
areas listed below EXCEPT:
a. Calculation of fluid resuscitation needs
b. Development of electrolyte abnormalities
c. Surgical management of injured areas
d. Immune response to the burn injury
e. Impact of the injury on cardiac function
86. You are evaluating a 45-year-old woman for a palpable right neck mass identified
on physical
exam by her gynecologist. She has no family history of thyroid cancer or radiation
exposure. Her TSH
is normal. What is the least invasive test to make a definitive diagnosis?
a. Radioactive iodine scan
b. Thyroglobulin level
c. CEA level
d. Fine-needle aspiration biopsy
e. USG
87. You are evaluating a 60-year-old woman who presents with a 3-cm left thyroid
nodule. Fineneedle biopsy is suggestive of follicular neoplasm. USG demonstrates a
normal right thyroid lobe. What is your recommendation for further treatment?
a. Repeat USG and biopsy in 6 months
b. Thyroid hormone suppression
c. Diagnostic left thyroid lobectomy
d. Total thyroidectomy
e. Total thyroidectomy and left modified radical neck dissection
88. You are evaluating a 25-year-old man with a new thyroid nodule. He reports a
longstanding history of HTN and episodic headaches. His family history is significant
for a mother and sister who had their thyroids removed for cancer, and his sister
died on the operating room table. What additional test should you obtain prior to a
fine-needle biopsy?
a. Calcium
b. Plasma metanephrines
c. 24-hour urine cortisol levels
d. CA 19-9 levels
e. Thyroglobulin
89. A 2.8-kg neonate with excessive salivation develops respiratory distress.
Attempts to pass an urogastric catheter fail as the catheter coils in the back of the
throat. X-ray shows right upper lobe atelectasis and a gasless abdomen. The most
likely diagnosis is:
a. Proximal esophageal atresia without a fistula
b. Proximal esophageal atresia with a distal tracheo-esophageal fistula (TEF)
c. “H-type” TE fistula
d. Esophageal atresia with both proximal and distal TEF
90. In infants with duodenal atresia, all the following statements are true EXCEPT:
a. There is an increased risk for Down syndrome
b. It can be detected by prenatal ultrasound examination
c. It may occur in infants with situs inversus, malrotation, annular pancreas, and
anterior
portal vein
d. It is best treated by gastroenterostomy
e. There is a high incidence of associated cardiac defects
91. The treatment of choice for neonates with uncomplicated meconium ileus is:
a. Observation
b. Emergency laparotomy, bowel resection, and Bishop-Koop enterostomy
c. Intravenous hydration and a Gastrografin® enema
d. Emergency laparotomy, bowel resection, and anastomosis
e. Sweat chloride test and pancreatic enzyme therapy
92. Which of the following statements about Hirschsprung disease is not true?
a. There are no ganglion cells seen in Auerbach plexus.
b. There is an increased incidence of Down syndrome.
c. It is more common in girls.
d. It may be associated with enterocolitis.
e. It may involve the small intestine.
93. Polyhydramnios is frequently observed in all the following conditions EXCEPT:
a. Esophageal atresia
b. Duodenal atresia
c. Pyloric atresia
d. Hirschsprung disease
e. Congenital diaphragmatic hernia
94. In infants with gastroschisis, which of the following statements is not true?
a. It is associated with malrotation.
b. There is a high incidence of associated anomalies.
c. There is prolonged adynamic ileus following repair.
d. It is complicated by intestinal atresia in 10% to 12% of cases.
e. It is not associated with chromosomal syndromes.
95. In neonates with necrotizing enterocolitis, which of the following findings is an
indication of significant bowel ischemia?
a. Increased gastric residuals
b. Septic shock
c. Cardiac failure due to a patent ductus arteriosus
d. Elevated platelet count
e. Erythema of the abdominal wall
96. All the following conditions are derived from the primitive embryologic foregut
EXCEPT:
a. Bronchogenic cyst
b. Cystic adenomatoid malformation
c. Gastric duplication
d. Mesenteric cyst
e. Pulmonary sequestration
97. A 25-year-old male undergoes 4 cycles of etoposide and platinum following a
right orchiectomy for stage IIb non-seminoma germ cell tumor (NSGCT). Serum
tumor markers and chest CT are normal. Abdominal CT scan is shown below. Which
of the following is most appropriate?
a. Observation
b. Post-chemo RPLND
c. Radiation
d. Further chemo
e. Insufficient information

98. A 32-year-old alcoholic male presents to the ED with nausea, vomiting, and
epigastric pain. He is diagnosed with pancreatitis. Which of the following is NOT one
of the Ranson criteria, which can help you decide on the severity of the attack?
a. Calcium level <8 mg/dL at admission
b. WBC >16,000 at admission
c. LDH >350 IU/L at admission
d. Glucose >200 at admission
e. Age >55 years
99. Which of the following statements regarding groin hernias is false?
a. Men are much more likely to develop inguinal hernias than are women.
b. In women with inguinal hernias, there is an equal incidence of direct and indirect
hernias.
c. Femoral hernias are the most likely to strangulate of any groin hernias.
d. A combined direct and indirect inguinal hernia is known as a “pantaloon” hernia.
e. Inguinal hernias are more common in women than are femoral hernias.
100. Which of the following statements regarding the repair of inguinal hernias is
correct?
a. Applying considerable tension to the sutures is important to prevent bulges from
occurring.
b. Recurrence risk from open and laparoscopic hernia repairs are the same.
c. The lowest recurrence rates are reported after tension-free repairs with mesh.
d. The Bassini repair involves suturing transversus abdominis to the Cooper
ligament.
e. If the spermatic (testicular) artery is inadvertently transected during an inguinal
hernia
repair, the testicle must be removed.

Pancreas
1. In pancreas divisum all the following are true except
A. occur in 5% of the population
B. may lead to recurrent pancreatitis
C. duct of Wirsung is the main duct
D. there is failure of fusion
E. diagnosed by ERCP

2. Cystic fibrosis all the following are true except


A. transmitted as autosomal dominent
B. dysfunction of exocrine glands
C. there is poor appetite & growth,
D. patient has distension, steatorhea, finger clubbing present
E. diagnosis by DNA analysis

3. Sever attack of pancreatitis is


A. attack followed by pseudocyst
B. has four points on Ranson criteria
C. has 5 points on Balthazar grade
D. all the above
E. none of the above

4. Antibiotics in acute pancreatitis:


A. not indicated
B. second generations cephalosporins are the best
C. indicated in all cases
D. b and c only
E. none of the above

5. CT in acute pancreatitis :
A. should be done on admission
B. should be done 48 hours later
C. should be done one week later
D. not indicated
E. none of the above

6. Pancreatic pseudocyst
A. usually appears after two weeks of the attack
B. has one layer of epithelial lining
C. may be treated by endoscopy
D. all the above
E. none of the above

7. Percutaneous drainage of pancreatic cyst is indicated in all the following except :


A. after three weeks in order to mature the wall
B. rapidly enlarging cyst
C. difficult sites
D. presence of infection
E. unfit patients

8. The following are known complications of acute pancreatitis except


A. respiratory failure
B. renal failure
C. cardiac failure
D. coagulopathy
E. bleeding gastric erosion

9. The following can cause high serum amylase :


A. perf. GB
B. rupture ectopic pregnancy
C. rupture abdominal aortic aneurysm
D. afferent loop obstruction
E. all the above

10. Carcinoma of the pancreas:


A. 50% adeocarcinoma of duct origin
B. slightly more in the head
C. 10% cystadenocarcioma
D. CIS about 40 %
E. none of the above
11. Insolinoma :
A. age usually above 40
B. hyper glycemia in the early morning is classical
C. clinically may simulate DU
D. insulin / glucose ratio should equal one
E. all of the above

Orthopedic

12. Bennet's fracture is :


A. fracture of the base of first meta tarsal bone
B. fracture of the base first meta carpal bone
C. fracture of the head of first meta tarsal bone
D. fracture of the head first meta carpal bone
E. fracture of the head second meta carpal bone

13. Thomas test is positive in :


A. Paget's disease of the hip
B. Perthe's disease of the hip
C. osteo arthritis of the knee
D. Sudeck's atrophy of the shoulder
E. none of the above

14. All the following are complications of cast except


A. compartment syndrome
B. pressure sore
C. blisters with 2ry infection
D. immobilization of joints
E. crush syndrome

15. Volkmann's ischemic contracture :


A. may follows fracture of radius and ulna
B. fracture neck of femur
C. supracondylar fracture of humerus
D. all the above
E. none of the above

16. Compartment syndrome


A. common in fracture of the leg
B. passive flexing of the toes increase the pain
C. compartment pressure should exceed 50 mmHg
D. treated by urgent fasiectomy
E. all the above

17. Indication of operative treatment in fractures :


A. compound fracture
B. intertrochanteric fracture of femur in the elderly
C. associated complications
D. a and c only
E. all the above

18. The following are methods of stabilization of fractures except:


A. External splint e.g. POP
B. Internal fixation
C. Hanging plaster
D. Skin traction
E. a and b only

19. stress fracture :


A. may affect the shaft of tibia
B. may affect the second metatarsal bone
C. occur in marathon runners
D. all the above

20. Mallet finger is due to:


D. avulsion fracture of the base of terminal phalanx

21. operative approach to the hip joint may be :


A. anterior
B. posterior
C. lateral
E. all the above

22. Salter and Harris classification :


A. classification for fracture tibial platue
B. classification for fracture neck femur
C. classification for fracture growth plate

VASCULAR
23. In critical ischemia there is:
A. rest pain
B. oedema
C. color changes
D. hyperesthesia
E. all the above

24. popliteal artery aneurysm


A. accounts for 70% of all peripheral aneurysms
B. 25% are bilateral
C. 30% develop complication within 5years
D. All of the above

25. Beurger' diseases has :


A. occlusion of small and medium arteries
B. thrombophlebitis
C. Reynold's phenomenon
D. all the above
26. abdominal aortic aneurysm
A. usually due to atherosclerosis
B. usually at the level of L1
C. incidence of rupture is 50% for 5 cm aneurysms
D. elective surgery careies 20% mortality rate
E. all the above

27. Berry's aneurysms


A. are congenital
B. affect medium sized arteries
C. fusiform in shape
D. all the above

28. Secondary varicose viens


A. due to valve destruction
B. due to incompetent perforators
C. has lower complication rate than the primary varicose
D. may lead to venous ulcer
E. none of the above

29. Greenfield filter


A. reduce incidence of DVT
B. inserted in the common iliac vien
C. inserted between L1 and L2 levels
D. can be inserted percutaneous

30. lymph edema tarda


A. usually before the age of 35 years
B. affects upper limbs only
C. affect lower limbs only
D. usually pitting in early stage

BASIC SURGICAL SCIENCES

31. Abduction of the vocal cords results from contraction of the:


A. crico-thyroid muscles
B. posterior crico-arytenoid muscles
C. vocalis muscles
D. thyro-ary-epiglottic muscles
E. lateral crico-arytenoid and transverse arytenoids muscles

32. The rectum:


A. is devoid of peritoneum
B. is surrounded by peritoneum
C. has peritoneum on its lateral surfaces for its upper two- thirds, and on its anterior
surface for its upper one-third
D. has pritoneum on its anterior surface for its upper two-thirds, and on its lateral
surfaces for its upper one-third
E. has peritoneum on its anterior surface only

33. The umbilicus:


A. lies near the to the xiphoid than to the pubis
B. derives its cutaneous innervation from the eleventh thoracic nerve
C. transmits, during development, the umbilical cord two arteries and two veins
D. usually lies at about the level between the third and fourth lumbar vertebra
E. emberiologicall, may transmit urine but never bowel contents

34. The superficial perineal pouch:


A. is limited inferiorly by the urogenital diaphragm
B. is not continuous with the space in the scrotum occupied by the testes
C. has a membranous covering which provides a fascial sheath around the penis
D. is traversedby the urethera in the male but not the urethera and vagina in the
female
E. in the female, the greater vestibular glands are situated outside this pouch

35. The tongue:


A. has a foramen caecum at the base of the frenulum
B. is separated from the epiglottis by the valleculae on each side of the midline
C. has 7-12 circumvallate papillae situated behind the sulcus terminalis
D. is attached to the hyoid bone by the genioglossus muscle
E. is supplied only by hypoglossal nerve

36. Hypovolaemic shock is characterized by:


A. a low central venous pressure , low cardiac output , low peripheral resistance
B. a high central venous pressure , high cardiac output , low peripheral resistance
C. a low central venoys pressure , low cardiac output , high periphera resistance
D. a low central venous pressure , high cardiac output , high peripheral resistance
E. a high central venous pressure , low cardiac output , low peripheral resistance
37. An oxygen debt is:
A. the amount of oxygen in excess of the resting metabolic needs that must be
consumed after completion of exercise
B. build up because the pulmonary capillaries limit the uptake of Oxygen at high rates
of oxygen consumption
C. related to the fact that skeletal muscle cannot function temporarily in the absence
of oxygen
D. associated with a decrease in blood lactate
E. associated with alkalosis

38. Pulmonary embolism may be a complication of except :


A. prolonged bed rest
B. a surgical operation
C. vitamine K deficiency
D. oral contraceptive therapy
E. Antithrombin III deficiency

39. Which of the following statements regarding potassium metabolism is NOT


True?:
A. potassium deficiency commonly results from thiazide diuretic theraoy
B. the normal compensation for potassium deficiency is a metabolic extracellular
acidosis
C. aldosterone increases urinary potassium loss
D. hyperkalaemia causes bradycardia and loss of P waves on the ECG
E. hypokalaemia aggrevates the cardiac effects of digitalis toxicity

40. Cutaneous pain:


A. is due to overstimulation of receptors serving other sensory modalities
B. cannot be elicited more readily if the tissue has recently been injured
C. is due to exitation of receptors by pain-producing chemical substances in the
injured tissue
D. shows marked adaptation, i. e. decrease in severity in response to a constant
stimulus
E. is conducted through the medial spinothalamic tract

41. Which of the following is NOT associated with hyperthyroidism?:


A. increase size of the thyroid gland
B. increased amount of colloid in thyroid follicle
C. increased height of epithelium of the thyroid follicle
D. increased vascularity of the thyroid gland
E. increased uptake of iodine by the thyroid gland

42. Sarcomata may show all of the following EXCEPT:


A. production of myxomatous tissue
B. production of collagen
C. spindle shaped cells
D. signet ring cells
E. blood stream metastasis

43. Anaphylaxis is characterized by all of the following EXCEPT:


A. is a reaction either local or general , frequently occurs within five minutes
B. causes an urticarial eruption
C. is produced by IgA antibody
D. causes eosinophilia
E. causes degranulation of basophils and mast cells

44. Autoimmunity is characterized by the following EXCEPT:


A. occurs because of a breakdown in the ability of the body to distinguish between
self and non self
B. is involved in some forms of orchitis
C. is involved in formation of cryo globulin
D. is important in the pathogenesis of lupus erytheromatosus
E. does not result in immune complex disease

45. Pseudomembranous enterocolitis is caused by the following organisms:


A. Clostridium sporogenes
B. Clostridium defficile
C. Streptococcus faecalis
D. Penicillin sensitive staphylocci
E. Pseudomonas aeruginos
CLINICAL SUGERY

46. The “ white clot syndrome”:


A. is usually characterized with antithrombin III deficiency
B. most often present with arterial complicatios of heparin induced throbocytopenin
C. is best managed by loe molecular weight dextran
D. is best managed by halving the therapeutic dose of heparin sodium
E. results from nitric oxide deficiency of endothelial cells

47. A 21-year-old man who was the driver in a head-on collision has a pulse of
140/min , respiratory rate of 36 and blood pressure of 75 palpable. His trachea is
deviated to the left, with palpable subcutaneous emphysema and poor breath sounds
in the right hemithorax, The most appropriate initial treatment must be
A. immediate thoracotomy
B. catheter insertion in the subclavian vein for fluid resuscitation
C. intubation and ventilation
D. tube thoracostomy
E. immediate tracheostomy

48. The best test to monitor the adequacy of levothyroxin therapy is:
A. radioactive iodine uptake
B. thyroglobulin
C. free thyroxine index (T4)
D. triiodothyronine resin uptake (T3)
E. thyroid stimulating hormone (TSH)

49. Which of the following statements about fungal infection is NOT true ?:
A. Prior or synchronous culture positive for Candida at another site occurs in few
patients with candidimia
B. For critically ill patients nonhaematogenous sites of candida are appropriately
treated with systemic antifungal therapy
C. Mortality rates are similar regardlss of whether C. albicans fungmia is treated with
amphotericin B or flconazole
D. Intravenous catheters and the gastrointestinal tract are common portals for Candida
to gain blood stream access
E. Septic emboli are more common with fungal endocarditis than with bacterial
endocarditis

50. The maximum safe dose of local anaesthetic administered subcutaneously in a


70-kg man is:
A. 10 to 20 ml of 1% lidocaine
B. 40 to 50ml oh 2% lidocaine with epinephrine
C. 40 to 50 ml of 1% lidcaine with epinephrine
D. 40 to 50 ml of 1% bupivacaine (marcaine)
E. 40 to 50 ml of 1%lidocaine without epinephrine

51. Two days after right hemicolectomy for a Dukes B caecal carcinoma , the Patient
complains of sharp right-sided chest pain and dyspnea. HisPaO2 Is 64mmHg ,his
PaCo2 is 32mmHg. CVP is 26 cm water, and the blood pressure is 102/78mmHg. A
pulmonary embolus is suspected, The next step in management should be:
A. A ventilation- perfusion lung scan
B. A pulmonary arteriogram
C. Postrioanterior and lateral chest x-rays
D. Heparin sodium ,100 units/kg intravenously
E. Immediate duplex scanning of both lower extremities

52. The major cause of graft loss in heart and kidney allograft is:
A. acute rejection
B. hyperacute rejection
C. vascular thrombosis
D. chronic rejection
E. graft infection

53. All of the following are indicators of tumor aggressiveness and poor outcome for
papillary carcinoma of the thyroid gland EXCEPT:
A. age over 50 years
B. microscopic lymph node metstasis
C. tumor larger than 4 cm
D. poorly differentiated histological grade
E. invasion through capsule to adjacent tissues

54. A 40-year-old woman has extensive microcalcifications involving the entire upper
aspect of the right breast. Biopsy shows a commedo pattern of intraductal
carcinoma.The most appropriate treatment is :
A. wide local excision
B. radiation therapy
C. wide local excision plus radiation therapy
D. right total mastectomy
E. right modified radical mastectomy
55. In the conventional ventilator management of acute adult respiratory distress
syndrome (ARDS) , arterial O2 saturation is maintained above 90% by all the
following EXCEPT :
A. increasing the ventilatory rate
B. the use of positive end-expiratory pressure (PEEP)
C. increasing mean airway pressure
D. increasing tidal volumes
E. increasing FiO2

56. Which of the following statements about patients with abdominal compartment
syndrome is NOT true ?
A. Abdominal pressure is usually measured indirectly through inferior vena cava
B. Multiple contributing factors are commonly responsible
C. The chief manifestations are reflected in central venous pressure , ventilatory
function, and oliguria
D. Decopression of the abdomen is required to resverse the syndrome
E. Aggressive hemodynamic monitoring and management is required when the
abdomen is opened
57. The most appropriate treatment for histologically malignant cystadenoma
phylloides is :
A. total mastectomy without axillary node dissection
B. total mastectomy with axillary node dissection
C. wide margin (3) cm excision of the lesion
D. post operative hormonal manipulation
E. postoperative adjuvant chemotherapy

58. Deep venous thrombosis resulting from upper extremity central venous lines:
A. should be treated with catheter removal, heparin therapy, and long term
anticoagulants
B. is best with urokinase through the catheter
C. is innocuous and self limiting, and best treated with catheter removal only
D. is best treated with low-dose warfarin (coumadin, 1 mg / day) , without catheter
removal
E. is best managed by single systemic dose of low molecular weight heparin daily and
continued catheter use

59. Emergency surgery is indicated for all of the following complications of


ulcerative colitis EXCEPT:
A. colonic dilatation greater than 12 cm (toxic mega colon )
B. free perforation
C. complete intestinal obstruction
D. intractable haemorrhage
E. abscess formation

60. All the following statements concerning carcinoma of the oesophagus are true
EXCEPT that:
A. it has a higher incidence in males than females
B. alcohol has been implicated as a precipitating factor
C. adenocarcinoma is the most common type at the cardio esophageal junction
D. it occurs more commonly in patients with corrosive oesophagitis
E. surgical excision is the only effective treatment

61. Which of the following statements about pathology encountered at


esophagoscopy is/are correct?
A. Reflux esophagitis should be graded as mild, moderate, or severe, to promote
consistency among different observers.
B. An esophageal reflux stricture with a 2-mm. lumen is not dilatable and is best
treated with resection.
C. A newly diagnosed radiographic distal esophageal stricture warrants dilation and
antireflux medical therapy.
D. In patients with Barrett's mucosa, the squamocolumnar epithelial junction occurs 3
cm. or more proximal to the anatomic esophagogastric junction.

1 C 11 21 E 31 B 41 B 51 D 61
2 A 12 22 C 32 C 42 D 52 D
3 E 13 23 E 33 D 43 C 53 B
4 E 14 24 A 34 C 44 E 54 D
5 A 15 25 D 35 B 45 B 55 A
6 A 16 26 A 36 C 46 B 56 A
7 A 17 27 A 37 A 47 D 57 C
8 C 18 28 D 38 C 48 E 58 A
9 E 19 29 D 39 B 49 A 59 E
10 B 20 30 D 40 C 50 C 60 E

1. A 35-year-old male is struck on the lateral aspect of his right knee by the bumper
of a car travelling at low velocity. On examination he is unable to dorsiflex the ankle,
evert the foot and extend the toes. There is loss of sensation of the dorsum of the
foot. He is most likely to have damaged which structure?
A. Common peroneal nerve
B. Deep peroneal nerve
C. Saphenous nerve

2. Which one of the following suggest a diagnosis of Hirschsprung's disease?


A. A contrast-study showing dilatation of the aganglionic bowel segment.
B. Early presentation with vomiting.
C. neonatal large bowel obstruction.
D. Presentation after 1 year of age
E. Red current jelly stools.

3. A 46-year-old man presents after penetrating injuries to his arm and forearm. He
is unable to extend his fingers. There is no sensory disturbance and there is no
vascular injury. Which nerve has been damaged?
A. anterior interosseous nerve
B. median nerve
C. posterior interosseous nerve
D. radial nerve
E. ulna nerve

4. Which of the following regarding the anatomy of the heart is true?


A. The aortic valve is tricuspid
B. The ascending aorta is entirely outside the pericardial sac
C. The left atrial appendage is identified readily by transthoracic echocardiography.

D. The pulmonary trunk lies anterior to the ascending aorta


E. The right atrium is posterior to the left atrium.

5. Which of the following is not within the carpal tunnel?


A. median nerve
B. flexor digitorum profundus
C. flexor digitorum superficialis
D. flexor pollicis longus?
E. abductor pollicis longus

6. A 48-year-old woman with chronic pancreatitis due to gallstones is noted to have a


macrocytic anemia What is the most likely cause of the anemia?
A. Bone marrow dysfunction
B. Folate deficiency
C. Hyposplenism
D. Hypothyroidism
E. Vitamin B12 deficiency
7. Which of the following organs is in direct contact with the anterior surface of the
left kidney, without being separated from it by visceral peritoneum?
A. Duodenum
B. Jejunum
C. Pancreas
D. Spleen
E. Stomach
8. A 5-year-old boy presents to Accident and Emergency complaining of acute pain
over his upper tibia He is febrile and he refuses to move his leg. A diagnosis of
osteomyelitis is suspected The likely infecting organism is?
A. Clostridium difficile
B. Haemophilus influenzae
C. Pseudomonas
D. Salmonella
E. Staphylococcus aureus

9. Which is the principle root inervation for the small muscles of the hand?
A. C5
B. C6
C. C7
D. C8
E. T1

10. Which of the following is true concerning Scaphoid fractures?


A. Rarely occur in young adults
B. when complicated by avascular necrosis the proximal pole is usually affected

C. should be treated by bone grafting and internal fixation even if undisplaced

11. A 17-year-old girl underwent emergency splenectomy after a domestic accident.


Which one of the following organisms is most likely to cause life-threatening
infection in the future?
A. Actinomycosis
B. Haemophilus influenzae
C. Pseudomonas aeruginosa
D. Staphylococcus aureus
E. Streptococcus pneumonia

12. Which vertebral level and corresponding structure is correct?


A. C4 and bifurcation of the carotid artery
B. T2 and manubriosternal joint
C. T10 and opening for vena cava in diaphragm
D. T12 and oesophageal opening in the diaphragm
E. T8 and aortic opening in the diaphragm
13. Which of the following statements is true of Compartment syndrome:
A. Only occurs following fractures.
B. Loss of distal pulse is an early sign.
C. The presence of pain is unhelpful in diagnosis.
D. Passive stretch of affected muscles exacerbates pain.

14. The axillary nerve passes directly inferior to which muscle as it leaves the axilla?
A. Long head of triceps
B. Long head of biceps
C. Pectoralis major
D. Subscapularis
E. Teres minor

15. Which of the following is not a branch of the pudendal nerve?


A. Perineal nerve
B. Dorsal nerve of the penis.
C. Inferior rectal nerve
D. Genitofemoral nerve
E. Posterior scrotal nerve

16. A patient presents with a history of low back pain and sciatica The pain radiates
to the little toe, the ankle reflex is absent and the patient has difficulty in everting
the foot. Which nerve root is likely to be trapped
A. L3
B. L4
C. L5
D. S1
E. S2

17. The commonest clinical manifestation of primary hyperparathyroidism is:


A. Renal stone disease
B. Bone disease
C. Peptic ulceration
D. Constipation
E. Polyuria

18. Which of the following structures accompany the median nerve in the carpal
tunnel?
A. Flexor carpi ulnaris.
B. Flexor digitorum profundis.
C. The ulnar artery.
D. All of the above
E. None of the above
19. Which of the following is a branch of the LATERAL CORD of the brachial
plexus?
A. Suprascapular nerve
B. Lower subscapular nerve
C. Medial pectoral nerve
D. Musculo-cutaneous nerve
E. Upper subscapular nerve
20. Which of the following vertebrae has the most prominent spinous process?
A. T1.
B. T2.
C. C7.
D. T11.

21. The ulnar nerve all are true except :


A. innervates the first dorsal interosseus muscle
B. originates from the laterall cord of the brachial plexus
C. has no branches above the elbow
D. innervates the medial half of the flexor digitorum profundus
E. innervates the adductor pollicis muscle

22. A 9 month old boy presents with an acute scrotal swelling. The following
diagnoses are likely:
A. Epididymitis
B. Orchitis
C. Torsion of the testicular appendage
D. Irreducible inguinal hernia
E. Acute idiopathic scrotal oedema

23. Congenital pyloric stenosis:


A. Presents in the first few daies after birth.
B. Can cause hyperchloraemic alkalosis.
C. Is inherited as an autosomal recessive
D. Typically presents with bile stained vomiting after feeds
E. Is often diagnosed by feeling a mass in the right hypochondrium.

24. If torsion of the testicle is suspected, surgical exploration:


A. Should be immediate and include the asymptomatic side

25. Meckel's diverticulum all are true except :


A. Can present with chronic anaemia
B. Is present in 2% of the population.
C. Can be diagnosed by hydrogen breath test.
D. May contain ectopic pancreatic tissue
E. May be implicated in Littre's hernia
26. Which of the following statements is true of upper limb nerve injuries?
A. Injury to the median nerve results in a wrist drop
B. Injury to the radial nerve results in loss of sensation over the palmar aspect of the
index finger
C. Injury to the median nerve results in loss of sensation in the anatomical snuffbox
D. Injury to the ulnar nerve results in a claw hand
E. Injury to the ulnar nerve results in loss of sensation over the thumb

27. Concerning the inguinal canal:


A. It transmits the ilio-inguinal nerve
B. The deep inguinal ring lies below the mid-point of inguinal ligament
C. The superficial inguinal ring overlies the pubic tubercle
D. Laterally , the anterior wall is made up of the external oblique aponeurosis
E. Laterally, the posterior wall is formed by the conjoint tendon

28. In chest trauma, urgent cardiothoracic surgical referral is necessary for all
except:
A. Continuing massive air leak following insertion of chest drain
B. Cardiac tamponade
C. Disruption of the great vessels
D. Severe pulmonary contusion
E. Continuing haemorrhage following insertion of chest drain

29. A raised titre of anti-HBs in the bloodallare true except :


A. signifies previous hepatitis B infection
B. is produced after hepatitis B vaccination
C. indicates immunity to hepatitis infection
D. indicates active hepatitis B infection

30. The following statements regarding intravenous solutions is correct:


A. Normal saline contains 180mmol/l of sodium
B. Ringer's lactate solutions is designed for intracellular fluid replacement
C. Sodium bicarbonate 8.4% is a hyperosmolar solution
D. Normal saline with added potassium is appropriate therapy to correct a respiratory
alkalosis

31. The following is true of the blood supply of the rectum all are true except:
A. The principle blood supply is derived from the inferior rectal artery
B. The median sacral vessels contribute to the blood supply
C. As the inferior mesenteric artery crosses the pelvic brim it becomes known as the
superior rectal artery
D. The internal venous rectal plexus is continuous with the vascular cushions of the
anal canal
E. The superior rectal vein drains to the portal system

32. Which of the following concerning the Femoral sheath is false:


A. Contains the femoral artery
B. Contains lymph nodes
C. Contains the femoral canal
D. Contains femoral nerve

33. The following is true of the spleen:


A. Is the largest lymphoid organ in the body
B. Lies obliquely between the seventh and tenth rib
C. The lower pole extends beyond the mid-axillary line
D. Is usually palpable when normal
E. Usually measures 16cm in maximum length when healthy

34. A knife stabbed horizontally through the fourth intercostals space to the right of
the sternum is likely to damage the:
A. Right brachiocephalic vein-
B. Hemiazygos vein
C. Descending aorta
D. Right pulmonary artery
E. Thoracic duct

35. Breast cancer risk is increased in association with the following factors except:
A. Nulliparity
B. Immediately after pregnancy
C. Early menarche
D. Early age at first pregnancy
E. Late menopause

36. The Appendix all true except:


A. Is typically less than 10 cm in length in the adult.
B. Is located in the retrocaecal recess.
C. Macburneys point, lies 2/3 laterally from a line from umbilicus to the anterior
superior iliac spine
D. The longitudinal coat of the appendix is derived from the three bands of taenia coli.
E. Is supplied by branches of the inferior mesenteric artery

37. In tension pneumothorax the following signs are present except:


A. Hypoxia
B. Hyper resonance to percussion on the affected side
C. Tracheal deviation to the ipsilateral side
D. Distended neck veins
E. Tachycardia

38. The following are encountered during inguinal hernia repair :


A. Scarpas' fascia
B. Colle's fascia
C. Ilioinguinal nerve
D. Cremaster muscle
E. Genitofemoral nerve

39. Which of the following variables best predicts prognosis for patients with a
recent diagnosis of cutaneous melanoma and no clinical evidence of metastatic
disease?
A. Breslow thickness

40. The intravenous fluid that a 60 kg., 30-year-old woman with an 80% burn should
be given in the first 24 hours following burn injury is:
A. 19.2 liters of 5% glucose in lactated Ringer's.
B. 14.4 liters of lactated Ringer's.

41. All venous sinuses of the dura mater ultimately drain into :
A. Internal jugular vein

42. The tendon of biceps bronchii muscle inserted into the


A. Radial tuberosity

43. the saphenous nerve is terminal branch of the :


A. Ilioinguinal nerve
B. Femoral nerve
C. Genitofemoral nerve
D. Superior glutial nerve
E. popliteal nerve

44. All the following origenat on the scapula except


A. Trapezius
B. Teres major
C. Deltoid
D. Brachio-radialis
E. Coraco-brachialis
45. Total number of human vertebra is:
A. 33 vertebras.

46. Foot droop is usually associated with paralysis of the:


A. Common peroneal

47. The following pass through the aortic hiatus of the diaphragm
A. Aorta
B. Azygos vein
C. Thoracic duct
D. All of the above
E. Non of the above

48. At birth the umbilical vein become the:


A. Round ligament of the liver

49. Arrangement of structures at hilum of the right kidney as following from


anterior to posterior:
A. Vein ,artery , ureter

50. Shock can best be defined as:


A. Hypotension.
B. Hypo perfusion of tissues.
C. Hypoxemia
D. All of the above

51. All of the following are true about neurogenic shock except:
A. there is a decrease in systemic vascular resistance and an increase in venous
capacitance
B. tachycardia or bradycardia may be observed, along with hypotension.
C. the use of an alpha agonist such as phenylephrine is the mainstay of treatment.
D. Severe head injury, spinal cord injury, and high spinal anesthesia may all cause
neurogenic shock.

52. All of the following may be useful in the treatment of cardiogenic shock except:
A. Dobutamine
B. Sodium nitroprusside
C. Pneumatic antishock garment.
D. Intra-aortic balloon pump.

53. All of the following statements about hemorrhagic shock are true except:
A. Following hemorrhagic shock, there is an initial interstitial fluid volume
contraction.
B. Dopamine, or a similar inotropic agent, should be given immediately for
resuscitation from hemorrhagic shock, to increase cardiac output and improve oxygen
delivery to hypoperfused tissues.
C. The use of colloid solutions or hypertonic saline solutions is indicated for treatment
of hemorrhagic shock.
D. In hemorrhagic shock, a narrowed pulse pressure is commonly seen before a fall in
systolic blood pressure

54. Which of the following statements about head injury and concomitant
hyponatremia is true?
A. There are no primary alterations in cardiovascular signs.
B. Signs of increased intracranial pressure may be masked by the hyponatremia
C. Oliguric renal failure is an unlikely complication.
D. Rapid correction of the hyponatremia may prevent central pontine injury.
E. This patient is best treated by restriction of water intake

55. Which of the following statements about extracellular fluid is true?


A. The total extracellular fluid volume represents 40% of the body weight.
B. The plasma volume constitutes 20%of the total extracellular fluid volume
C. Potassium is the principal cation in extracellular fluid
D. The protein content of the plasma produces a lower concentration of cations than in
the interstitial fluid
E. The interstitial fluid equilibrates slowly with the other body compartments.

56. Which of the following statements is true of a patient with hyperglycemia and
hyponatremia?
A. Insulin administration will increase the potassium content of cells.
B. The sodium concentration must be corrected by 5 mEq. per 100 mg. per 100 ml.
elevation in blood glucose
C. With normal renal function, this patient is likely to be volume overloaded
D. Proper fluid therapy would be unlikely to include potassium administration.

57. Which of the following is not associated with increased likelihood of infection
after major elective surgery?
A. Age over 70 years.
B. Chronic malnutrition.
C. Controlled diabetes mellitus.
D. Long-term steroid use
E. Infection at a remote body site

58. The transfusion of fresh frozen plasma (FFP) is indicated for which of the
following reasons?
A. For the correction of abnormal PT secondary to warfarin therapy, vitamin K
deficiency, or liver disease
B. Volume replacement.
C. As a nutritional supplement.
D. Treatment of prolonged bleeding time

59. In patients receiving massive blood transfusion for acute blood loss, which of the
following is correct?
A. Packed red blood cells and crystalloid solution should be infused to restore
oxygen-carrying capacity and intravascular volume
B. Two units of FFP should be given with every 5 units of packed red blood cells in
most cases.
C. A “six pack” of platelets should be administered with every 10 units of packed red
blood cells in most cases.
D. One to two ampules of sodium bicarbonate should be administered with every 5
units of packed red blood cells to avoid acidosis.

60. Hemostasis and the cessation of bleeding require which of the following
processes?
A. Adherence of platelets to exposed subendothelial glycoproteins and collagen with
subsequent aggregation of platelets and formation of a hemostatic plug.
B. Interaction of tissue factor with factor VII circulating in the plasma
C. The production of thrombin via the coagulation cascade with conversion of
fibrinogen to fibrin.
D. Cross-linking of fibrin by factor XIII.
E. All of the above

61. Advantages of epidural analgesia include:


A. Earlier mobilization after surgery.
B. Earlier return of bowel function.
C. Shorter hospitalizations.
D. Decreased stress response to surgery.
E. All of the above

62. Factors that decrease collagen synthesis include all of the following except:
A. Protein depletion.
B. Infection.
C. Anemia
D. Advanced age
E. Hypoxia

63. Which of the following statement is true concerning excessive scarring


processes?
A. Keloids occur randomly regardless of gender or race
B. Hypertrophic scars and keloid are histologically different
C. Keloids tend to develop early and hypertrophic scars late after the surgical injury
D. Simple reexcision and closure of a hypertrophic scar can be useful in certain
situations such as a wound closed by secondary intention

64. Which of the following statement is true concerning the vascular response to
injury?
A. Vasoconstriction is an early event in the response to injury
B. Vasodilatation is a detrimental response to injury with normal body processes
working to avoid this process
C. Vascular permeability is maintained to prevent further cellular injury
D. Histamine, prostaglandin E2 (PGE2) and prostacyclin (PGI2) are important
mediators of local vasoconstriction

65. All are true ligamentous attachments of liver except


A. Falciform ligament
B. Coronary Ligaments
C. Hepatoduodenal ligaments
D. Glisson's capsul

66. The appropriate surgical treatment for suspected carcinoma of the testis is:
A. Inguinal exploration, control of the spermatic cord, biopsy, and radical orchectomy
if tumor is confirmed

67. Nasotracheal intubation all true except :


A. Is preferred for the unconscious patient without cervical spine injury.
B. Is preferred for patients with suspected cervical spine injury.
C. Maximizes neck manipulation.
D. Is contraindicated in the patient who is breathing spontaneously.

68. The radiographic findings indicating a torn thoracic aorta include all except :
A. Widened mediastinum.
B. Presence of an apical “pleural cap.”
C. Tracheal deviation to the right.
D. right hem thorax.

69. Which of the following steps is not a part of the primary survey in a trauma
patient?
A. Insuring adequate ventilatory support
B. Measurement of blood pressure and pulse
C. Neurologic evaluation with the Glasgow Coma Scale
D. Examination of the cervical spine
70. Which of the following statement(s) is/are true concerning the Advanced Trauma
Life Support (ATLS) classification system of hemorrhagic shock?
A. Class I shock is equivalent to voluntary blood donation
B. In Class II shock there will be evidence of change in vital signs with tachycardia,
tachypnea and a significant decrease in systolic blood pressure
C. Class IV hemorrhage can usually be managed by simple administration of
crystalloid solution
D. Class III hemorrhage involves loss of over 40% of blood volume loss and can be
classified as life-threatening

71. The clinical picture of gallstone ileus includes all except ?


A. Air in the biliary tree
B. Small bowel obstruction.
C. A stone at the site of obstruction.
D. Acholic stools.
E. Associated bouts of cholangitis.

72. The evaluation of a comatose patient with a head injury begins with:
A. Establishment of an airway.

73. Which of the following signs does Horner's syndrome include?


A. Ptosis.
B. Facial hyperhidrosis.
C. Exophthalmos.
D. Mydriasis.

74. The goals of proper fracture reduction include all except?


A. Providing patient comfort and analgesia
B. Allowing for restoration of length of the extremity.
C. Correcting angular deformity and rotation.
D. Enabling immediate motion of all fractured extremities.

75. The neurovascular structure most commonly injured as a result of an anterior


dislocation of the shoulder is the:
A. Musculocutaneous nerve
B. Axillary nerve
C. Axillary artery.
D. Median nerve

76. The radial nerve is at greatest risk for injury with which fracture?
A. Fracture of the surgical neck of the humerus.
B. Fracture of the shaft of the humerus.
C. Supracondylar fracture of the humerus.

77. The most consistent sign of a fracture of the carpal scaphoid is:
A. Wrist pain during attempted push-ups.
B. Diffuse swelling on the dorsum of the wrist.
C. Localized tenderness in the anatomic snuffbox.
D. Wrist popping on movement.

78. Hematogenous osteomyelitis most frequently affects:


A. The diaphysis of long bones.
B. The epiphysis.
C. The metaphysis of long bones.
D. Flat bones.
E. Cuboidal bones.
79. The most common hernia in females is:
A. Femoral hernia
B. Direct inguinal hernia
C. Indirect inguinal hernia

80. Which of the following most often initiates the development of acute
appendicitis?
A. A viral infection.
B. Acute gastroenteritis.
C. Obstruction of the appendiceal lumen.

81. The most helpful diagnostic radiographic procedure in small bowel obstruction
is:
A. CT of the abdomen.
B. Contrast study of the intestine
C. Supine and erect x-rays of the abdomen.
D. Ultrasonography of the abdomen.

82. Which of the following treatments should never be recommended to a patient


with purely intraductal carcinoma?
A. Modified radical mastectomy.
B. Lumpectomy to clear surgical margins, followed by observation.
C. Incisional biopsy with an involved margin, followed by radiation.
D. Excisional biopsy to clear margins, followed by radiation.

83. In role of nine extent of burn if entire trunk is burned it will be equal to:
A. 9% body surface area
B. 18% body surface area
C. 36% body surface area
D. 27% body surface area

84. Which of the following does not describe intermittent claudication?


A. Is elicited by reproducible amount of exercise
B. Abates promptly with rest.
C. Is often worse at night.
D. May be an indication for bypass surgery.

85. Which of the following statements are true?


A. Patients with critical limb ischemia have paralysis and paresthesias.
B. All arterial injuries are associated with absence of a palpable pulse
C. Preoperative arteriography is required to diagnose an arterial injury.
D. The presence of Doppler signals indicates that an arterial injury has not occurred
E. In all patients with multiple trauma, arterial injuries should be repaired before other
injuries are addressed

86. A patient with acute urinary tract infection (UTI) usually presents with:
A. Chills and fever.
B. Flank pain.
C. Nausea and vomiting.
D. Painful urination.
87. The most precise diagnostic screening procedure for differentiating benign
thyroid nodules from malignant ones
A. Fine-needle-aspiration biopsy (FNAB).

88. Hyperthyroidism can be caused by all of the following except:


A. Graves' disease
B. Plummer's disease
C. Struma ovarii.
D. Hashimoto's disease
E. Medullary carcinoma of the thyroid
1 A 21 B 41 A 61 E 81 C
2 C 22 D 42 A 62 C 82 C
3 C 23 E 43 B 63 D 83 C
4 A 24 A 44 A 64 A 84 C
5 E 25 C 45 A 65 D 85 A
6 E 26 D 46 A 66 A 86 D
7 C 27 A 47 D 67 B 87 A
8 E 28 D 48 A 68 D 88 D
9 E 29 D 49 A 69 D
10 B 30 C 50 B 70 A
11 E 31 A 51 C 71 D
12 A 32 D 52 C 72 A
13 D 33 A 53 B 73 A
14 D 34 D 54 A 74 D
15 D 35 D 55 B 75 B
16 D 36 E 56 A 76 B
17 A 37 C 57 C 77 C
18 B 38 B 58 A 78 C
19 D 39 A 59 A 79 C
20 C 40 B 60 E 80 C

1. Which of the following anatomic features of the biliary system are important
considerations in operative cholangiography?
A. The left hepatic duct comes off farther anterior than the right one.
B. At the confluence there may be more than just a right and a left hepatic duct.
C. Dissection of the triangle of Calot is more important than cholangiography in
preventing bile duct injury.
D. Segments V, VII, or VIII sometimes join the biliary system below the confluence.
E. All of the above are correct.

2. Which of the following statements characterize amebic abscess?


A. Mortality is higher than that for similarly located pyogenic abscesses.
B. The diagnosis of amebic abscess may be based on serologic tests and resolution of
symptoms.
C. In contrast to pyogenic abscess, the treatment of amebic abscess is primarily
surgical.
D. Patients with amebic abscess tend to be older than those with pyogenic abscess.

3. Which of the following statement(s) is/are true about benign lesions of the liver?
A. Adenomas are true neoplasms with a predisposition for complications and should
usually be resected.
B. Focal nodular hyperplasia (FNH) is a neoplasm related to birth control pills (BCPs)
and usually requires resection.
C. Hemangiomas are the most common benign lesions of the liver that come to the
surgeon's attention.
D. Nodular regenerative hyperplasia does not usually accompany cirrhosis.
4. Which of the following statements about hemobilia are true?
A. Tumors are the most common cause.
B. The primary treatment of severe hemobilia is an operation.
C. Percutaneous cholangiographic hemobilia is usually minor.
D. Ultrasonography usually reveals a specific diagnosis.

5. Ligation of all of the following arteries usually causes significant hepatic enzyme
abnormalities except:
A. Ligation of the right hepatic artery.
B. Ligation of the left hepatic artery.
C. Ligation of the hepatic artery distal to the gastroduodenal branch.
D. Ligation of the hepatic artery proximal to the gastroduodenal artery

6. Which of the following is the most common acid-base disturbance in patients with
cirrhosis and portal hypertension?
C. Metabolic alkalosis.

7. Cytokines are endogenous signals that stimulate:


A. Septic shock
B. The central nervous system to initiate fever.
C. The production of “acute-phase proteins.”
D. Hypoferremia.
E. All of the above are correct.
8. Which of the following statements about maxillofacial trauma is/are false?
A. Asphyxia due to upper airway obstruction is the major cause of death from facial
injuries.
B. The mandible is the most common site of facial fracture.
C. The Le Fort II fracture includes a horizontal fracture of the maxilla along with
nasal bone fracture.
D. Loss of upward gaze may indicate either an orbital floor or orbital roof fracture.

9. A 28-year-old male was injured in a motorcycle accident in which he was not


wearing a helmet. On admission to the emergency room he was in severe respiratory
distress and hypotensive (blood pressure 80/40 mm. Hg), and appeared cyanotic . He
was bleeding profusely from the nose and had an obviously open femur fracture
with exposed bone . Breath sounds were decreased on the right side of the chest. The
initial management priority should be:
A. Control of hemorrhage with anterior and posterior nasal packing.
B. Tube thoracostomy in the right hemithorax.
C. Endotracheal intubation with in-line cervical traction.
D. Obtain intravenous access and begin emergency type O blood transfusions.
E. Obtain cross-table cervical spine film and chest film.
10. Regarding the diagnosis and treatment of cardiac tamponade, which of the
following statements is/are true?
A. Accumulation of greater than 250 ml. of blood in the pericardial sac is necessary to
impair cardiac output.
B. Beck's classic triad of signs of cardiac tamponade include distended neck veins,
pulsus paradoxicus, and hypotension.
C. Approximately 15% of needle pericardiocenteses give a false-negative result.
D. Cardiopulmonary bypass is required to repair most penetrating cardiac injuries.

11.Which of the following statement(s) is true concerning the diagnosis of a


peripheral vascular injury?
A. The presence of a Doppler signal over an artery in an extremity essentially rules
out an arterial injury
B. Doppler examination is a valuable tool in the diagnosis of venous injuries
C. A gunshot wound in the proximity of a major vessel is an absolute indication for
arteriography
D. Both the sensitivity and specificity of arteriography of the injured extremity
approaches 100%

12. An 18-year-old male suffers a gunshot wound to the abdomen, resulting in


multiple injuries to the small bowel and colon. Which of the following statement(s)
is/are true concerning this patient’s perioperative management?
A. A multi-agent antibiotic regimen is indicated
B. Antibiotics should be continued postoperatively for at least 7 days
C. Laparotomy, as a diagnostic test for postoperative sepsis, should be considered
D. The incidence of postoperative wound or intraabdominal infection would be
increased in association with a colon injury

13. A 75-year-old man is involved in a motor vehicle accident. Which of the following
statement(s) is/are true concerning this patient’s injury and management?
A. Acceptable vital sign parameters are similar across all age groups
B. Hypertonic solutions should not be used for resuscitation due to concerns for fluid
overload
C. The patient would be more prone to a subdural hematoma than a younger patient
D. There is no role for inotropic agents in the management of this patient

14. Valid points in the management of burns on special areas include:


A. The large majority of genital burns are best managed by immediate excision and
autografting
B. All digits with deep dermal and full-thickness burns should be immobilized with
six weeks of axial Kirschner wire fixation
C. Deep thermal burns of the central face are best managed with immediate excision
and autografting
D. Burns of the external ear are commonly complicated by acute suppurative
chondritis if topical mafenide acetate is not applied

15. Which of the following statements regarding unusual hernias is incorrect?


A. An obturator hernia may produce nerve compression diagnosed by a positive
Howship-Romberg sign.
B. Grynfeltt's hernia appears through the superior lumbar triangle, whereas Petit's
hernia occurs through the inferior lumbar triangle.
C. Sciatic hernias usually present with a painful groin mass below the inguinal
ligament.
D. Littre's hernia is defined by a Meckel's diverticulum presenting as the sole
component of the hernia sac.
E. Richter's hernia involves the antimesenteric surface of the intestine within the
hernia sac and may present with partial intestinal obstruction.

16. The following statement(s) is/are true concerning umbilical hernias in adults.
A. Most umbilical hernias in adults are the result of a congenital defect carried into
adulthood
B. A paraumbilical hernia typically occurs in multiparous females
C. The presence of ascites is a contraindication to elective umbilical hernia repair.
D. Incarceration is uncommon with umbilical hernias

17. Which of the following statements about esophageal anatomy is correct?


A. The esophagus has a poor blood supply, which is segmental in distribution and
accounts for the high incidence of anastomotic leakage.
B. The esophageal serosa consists of a thin layer of fibroareolar tissue.
C. The esophagus has two distinct muscle layers, an outer, longitudinal one and an
inner, circular one, which are striated in the upper third and smooth in the distal two
thirds.
D. Injury to the recurrent laryngeal nerve results in vocal cord dysfunction but does
not affect swallowing.
E. The lymphatic drainage of the esophagus is relatively sparse, localized primarily to
adjacent paraesophageal lymph nodes.

18. Which of the following is most reliable for confirming the occurrence of a
significant esophageal caustic injury?
A. History of the event.
B. Physical examination of the patient.
C. Barium esophagraphy.
D. Endoscopy.
19. Numerous epidemiologic associations have been made between (1) environmental
and dietary factors and (2) the incidence of gastric cancer, including all except:
A. Dietary nitrites.
B. Dietary salt.
C. Helicobacter pylori infection.
D. Dietary ascorbic acid.

20. Complete mechanical small bowel obstruction can cause dehydration by:
A. Interfering with oral intake of water.
B. Inducing vomiting.
C. Decreasing intestinal absorption of water.
D. Causing secretion of water into the intestinal lumen.
E. All of the above

21. Meckel's diverticulum most commonly presents as:


A. Gastrointestinal bleeding.
B. Obstruction.
C. Diverticulitis.
D. Intermittent abdominal pain.

22. A 45-year-old man with a history of previous right hemicolectomy for colon
cancer presents with colicky abdominal pain which has become constant over the
last few hours. He has marked abdominal distension and has had only minimal
vomiting of a feculent material. His abdomen is diffusely tender. Abdominal x-ray
shows multiple air fluid levels with dilatation of some loops to greater than 3 cm in
diameter. The most likely diagnosis is:
A. Proximal small bowel obstruction
B. Distal small bowel obstruction
C. Acute appendicitis
D. Closed-loop small bowel obstruction

23. Which of the following statement(s) is true concerning laboratory tests which
might be obtained in the patient discussed above?
A. The presence of a white blood cell count > 15,000 would be highly suggestive of a
closed-loop obstruction
B. Metabolic acidosis mandates emergency exploration
C. An elevation of BUN would suggest underlying renal dysfunction
D. There is no rapidly available test to distinguish tissue necrosis from simple bowel
obstruction

24. Recurrence after operation for Crohn's disease:


A. Occurs after operations for ileal Crohn's but not colonic Crohn's.
B. Is usually found just proximal to an enteric anastomosis.
C. Rarely requires reoperation.
D. Occurs in 1% of patients at risk per year during the first 10 years after the
operation.
E. Is prevented by maintenance therapy with corticosteroids.

25. The most common indication for surgery secondary to acute diverticulitis is:
A. Abscess.

26. Many patients with ulcerative colitis are operated upon electively with total
abdominal colectomy, rectal mucosectomy, formation of a small intestinal reservoir,
and ileoanal anastomosis. The most common postoperative complication after this
operation is which of the following?
B. Small bowel obstruction

27. A 30-year-old male two years postoperative after total abdominal colectomy with
ileoanal anastomosis reports a sudden increase in stool frequency, nocturnal leakage,
and low-grade fevers. Physical examination is unremarkable . Flexible endoscopic
examination of the small intestinal pouch reveals a friable erythematous mucosa
.Biopsies of the mucosa are obtained. While awaiting biopsy results, which of the
following is the most appropriate empiric therapy?
C. Oral metronidazole

28. Useful methods for detection of splenic injury, in descending order of sensitivity,
are:
A. Magnetic resonance imaging (MRI)
B. CT.
C. Ultrasonography.
D. Isotope scan.

29. Which of the following comments does not describe hypersplenism?


A. It may occur without underlying disease identification.
B. It may be secondary to many hematologic illnesses.
C. It is associated with work hypertrophy from immune response.
D. It requires evaluation of the myeloproliferation.
E. It is associated with antibodies against platelets.

30.You are consulted regarding a 50-year old male with Laennec’s cirrhosis, portal
hypertension and hypersplenism. He has no history of gastrointestinal bleeding. You
would recommend which of the following?
D. Observation
31. Which of the following statements about the differential diagnosis of
hypercalcemia is correct.
A. Malignant tumors typically cause hypercalcemia by ectopic production of
parathyroid hormone (PTH).
B. The diagnosis of primary hyperparathyroidism is supported by these serum levels:
calcium, 10.8 mg. per dl.; chloride, 104 mmol. per liter; bicarbonate 21 mmol. per liter;
phosphorus, 2.4 mg. per dl.; elevated parathyroid hormone.
C. Familial hypocalciuric hypercalcemia is distinguished from primary
hyperparathyroidism by parathyroid imaging.
D. Although serum albumin binds calcium, the measured total calcium value is
usually unaffected in patients with severe hypoproteinemia.
E. Thiazide diuretics are a good treatment for hypercalcemia and can be given to
patients with apparent hypercalcemia of malignancy.

32. The parathyroid glands:


A. Develop from the second and third pharyngeal pouches, along with the palatine
tonsil and the thymus.
B. Migrate caudally in the neck in normal development but can be found anywhere
from the pharyngeal mucosa to the deep mediastinum.
C. Secrete PTH and calcitonin to manage calcium homeostasis.
D. Usually number four, but frequently number only two or three.
E. Contain enzymes that catalyze the conversion of 25(OH) vitamin D 3 to 1,25(OH)
2 vitamin D 3.

33. Which of the following breast lesions are noninvasive malignancies?


A. Intraductal carcinoma of the comedo type.
B. Tubular carcinoma and mucinous carcinoma.
C. Infiltrating ductal carcinoma and lobular carcinoma.
D. Medullary carcinoma, including atypical medullary lesions.

34. A 45-year-old woman presents with a weeping eczematoid lesion of her nippl
E. Which of the following statement(s) is/are true concerning her diagnosis and
management?
A. Treatment is with warm compresses and oral antibiotics
B. Biopsy of the nipple revealing malignant cells within the milk ducts is invariably
associated with an underlying invasive carcinoma
C. The appropriate treatment is mastectomy
D. The lesion always represents a high-risk disease with a significant risk of
subsequent metastatic disease

35. The effect of high positive end-expiratory pressures (PEEP) on cardiac output is:
A. None.
B. Increased cardiac output.
C. Decreased cardiac output because of increased afterload to the left ventricle.
D. Decreased cardiac output because of decreased effective preload to the left
ventricle.

36. The diagnosis of myasthenia gravis can be confirmed most reliably using:
D. Single-fiber EMG.

37. In an infant with suspected PDA, which of the following would be the optimal
method of confirming the diagnosis?
D. Two-dimensional echocardiography with continuous-wave and color-flow Doppler
echocardiography.

38. A 5-year-old girl is found on routine examination to have a pulmonic flow


murmur, fixed splitting of P2 and a right ventricular lift. The following is/are true:
A. Cardiac catheterization is indicated if the chest film shows cardiomegaly
B. Radiology report of “scimitar syndrome” findings on the chest film would indicate
need for an arteriogram
C. If the catheterization report is “ostium secondum defect,” at least one pulmonary
vein drains anomalously
D. Measured pulmonary vascular resistance of 14 Woods units/m2 with an ASD
mandates early repair
E. An ASD with Qp/Qs of 1.8 can be observed until symptoms occur

39. The single most important indication for emergency pulmonary embolectomy is:
C. The presence of persistent and intractable hypotension.

40. A 67-year-old man with acute popliteal arterial embolism has a negative cardiac
echo for source of the thrombus. The following is/are true:
A. Most likely non-cardiac source is a thoracic aortic aneurysm
B. Embolism is more common from femoral than popliteal arterial aneurysms
C. Emboli from popliteal aneurysms are often clinically silent
D. Embolism is rare from subclavian artery aneurysms

41. Which of the following statements about allograft rejection are true?
A. In the absence of immunosuppression, the time and intensity of rejection of
transplants between unrelated donors and recipients is highly variable.
B. Allograft rejection may be mediated by antibodies or by cells.
C. Allograft rejection is thought to be caused by Th2 cells.
D. Acute cellular rejection is the major cause for loss of clinical organ transplants.
E. An individual with “tolerance” is unable to reject an allograft.

42. Which of the following are contraindications to lung transplantation?


A. Age 65 years or older.
B. Current corticosteroid therapy.
C. History of thoracotomy.
D. Ventilator-dependent respiratory failure.

43. Familial hypercholesterolemia has been proposed as a disease to be treated by


gene therapy. The molecular basis of familial hypercholesterolemia is which of the
following?
A. Absence of hepatic low density lipoprotein receptors
B. Overproduction of high density lipoprotein
C. Absence of lipoprotein lipase
D. Overproduction of hepatic ornithine transcarbamylase

44. The pentalogy of Cantrell includes all of the following except:


A. Epigastric omphalocele.
B. Sternal cleft.
C. Intracardiac defect.
D. Pericardial cyst.
E. Ectopia cordis.

45. Patients with Wilms’ tumors most frequently present with which of the
following?
E. A unifocal, unilateral lesion

46. Renal adenocarcinomas:


A. Are of transitional cell origin.
B. Usually are associated with anemia.
C. Are difficult to diagnose.
D. Are extremely radiosensitive.
E. Frequently are signaled by gross hematuria.

47. A 55-year-old male presents with severe flank pain radiating to the groin
associated with nausea and vomiting. Urinalysis reveals hematuria .A plain
abdominal film reveals a radiopaque 5 mm stone in the area of the ureterovesical
junction. Which of the following statement(s) is/are true concerning this patient’s
diagnosis and management?
A. A likely stone composition for this patient would be uric acid
B. The stone will likely pass spontaneously with the aid of increased hydration
C. Stone analysis is of relatively little importance
D. Patients with a calcium oxalate stone and a normal serum calcium level should
undergo further extensive metabolic evaluation
48. The most common physical findings in a patient with median nerve compression
at the wrist (carpal tunnel syndrome) are:
A. Diminished two-point discrimination and dryness of the index and long fingers.
B. Atrophy of the abductor pollicis brevis and opponens pollicis.
C. A positive percussion test at the wrist and a positive wrist flexion test producing
paresthesias at the thumb, index, and long fingers.
D. A weak grip in addition to hand cramping and difficulty writing.

49. Isolated flexor digitorum superficialis tendon function is determined by


assessing:
A. Flexion of the metacarpophalangeal joint.
B. Flexion of the proximal interphalangeal joint with the adjacent digits held in
extension.
C. Flexion of the distal interphalangeal joint.
D. Flexion of the proximal interphalangeal joint.
50. Which of the following statements are true?
A. The Hoffmann-Tinel sign localizes the level of a nerve injury.
B. Causalgia is a term used to denote the etiology of pain.
C. Secondary repair of a lacerated nerve 3 to 8 weeks after injury has several
advantages.
D. A surgeon who finds at delayed (3 to 8 weeks) exploration that a clinically
nonfunctioning nerve is in continuity should resect the injured portion of the nerve and
suture together the ends.
E. If a nerve is found to be disrupted at delayed (3 to 8 weeks) exploration, the
surgeon should find the two ends of the nerve and suture them together.

51. What is the critical difference between frame-based and frameless stereotactic
procedures?
A. The use of digitized imaging studies such as CT and MRI.
B. The use of rendered three-dimensional images and a three-dimensional digitizer.
C. Rigid fixation of the patient's head to the operating room table.
D. The presence of a lesion in the brain on digitized imaging studies.
E. The absence of a lesion in the brain on digitized imaging studies.

52. The following are absorbable sutures


A. Catgut
B. Silk
C. Polyamide (Nylon)
D. Polyglyconate (Maxon)
E. Polyglactin (Vicryl)

53. During surgery on the submandibular gland


A. An incision on the lower border of the mandible is safe
B. The submandibular gland is seen to wrap around the posterior border of mylohyoid
C. The facial artery and vein are divided as they course through the deep part of the
gland
D. The hypoglossal nerve is seen to loop under the submandibular duct
E. Damage to the lingual nerve will cause loss of sensation to the posterior third of the
tongue

54. Regarding pancreatic carcinoma false


A. 90% are ductal adenocarcinomas
B. Less than 20% occur in the head of the gland
C. The usual presentation is with pain, weight loss and obstructive jaundice
D. Ultrasound has a sensitivity of 80-90% in the detection of the tumour
E. Less than 20% of patients are suitable for curative surgery

55. Regarding the management of major trauma


A. Deaths follow a trimodal distribution
B. X-rays after the primary survey should be of AP Cervical spine, chest and pelvis
C. Cardiac tamponade is characterised by raised BP, low JVP and muffled heart
sounds
D. Assessment of uncomplicated limb fractures should occur during the primary
survey
E. Deterioration of the casualty during the primary survey should lead to the
secondary survey

56. Regarding appendicitis


A. The risk of developing the illness is greatest in childhood
B. Mortality decreases with age .
C. 20% of appendices are extraperitoneal in a retrocaecal position
D. Faecoliths are present in 75-80% of resected specimens
E. Appendicitis is a possible diagnosis in the absence of abdominal tenderness

57. Regarding stones in the gallbladder


A. Cholesterol stones are the least common
B. Pigment stones are due increased excretion of polymerised conjugated bilirubin
C. Are not a risk factor for the development of gallbladder carcinoma
D. 90% of gallstones are radio-opaque
E. A mucocele of the gallbladder is caused by a stone impacted in Hartmann's pouch

58. Stones in the common bile duct the false is


A. Are found in 30% of patients undergoing cholecystectomy (Without pre-op ERCP)
B. Can present with Charcot's Triad
C. Are suggested by an bile duct diameter >8mm on ultrasound
D. ERCP, sphincterotomy and balloon clearance is now the treatment of choice
E. If removed by exploration of the common bile duct the T-tube can be removed after
3 days
59. Regarding crystalloid solutions the false is
A. Normal saline contains 154 mmol sodium and 154 mmol of chloride
B. 3 litres of dextrose saline in a day will provide 90 mmol of sodium
C. 2 grams of potassium chloride is equal to 57 mmol of the salt
D. Hartmann's solution contains calcium and bicarbonate
E. The daily maintenance potassium requirement of a 40 Kg woman is about 40 mmol

60. Solitary thyroid nodules the false is


A. Are more prevalent in women
B. In the adult population less than 10% are malignant
C. Less than 20% of scintigraphically cold nodules are malignant
D. The risk of a hot module being malignant is negligible
E. Should be surgically removed in all patients

61. Regarding abdominal wall hernias the false is


A. Almost 100,000 hernia operations are performed annually in the United Kingdom
B. Over 60% of inguinal hernias are indirect
C. In women inguinal hernias are as common as femoral hernias
D. The mortality assocaited with strangulation is over 10%
E. The mortality has reduced dramatically over the past 30 years

62. The femoral canal the false is


A. Lies medial to the femoral vein
B. Has the inguinal ligament as its anterior border
C. Has the lacunar ligament as its lateral border
D. Has the pectineal ligament as its posterior border
E. Contains the lymph node of Cloquet

63. Intermittent claudication the false is


A. Affects less than 1% of men over the age of 50 years
B. At 5 years 10% of claudicants will have progressed to an amputation
C. At 5 years 20% of claudicants will have died from ischaemic heart disease
D. Is usually associated with an ankle / brachial pressure index (ABPI) 0.3- 0.7
E. Is associated with a fall in the ABPI on exercise with delayed recovery

64. The pathology of ulcerative colitis the false is


A. Shows full thickness inflammation
B. The rectum is almost always involved
C. 10% patients have terminal ileal disease
D. Enterocutaneous or intestinal fistulae are less common
E. The serosa is usually normal
65. Regarding benign breast disease the false is
A. Cyclical mastalgia is the commonest reason for referral to the breast clinic
B. Fibroadenomas are derived from the breast lobule
C. Lactational breast abscesses are usually due to Staph aureus
D. Duct ectasia is less common in smokers
E. Atypical lobular hyperplasia is associated with an increased risk of breast cancer

66. Regarding anal fissures the false is


A. 10% occur in the posterior midline
B. Multiple fissures suggest a diagnosis of tuberculosis or Crohn's Disease
C. 50% of acute fissures heal with the use of a bulking agent
D. Sphincterotomy has a success rate of over 90%
E. Sphincterotomy is associated with minor faecal incontinence in over 15% of
patients

67. Fibroadenomas the false is


A. Are benign monoclonal neoplasms
B. Most commonly present in late adolescence or the early 20s
C. Should be diagnosed by triple assessment
D. At least 30% reduce in size over a 2 year period
E. Can be safely managed conservatively

68. Warfarin the false is


A. Reduces the concentration of vitamin A dependent clotting factors
B. Has a half life of about 36 hours
C. Crosses the placenta and should be avoided in pregnancy
D. Doses should be reduced in liver disease
E. An INR of between 2.0 and 3.0 is appropriate in the treatment of DVT

69. Heparin the false is


A. Is a heterogeneous mixture of sulphated polysaccharides
B. Potentiates the actions of antithrombin III
C. Has a half life of 12 hours
D. Can be reversed by protamine sulphate
E. Can induce an idiosyncratic thrombocytopenia

70. The following cause hypercalcaemia except


A. Sarcoidosis
B. Primary hyperparathyroidism
C. Acute pancreatitis
D. Metastatic bronchial carcinoma
E. Milk-Alkali syndrome
71. Regarding oesophageal cancer the false is
A. Squamous carcinomas predominate in the upper 2/3 of the oesophagus
B. Overall 5 year survival is greater than 50%
C. Tylosis genetically predisposes to the disease
D. 15% of adenocarcinomas are associated with Barrett's oesophagus
E. For palliation an Atkinson tube can be inserted endoscopically

72. Infantile hypertrophic pyloric stenosis the false is


A. Occurs with a male : female ratio of 4:1.
B. Sons of affected mothers have a 20% risk of developing the lesion
C. Invariably presents between three and four weeks of age
D. Typically presents with nonbilious vomiting
E. Surgical treatment is by Heller's Cardiomyotomy

73. Oesophageal atresia the false is


A. Is often associated with a distal trachea-oesophageal fistula
B. Polyhydramnios is often present late in pregnancy
C. 50% have other associated congenital abnormalities
D. Contrast X-ray studies are necessary to confirm the diagnosis
E. Post-operatively over 30% develop oesophageal strictures

74. The following are features of Fallot's Tetralogy except


A. An atrial septal defect
B. Pulmonary stenosis
C. Right ventricular hypertrophy
D. A right to left cardiac shunt
E. Cyanotic attacks during feeding and crying

75. The functions of a tracheostomy are to except


A. Bypass an upper airway obstruction
B. Reduce the anatomical dead space
C. Increase airway resistance
D. Protect against aspiration
E. Allow frequent airway suction

76. Medullary carcinoma of the thyroid the false is


A. Is a tumour of the parafollicular C cells
B. Produce thyroxine as the principle hormone
C. 90% of cases are sporadic
D. Can occur as part of the MEN type II syndrome
E. Total thyroidectomy is the surgical treatment of choice
77. The following are features of thyrotoxicosis except
A. Weight gain
B. Palpitations
C. Proximal myopathy
D. Increased skin pigmentation
E. Pretibial myxoedema

78. Regarding abdominal aortic aneurysms the false is


A. They commonly remain symptomless until they rupture
B. The risk of rupture increases with aortic diameter
C. Elective repair should have a 30-day mortality of less than 5%
D. Emergency repair has a 30-day mortality of less than 10%
E. The benefits of surgery for small (4.0-5.5 cm) is unproven

79. In patients with carotid artery disease


A. A bruit is a reliable sign of the degree of stenosis
B. Atheroma is most commonly seen in the external carotid artery
C. An embolic event often results in an ipsilateral hemiplegia
D. Prophylactic aspirin reduces the risk of a stroke
E. Surgery is of proven benefit in those with asymptomatic stenoses

80. Hepatocellular carcinoma the false is


A. Has a high incidence in East Africa and South-east Asia
B. Its worldwide incidence parallels the prevalence of Hepatitis B
C. Mycotoxins (e.g. aflatoxin) are an important aetiological factor
D. Over 80% of tumours are surgically resectable
E. Liver transplantation offers the only chance of cure in those with irresectable
disease

81. Serum alpha fetoprotein the false is


A. Is increased in testicular tumours
B. In testicular tumours is produced by trophoblastic elements
C. Is increased in over 70% patients with hepatocellular carcinoma
D. In patients with hepatocellular carcinoma levels correlate well with size of the
tumour
E. In patients with hepatocellular carcinoma levels fall following resection of the
tumour

82. Regarding testicular tumours


A. They are the commonest malignancy in young men
B. Peak incidence for teratomas is seen at the age of 25 years
C. Seminomas are radiosensitive
D. Over 75% of Stage I teratomas are cured by surgery alone
E. Chemotherapy rarely produces a cure in those with metastatic disease

83. In patients with ascites the false is


A. A exudate has a protein content of greater than 30 g per litre
B. Both malignancy and infection result in a transudate
C. Ovarian carcinoma is the commonest malignant cause of ascites
D. Cirrhosis results in marked sodium retention
E. Tumour cells increase the permeability of the peritoneum

84. Regarding opiate analgesia the false is


A. Patient controlled analgesia (PCA) is more effective than intermittent parenteral
dosing
B. The total opiate dose is usually reduced with a PCA
C. Fentanyl is more lipid soluble than morphine
D. Epidural morphine can result in late respiratory depression
E. Epidural and parenteral morphine can be safely administered together

85. Intussusception the false is


A. Is most common in children from 6 to 12 years
B. Presents with colicky abdominal pain, rectal bleeding and an abdominal mass
C. 10% present with diarrhoea and vomiting suggestive of gastroenteritis
D. If no shock or peritonitis hydrostatic reduction can be attempted
E. A Meckel's diverticulum can induce an intussusception

86. Hirschsprung's disease the false is


A. Is due to absent ganglion cells in Auerbach's plexus
B. 10% cases have involvement of the recto-sigmoid segment
C. 80% cases present with delayed passage of meconium in the first 24 hours after
birth
D. The affected segment of bowel appears as cone-shaped contracted zone on barium
enema
E. On rectal biopsy there increased acetylcholinesterase containing cells in the
muscularis

87. Laparoscopic cholecystectomy the false is


A. Is usually performed using a four port technique
B. The Veress needle is an 'open' technique for inducing the pneumoperitoneum
C. A supraumbilical abdominal scar is a contraindication to laparoscopic
cholecystectomy
D. Dissection of Calot's triangle should be performed before the cystic duct is clipped
E. Most series report a conversion rate of less than 5%
88. In obstructive jaundice the false is
A. Urinary conjugated bilirubin is increased
B. Serum unconjugated bilirubin is increased
C. Urinary urobilinogen is reduced
D. Serum conjugated bilirubin is increased
E. Faecal stercobilinogen is reduced

89. Regarding the anatomy of the inguinal canal the false is


A. The internal ring lies midway between the symphysis pubis and anterior superior
iliac spine
B. The internal ring lies medial to the inferior epigastric vessels
C. The external oblique aponeurosis forms the anterior boundary
D. The inguinal ligament forms the inferior boundary
E. The conjoint tendon forms the medial part of the posterior wall

90. Papillary carcinoma of the thyroid the false is


A. Can be reliably diagnosed using fine needle aspiration cytology
B. Is almost always unifocal
C. Histologically displays Psammoma bodies
D. Typically spread to the cervical lymph nodes
E. Requires a total thyroidectomy for large tumours

91. Regarding bladder tumours the false is


A. 90% are squamous carcinomas
B. Painless haematuria is the commonest presentation
C. Cigarette smoking is an important aetiological factor
D. 80% of tumours are superficial (i.e no muscle invasion)
E. Superficial tumours are often well controlled by transurethral resection

92. Regarding ureteric calculi the false is


A. Are most often composed of calcium oxalate or phosphate
B. Most stones of those less than 5 mm in diameter pass spontaneously
C. Extracorporeal lithotripsy is useful for stones in the upper third of the ureter
D. About 30% of patients require open surgery to remove the stone
E. An obstructed ureter in the presence of infection is a surgical emergency

93. Regarding bladder calculi the false is


A. The incidence has fallen markedly in this country since the late 19th century
B. They may be totally asymptomatic
C. They are more prevalent in patients with chronic urinary sepsis
D. They are associated with squamous metaplasia of the bladder mucosa
E. They increase the risk of transitional cell carcinoma
94. Ectopic pregnancy the false is
A. Occurs in about 1% of pregnancies
B. The risk is increased in those with a history or pelvic inflammatory disease
C. Usually presents between 2 and 4 months of gestation
D. Patients usually have a positive pregnancy test
E. if shocked early laparotomy is essential

95. Tetanus the false is


A. Is due to an infection with a gram-positive spore forming rod
B. The organism produces a powerful endotoxin
C. The toxin prevents the release of inhibitory neurotransmitter
D. Clostridium tetani is sensitive to penicillin

96. Regarding pathological terms


A. Hypertrophy is an increase in tissue size due to increased cell number
B. Hyperplasia is an increase in tissue size due to an increase in cell size
C. Atrophy is an increase in tissue size to disuse
D. Metaplasia is a change form one abnormal tissue type to another
E. A hamartoma is a developmental abnormality

1 E 16 B 31 B 46 E 61 E 76 B 91 A
2 B 17 C 32 B 47 B 62 C 77 A 92 D
3 A 18 D 33 A 48 C 63 A 78 E 93 E
4 C 19 D 34 C 49 B 64 A 79 D 94 C
5 D 20 E 35 D 50 C 65 D 80 D 95 B
6 C 21 A 36 D 51 B 66 A 81 B 96 E
7 E 22 B 37 D 52 A+D+E 67 A 82 E
8 B 23 D 38 B 53 B 68 A 83 B
9 C 24 B 39 C 54 B 69 C 84 E
10 C 25 A 40 C 55 A 70 C 85 A
11 D 26 B 41 B 56 B 71 B 86 B
12 D 27 C 42 D 57 E 72 E 87 B
13 C 28 B 43 A 58 E 73 D 88 B
14 D 29 E 44 D 59 C 74 A 89 B
15 C 30 D 45 E 60 E 75 C 90 B

1. The ilio-inguinal nerve:


A. supplies the rectus abdominis muscle
B. supplies skin on inner side of thigh
C. supplies the cremasteric muscle
D. supplies the urethra
E. does none of the above

2. The skin of the tip of the index finger is supplied by the:


A. Radial nerve only
B. Radial & median nerves
C. Median & ulnar nerves
D. Ulnar nerve only
E. Median nerve only

3. Hypovolaemic shock is characterized by:


A. A low central venous pressure , high cardiac output, low peripheral resistance
B. A high central venous pressure, high cardiac output, high peripheral resistance
C. A low central venous pressure , low cardiac output, high peripheral resistance
D. A low central venous pressure , high cardiac output, high peripheral resistance
E. A high central venous pressure, low cardiac output, low peripheral resistance

4. Which of the following in NOT true of Hodgkin's disease?


A. Usually starts from several groups of nodes simultaneously
B. Usually involves liver & spleen
C. Sometimes manifests itself as pyrexia of unknown origin
D. Severe pain follows ingestion of alcohol
E. Shows increased susceptibility to opportunistic infection

5. Tetanus toxoid:
A. Is produced by injecting animals with antititanic serum
B. Is administered to previously immunized subjects with potentially infected
wounds
C. Frequently gives rise to anaphylactic reaction
D. Is used to induce active immunity against tetanus

6. The most probable cause of shock in a patient with multiple injuries &
craniocerebral trauma is:
A. Depression of vital medullary centres
B. Hypoperfuion control over subcortical areas
C. Loss of cortical control over subcortical areas
D. Hypovolaemia
E. Inadequate ADH response

7. The most sensitive guide to acute changes in fluid balance in a surgical patient is:
A. Accurate daily weight
B. Serial serum Na concentration
C. Fluid balance sheets recording inputs & outputs
D. Daily urine output
E. Serial anion gap measurements
8. cellullitis is:
A. Inflammation of the bone marrow
B. Inflammation of the mastoid cells
C. Inflammation of the subcutaneous tissues
D. Infiltration of the skin by gaint cells
E. A malignant condition
9. secondary haemorrhage occurs:
A. Within 6 hours of operation
B. 7-14 days after operation
C. As a result of violent coughing on recovery from anaesthesia
D. Due to a blood transfusion line disconnected
E. When a ligature slips

10. the minimum urine output for 24 hours required to excrete end products of
protein metabolism is:
A. 200 ml
B. 300 ml
C. 400 ml
D. 500 ml
E. 600 ml

11. Potassium deficiency is present if the plasma-potassium level is:


A. 6.0 mmol/l
B. 5.0 mmol/l
C. 4.5 mmol/l
D. 4.0 mmol/l
E. 3.0 mmol/l

12. in health the pH of the blood lies between the range:


A. pH 7.05-7.19
B. pH 7.20-7.35
C. pH 7.36-744
D. pH 7.45-7.59
E. pH 7.60-7.80

13. TNM classification of a malignant tumour was designed as:


A. An histological staging
B. A clinical staging
C. A staging carried out at operation
D. A staging dependent upon radio scanning & skeletal survey
E. A staging dependent upon ultrasound

14. a blue-green discharge from an ulcer will be seen to contain:


A. Pseudomonas pyocyaneus
B. Streptococcus viridians
C. Candida albicans
D. Staphylococcus aureus
E. Haemophilius influenzae
15. a rodent ulcer is:
A. A squamous cell carcinoma
B. A basal cell carcinoma
C. Only occur on the face
D. Contains epithelial pearls
E. A venous ulcer

16. the space of Parona is:


A. In the wrist between the deep flexor tendons & the pronator quadratus
B. Above the patella between the quadriceps muscle & the femur
C. Benath the tendon of the iliopsos
D. Between the achills tendon & the posterior aspect of the tibia
E. The web space of the palm

17. 'rest pain' occurs:


A. Anywhere in the body at rest
B. In the thigh of a patient with Buerger's disease
C. In the calf of a patient with intermittent claudicating
D. In the foot of a patient with severe vascular disease
E. In the back

18. ischaemia means:


A. Pain in the ischial tuberosities
B. Anaemia due to malignant seconderies in the ischial part of the pelvis
C. Lack of blood flow
D. Increased blood flow
E. Polycythaemia

19. Colles' fracture is:


A. A common in adolescence
B. A fracture about the ankle joint
C. Common in elderly women
D. A fracture of the head of the radius
E. A fracture of scaphoid

20. Pott's disease is;


A. A fracture dislocation about the ankle
B. A neuropathic joint
C. Traumatic ostechondritis of the spine
D. Tuberculosis of the spine
E. A secondary tumour in the skull

21. Vincent's angina is a form of angina associated with:


A. Spasm of the oesophagus
B. Diphtheria
C. An infection of the mouth
D. Coronary artery spasm
E. Carcinoma of the bronchus
22. Ludwig's angina is due to :
A. A type of coronary artery spasm
B. Oesophageal spasm
C. Retropharyngeal infection
D. A virulent infection of the cellular tissue around the submandibular salivary gland
E. Infection with candida

23. in simple nodular goiter:


A. Carcinoma occurs in 30% of cases
B. The nodular stage is irreversible
C. Operation is contraindicated
D. The patient does not develop hyperthyroidism
E. Cretinism is the presenting feature

24. Hashimoto's disease is:


A. A granulomatous thyroiditis
B. An auto-immune thyroiditis
C. An infiltrating fibrosis of the thyrois & the adjacent muscles
D. Focal thyroiditis
E. A parathyroid tumour

25. A thyroglossal fistula:


A. Is never congenital
B. Follows inadequate removal of a thyroglossal cyst
C. Has a hood of skin with its concavity upwards
D. Is lined throughout by squamous epithelium
E. Occurs in carcinoma of the tongue

26. The following are clinical signs supporting an early diagnosis of carcinoma of the
breast:
A. A prickling sensation in a breast lump
B. Peau d'ornge
C. Brawny arm
D. Cancer en cuirasse
E. A krukenderg tumour

27. The gastroduodenal artery is a branch of the:


A. Celiac axis
B. Hepatic artery
C. Superior mesenteric artery
D. Gastroepiploic artery
E. Splenic artery
28. Chronic gastric ulcers most often occur in patients with:
A. Blood groub A
B. Tend to occur in alkaline mucosa
C. Muscularis mucosae is separated from the muscularis at the edge of the ulcer
D. Are malignant when there is epithelial proliferation & downgrowths
E. Are never large
29. Meckel's diverticulum:
A. Is present in 20% of the human race
B. Arises from the mesenteric border of the jejunum
C. May contain heterotopic pancreas
D. Is only present in the male sex
E. Is a diverticulum of the bladder

30. Intussusception is related to:


A. Mucoviscidosis
B. Swollen Peyer's patches
C. Volvulus
D. A littre's hernia
E. A patent vitello intestinal duct

31. The site of the neck of a femoral hernia is the:


A. Transversalia fascia
B. Iliopectineal ligament
C. Femoral ring
D. Cribriform fascia
E. Obturator foramen

32. Regarding operation for an indirect inguinal hernia:


A. It should not be performed on patients who have chronic bronchitis
B. General anaesthesia has to be used
C. In infants the posterior inguinal wall should be repaired
D. In adults the internal inguinal ring usually needs to be strengthened
E. Mesh implants are mandatory

33. Cystic fibrosis of the pancreas


A. is inherited as autosomal recessive
B. islets of Langerhans are affected
C. diagnosis is pssible by DNA analysis
D. all the above
E. a and c only

34. the following are clinical criteria of Bank ad Wise


A. pulmonary
B. neurological
C. renal
D. all the above
E. a and c only

35. antibiotics in acute pancreatitis


A. must be given to all cases
B. should not be given
C. given only to modarate & sever cases
D. should cover anaerobic & Gam positive bactria
E. none of the above
36. pancreatic carcinoma
A. occure usually at the age of 50years
B. usually is cystadenocarcinoma
C. more favaorable in the body and tail
D. less favourable if occure with back pain
E. all the above

37. infected pancreatic necrosis


A. is a collection of pus around the pancreas
B. diagnosed by CT scan
C. more serious than pancreatic abscess
D. all the above
E. none of the above

38. secondarey survay in polytrauma


A. should be done in the first hour
B. done as soon as the patient is stable
C. detects life threateing problems
D. a and c
E. none of the above

39. spontaneous ruptur of the spleen


A. occure less in tropical countries
B. the spleen may be enlarged
C. malaria is a common cause
D. all the above

40. spleic aretry aneurysm


A. incidence is around 5%
B. usually symptomless
C. more conmmon in males
D. all the above
41. wolf skin graft
A. used to cover large area of burn
B. success rate is less than split thckness graft
C. both thighs are common donor areas
D. usually contracts

42. medullary carcinoma of the thyroid


A. is a tumor of the G cells
B. familial in 70% of the cases
C. diagosed by low level of calcitonin
D. amyloid stroma is charactrestic

43. solitary thyroid nodule


A. 70% are follicular adenoma
B. 30% are malignant
C. more favourable in young males
D. all the above
E. none of the above

44. rupture of the diaphragm


A. is a life threatenig problem
B. usually missed
C. poly galctan suture is used for repair
D. all the above
E. noe of the above

45. Complication of vascular graft includes All the following except:


A. Infection
B. Aneurysm
C. Graft failure
D. Hemorrhage
E. a & c only

46. Gangrene:
A. is necrosis of tissue
B. The cause may be venous occlusion
C. usually painful
D. All the above
E. None of the above

47. Lynphedema:
A. may be Congenital
B. should be bilateral
C. may be pitting in early stage
D. A & C only
E. None of the above

48. Suprapubic cystostomy:


A. indicated in case of bladder out flow obstruction
B. indicated in case of urethral injury
C. indicated in case of urethral stricture
D. All the above
E. B& C only

49. differential diagnosis of Acute scrotal swelling in young adult includes all the
follow except:
A. Incarcerated inguinal hernia .
B. Torsion of testes
C. Acute epididymo- orchitis
D. Teratoma
E. Seminoma
50. Horizontal spread of infection across the external sphincter can result in which
type of anorectal abscess:
A. ischiorectal
B. perianal
C. supralevator
D. intersphincteric
E. intermuscular

51. All of the following statements are true of diffuse esophageal spasm EXCEPT:
A. chest pain is frequently seen
B. high amplitude esophageal contraction are present.
C. it is best diagnosed with barium esophogram.
D. usual surgical treatment is long esohagomyotomy.
E. most patients do not have significant coronary artery disease.

52. The treatment of an esophageal burn with a caustic agent may include all of the
following EXCEPT:
A. expeditious administration of an antidote.
B. induction of vomiting.
C. steroids and antibiotics.
D. bougienage.
E. gastrectomy.

53. All of the following substances are irritating to the peritoneum EXCEPT:
A. bile.
B. meconium.
C. blood.
D. gastric content.
E. pus.

54. Complications of truncal vagotomy and pyloroplasty include all of the following
EXCEPT:
A. dumping syndrome.
B. recurrent ulcer.
C. diarrhea.
D. alkaline reflux gastritis.
E. steatorrhea.

55. Gastric polyps:


A. are most commonly adenomatous.
B. require gastrotomy and removal if greater than 2 cm and are pedunculated.
C. are rarely multiple.
D. are clearly premalignant.
E. are more frequent in achlorhydric patients.

56. Vascular compression of the duodenum resulting in obstruction:


A. is present primarily in patients who are overweight.
B. should be given a trial of conservative management.
C. is common in pediatric patients.
D. is best diagnosed by identifying a "double bubble" sign on abdominal x-ray.
E. includes as medical therapy lying in the supine position after meals
57. Conditions associated with gastric cancer include all of the following EXCEPT:
A. higher socioeconomic groups.
B. pernicious anemia.
C. chronic atrophic gastritis.
D. adenomatous polyps.
E. a high intake of dietary nitrates

58. Patients with morbid obesity have an increased incidence of all of the following
EXCEPT:
A. gastric carcinoma.
B. diabetes.
C. stroke
D. gallbladder disease.
E. joint deterioration.

59. All of the following contribute to malabsorption following truncal vagotomy and
antrectomy EXCEPT:
A. increased rate of gastric emptying.
B. poor mixing of pancreatic secretions and bile salts with food.
C. increased release of secretions and bile salts with food.
D. decreased small intestinal transit time.
E. malabsorption of fat and carbohydrates.

60. All of the following statements are true about patients with carcinoid tumors
EXCEPT:
A. they often have evidence of serotonin production.
B. tumor growth is often slow.
C. the majority have carcinoid syndrome.
D. they have a much better prognosis if the tumors are less than 2 cm.
E. the combination of streptozotocin and 5-fluorouracil can often result in objective
response.

61. The followings are usually associated with acute appendicitis EXCEPT
A. Abdominal pain and nausea
B. CT scan with water soluble enema is the most helpful imaging study
C. Deep right lower abdominal tenderness by rectal examination
D. Positive Rovsing sign
E. WBCs around 20,000/mm3

62. The least problem that cause right lower abdominal pain in a 18 years female is
A. Ectopic pregnancy
B. Acute appendicitis
C. Ovarian torsion
D. Perforated peptic ulcer
E. Mittleschmerz

63. A patient with high hichitic fever, severe tenderness and rigidity at the right ileac
fossa, WBCs 18000/mm3 and abdominal ultrasound showed a heterogeneous mass in
the right iliac fossa with a central fluid collection. Management may include any of
the followings EXCEPT
A. Open drainage
B. I.V antibiotics
C. Appendix should be resected in the open drainage
D. Percutaneous drainage under U/S or CT guidance
E. Oral feeding shouldn’t be delayed in the absence of ileus

64. Regarding the lower esophageal sphincter the following are correct EXCEPT
A. It is a physiological sphincter
B. Located in the distal 3-5cm of the esophagus
C. Has a resting pressure of 20-60 mmHg
D. Abdominal pressure play a role
E. Its pressure can be estimated by esophageal manometry
65. In esophageal perforation, the most sensitive diagnostic study is
A. Plain film abdomen
B. Plain film chest and neck
C. Esophagogram
D. Esophagoscopy
E. CT chest and neck

66. Achalasia can be presented with all of the followings EXCEPT


A. Recurrent pulmonary infections
B. Weight loss
C. Regurgitation
D. Irregular narrowing of the distal esophagus by Ba. Swallow
E. Retrosternal chest pain

67. Complications of reflux esophagitis include the followings EXCEPT


A. Dysmotility
B. Schatzki's ring
C. Barrett's esophagus
D. Epiphrenic esophageal diverticulum
E. Hemorrhage

68. The pressure in the lower esophageal sphincter decreases by all of the following
EXCEPT
A. Alcohol
B. Nitroglycerin
C. Anticholinergics
D. Alpha adrenergics
E. Cholecystokinin hormone

69. The genetic predisposing factors to gastric cancer include the followings
EXCEPT
A. Family history of gastric cancer
B. Black race
C. P 53 mutation
D. Germline mutation of e-cadherin
E. BRCA2 mutation

70. Regarding the diffuse gastric cancer, the followings are true EXCEPT
A. The commonest type of gastric adenocarcinoma
B. Not associated with intestinal metaplasia
C. More incidence in young ages
D. Less related to environmental influences
E. Results from single cell mutation

71. The best diagnostic study for gastric adenocarcinoma


A. Upper endoscopy
B. Endoscopic ultrasound
C. Upper gastrointestinal double contrast barium study
D. Laparoscopy
E. Abdominal CT scanning

72. In primary gastrointestinal stromal tumors (GIST), which is NOT true


A. The most common site is the stomach
B. Bleeding is the commonest manifestation
C. Almost never metastasize to regional lymph nodes
D. The traditional cytotoxic chemotherapy greatly suppresses its growth
E. Endoscopic ultrasound guided FNAC gives the definitive diagnosis

73. Primary gastric lymphoma


A. 2% of all hodgkin's lymphoma
B. Greatly differs in presentation from gastric adenocarcinoma
C. One third of all gastrointestinal lymphomas
D. The most common extranodal lymphoma
E. Worse prognosis than adenocarcinoma

74. In gastric adenocarcinoma which is NOT true


A. Risk increased 3—6 times in patient with gastric H pylori infection
B. Blumer's shelf results from omental invasion in pelvic cavity
C. Krukenburg's tumor is due to ovarian metastasis
D. Weight loss and vague abdominal pain are the commonest presentation
E. Endoscopic ultrasound is 90% accurate in determining T stage

75. Mucosa associated lymphoid tissue ( MALT ) is found in all of the followings
EXCEPT
A. Small bowel ( Peyer's patches )
B. Waldeyer's ring
C. Appendix
D. Stomach
E. Bronchus
76. All of the following are associated with Barrett's esophagus EXCEPT
A. GERD
B. Squamous carcinoma
C. Esophageal mucosal dysplasia
D. Increased incidence of p53 mutations
E. adenocarcinoma
77. The best test to establish the presence of gastroesophageal reflux (GERD) disease
is
A. An upper gastrointestinal series
B. Bernstein test (acid perfusion)
C. 24-hour pH monitoring
D. Esophageal manometry
E. Endoscopic biopsy

78. The T and N status of esophageal carcinoma is most accurately assessed by


A. Upper gastrointestinal series
B. Computed tomographic scan of the chest with double contrast
C. Endoscopic ultrasound (EUS)
D. Positron emission tomography (PET scan)
E. Magnetic resonance imaging (MRI)

79. Salivary gland stones


A. Most arise in the sublingual gland
B. Usually present with persistent pain
C. Are composed predominantly of magnesium phosphate
D. Predispose to infection of the involved gland
E. Never occur in parotid duct

80. 35-year-old alcoholic is admitted with acute pancreatitis. He complains of


Numbness of his fingers and toes. On examination he has hyperactive Tendon
reflexes. The most likely cause of these symptoms is
A. Hyponatremia
B. Hypocalcemia
C. Hypophosphatemia
D. Hypermagnesemia
E. Hyperkalemia

81. Regarding benign salivary gland adenomas


A. Pleomorphic adenomas are only seen in parotid gland
B. Pleomorphic adenomas cannot undergo malignant change
C. Warthin's tumor is otherwise known as an adenolymphoma
D. Adenolymphomas usually occur in young men
E. Adenolymphomas are often bilateral

82. Massive lower gastrointestinal hemorrhage


A. Rarely stops after resuscitation
B. Is caused by large bowel lesions only
C. can be most accurately localized by colonoscopy
D. Is frequently related to right colon lesions
E. Is most commonly caused by adenocarcinoma of the large intestine
83. Regarding carcinoid tumors all of the following are true EXCEPT
A. Liver metastases can result in the carcinoid syndrome
B. The appendix is the commonest primary site for gastrointestinal tumors
C. Gastric carcinoid tumors produce little 5-hydroxyindoleacetic acid
D. If discovered in the appendix right hemicolectomy should always be done
E. Octreotide scintigraphy may identify both the primary and secondary lesions

84. A Meckel's diverticulum


A. Occurs in 10% of the population
B. Will be found on the mesenteric border of the small intestine
C. Consists of mucosa without a muscle coat
D. Usually found 20 cm from ileocecal valve
E. A fibrous band between the apex and umbilicus can cause intestinal obstruction

85. In overwhelming post-splenectomy infection, which is NOT true


A. Is usually due to unencapsulated bacterial infection
B. Strep. pneumonia is the commonest etiological agent
C. Despite aggressive therapy it can have a mortality of over 50%
D. The risk of infection can be reduced with pneumococcal and hemophilus
vaccination
E. Penicillin antibiotic prophylaxis should be considered in all children

86. Clostridium tetani


A. Is a gram-negative rod
B. Is sensitive to penicillin
C. Is available in the expired canned food
D. Releases a heat-resistant endotoxin
E. The toxin acts on the post-synaptic membrane of inhibitory nerve fibers

87. Grade I hypovolemic shock


A. Occurs when more than 40% of the circulating blood volume has been lost
B. Tachycardia is a reliable clinical sign
C. A fall in pulse pressure is observed
D. Urine output is markedly reduced
E. Capillary return is delayed

88. Regarding Hydatid disease the followings are true EXCEPT


A. Recently surgery is no more the treatment of choice in liver disease
B. Man is an accidental intermediate host
C. The liver is the commonest site of infection
D. Can be diagnosed by the ELISA test
E. Treatment by benzimidazoles alone is 30% successful

89. In peptic ulcer disease


A. H. pylori is a gram-positive bacillus
B. Surgery is always indicated in perforated ulcers
C. Sepsis after perforation is the commonest cause of death
D. Parietal cell vagotomy carries the highest rate of recurrence
E. Gastric drainage should be done in all types of vagotomies
90. Stones in the common bile duct, which is NOT true
A. Are mostly secondary stones
B. Can present with Charcot's Triad
C. Are suggested by a bile duct diameter >8mm on ultrasound
D. ERCP, sphincterotomy and balloon clearance is now the treatment of choice
E. If removed by exploration of the common bile duct the T-tube can be removed after
3 days

91. Regarding pancreatic carcinoma the followings are true EXCEPT


A. 90% are ductal adenocarcinomas
B. Less than 20% occur in the head of the gland
C. The usual presentation is with pain, weight loss and obstructive jaundice
D. Ultrasound has a sensitivity of 80-90% in the detection of the tumor
E. Less than 20% of patients are suitable for curative surgery

92. Regarding anal fissures all are true EXCEPT


A. 10% occur in the posterior midline
B. Multiple fissures suggest a diagnosis of tuberculosis or Crohn's Disease
C. 50% of acute fissures heal with the use of a bulking agent
D. Sphincterotomy has a success rate of over 90%
E. Sphincterotomy is associated with minor fecal incontinence in over 15% of patients

93. Medullary carcinoma of the thyroid


A. Concentrates Iodine 131
B. Produce thyroxine as the principle hormone
C. 10% of cases are sporadic
D. 90% can occur in association with MEN type II syndrome
E. All patients will need a 24-hour VMA level in urine

94. Which of the following is LEAST likely to be associated with the systemic
inflammatory response syndrome (SIRS)
A. Infection
B. Elevated/depressed temperature
C. Elevated heart rate
D. Elevated respiratory rate
E. Elevated/depressed WBC count

95. In papillary carcinoma of the thyroid the followings are true EXCEPT
A. Can be reliably diagnosed using fine needle aspiration cytology
B. Is almost always unifocal
C. Histologically displays Psammoma bodies
D. Typically spread to the cervical lymph nodes
E. Requires a total thyroidectomy for large tumors

96. Serum alpha-fetoprotein is increased in the following EXCEPT


A. Acute hepatitis
B. Hepatocellular carcinoma
C. Neuroblastoma
D. Teratomas
E. Bladder carcinoma
97. The following predispose to wound infection EXCEPT
A. Malnutrition
B. Hypovolemia
C. Malignancy
D. Obstructive jaundice
E. Steroid therapy

98. Regarding acute respiratory distress syndrome (ARDS) which is NOT true
A. Hypoxia in spite of high inspired oxygen
B. Increased lung compliance
C. Non-cardiac edema
D. Diffuse or patchy infiltrates in chest X ray
E. Deposition of proteinaceous fluid in the respiratory membrane

99. Which of the following statements about serum thyroglobulin is TRUE?


A. Elevation after total thyroidectomy justify the use of iodine131 therapy
B. It suppresses thyroid-stimulating hormone (TSH)
C. It suppresses thyroid-releasing hormone (TRH)
D. It is an effective tumor marker because it is specifically elaborated by the
malignant cells of papillary cancer
E. It is only about 50% effective in detecting recurrence

100. Women who have hereditary nonpolyposis colorectal cancer (HNPCC) should
also be screened for
A. Endometrial cancer
B. Papillary cancer of thyroid
C. Ampullary cancer
D. Pheochromocytoma
E. Hepatoma

1 B 21 C 41 B 61 E 81 C
2 E 22 D 42 D 62 D 82 D
3 C 23 D 43 E 63 C 83 D
4 A 24 D 44 B 64 C 84 E
5 D 25 A+B 45 E 65 D 85 A
6 D 26 B 46 B 66 D 86 B
7 D 27 B 47 D 67 D 87 B
8 C 28 D 48 D 68 D 88 A
9 B 29 C 49 E 69 E 89 D
10 E 30 B 50 A 70 A 90 E
11 E 31 C 51 C 71 A 91 B
12 C 32 D 52 B 72 D 92 A
13 B 33 E 53 C 73 D 93 E
14 A 34 D 54 E 74 B 94 A
15 B 35 C 55 D 75 D 95 B
16 A 36 D 56 B 76 B 96 E
17 D 37 C 57 A 77 C 97 D
18 C 38 E 58 A 78 C 98 B
19 D 39 C 59 C 79 D 99 A
20 D 40 B 60 C 80 B 100 A

OSCE , 6th yr Female group


2nd semester , 20/4/2005

1)- picture of varicose vein


a. Dx
b. name the structure involved
c. name 2 surgical treatment
d. 2 complications

2)- X- ray of tension pneumothorax


20 yrs old boy , presented with acute dyspnea & chest pain .
no Hx of TRAUMA
a. name 3 diagnostic features from X- ray
b. what is your initial invasive procedure
c. mention 2 causes of this condition

3)- X- ray of
a. name the study  Intraoperative cholangiogram
b. name the labeled structures
1- common hepatic duct
2- CBD
3- Dudenum
c. what is the pathology  stone in CBD

4)- 35 yrs old postpartum female complain of leg pain 3


days postpartum with tenderness
a. give 2 D.Dx
b. if calf ms was tender & hot what is your
spot Dx  DVT
c. Best investigation in this particular pt  Dopplar US
NOT VENOGRAM B/C LEG WAS ULCERATED
d. mention 4 risk factors

5)- Rt swelling in the groin


a. give 4 D.Dx
b. if cough impulse was +ve & non tender , what is the
complication ?
c. name 2 surgical procedure
d. name 2 non- pulmonary & non- systemic complication in
the 1st 24 hrs post op  1. acute urine retention
2. Bleeding

6)- picture of man with distended abdomen


Indonesian male , 40 yrs old , presented with progressive
abdominal distention
a. what is the most appropriate explanation for this case
 Ascitis d/t abdominal pathology
b. give 2 causes  1. TB
2. lymphoma

7)- X- ray of abdomen


a. spot Dx  small intestinal obstruction
b. name 4 imp managements other than bed rest

8)- picture of breast with tethered skin , retracted nipple &


erythema
a. spot Dx  Breast Ca
b. 3 loco-regional evaluative measures to Dx
c. 3 investigations to know extent of the disease
d. appropriate management

9)- Back of neck with erythema , diffuse swelling &


multiple sinuses discharge , pt is diabetic
a. Dx  Carbuncle
b. 3 imp managements for this pt :
1. admit
2. surgical excision
3. antibiotic + control sugar level

10)- picture of Single thyroid swelling progressive increase


in size
a. give 3 D.Dx of thyroid swelling
b. imp investigation
c. if TSH was normal , name 2 cost-effective investigatory
methods  1. US
2. FNA

GOOD LUCK

HANAN AL-JERAINI 2002040034


AMAL AL-MOHAWES 2002040047

Final 602 Surgical Exam


6th yr Female group
2nd semester 23/April /2005

 Most of the questions were about TRAUMA


 There were some repeated questions but not that much
as used B4

1)- symptoms & signs of pheochromocytoma except :


a. palpitation
b. ANHYDROSIS *
c. episodic hypertension
d. headach

2)- for purpose of trauma management , neck is divided


into how many zones :
a. 2
b. 3*
c. 4
d. 5

3)- one of the following is a consistent finding with acute


appendicitis :
a. tenderness in Rt iliac fossa *
b. oarasthesia in Rt iliac fossa
c. ??
d. ??

4) 20 yrs old male presented with dragging pain in Lt


hypochondrium , investigation ………
What is the cost-effective tool :
a. MRI
b. CT
c. US *
d. ??

5)- pt in Q 4 develop fever , palpitation …. 3 days later &


US shows hypoechogenicity , your Dx :
a. abscess *
b. cyst
c. ??
d. ??

6)- if this pt in Q4 will go for splenectomy , what is your


pre-op preparation :
a. ??
b. pneumococcal vaccine *
c. ??
d. ??

7)- which one of the following goes in the pathway of


spermatic cord :
a. femoral hernia
b. direct inguinal hernia
c. indirect inguinal hernia *
d. ??

8)- middle age female , P5+2 , presented with susage like


swelling , extending from xiphoid to umbilicus along linea
alba line , soft , reducible that is obvious when contracting
anterior abdominal wall . your Dx :
a. spiglian hernia
b. divercation of recti *
c. ??
d. ??

9)- middle age female presented with Rt hypochondrial


pain , jaundice , Temp = 38 C …….. , your clinical Dx is:
a. pancreatitis
b. ascending cholangitis *
c. cholilithiasis
d. ??

10)- what is not relevant for her case in the following


investigations :
a. serum amylase & lipase *
b. US
c. cholangiogram all needed in Dx
d. ??

11)- young male , bilharzias , presented with hematemesis


of 2 days duration …….
What do u think the cause of this ?
a. ??
b. Pre-sinusoidal portal HTN
c. Post sinusoidal portal HTN
d. ??

12)- 60 yrs old female presented with solitary thyroid


nodule that grows slowly over months .
What is the best investigation to know its nature ?
a. FNA
b. US
c. CT
d. ??

13)- pt in Q12 FNA was follicular cells what is your


management ?
a. total thyroidectomy
b. lumbectomy & isthmus resection
c. ??
d. ??

14)- which of the following doesn’t contribute in the


formation of psoas abscess :
a. origin
b. insertion
c. relation to appendix
d. relation to sigmoid

15)- commonest site of appendix is :


a. retro-cecal *
b. pelvic
c. para-iliac
d. pre-iliac

16)- fracture of 1st , 2nd , 3 rd ribs signifies what ?


a. severity of trauma * (Dr.‫ ييييييي‬note )
b. lung involvement
c. injury to large vessel
d. ??

17)- which one of the following is clean-contaminated :


a. cholecystectomy *
b. peritionitis
c. pus
d. appendix
1

A 44-year-old man presents with painless rectal bleeding of 1 month's duration. He


reports a history of constipation. He works in heavy labor.
For this patient, which of the following statements regarding internal hemorrhoids is
true?

Choose one answer


i. Stapled hemorrhoidectomy should be done for grade 1 and 2 hemorrhoids
j. All of the above
k. Internal hemorrhoids are located proximal to the dentate line and therefore are
usually painless
l. A grade 1 internal hemorrhoid represents bleeding with prolapse

39. A 34-year-old woman presents for evaluation of severe and frequent bloody
bowel movements, as well as abdominal pain, dehydration, and anemia. She has had
these symptoms for 2 days. She has not had any similar symptoms in the past, and
she has been in relatively good health.
If the patient has toxic megacolon, under what circumstances emergency surgical
management is indicated?

Choose one answer.


i. There is a perforation
j. Any of the above
k. The patient's clinical or radiographic status worsens
l. There is no improvement in 24 to 36 hours after aggressive medical therapy

Complications of untreated pancreatic pseudocysts include all of the following


EXCEPT:

Choose one answer.


k. intracystic hemorrhage
l. abscess
m. pancreatic necrosis
n. free rupture
o. gastrointestinal obstruction

Platelets in the wound form a hemostatic clot and release clotting factors to produce:

Choose one answer.


k. thrombin
l. Fibrin
m. Fibrinogen
n. Fibroblasts
o. thromboplastin

A 43-year-old man presents to the office for evaluation of recent weight loss and
frequent loose stools. He is concerned because his father was diagnosed with colon
cancer at the age of 50.

Besides family history, what are some other risk factors for colorectal cancer?

Choose one answer.


i. Hypertension
j. Diabetes
k. All of the above
l. Inflammatory bowel disease

A 40-year-old woman presents to the office for evaluation of yellowish skin. She
states that over the past few weeks, she has noticed that her eyes and skin have
developed a yellow tint. She also reports that she has dark urine and pale-colored
stools. Further history elicits periodic bouts of right upper quadrant pain after
eating. She is otherwise healthy. She denies using any medications. On physical
examination, a yellowish tint is observed on the patient's skin, sclera, and mucous
membranes.

On the basis of this patient's history and clinical examination, which type of
bilirubin would you expect to predominate?

Choose one answer.


i. Mixed
j. Conjugated
k. Unconjugated
l. Indirect
7

39. A 38-year-old man presents with a complaint of a slow-growing mass over his
right parotid gland. The lesion is fixed to the underlying structures and has recently
become painful.
Which of the following features strongly suggests that this patient's lesion is a
malignancy?

Choose one answer.


i. Ipsilateral numbness of the tongue
j. .All of the above
k. Overlying skin involvement
l. Facial nerve paralysis

Axillary lymph nodes are classified according to the relationship with the

Choose one answer.


k. pectoralis minor muscle
l. pectoralis major muscle
m. axillary vein
n. serratus anterior muscle
o. latissimus dorsi muscle

39. A 78-year-old man is recovering from abdomino-perineal (A-P) resection for Ca


rectum, which was performed 3 days ago. The patient is now complaining of mild
shortness of breath and chest pain. On physical examination, the patient's right leg
is slightly more swollen than his left. The pulse oximetry reading is 90%.
What is the principal method of diagnosing acute pulmonary embolism?

Choose one answer.


i. Magnetic resonance imaging
j. Chest x-ray
k. Ultrasound
l. Spiral computed tomography scanning

10

Compartment syndrome

Choose one answer.


k. Passive stretch decrease muscle pain
l. Due to decrease pressure in muscle compartments
m. Pulse is the first thing to disappear
n. Cause severe pain in the limb
o. Treatment is by delayed fasciotomy
11

The most significant risk factor for the development of adenocarcinoma of the
esophagus is:

Choose one answer.


k. lye stricture
l. alcohol abuse
m. Barrett's esophagus
n. long-standing achalasia
o. smoking

12

All of the following statements are true about patients with carcinoid tumors
EXCEPT:

Choose one answer.


k. the combination of streptozotocin and 5-fluorouracil (chemotherapy) can often
result in objective response.
l. tumor growth is often slow
m. they often have evidence of serotonin production
n. the majority have carcinoid syndrome
o. they have a much better prognosis if the tumors are less than 2 cm.

13

39. A 67-year-old man presents with left-lower-quadrant pain and low-grade fever.
He has had these symptoms for 1 day. The patient denies experiencing any rectal
bleeding, but for the past week, his bowel movements have been irregular.
For this patient, which of the following statements is true regarding diverticular
disease?

Choose one answer.


i. The sigmoid colon is the most common site of diverticula
j. All of the above
k. Most diverticula of the colon involve the muscular layer
l. Smoking does not seem to be related to the development of diverticular disease

14

All of the following are components of the MEN type 2B syndrome except:

Choose one answer.


i. Multiple neuromas on the lips, tongue, and oral mucosa
j. Medullary thyroid carcinoma
k. Pheochromocytoma
l. Hyperparathyroidism.

15
The risk of bilateral breast cancer is HIGHEST if the first breast shows:

Choose one answer.


k. inflammatory carcinoma
l. medullary carcinoma
m. infiltrating ductal carcinoma
n. paget’s disease
o. lobular carcinoma

16

The best initial therapy for deep venous thrombosis of the common femoral vein is:

Choose one answer.


k. warfarin
l. streptokinase
m. Heparin
n. venous thrombectomy
o. placement of a vena caval filter

17

Complications after thyroidectomy include all the following EXCEPT:

Choose one answer.


k. recurrent laryngeal nerve paralysis
l. parathyroid insufficiency
m. thyrotoxic crisis(storm) on operating on inadequitly prepared thyrotoxic patient
n. tracheomalacia
o. hypercalcemia

18

A 55-year-old man presents with hematemesis that began 2 hours ago. He is


hypotensive and has altered mental status. No medical history is available.
For this patient, which of the following statements regarding nasogastric aspiration
is true?

Choose one answer.


i. None of the above
j. A clear, nonbilious aspirate rules out the need for EGD
k. A clear, bilious aspirate rules out the need for EGD
l. A bloody aspirate is an indication for esophagogastroduodenoscopy (EGD)

19

The major cause of impaired wound healing is:

Choose one answer.


k. steroid use
l. malnutrition
m. diabetes mellitus
n. local tissue infection
o. anemia

20

Common presenting conditions in patients with pancreatic carcinoma include all of


the following EXCEPT:

Choose one answer.


i. esophageal varices.
j. palpable gallbladder
k. weight loss.
l. abdominal pain

21

All of the following statements about keloids are true EXCEPT:

Choose one answer.


k. Keloid tissue contains an abnormally large amount of collagen
l. A keloid does not regress spontaneously
m. Keloid tissue contains an unusually large amount of soluble collagen
n. A keloid extends beyond the boundaries of the original wound
o. Keloids or hypertrophic scars are best managed by excision and careful
reapproximation of the wound

22

The treatment of choice for a 40-year-old man who is found on endoscopy and
biopsy to have a gastric lymphoma would be:

Choose one answer.


k. wide local excision
l. subtotal gastrectomy
m. chemotherapy
n. subtotal gastrectomy and radiotherapy
o. Radiotherapy

23

The treatment of an esophageal burn with a caustic agent may include all of the
following EXCEPT:

Choose one answer.


k. boogieing. (dilatation)
l. induction of vomiting
m. gastrectomy
n. expeditious administration of an antidote
o. steroids and antibiotics.

24

A 32-year-old man with a family history of familial adenomatous polyposis (FAP)


presents with hematochezia. He denies having any diarrhea, abdominal pain, or
fever.

For this patient, which of the following statements regarding FAP is true?

Choose one answer.


a. CRC does not occur in patients with FAP if they are given adequate medical treatment
b. Total proctocolectomy (TPC) is considered the only option for the surgical
management of FAP
c. For patients with FAP, there is a 10% risk of CRC by age 40 if prophylactic colectomy
is not performed
d. In the setting of FAP, colorectal cancer (CRC) is more commonly located on the left
side

25

When stage I breast cancer is treated by partial mastectomy and axillary dissection,
further therapy should include:

Choose one answer


a. antiestrogen agents.
b. radiation of the affected breast.
c. oophorectomy if premenopausal.
d. nothing
e. chemotherapy

26

For the patient in Question 68, which of the following statements is true regarding
an esophageal varix as the site of bleeding?

Choose one answer.


i. I.V. propranolol should be administered first
j. Balloon tamponade should be performed first
k. Rubber banding or intravariceal sclerotherapy should be performed first
l. I.V. somatostatin should be administered first

27

Drugs which may produce gynecomastia include all of the following EXCEPT:

Choose one answer.


k. furosemide
l. cimetidine
m. Verapamil
n. Diazepam
o. Tamoxifen

28

Splenectomy is commonly indicated for the following EXCEPT:

Choose one answer.


k. hypersplenism associated with cirrhosis
l. hereditary spherocytosis
m. splenic tumor
n. immune thrombocytopenic purpura
o. grade four splenic injury in trauma

29

An ischiorectal abscess is characterized by all of the following EXCEPT:

Choose one answer.


k. Requires deroofing
l. Should be treated entirely by antibiotics
m. May be tuberculous in origin
n. Can be followed by anal fistula
o. Is an infective necrosis of the fat of the ischiorectal fossa

30
Marks: 1

The most common presentation of Meckel’s diverticulum in an adult is:

Choose one answer.


i. intussuception
j. Littre’s hernia
k. Gastrointestinal bleeding
l. diverticulitis

31

For the patient in Question 65, which of the following is an indication for immediate
surgery?

Choose one answer.


i. Closed-loop obstruction
j. All of the above
k. Complete bowel obstruction
l. Incarcerated hernia

32
Choledocholithiasis in a patient who previously had cholecystectomy is BEST
treated with:

Choose one answer.


k. endoscopic sphincterotomy
l. choledochoduodenostomy
m. choledochojejunostomy.
n. dissolution with mono-octanoin
o. open common bile duct exploration with stone removal

33

39. A 77-year-old man undergoes endoscopic ultrasonography as part of a workup


for jaundice. He is found to have a tumor in the head of the pancreas.
For this patient, which of the following findings would indicate that the tumor is
unresectable?

Choose one answer.


i. All of the above
j. Peritoneal metastases
k. Invasion of the superior mesenteric artery
l. Metastases to celiac lymph nodes

34

Regarding polyps of the colon

Choose one answer.


k. Villous polyps are usually pedunculated
l. Villous polyps occur more proximal in colon
m. Adenomatous polyps are usually solitary
n. Cancer risk is not related to size of polyp
o. Metaplastic polyps are not precancerous

35

The most likely diagnosis in elderly patient with abdominal pain and colonoscopy
finding of patchy mucosal ulceration at the splenic flexure of the colon is :

Choose one answer.


k. ulcerative colitis
l. crohns disease
m. ischemic colitis
n. diverticulitis
o. lymphogranuloma venerum

36
39. A 54-year-old man presents with a neck mass of 2 weeks' duration. He has no
significant medical history. He smokes two packs of cigarettes a day and has been
doing so since he was 21 years of age.
For this patient, which of the following statements is true?

Choose one answer.


i. Low cervical nodes are more likely to contain metastases from a primary source
other than the head and neck, whereas upper cervical nodes are more likely to
contain metastases from the head and neck
j. Soft or tender nodes are more likely to derive from an inflammatory or infectious
condition, whereas hard, fixed, painless nodes are more likely to represent
metastatic cancer
k. All of the above
l. Enlarged lymph nodes are by far the most common neck masses encountered

37

The first-choice diagnostic study for suspected deep venous thrombosis of the lower
extremity is:

Choose one answer.


k. real-time Doppler imaging
l. contrast sonography
m. radioactive labeled fibrinogen uptake
n. impedance plethysmography
o. isotope injection with gamma scintillation scanning

38

Decreased PaCO2 levels should be attained in a patient at serious risk for cerebral
edema secondary to a head injury in order to :

Choose one answer.


k. prevent neurogenic pulmonary edema
l. prevent increased capillary permeability
m. prevent metabolic acidosis
n. allow reciprocally high levels of PaO2 in the brain
o. prevent cerebral vasodilation

39

Causes of third space loss include all except:

Choose one answer.


k. Acute pancreatitis
l. Pancreatic fistula
m. Necrotizing fasciitis
n. Site of major surgery
o. Crush syndrome
40

Causes of metabolic acidosis include all except:

Choose one answer.


k. Small bowel fistula
l. Shock
m. CO poisoning
n. severe anemia
o. All of the above

41

A 39-year-old man comes in for evaluation of intermittent anal pain and bleeding
after bowel movements. He also has hard stools. He has had these symptoms for over
1 year .
For this patient, which of the following is included in the classic triad of signs of
chronic anal fissures?

Choose one answer.


i. An anal fissure
j. All of the above
k. A sentinel skin tag
l. Hypertrophy of the anal papilla

42

All the following statements about achalasia are true EXCEPT:

Choose one answer.


k. it occurs most commonly in persons between the ages of 30 and 50 years
l. in most affected persons, ganglion cells in the body of the esophagus either are
absent or have degenerated
m. esophageal cancer is seven times as common in affected persons as in the general
population
n. pressure in the body of the esophagus is lower than normal
o. affected persons usually experience more difficulty swallowing cold foods than
warm foods

43

A patient with the Zollinger-Ellison syndrome is found to have the multiple


endocrine neoplasia type I (MEN-I) syndrome. Appropriate management for the
ulcer symptoms should be:

Choose one answer.


k. Omeprazole
l. pancreatic resection
m. streptozocin
n. cimetidine
o. total gastrectomy

44

Fat absorption occurs primarily in the:

Choose one answer.


k. Ileum
l. third portion of the duodenum
m. Stomach
n. Jejunum
o. first portion of the duodenum

45

39. A 56-year-old woman has been experiencing abdominal pain for 4 hours. The
pain is in right upper quadrant and radiates into the scapular region. She has had
multiple episodes of vomiting.
For this patient, which of the following signs on physical examination is associated
with acute cholecystitis?

Choose one answer.


i. Carnett sign
j. Kehr sign
k. Murphy sign
l. Rovsing sign

46

Mammary duct ectasia is characterized by the following EXCEPT :

Choose one answer.


k. Is treated usually by simple mastectomy
l. May present with nipple retraction and Peau d'orange picture
m. Anaerobic superinfection cmmmonly occurs in this recurrent periductal plasma
cell mastitis
n. Is defined as primary dilatation of major ducts of breast in middle aged women
o. Iscommonly pre-malignant

47

Regarding veins of lower limbs all are true except:

Choose one answer.


i. Valves allow flow from deep to superficial system Venous return from lower limbs
is aided by respiratory movements
j. Superficial veins lie in subcutaneous tissue
k. The pressure in veins of the foot while standing is 100mmHg
l. a) Stasis of blood is important factor in developing varicose veins
48

Most common complication of central venous access is:

Choose one answer.


k. major artery damage.
l. Catheter problems.
m. Thrombosis of central vein
n. Catheter related sepsis
o. Pleural space damage, pneumothorax
49

Factors associated with increase risk of death in acute pancreatitis include all
except:

Choose one answer.


k. Ranson score more than five
l. Obesity
m. High APACHE_II score
n. Age more than 70 years
o. sterile necrosis

50

For the patient in Question 56, which of the following chronic conditions can cause a
neck mass?

Choose one answer.


i. Tuberculosis
j. Sarcoidosis
k. AIDS
l. All of the above

51

For the patient in Question 56, if metastatic cancer is suspected initially, which of the
following would be the most appropriate step to take next in the workup?

Choose one answer.


i. CT scan
j. Empirical therapy with antibiotics
k. Fine-needle aspiration (FNA)
l. Observation only

52

The largest component of intestinal gas is:

Choose one answer.


k. Nitrogen
l. carbon dioxide
m. Oxygen
n. ammonia
o. hydrogen

53

A 66-year-old woman presents to the office complaining of a sharp, constant pain in


her lower abdomen. She has had this pain for the past 2 weeks. Examination of her
abdomen is normal. However, subsequent ultrasound reveals a 4.5 cm Aortic
abdominal aneurysm (AAA). Spiral computed tomography confirms the ultrasound
findings.

Aside from rupture, which of the following is a complication associated with


(AAA) ?

Choose one answer.


i. Aortoenteric fistula
j. Lower-extremity atheroemboli
k. All of the above
l. Thrombosis

54

Diagnosis of esophageal perforation is best established by:

Choose one answer.


k. transesophageal ultrasound study
l. esophagoscopy with a flexible esophagoscope
m. contrast esophagograms
n. esophagoscopy with a rigid esophagoscope
o. upright X-rays of the chest including lateral and oblique films

55

For the patient in Question 68, which of the following is an indication for surgery?

Choose one answer.


i. Ongoing hemorrhage occurs from a gastric ulcer in a hemodynamically unstable
patient
j. All of the above
k. Bleeding continues from either a duodenal ulcer or a gastric ulcer despite medical
and endoscopic therapy
l. Substantial bleeding occurs from a duodenal ulcer that is not controlled by EGD

56

Yesterday, a 38-year-old woman underwent a laparoscopic cholecystectomy for


cholelithiasis and was discharged home 8 hours after surgery. She returns this
morning complaining of worsening abdominal pain. The oral narcotics that the
patient was prescribed are ineffective in controlling the pain. The patient's
temperature is (38.3 C). Laboratory studies reveal an elevated white blood cell
count. Abdominal ultrasonography shows a large subhepatic fluid collection. The
fluid is percutaneously aspirated and reveals enteric contents.

What step should be taken next in the management of this patient?

Choose one answer.


i. Immediate laparotomy
j. I.V. antibiotics and close observation
k. None of the above
l. Observation in the hospital until pain improves

57

The most common symptom after major pulmonary embolism is:

Choose one answer.


k. cough
l. Dyspnea
m. Hemoptysis
n. pleural pain
o. fear of death

58

Treatment of paralytic ileus includes all of the following


EXCEPT :

Choose one answer.


k. intravenous fluids
l. cessation of oral intake
m. correction of electrolyte imbalance
n. early operation
o. nasogastric suction

59
Lymphedema :
Choose one answer
k. e) None of the above
l. b) should be bilateral
m. c) may be pitting in early stage
n. d) A & C only
o. a) may be Congenital

60
The level of consciousness for a head injury patient is BEST evaluated by :

Choose one answer.


k. CT scan
l. visual evoked potentials
m. Glasgow coma scale
n. papillary responses
o. response to pain

61

A 52-year-old male weighing 70 kg, sustained a 65% total body surface area (TBSA)
burn. What are his fluid requirements?

Choose one answer.


a. 18,200 cc in 16 h
b. 9100 cc in the first 8 h
c. 8000 cc in 24 h
d. 12,800 cc in 24 h

62

Management of cholangitis may include all of the following EXCEPT:

Choose one answer.


a. decomperession of the common bile duct.
b. cholecystostomy
c. IV antibiotics.
d. correct underlying cause.
e. percutaneous transhepatic cholangiography.

63

A 48-year-old woman presents to the emergency department complaining of right


upper quadrant pain, which began 4 hours ago. She reports the pain as being
spasmodic and sharp and that it radiates to her right shoulder blade. She says that
she has had similar episodes over the past few months, especially after eating large
meals. Associated with the pain is nausea and vomiting. Her blood pressure is 120/85
mm Hg, and her pulse is 100 beats/min. On physical examination, the patient is
found to have a nontender abdomen with no palpable masses. Her chest and
cardiovascular examinations are normal. The nurse notices that her sclerae are
slightly icteric. On subsequent laboratory studies, her serum bilirubin level is found
to be 10 mg/dl.

What imaging study should be performed next for this patient with presumed
posthepatic jaundice?

Choose one answer.


a. Percutaneous transhepatic cholangiography (PTC)
b. Ultrasonography
c. Endoscopic retrograde cholangiopancreatography (ERCP)
d. Magnetic resonance imaging

64

39. An HIV-positive man presents for evaluation of new oral cavity lesions he
discovered last month. Physical examination reveals purple exophytic masses
involving the palate mucosa and gingiva.
What is the most likely diagnosis of these oral cavity lesions?

Choose one answer.


a. Oral hairy leukoplakia
b. Non-Hodgkin lymphoma
c. Syphilis
d. Kaposi sarcoma

65

Brain injury alone

Choose one answer.


a. causes shock only if the skull is intact
b. rarely causes shock
c. causes shock that is reversed by very simple measures
d. frequently causes shock
e. causes shock if hypoxia is superimposed

66

All of the following substances are irritating to the peritoneum EXCEPT:

Choose one answer.


a. bile.
b. blood
c. gastric content.
d. meconium
e. pus

67

Regarding volvulus of the sigmoid colon, each of the following is true except :

Choose one answer.


a. likely results from redundant sigmoid colon with an elongated narrow mesocolon
b. there appears to be a congenital predisposition sigmoid volvulus.
c. diagnostic barium enema for sigmoid colon is essential
d. diagnostic x-ray for sigmoid volvulus shows a dilated loop of colon which points
toward the right upper quadrant.
68

Gallstones are characterized by all the following EXCEPT:

Choose one answer.


a. cause mucocoele of the gall bladder
b. are present in the common bile duct in 40% of patients with stones in the gall bladder
c. are becoming common in post-partum primipara who were pre pregnancy ‘Pill’ takers
d. are frequently the cause of flatulent dyspepsia
e. may be present in the newborn

69

Following a burn, the agent responsible for early increased capillary permeability is

Choose one answer.


a. thromboxane A2
b. serotonin
c. histamine
d. prostacyclin PGI2
e. bradykinin

70

Incisonal hernias all are true except:

Choose one answer.


a. 10 % of all hernias
b. Usually easy to reduce
c. Multiloculated sac
d. Operative technique is most important cause
e. Complication is common

71

In the treatment of gastric cancer, all of the following are true EXCEPT:

Choose one answer.


a. five-year survival rates in the continue to be between 10% and 25%.
b. Palliative resection is frequently helpful with advanced disease.
c. Lymph node involvement is associated with a poorer prognosis.
d. Finding early disease at the time of operation is associated with a better prognosis.
e. total gastrectomy is mandated in most patient. ***

72

of the involved area and : Severe cases of hidradenitis suppurativa in the groin area
are best managed by excision
Choose one answer.
a. transfer of a rectus abdominus muscle flap
b. split thickness skin grafting
c. primary closure
d. delayed primary closure
e. closure by secondary intention

73

Patients at increased risk for gastric carcinoma include all the following EXCEPT:

Choose one answer.


a. those who have undergone gastric bypass for morbid obesity
b. those who have undergone gastric resection for duodenal ulcer
c. those with pernicious anemia
d. those with a high consumption of smoked fish
e. those with blood group A

74

A 56-year-old woman presents with symptoms of abdominal pain, weight loss, and
rectal bleeding. She is anemic and hypotensive, but she is stable.
For this patient, which of the following should be done first if the gastric lavage
yields copious amounts of bile?

Choose one answer.


a. Arteriography
b. Emergency laparotomy
c. Colonoscopy
d. Esophagogastroduodenoscopy

75

A 39-year-old man presents with lower GI bleeding. He has no abdominal


discomfort and has experienced no loss of weight.
For this patient, which of the following statements regarding the etiology of lower GI
bleeding is true?

Choose one answer.


a. Lower GI bleeding from diverticulosis often requires surgery
b. Arteriovenous malformations (AVMs) are the most common cause of lower GI
bleeding
c. 6% to 10% of patients with ulcerative colitis have lower GI bleeding severe enough to
necessitate emergency surgical resection
d. None of the above

76

For the patient in Question 65, which of the following statements regarding
abdominal radiographic findings is true?
Choose one answer.
a. In gastric outlet obstruction, no gastric air will be seen, but large amounts of air will be
seen in the small bowel and colon
b. Mechanical small bowel obstruction usually shows no air-fluid levels, nor will
distended bowel loops of similar sizes be seen
c. All of the above
d. High-grade obstruction of the colon in association with an incompetent ileocecal valve
may mimic small bowel obstruction on x-ray

77

A 41-year-old female presents to the emergency department after sustaining a


gunshot wound to the abdomen, with injuries to the liver and large bowel. Despite
successful resuscitation and operative intervention, the patient dies 2 weeks later of
multisystem organ failure in the intensive care unit. Which organ most likely first
experienced dysfunction?

Choose one answer.


a. lung
b. heart
c. liver
d. gastrointestinal tract
e. kidney

78

39. A 35-year-old man presents with severe acute abdominal pain of sudden onset.
The pain initially began in the upper abdomen and has now settled in the whole
abdomen. On examination, the abdomen was rigid. Chest X-Ray showed free gas
under diaphragm.
Which of the following is the most appropriate course of management for this
patient?

Choose one answer.


a. Observation
b. Urgent laparotomy
c. Nonurgent laparotomy
d. Laparoscopy

79

All are true about the dumping syndrome except:

Choose one answer.


a. Symptoms can be controlled with a somatostatin analogue.
b. Early postoperative dumping after vagotomy often resolves spontaneously.
c. Flushing and tachycardia are common features of the syndrome.
d. Diarrhea is always part of the dumping syndrome.
e. Separating solids and liquids in the patient's oral intake alleviates some of the
symptoms of the syndrome.

80

An anal fissure is characterized by the following EXCEPT:

Choose one answer.


a. Is more common in women.
b. Is an ulcer of the anal mucosa.
c. Can be treated coservatively.
d. Usually lies anteriorly.

81

All of the following statements are true of esophageal carcinoma


EXCEPT

Choose one answer.


a. squamous cell tumor is the most frequent histology.
b. operation is frequently curative.
c. patient often require a pyloroplasty with operation.
d. patients usually die within one year of diagnosis
e. patient can have intestinal continuity reestablished using the stomach after esophageal
resection.

82

A serum tumor marker correlated with recurrence after management of colon


cancer is:

Choose one answer.


a. carcinoembryonic antigen (CEA)
b. Calcitonin
c. 5-hydroxyindoleacetic acid
d. CA 15-3
e. Alpha-fetoprotein

83

Conditions associated with gastric cancer include all of the following EXCEPT:

Choose one answer.


a. adenomatous polyps.
b. a high intake of dietary nitrates.
c. pernicious anemia.
d. higher socioeconomic groups.
e. chronic atrophic gastritis.

84
Which of the following is required for addressing any pressure sore?

Choose one answer.


a. pressure reduction
b. hyperbaric oxygen
c. antibiotics
d. skin flap coverage

85

The most effective treatment of achalasia is:

Choose one answer.


a. antireflux surgical procedures
b. Esophagomyotomy
c. dilation of the lower esophageal sphincter
d. antispasmodic medication
e. resection of the cardioesophageal junction

86

Regarding desmoid tumors all are true except:

Choose one answer.


a. Treatment is wide local excision with safety margins
b. Has high rate of local recurrence
c. Is a low grade fibro sarcoma
d. More common in females
e. Is radio resistant

87

Which of the following variables best predicts prognosis for patients with a recent
diagnosis of cutaneous melanoma and no clinical evidence of metastatic disease?

Choose one answer.


a. Ulceration.
b. Clark's level.
c. Gender
d. Bleeding
e. Breslow thickness.

88
In the treatment of acute cholecystitis, most patient are BEST served with:
Choose one answer.
a. endoscopic sphincterotomy.
b. cholecystostomy.
c. percutaneous drainage of the gallbladder.
d. early cholecystectomy (within 3 days of onset of symptoms).
e. IV antibiotics and cholecystectomy in 6 to 8 weeks.

89

Regarding crohn's disease all are true except

Choose one answer.


a. Cause non caseating granuloma
b. commonly affect small bowel
c. Causes sub mucosal fibrosis
d. It is full thickness inflammation
e. Cause depletion of goblet cell mucin

90

Advantages of laparoscopic versus open cholecystectomy include all of the following


EXCEPT:

Choose one answer.


a. decreased pain.
b. less risk of bile duct injury.
c. reduced hospitalization.
d. improved cosmetic.
e. reduced ileus.

91

The most common cause of massive hemorrhage in the lower gastrointestinal tract
is:

Choose one answer.


a. carcinoma
b. Diverticulosis
c. Diverticulitis
d. Polyp
e. ulcerative colitis

92

Carcinoma of the gallbladder is

Choose one answer.


a. rarely associated with jaundice.
b. associated with a good prognosis.
c. usually not diagnosed preoperatively.
d. most commonly metastatic to the lung.
e. best treated with radiation and chemotherapy

93
All the following statements concerning nipple discharges are true EXCEPT:

Choose one answer.


a. benign duct papillomas are the most common cause of bloody discharges.
b. when bloody , the discharge is due to a malignancy 70% of the time
c. a) excision of involved duct may be necessary to determine the etiology
d. they may be caused by multiple lesions.
e. a milky discharge may be due to a pituitary adenoma .

94

39. A 28-year-old woman presents with a complaint of a growing, painless mass in


her neck. Physical examination reveals a firm, fixed nodule measuring 2 cm on the
right lobe of her thyroid. The surgeon recommends fine-needle aspiration (FNA) of
the lesion instead of excisional biopsy.
What are the advantages of FNA over excisional biopsy?

Choose one answer.


a. FNA is associated with a decreased risk of tumor seeding
b. FNA requires only an office visit
c. Anesthesia is not necessary
d. All of the above

95

For the patient in Question 74, which of the following is a sign of a colovesical fistula
associated with diverticulitis, as seen on computed tomography with contrast?

Choose one answer.


a. Thickening of the bladder and the colon
b. Sigmoid diverticula
c. All of the above
d. Air in the bladder

96

All of the following are true statements concerning paget ’ s disease of the nipple
EXCEPT:

Choose one answer.


a. it is an eczematoid lesion.
b. it is very uncommon, accounting for only 2% of all breast cancers.
c. it is an in situ squamous cell malignancy of the nipple.
d. it can be confused with malignant melanoma histologically
e. it has a better prognosis than the majority of other breast cancers.

97

The classical picture of Acute arterial embolism include all the following except:
Choose one answer.
a. peripheral pulses
b. Parasthesia
c. All the above
d. Pain
e. Pallor

98

Extra colonic manifestation of inflammatory bowel disease include all except:

Choose one answer.


a. Erythema nodosum
b. Mixed connective tissue disease
c. Primary Sclerosing cholangitis
d. Polyarthopathy
e. Episcleritis

99

A 45-year-old woman presents with abdominal pain and vomiting of 1 day's


duration. The patient underwent an exploratory laparotomy after a motor vehicle
accident 8 years ago.
Which of the following statements is true for this patient?

Choose one answer.


a. Pain from mechanical obstruction is usually more severe than pain from ileus
b. The severity of pain from mechanical obstruction may decrease over time
c. Pain from mechanical obstruction is usually localized in the middle of the abdomen,
whereas pain from ileus or pseudo-obstruction is diffuse
d. All of the above

100

A 4-year-old girl has been experiencing dyspnea on exertion. Chest x-ray shows a
Morgagni hernia.
For this patient, which of the following statements is true?

Choose one answer.


a. Morgagni hernias can be repaired with a subcostal, a paramedian, or a midline incision
b. Morgagni hernias are most commonly seen on the right side
c. All of the above
d. The average age at diagnosis is typically greater for patients with Morgagni hernia than
for patients with Bochdalek hernia

A 19 year old female is admitted with suspected meningitis. The House


Officer is due to perform a
lumbar puncture. What is the most likely structure first encountered
when the needle is inserted?
A. Ligamentum flavum
B. Denticulate ligament
C. Dural sheath
D. Pia Mater
E. Supraspinous ligament

A 34 year old male is being examined in the pre-operative assessment


clinic. A murmur is identified in
the 5th intercostal space just next to the left side of the sternum. From
where is it most likely to have
originated?
A. Mitral valve
B. Aortic valve
C. Pulmonary valve
D. Right ventricular aneurysm
E. Tricuspid valve

What is the correct embryological origin of the stapes?


A. First pharyngeal arch
B. Second pharyngeal arch
C. Third pharyngeal arch
D. Fourth pharyngeal arch
E. Fifth pharyngeal arch

Which muscle initiates abduction of the shoulder?


A. Infraspinatus
B. Latissimus dorsi
C. Supraspinatus
D. Deltoid
E. Teres major

What is the most important structure involved in supporting the uterus?


A. Round ligament
B. Broad ligament
C. Uterosacral ligaments
D. Cardinal ligaments
E. Central perineal tendon

78 year old man is due to undergo an endarterectomy of the internal


carotid artery. Which of the
following nervous structures are most at risk during the dissection?
A. Recurrent laryngeal nerve
B. Sympathetic chain
C. Hypoglossal nerve
D. Phrenic nerve
E. Lingual nerve
Nerves at risk during a carotid
endarterectomy:
Hypoglossal nerve
Greater auricular nerve
Superior laryngeal nerve

Which of the following structures is not transmitted by the jugular


foramen?
A. Hypoglossal nerve
B. Accessory nerve
C. Internal jugular vein
D. Inferior petrosal sinus
E. Vagus nerve

With regards to the jugular vein, which of the following statements is


untrue?
A. It lies within the carotid sheath
B. It is the continuation of the sigmoid sinus
C. The terminal part of the thoracic duct crosses anterior to it to insert
into the right subclavian vein
D. The hypoglossal nerve is closely related to it as it passes near the
atlas
E. The vagus nerve is closely related to it within the carotid sheath
A 28 year old man requires a urethral catheter to be inserted prior to
undergoing a splenectomy. Where is the
first site of resistance to be encountered on inserting the catheter?
A. Bulbar urethra
B. Membranous urethra
C. Internal sphincter
D. Prostatic urethra
E. Bladder neck

A 23 year old man undergoes an orchidectomy. The right testicular vein


is ligated; into which structure does it
drain?
A. Right renal vein
B. Inferior vena cava
C. Common iliac vein
D. Internal iliac vein
E. External iliac vein

A 24 year old man falls and sustains a fracture through his scaphoid
bone. From which of the following areas
does the scaphoid derive the majority of its blood supply?
A. From its proximal medial border
B. From its proximal lateral border
C. From its proximal posterior surface
D. From the proximal end
E. From the distal end

Which of the following forms the medial wall of the femoral canal?
A. Pectineal ligament
B. Adductor longus
C. Sartorius
D. Lacunar ligament
E. Inguinal ligament

A 67 year old man is undergoing a transurethral resection of a bladder


tumour using diathermy. Suddenly during the
procedure the patients leg begins to twitch. Stimulation of which of the
following nerves is the most likely cause?
A. Femoral
B. Pudendal
C. Sciatic
D. Obturator
E. Gluteal

A 5 year old boy is playing with some small ball bearings. Unfortunately
he inhales one. To which of the following lung
regions is the ball most likely to settle?
A. Right lower lobe
B. Left main bronchus
C. Right upper lobe
D. Left lower lobe
E. None of the above

An 18 year old man is cutting some plants when a small piece of


vegetable matter enters his eye. His eye
becomes watery. Which of the following is responsible for relaying
parasympathetic neuronal signals to the
lacrimal apparatus?
A. Pterygopalatine ganglion
B. Otic ganglion
C. Submandibular ganglion
D. Ciliary ganglion
E. None of the above

A 43 year old lady is undergoing a total thyroidectomy for an extremely


large goitre. The surgeons decide that
access may be improved by division of the infra hyoid strap muscles. At
which of the following sites should
they be divided?
A. In their upper half
B. In their lower half
C. In the middle
D. At their origin from the hyoid
E. At the point of their insertion
A 22 year old women has recently undergone a surgical excision of the
submandibular gland. She presents to the follow
up clinic with a complaint of tongue weakness on the ipsilateral side to
her surgery. Which nerve has been damaged?
A. Hypoglossal nerve
B. Lingual nerve
C. Inferior alveolar nerve
D. Facial nerve
E. Lesser petrosal nerve
Three cranial nerves may be injured during
submandibular gland excision.
Marginal mandibular branch of the facial nerve
Lingual nerve
Hypoglossal nerve

You decide to take an arterial blood gas from the femoral artery. Where
should the needle be inserted to gain the sample?
A. Mid point of the inguinal ligament
B. Mid inguinal point
C. 2cm inferomedially to the pubic tubercle
D. 2cm superomedially to the pubic tubercle
E. 3cm inferolaterally to the deep inguinal ring

A 67 year old man undergoes a carotid endarterectomy and seems to


recover well following surgery. When he is reviewed
on the ward post operatively he complains that his voice is hoarse. What
is the most likely cause?
A. Damage to the accessory nerve
B. Damage to the cervical plexus
C. Damage to the glossopharyngeal nerve
D. Damage to the hypoglossal nerve
E. Damage to the vagus

A 25 year old man has an inguinal hernia, which of the following


structures must be divided (at open surgery)
to gain access to the inguinal canal?
A. Transversalis fascia
B. External oblique aponeurosis
C. Conjoint tendon
D. Rectus abdominis
E. Inferior epigastric artery

A 34 year old man is shot in the postero- inferior aspect of his thigh.
Which of the following lies at the most lateral aspect
of the popliteal fossa?
A. Popliteal artery
B. Popliteal vein
C. Common peroneal nerve
D. Tibial nerve
E. Small saphenous vein

A 67 year old man has an abdominal aortic aneurysm which displaces


the left renal vein. Which branch of the
aorta is most likely to affected at this level?
A. Inferior mesenteric artery
B. Superior mesenteric artery
C. Coeliac axis
D. Testicular artery
E. None of the above

A 12 year old boy undergoes surgery for recurrent mastoid infections.


Post operatively he complains of an altered taste
sensation. Which of the following nerves has been injured?
A. Glossopharyngeal
B. Greater petrosal
C. Olfactory
D. Trigeminal
E. Chorda tympani

What is the anatomical level of the transpyloric plane?


A. T11
B. T12
C. L1
D. L4
E. T10
A 35 year old man falls and sustains a fracture to the middle third of his
clavicle. Which vessel is at greatest risk
of injury?
A. Subclavian vein
B. Subclavian artery
C. External carotid artery
D. Internal carotid artery
E. Vertebral artery

A 33 year old man sustains an injury to his forearm and wrist. When
examined in clinic he is unable to adduct
his thumb. What is the most likely underlying nerve lesion?
A. Radial nerve
B. Superficial branch of the ulnar nerve
C. Median nerve
D. Posterior interosseous nerve
E. Deep branch of the ulnar nerve

A 63 year old man who smokes heavily presents with dyspepsia. He is


tested and found to be positive for
helicobacter pylori infection. One evening he has an episode of
haematemesis and collapses. What is the most
likely vessel to be responsible?
A. Portal vein
B. Short gastric arteries
C. Superior mesenteric artery
D. Gastroduodenal artery
E. None of the above

A 22 year old man is stabbed in the chest at the level of the junction
between the sternum and manubrium.
Which structure is at greatest risk?
A. Left atrium
B. Oesophagus
C. Thyroid gland
D. Inferior vena cava
E. Aortic arch
Which of the following nerves passes through the greater and lesser
sciatic foramina?
A. Pudendal nerve
B. Sciatic nerve
C. Superior gluteal nerve
D. Inferior gluteal nerve
E. Posterior cutaneous nerve of the thigh

A 73 year old lady is admitted with brisk rectal bleeding. Despite


attempts at resuscitation the bleeding proceeds
to cause haemodynamic compromise. An upper GI endoscopy is normal.
A mesenteric angiogram is performed
and a contrast blush is seen in the region of the sigmoid colon. The
radiologist decides to embolise the vessel
supplying this area. At what spinal level does it leave the aorta?
A. L2
B. L1
C. L4
D. L3
E. T10

In which space is a lumbar puncture performed?


A. Subdural space
B. Epidural space
C. Subarachnoid space
D. Extradural space
E. Intraventricular space

A 21 year old man is stabbed in the antecubital fossa. A decision is made


to surgically explore the wound. At operation the surgeon dissects down
onto the brachial artery. A nerve is identified medially, which nerve is it
likely to be?
A. Radial
B. Recurrent branch of median
C. Anterior interosseous
D. Ulnar
E. Median

A 65 year old man with long standing atrial fibrillation develops an


embolus to the lower leg. The decision is made to
perform an embolectomy, utilising a trans popliteal approach. After
incising the deep fascia, which of the following
structures will the surgeons encounter first on exploring the central
region of the popliteal fossa?
A. Popliteal vein
B. Common peroneal nerve
C. Popliteal artery
D. Tibial nerve
E. None of the above

A man sustains a laceration between the base of the little finger and
wrist. Several weeks after the injury there is
loss of thumb adduction power. Which nerve is most likely to have been
injured?
A. Superficial ulnar nerve
B. Deep ulnar nerve
C. Median nerve
D. Radial nerve
E. Recurrent branch of median nerve

What is the course of the median nerve relative to the brachial artery in
the upper arm?
A. Medial to anterior to lateral
B. Lateral to posterior to medial
C. Medial to posterior to lateral
D. Medial to anterior to medial
E. Lateral to anterior to medial

A 43 year old man is due to undergo an excision of the sub mandibular


gland. Which of the following incisions
is the most appropriate for this procedure?
A. A transversely orientated incision 4cm below the mandible
B. A transversely orientated incision immediately inferior to the
mandible
C. A vertical incision 3 cm anterior to the angle of the mandible and
extending inferiorly
D. A transversely orientated incision 2cm above the mandible
E. A transversely orientated incision 12cm below the mandible
A 5 year old boy presents with recurrent headaches. As part of his
assessment he undergoes an MRI scan of his
brain. This demonstrates enlargement of the lateral and third ventricles.
Where is the most likely site of
obstruction?
A. Foramen of Luschka
B. Foramen of Magendie
C. Foramen of Munro
D. Aqueduct of Sylvius
E. None of the above

Where does the spinal cord terminate in neonates?


A. L1
B. L2
C. L3
D. L4
E. L5

A 73 year old lady is admitted with acute mesenteric ischaemia. A CT


angiogram is performed and a stenotic lesion is noted at the origin of the
superior mesenteric artery. At which of the following levels does this
branch from the aorta?
A. L1
B. L2
C. L3
D. L4
E. L5

Following an oesophagogastrectomy the surgeons will anastomose the


oesophageal remnant to the stomach, which of the following is not part
of the layers that comprise the oesophageal wall?
A. Serosa
B. Adventitia
C. Muscularis propria
D. Submucosa
E. Mucosa

Where is the 'safe triangle' for chest drain insertion located?


A. 4th intercostal space, mid axillary line
B. 5th intercostal space, mid axillary line
C. 4th intercostal space, mid scapular line
D. 5th intercostal space, mid scapular line
E. 4th intercostal space, mid clavicular line

Your consultant decides to perform an open inguinal hernia repair


under local anaesthesia. Which of the following dermatomal levels will
require blockade?
A. T10
B. T12
C. T11
D. S1
E. S2

Which of the following does not pass through the superior


orbital fissure?
A. Inferior opthalmic vein
B. Abducens nerve
C. Opthalmic artery
D. Trochlear nerve
E. Superior opthalmic vein

A patient is found to have an ischaemic left colon. Which artery arising


from the aorta at around the level of L3
is most likely to account for this situation?
A. Superior mesenteric artery
B. Inferior mesenteric artery
C. Superior rectal artery
D. Ileocolic artery
E. Middle colic artery

At which level does the aorta perforate the diaphragm?


A. T10
B. T9
C. T8
D. T11
E. T12
A 24 year old lady is stabbed in the buttock. Following the injury the
wound is sutured in the emergency department. Eight weeks later she
attends the clinic, as she walks into the clinic room she has a waddling
gait and difficulty with thigh abduction. On examination she has
buttock muscle wasting. Which nerve has been injured?
A. Superior gluteal nerve
B. Obturator nerve
C. Sciatic nerve
D. Femoral nerve
E. Inferior gluteal nerve

At which level is the hilum of the left kidney located?


A. L1
B. L2
C. T12
D. T11
E. L3

A 73 year old lady presents with symptoms of faecal incontinence. On


examination she has weak anal sphincter muscles. What are the main
nerve root values of the nerves supplying the external anal sphincter?
A. S2,3
B. L5, S1
C. S4,5
D. S5
E. S2,3,4

A 72 year old man has a fall. He is found to have a fractured neck of


femur and goes on to have a left hip hemiarthroplasty. Two months post
operatively he is found to have an odd gait. When standing on his left leg
his pelvis dips on the right side. There is no foot drop. What is the
cause?
A. Sciatic nerve damage
B. L5 radiculopathy
C. Inferior gluteal nerve damage
D. Previous poliomyelitis
E. Superior gluteal nerve damage
A 28 year old lady requires an episiotomy for a ventouse vaginal
delivery. Which of the nerves listed below will usually be anaesthetised
to allow the episiotomy?
A. Femoral
B. Ilioinguinal
C. Pudendal
D. Genitofemoral
E. Sacral plexus

A 48 year old lady is undergoing an axillary node clearance for breast


cancer. Which of the structures listed below are most likely to be
encountered during the axillary dissection?
A. Cords of the brachial plexus
B. Thoracodorsal trunk
C. Internal mammary artery
D. Thoracoacromial artery
E. None of the above

A 53 year old lady is recovering following a difficult mastectomy and


axillary nodal clearance for carcinoma of the breast. She complains of
shoulder pain and on examination has obvious winging of the scapula.
Loss of innervation to which of the following is the most likely
underlying cause?
A. Latissimus dorsi
B. Serratus anterior
C. Pectoralis minor
D. Pectoralis major
E. Rhomboids

A 56 year old lady is referred to the colorectal clinic with symptoms of


pruritus ani. On examination a polypoidal mass is identified inferior to
the dentate line. A biopsy confirms squamous cell carcinoma. To which
of the following lymph node groups will the lesion potentially
metastasise?
A. Internal iliac
B. External iliac
C. Mesorectal
D. Inguinal
E. None of the above

A 20 year old man is hit with a hammer on the right side of the head. He
dies on arrival in the emergency department. Which of these features is
most likely to be found at post mortem?
A. Hydrocephalus
B. Supra tentorial herniation
C. Laceration of the middle meningeal artery
D. Sub dural haematoma
E. Posterior fossa haematoma

Which of the following ligaments contains the artery supplying the head
of femur in children?
A. Transverse ligament
B. Ligamentum teres
C. Iliofemoral ligament
D. Ischiofemoral ligament
E. Pubofemoral ligament

A 63 year old man is undergoing a coronary artery bypass procedure.


During the median sternotomy which structure would routinely require
division?
A. Parietal pleura
B. Interclavicular ligament
C. Internal mammary artery
D. Brachiocephalic vein
E. Left vagus nerve

A man undergoes a high anterior resection for carcinoma of the upper


rectum. Which of the following vesselswill require ligation?
. Superior mesenteric artery
B. Inferior mesenteric artery
C. Coeliac axis
D. Perineal artery
E. Middle colic artery

What are the boundaries of the 'safe triangle' for chest drain insertion?
A. Bounded by trapezius, latissimus dorsi, and laterally by the vertebral
border of the scapula
B. Bounded by latissimus dorsi, pectoralis major, line superior to the
nipple and apex at the axilla
C. Bounded by latissimus dorsi, serratus anterior, line superior to the
nipple and apex at the axilla
D. Bounded by trapezius, deltoid, rhomboid major and teres minor
E. Bounded by trapezius, deltoid and latissimus dorsi

A 60 year old female attends the preoperative hernia clinic. She reports
some visual difficulty. On examination she is noted to have a
homonymous hemianopia. Where is the lesion most likely to be?
A. Frontal lobe
B. Pituitary gland
C. Parietal lobe
D. Optic chiasm
E. Optic tract

A 23 year old climber falls and fractures his humerus. The surgeons
decide upon a posterior approach to the middle third of the bone. Which
of the following nerves is at greatest risk in this approach?
A. Ulnar
B. Antebrachial
C. Musculocutaneous
D. Radial
E. Intercostobrachial

Following a carotid endarterectomy a man notices that he has a


weakness of his tongue. Damage to which of the following nerves is the
most likely explanation for this process?
A. Hypoglossal
B. Accessory
C. Ansa cervicalis
D. Vagus
E. Cervical plexus

At which of the following levels does the inferior vena cava exit the
abdominal cavity?
A. T6
B. T7
C. T10
D. T8
E. T12

Which of the following structures lies deepest in the popliteal fossa?


A. Popliteal artery
B. Popliteal vein
C. Tibial nerve
D. Common peroneal nerve
E. Popliteal lymph nodes

What is the nerve root value of the external urethral sphincter?


A. S4
B. S1, S2, S3
C. S2, S3, S4
D. L3, L4, L5
E. L5, S1, S2

A patient presents to the clinic following a surgical procedure. She


complains that she is unable to shrug her shoulder. What is the most
likely underlying nerve injury?
A. Accessory nerve
B. Cervical plexus
C. Ansa cervicalis
D. Long thoracic nerve
E. Axillary nerve

A 32 year old man is undergoing a splenectomy. Division of which of the


following will be necessary during the procedure?
A. Left crus of diaphragm
B. Short gastric vessels
C. Gerotas fascia
D. Splenic flexure of colon
E. Marginal artery

A patient has a chest drain insertion. There is fresh blood at the chest
drain insertion area. Which vessel has been damaged?
A. Pericardiophrenic artery
B. Intercostal vein
C. Right ventricle
D. Vagus artery
E. Intercostal artery

Two teenagers are playing with an airgun when one accidentally shoots
his friend in the abdomen. He is brought to the emergency department.
On examination there is a bullet entry point immediately to the right of
the rectus sheath at the level of the 1st lumbar vertebra. Which of the
following structures is most likely to be injured by the bullet?
A. Head of pancreas
B. Right ureter
C. Right adrenal gland
D. Fundus of the gallbladder
E. Gastric antrum

Which of the following nerves is not contained within the posterior


triangle of the neck?
A. Accessory nerve
B. Phrenic nerve
C. Greater auricular nerve
D. Ansa cervicalis
E. Lesser occiptal nerve

A 42 year old lady is reviewed in the outpatient clinic following a routine


surgical procedure. She complains of diminished sensation at the lateral
aspect of her foot. Which of the following nerves is likely to be affected?
A. Sural
B. Superficial peroneal
C. Deep peroneal
D. Medial plantar
E. Lateral plantar

A sprinter attends A&E with severe leg pain. He had forgotten to warm
up and ran a 100m sprint race. Towards the end of the race he
experienced pain in the posterior aspect of his thigh. The pain worsens,
localising to the lateral aspect of the knee. The sprinter is unable to flex
the knee. What structure has been injured?
A. Anterior cruciate ligament
B. Posterior cruciate ligament
C. Semimembranosus tendon
D. Semiteninosus tendon
E. Biceps femoris tendon

Which nerve supplies the interossei of the fourth finger?


A. Radial
B. Median
C. Superficial ulnar
D. Deep ulnar
E. Posterior interosseous

A 45 year man presents with hand weakness. He is given a piece of paper


to hold between his thumb and index finger. When the paper is pulled,
the patient has difficulty maintaining a grip. Grip pressure is
maintained by flexing the thumb at the interphalangeal joint. What is
the most likely nerve lesion?
A. Posterior interosseous nerve
B. Deep branch of ulnar nerve
C. Anterior interosseous nerve
D. Superficial branch of the ulnar nerve
E. Radial nerve

Which of the following cranial foramina pairings are incorrect?


A. The foramen lacerum and internal carotid artery.
B. Foramen ovale and mandibular nerve.
C. Optic canal and ophthalmic artery.
D. Optic canal and ophthalmic nerve.
E. Foramen rotundum and maxillary nerve.

A man has an incision sited than runs 8cm from the deltopectoral groove
to the midline. Which of the following is not at risk of injury?
A. Cephalic vein
B. Shoulder joint capsule
C. Axillary artery
D. Pectoralis major
E. Trunk of the brachial plexus

A surgeon is due to perform a laparotomy for perforated duodenal ulcer.


An upper midline incision is to be performed. Which of the following
structures is the incision most likely to divide?
A. Rectus abdominis muscle
B. External oblique muscle
C. Linea alba
D. Internal oblique muscle
E. None of the above

A 23 year old man is injured during a game of rugby. He suffers a


fracture of the distal third of his clavicle, it is a compound fracture and
there is evidence of arterial haemorrhage. Which of the following vessels
is most likely to be encountered first during subsequent surgical
exploration?
A. Posterior circumflex humeral artery
B. Axillary artery
C. Thoracoacromial artery
D. Sub scapular artery
E. Lateral thoracic artery

What is the most useful test to clinically distinguish between an upper


and lower motor neurone lesion of the facial nerve?
A. Blow cheeks out
B. Loss of chin reflex
C. Close eye
D. Raise eyebrow
E. Open mouth against resistance

A 23 year old man is involved in a fight, during the dispute he sustains a


laceration to the posterior aspect of his right arm, approximately 2cm
proximal to the olecranon process. On assessment in the emergency
department he is unable to extend his elbow joint. Which of the
following tendons is most likely to have been cut?
A. Triceps
B. Pronator teres
C. Brachioradialis
D. Brachialis
E. Biceps

During the course of a radical gastrectomy the surgeons detach the


omentum and ligate the right gastro-epiploic
artery. From which vessel does it originate?
A. Superior mesenteric artery
B. Inferior mesenteric artery
C. Coeliac axis
D. Common hepatic artery
E. Gastroduodenal artery

A 43 year old man is reviewed in the clinic following a cardiac operation.


A chest x-ray is performed and a circular radio-opaque structure is
noted medial to the 4th interspace on the left. Which of the following
procedures is the patient most likely to have undergone?
A. Aortic valve replacement with metallic valve
B. Tricuspid valve replacement with metallic valve
C. Tricuspid valve replacement with porcine valve
D. Pulmonary valve replacement with porcine valve
E. Mitral valve replacement with metallic valve

A 63 year old lady is diagnosed as having an endometrial carcinoma


arising from the uterine body. To which nodal region will the tumour
initially metastasise?
A. Para aortic nodes
B. Iliac lymph nodes
C. Inguinal nodes
D. Pre sacral nodes
E. Mesorectal lymph nodes

A 23 year old lady is undergoing a trendelenberg procedure for varicose


veins. During the dissection of the saphenofemoral junction, which of
the structures listed below is most liable to injury?
A. Superficial circumflex iliac artery
B. Superficial circumflex iliac vein
C. Femoral artery
D. Femoral nerve
E. Deep external pudendal artery

Which of the following nerves is responsible for the motor innervation of


the sternocleidomastoid muscle?
A. Ansa cervicalis
B. Accessory nerve
C. Hypoglossal nerve
D. Facial nerve
E. Vagus nerve

A 42 year old lady has had an axillary node clearance for breast
malignancy. Post operatively she reports weakness of the shoulder. She
is unable to push herself forwards from a wall with the right arm and
the scapula is pushed out medially from the chest wall. What is the most
likely nerve injury?
A. C5, C6
B. C8, T1
C. Axillary nerve
D. Long thoracic nerve
E. Spinal accessory nerve

A 36 year old male is admitted for elective surgery for a lymph node
biopsy in the supraclavicular region. Post operatively the patient has
difficulty shrugging his left shoulder. What nerve has been damaged?
A. Phrenic nerve
B. Axillary nerve
C. C5, C6 lesion
D. C8, T1 lesion
E. Accessory nerve

A 17 year old male presents to the clinic. He complains of difficulty


using his left hand. It has been a persistent problem since he sustained a
distal humerus fracture as a child. On examination there is diminished
sensation overlying the hypothenar eminence and medial one and half
fingers. What is the most likely nerve lesion?
A. Anterior interosseous nerve
B. Posterior interosseous nerve
C. Ulnar nerve
D. Median nerve
E. Radial nerve

A 56 year old man is undergoing a pancreatectomy for carcinoma.


During resection of the gland which of the following structures will the
surgeon not encounter posterior to the pancreas itself?
A. Left crus of the diaphragm
B. Superior mesenteric vein
C. Common bile duct
D. Portal vein
E. Gastroduodenal artery

Which of the following bones is related to the cuboid at its distal


articular surface?
A. All metatarsals
B. 5th metatarsal
C. Calcaneum
D. Medial cuneiform
E. 3rd metatarsal

A 34 year old lady presents with symptoms of faecal incontinence. Ten


years previously she gave birth to a child by normal vaginal delivery.
Injury to which of the following nerves is most likely to account for this
process?
A. Genitofemoral
B. Ilioinguinal
C. Pudendal
D. Hypogastric autonomic nerve
E. Obturator

A 56 year old man undergoes an abdomino-perineal excision of the


rectum. He is assessed in the outpatient clinic post operatively. His
wounds are well healed. However, he complains of impotence.
Which of the following best explains this problem?
A. Sciatic nerve injury
B. Damage to the internal iliac artery
C. Damage to the hypogastric nerve plexus
D. Damage to the vas
E. Damage to the genitofemoral nerve

A woman develops winging of the scapula following a Patey mastectomy.


What is the most likely cause?
A. Division of pectoralis minor to access level 3 axillary nodes
B. Damage to the brachial plexus during axillary dissection
C. Damage to the long thoracic nerve during axillary dissection
D. Division of the thoracodorsal trunk during axillary dissection
E. Damage to the thoracodorsal trunk during axillary dissection

Which nerve directly innervates the sinoatrial node?


A. Superior cardiac nerve
B. Right vagus nerve
C. Left vagus nerve
D. Inferior cardiac nerve
E. None of the above

A 30 year old man presents with back pain and the surgeon tests the
ankle reflex. Which of the following nerve roots are tested in this
manoeuvre?
A. S3 and S4
B. L4 and L5
C. L3 and L4
D. S1 and S2
E. S4 only

An 18 year old male presents to casualty with a depressed skull fracture.


This is managed surgically. Over the next few days he complains of
double vision on walking down stairs and reading. On examination the
left eye cannot look downwards and medially. Which of the nerves listed
below is most likely to be responsible?
A. Facial
B. Oculomotor
C. Abducens
D. Trochlear
E. Trigeminal nerve
A 16 year old boy is hit by a car and sustains a blow to the right side of
his head. He is initially conscious but on arrival in the emergency
department is comatose. On examination his right pupil is fixed and
dilated. The neurosurgeons plan immediate surgery. What type of initial
approach should be made?
A. Left parieto-temporal craniotomy
B. Right parieto-temporal craniotomy
C. Posterior fossa craniotomy
D. Left parieto-temporal burr holes
E. None of the above

A 56 year old man is having a long venous line inserted via the femoral
vein into the right atrium for CVP measurements. The catheter is
advanced through the IVC. At which of the following levels does
this vessel enter the thorax?
A. L2
B. T10
C. L1
D. T8
E. T6

A 23 year old man falls and injures his hand. There are concerns that he
may have a scaphoid fracture as there is tenderness in his anatomical
snuffbox on clinical examination. Which of the following forms the
posterior border of this structure?
A. Basilic vein
B. Radial artery
C. Extensor pollicis brevis
D. Abductor pollicis longus
E. Extensor pollicis longus

The integrity of which muscle is assessed by the Trendelenberg test?


A. Sartorius
B. Quadratus femoris
C. Semimembranosus
D. Gluteus medius
E. Piriformis
Which of the following regions of the male urethra is entirely
surrounded by Bucks fascia?
A. Preprostatic part
B. Prostatic part
C. Membranous part
D. Spongiose part
E. None of the above

A 73 year old man has a large abdominal aortic aneurysm. During a


laparotomy for planned surgical repair the surgeons find the aneurysm
is far more proximally located and lies near the origin of the SMA.
During the dissection a vessel lying transversely across the aorta is
injured. What is this vessel most likely to be?
A. Left renal vein
B. Right renal vein
C. Inferior mesenteric artery
D. Ileocolic artery
E. Middle colic artery

A 56 year old machinist has his arm entrapped in a steel grinder and is
brought to the emergency department. On examination, he is unable to
extend his metacarpophalangeal joints and abduct his shoulder. He has
weakness of his elbow and wrist. What has been injured?
A. Ulnar nerve
B. Axillary nerve
C. Medial cord of brachial plexus
D. Lateral cord of brachial plexus
E. Posterior cord of brachial plexus

A 60 year old female is undergoing a Whipples procedure for


adenocarcinoma of the pancreas. As the surgeons begin to mobilise the
pancreatic head they identify a large vessel passing inferiorly over the
anterior aspect of the pancreatic head. What is it likely to be?
A. Superior mesenteric artery
B. Coeliac axis
C. Inferior mesenteric artery
D. Aorta
E. Left gastric artery
Which of the following structures does not lie posterior to the right
kidney?
A. Psoas major
B. Transversus abdominis
C. Quadratus lumborum
D. Medial artcuate ligament
E. 10th rib

Which of the following muscles is not within the posterior compartment


of the lower leg?
A. Peroneus brevis
B. Flexor digitalis longus
C. Soleus
D. Popliteus
E. Flexor hallucis longus

A 20 year old man undergoes an open appendicectomy performed via a


lanz incision. This surgeon places the incision on a level of the anterior
superior iliac spine in an attempt to improve cosmesis. During the
procedure the appendix is found to be retrocaecal and the incision is
extended laterally. Which of the following nerves is at greatest risk of
injury?
A. Genitofemoral
B. Ilioinguinal
C. Obturator
D. Lateral femoral cutaneous
E. Femoral

A 32 year old man is stabbed in the neck and the inferior trunk of his
brachial plexus is injured. Which of the modalities listed below is least
likely to be affected?
A. Initiating abduction of the shoulder
B. Abduction of the fingers
C. Flexion of the little finger
D. Sensation on the palmar aspect of the little finger
E. Gripping a screwdriver
A 23 year old man presents with delayed diagnosis of appendicitis. The
appendix is retrocaecal and has perforated causing a psoas abscess. Into
which structure does the psoas major muscle insert?
A. Greater trochanter of the femur
B. Linea aspera of the femur
C. Lesser trochanter of the femur
D. Iliac crest
E. None of the above

A 63 year old man is due to undergo a splenectomy. Which splenic


structure lies most posteriorly?
A. Gastrosplenic ligament
B. Splenic vein
C. Splenic artery
D. Splenic notch
E. Lienorenal ligament

A 62 year old man presents with arm weakness. On examination he has


a weakness of elbow extension and loss of sensation on the dorsal aspect
of the first digit. What is the site of the most likely underlying defect?
A. Axillary nerve
B. Median nerve
C. Ulnar nerve
D. Radial nerve
E. Musculocutaneous nerve

44. A 43 year old typist presents with pain at the dorsal aspect of the
upper part of her forearm. She alsocomplains of weakness when
extending her fingers. On examination triceps and supinator are both
functioning normally. There is weakness of most of the extensor muscles.
However, there is no sensory deficit.
A. Median nerve
B. Ulnar nerve
C. Radial nerve
D. Anterior interosseous nerve
E. Posterior interosseous nerve
45. A 28 year teacher reports difficulty with writing. There is no sensory
loss. She is known to have an aberrant Gantzer muscle.
A. Median nerve
B. Ulnar nerve
C. Radial nerve
D. Anterior interosseous nerve
E. Posterior interosseous nerve

46. A 35 year tennis player attends reporting tingling down his arm. He
says that his 'funny bone' was hit very hard by a tennis ball. There is
weakness of abduction and adduction of his extended fingers.
A. Median nerve
B. Ulnar nerve
C. Radial nerve
D. Anterior interosseous nerve
E. Posterior interosseous nerve

In relation to the middle cranial fossa, which of the following statements


relating to the foramina are incorrect?
A. The foramen rotundum transmits the maxillary nerve
B. The foramen lacerum transmits the internal carotid artery
C. The foramen spinosum lies posterolateral to the foramen ovale
D. The foramen ovale transmits the middle meningeal artery
E. The foramen rotundum lies anteromedial to the foramen ovale

During an operation for varicose veins the surgeons are mobilising the
long saphenous vein. Near its point of entry to the femoral vein an
artery is injured and bleeding is encountered. From where is
the bleeding most likely to originate?
A. Femoral artery
B. Profunda femoris artery
C. Superficial circumflex iliac artery
D. Superficial epigastric artery
E. Deep external pudendal artery

What is embryological origin of the pulmonary artery?


A. First pharyngeal arch
B. Second pharyngeal arch
C. Fourth pharyngeal arch
D. Fifth pharyngeal arch
E. Sixth pharyngeal arch

A 53 year old lady presents with pain and discomfort in her hand. She
works as a typist and notices
that the pain is worst when she is working. She also suffers symptoms at
night. Her little finger is less affected by the pain. Which of the nerves
listed below is most likely to be affected?
A. Radial
B. Median
C. Ulnar
D. Anterior interosseous nerve
E. Posterior interosseous nerve

Which of the following muscles lies medial to the long thoracic nerve?
A. Serratus anterior
B. Latissimus dorsi
C. Pectoralis major
D. Pectoralis minor
E. None of the above

A 25 year old man is being catheterised, prior to a surgical procedure.


As the catheter enters the
prostatic urethra which of the following changes will occur?
A. Resistance will increase significantly
B. Resistance will increase slightly
C. It will lie horizontally
D. Resistance will decrease
E. It will deviate laterally

Which of the following nerves is the primary source of innervation to


the anterior scrotal skin?
A. Genital branch of the genitofemoral nerve
B. Pudendal nerve
C. Ilioinguinal nerve
D. Femoral branch of the genitofemoral nerve
E. Obturator nerve

A 38 year old man presents to the clinic with shoulder weakness. On


examination he has an inability to initiate shoulder
abduction. Which of the nerves listed below is least likely to be
functioning normally?
A. Suprascapular nerve
B. Medial pectoral nerve
C. Axillary nerve
D. Median nerve
E. Radial nerve

A 72 year old lady with osteoporosis falls and sustains an intracapsular


femoral neck fracture. The fracture is completely
displaced. Which of the following vessels is the main contributor to the
arterial supply of the femoral head?
A. Deep external pudendal artery
B. Superficial femoral artery
C. External iliac artery
D. Circumflex femoral arteries
E. Superficial external pudendal artery

A 21 year old man is hit with a hammer and sustains a depressed skull
fracture at the vertex. Which of the following sinuses is
at risk in this injury?
A. Superior sagittal sinus
B. Inferior petrosal sinus
C. Transverse sinus
D. Inferior sagittal sinus
E. Straight sinus

A 19 year old man is playing rugby when he suddenly notices a severe


pain at the posterolateral aspect of his right thigh. Which
of the following muscle groups is most likely to have been injured?
A. Semimembranosus
B. Semitendinosus
C. Long head of biceps femoris
D. Gastrocnemius
E. Soleus

A 23 year old is stabbed in the groin and develops hypovolaemic shock.


What is the most likely finding on analysis of his urine?
A. Decreased specific gravity
B. Increased specific gravity
C. Increased urinary glucose
D. Increased urinary protein
E. Increased red blood cells in the urine

A 45 year old male is diagnosed with carcinoma of the head of the


pancreas. He reports that his stool sticks to the commode and will not
flush away. Loss of which of the following enzymes is most likely to
be responsible for this problem?
A. Lipase
B. Amylase
C. Trypsin
D. Elastase
E. None of the above

A 34 year old man receives morphine following an appendicectomy. He


develops constipation as a result. Which of the following best accounts
for this process?
A. Stimulation of DOPA receptors
B. Inhibition of DOPA receptors
C. Stimulation of μ receptors
D. Stimulation of serotonin release
E. Inhibition of serotonin release

A 56 year old male presents to the acute surgical take with severe
abdominal pain. He is normally fit and well. He has no malignancy. The
biochemistry laboratory contacts the ward urgently, his corrected
calcium result is 3.6 mmol/l. What is the medication of choice to treat
this abnormality?
A. IV Pamidronate
B. Oral Alendronate
C. Dexamethasone
D. Calcitonin
E. IV Zoledronate

An over enthusiastic medical student decides to ask you questions about


ECGs. Rather than admitting your dwindling knowledge on this topic,
you bravely attempt to answer her questions! One question is
what segment of the ECG represents ventricular repolarization?
A. QRS complex
B. Q-T interval
C. P wave
D. T wave
E. S-T segment

A 28 year old man is shot in the abdomen and haemorrhages. Which of


the following substances will
produce vasoconstriction in response to this process?
A. Renin
B. Angiotensin I
C. Angiotensin II
D. Aldosterone
E. None of the above

A 43 year old lady is recovering on the intensive care unit following a


Whipples procedure. She has a central venous line in situ. Which of the
following will lead to the "y" descent on the waveform trace?
A. Ventricular contraction
B. Emptying of the right atrium
C. Emptying of the right ventricle
D. Opening of the pulmonary valve
E. Cardiac tamponade

A 25 year old man is undergoing respiratory spirometry. He takes a


maximal inspiration and maximally exhales. Which of the following
measurements will best illustrate this process?
A. Functional residual capacity
B. Vital capacity
C. Inspiratory capacity
D. Maximum voluntary ventilation
E. Tidal volume

What is the typical stroke volume in a resting 70 Kg man?


A. 10ml
B. 150ml
C. 125ml
D. 45ml
E. 70ml

A 25 year old man undergoes an appendicetomy for appendicitis. The


appendix is submitted for histopathological evaluation. Which of the
following is most likely to be identified microscopically?
A. Macrophages
B. Neutrophils
C. Fibroblasts
D. Lymphocytes
E. Stem cells

A 73 year old man has an arterial line in situ. On studying the trace the
incisura can be seen. What is the
physiological event which accounts for this process?
A. Atrial repolarisation
B. Mitral valve closure
C. Ventricular repolarisation
D. Elastic recoil of the aorta
E. Tricuspid valve closure

A 22 year old man suffers a blunt head injury. He is drowsy and has a
GCS of 7 on admission. Which of the following is the major determinant
of cerebral blood flow in this situation?
A. Systemic blood pressure
B. Mean arterial pressure
C. Intra cranial pressure
D. Hypoxaemia
E. Acidosis
A 43 year old man has recurrent episodes of dyspepsia and treatment is
commenced with oral antacids. Which of the hormones listed below is
released in response to increased serum gastrin levels and decreases
intra gastric pH?
A. Cholecystokinin
B. Histamine
C. Somatostatin
D. Insulin
E. Vasoactive intestinal peptide

Which of the following cell types is least likely to be found in a wound 1


week following injury?
A. Macrophages
B. Fibroblasts
C. Myofibroblasts
D. Endothelial cells
E. Neutrophils
Theme from April 2012 Exam

Which of the following best accounts for the action of PTH in increasing
serum calcium levels?
A. Activation of vitamin D to increase absorption of calcium from the
small intestine.
B. Direct stimulation of osteoclasts to absorb bone with release of
calcium.
C. Stimulation of phosphate absorption at the distal convoluted tubule
of
the kidney.
D. Decreased porosity of the vessels at Bowmans capsule to calcium.
E. Vasospasm of the afferent renal arteriole thereby reducing GFR and
calcium urinary loss.

Theme from April 2012 Exam
PTH increases the activity of 1-α-hydroxylase enzyme, which converts
25-
hydroxycholecalciferol to 1,25-dihydroxycholecalciferol, the active form
of vitamin D.
Osteoclasts do not have a PTH receptor and effects are mediated via
osteoblasts.

What is the most likely cause for this patients deterioration?


A. Acute respiratory alkalosis secondary to hyperventilation
B. Over administration of oxygen in a COPD patient
C. Metabolic acidosis secondary to severe pancreatitis
D. Metabolic alkalosis secondary to hypokalaemia
E. Acute respiratory acidosis secondary to pneumonia

Theme from April 2012 exam

A 43 year old man has a nasogastric tube inserted. The nurse takes a
small aspirate of the fluid
from the stomach and tests the pH of the aspirate. What is the normal
intragastric pH?
A. 0.5
B. 2
C. 4
D. 5
E. 6

Theme from January 2012 Exam

Theme: Critical care


A. Hypovolaemia
B. Normal
C. Cardiogenic shock
D. Septic shock
For each of the scenarios outlined in the tables below, please select the
most likely diagnosis from the
list. Each option may be used once, more than once or not at all.
51. A 45 year old man is admitted to the intensive care unit following a
laparotomy. He has a central
line, pulmonary artery catheter and arterial lines inserted. The
following results are obtained:
Pulmonary artery occlusion pressure Cardiac output Systemic vascular
resistance
Low Low High
Hypovolaemia
Theme from April 2012 Exam
Cardiac output is lowered in hypovolaemia due to decreased preload.
52. A 75 year old man is admitted to the intensive care unit following a
laparotomy. He has a central
line, pulmonary artery catheter and arterial lines inserted. The
following results are obtained:
Pulmonary artery occlusion pressure Cardiac output Systemic vascular
resistance
High Low High
Cardiogenic shock
In cardiogenic shock pulmonary pressures are often high. This is the
basis for the use of
venodilators in the treatment of pulmonary oedema.
53. A 22 year old lady is admitted to the intensive care unit following a
laparotomy. She has a central
line, pulmonary artery catheter and arterial lines inserted. The
following results are obtained:
Pulmonary artery occlusion pressure Cardiac output Systemic vascular
resistance
Low High Low
Septic shock
Decreased SVR is a major feature of sepsis. A hyperdynamic circulation
is often present. This is
the reason for the use of vasoconstrictors.

A 22 year old lady receives intravenous morphine for acute abdominal


pain. Which of the following best
accounts for its analgesic properties?
A. Binding to δ opioid receptors in the brainstem
B. Binding to δ opioid receptors at peripheral nerve sites
C. Binding to β opioid receptors within the CNS
D. Binding to α opioid receptors within the CNS
E. Binding to μ opioid receptors within the CNS
4 Types of opioid receptor:
δ (located in CNS)- Accounts for analgesic and antidepressant
effects
k (mainly CNS)- analgesic and dissociative effects
μ (central and peripheral) - causes analgesia, miosis, decreased gut
motility
Nociceptin receptor (CNS)- Affect of appetite and tolerance to μ
agonists.
Theme from April 2012 Exam

Which of the following areas is predominantly concerned with


thermoregulation?
A. Hypothalamus
B. Anterior pituitary
C. Cerebellum
D. Brain stem
E. Temporal lobe
Theme from 2012 Exam

A 19 year old man is attacked outside a club and beaten with a baseball
bat. He sustains a blow to the
right side of his head. He is brought to the emergency department and a
policy of observation is adopted.
His glasgow coma score deteriorates and he becomes comatose. Which
of the following haemodynamic
parameters is most likely to be present?
A. Hypertension and bradycardia
B. Hypotension and tachycardia
C. Hypotension and bradycardia
D. Hypertension and tachycardia
E. Normotension and bradycardia
Theme from April 2013 Exam

Which substance can be used to achieve the most accurate measurement


of the glomerular filtration
rate?
A. Glucose
B. Protein
C. Inulin
D. Creatinine
E. Para-amino hippuric acid
Theme from January 2013 exam
Creatinine declines with age due to decline in renal function and muscle
mass. Glucose, protein (amino
acids) and PAH are reabsorbed by the kidney.

A 17 year old lady with long standing anorexia nervosa is due to


undergo excision of a lipoma. Which of
the following nutritional deficiencies is most likely to be implicated in
poor collagen formation as the
wound heals?
A. Deficiency of copper
B. Deficiency of iron
C. Deficiency of ascorbic acid
D. Deficiency of phosphate
E. None of the above
Theme from January 2011

A 45 year old man is undergoing a small bowel resection. The


anaesthetist decides to administer an
intravenous fluid which is electrolyte rich. Which of the following most
closely matches this
requirement?
A. Dextrose / Saline
B. Pentastarch
C. Gelofusine
D. Hartmans
E. 5% Dextrose with added potassium 20mmol/ L
Theme from April 2012 Exam

A 16 year old girl develops pyelonephritis and is admitted in a state of


septic shock. Which of
the following is not typically seen in this condition?
A. Increased cardiac output
B. Increased systemic vascular resistance
C. Oliguria may occur
D. Systemic cytokine release
E. Tachycardia
Cardiogenic Shock:
e.g. MI, valve abnormality
increased SVR (vasoconstriction in response to low BP)
increased HR (sympathetic response)
decreased cardiac output
decreased blood pressure
Hypovolaemic shock:
blood volume depletion
e.g. haemorrhage, vomiting, diarrhoea, dehydration, third-space losses
during major operations
increased SVR
increased HR
decreased cardiac output
decreased blood pressure
Septic shock:
occurs when the peripheral vascular dilatation causes a fall in SVR
similar response may occur in anaphylactic shock, neurogenic shock
reduced SVR
increased HR
normal/increased cardiac output
decreased blood pressure

Theme from January 2012 Exam
The SVR is reduced in sepsis and for this reason a vasoconstricting
inotrope such as
noradrenaline may be used if hypotension and oliguria remain a
concern despite administration
of adequate amounts of intravenous fluids.

Which receptor does noradrenaline mainly bind to?


A. α 1 receptors
B. α 2 receptors
C. β 1 receptors
D. β 2 receptors
E. G receptors

Theme from 2009 Exam
--------------------------------------------------------------------------------------------
--------------------------------------------------------------------------------------------
--------------------------------------------------------------------------------------------
--------------------------------------------------------------------------------------------
--------------------------------------------------------------------------------

A 12 year old child is admitted with a 12 hour history of colicky right


upper quadrant pain. On
examination the child is afebrile and is jaundiced. The abdomen is soft
and non tender at the time of
examination. What is the most likely cause?
A. Infectious hepatitis
B. Acute cholecystitis
C. Cholangitis
D. Hereditary spherocytosis
E. Gilberts syndrome
Theme from September 2012 Exam

A 40 year old man presents with obstructive jaundice. Twenty years


previously he underwent a right
hemicolectomy for a mucinous right sided colonic carcinoma. He was
subsequently diagnosed as having
Lynch syndrome. What is the most likely cause of his jaundice?
A. Hepatocellular carcinoma
B. Liver metastasis from colonic cancer
C. Pancreatic carcinoma
D. Duodenal carcinoma
E. Gastric carcinoma
Theme from April 2013

Theme: Breast disease


A. Tuberculosis
B. Actinomycosis
C. Duct ectasia
D. Fibroadenoma
E. Fat necrosis
F. Intraductal papilloma
G. Breast abscess
H. Breast cancer
What is the most likely diagnosis for each scenario given? Each
diagnosis may be used once, more than
once or not at all.
6. A 32 year old woman presents with a tender breast lump. She has a 2
month old child. Clinically
there is a tender, fluctuant mass of the breast.
Breast abscess
Theme from January 2013 Exam
Theme from September 2011 Exam
This lady is likely to be breast feeding and is at risk of mastitis. This
may lead to an abscess if not
treated. Staphylococcus aureus is usually the causative organism.
7. A 53 year old lady presents with a creamy nipple discharge. On
examination she has discharge
originating from multiple ducts and associated nipple inversion.
Duct ectasia
Duct ectasia is common during the period of breast involution that
occurs during the menopausal
period. As the ducts shorten they may contain insipiated material. The
discharge will often
discharge from several ducts.
8. A 52 year old lady presents with an episode of nipple discharge. It is
usually clear in nature. On
examination the discharge is seen to originate from a single duct and
although it appears clear,
when the discharge is tested with a labstix it is shown to contain blood.
Imaging and examination
shows no obvious mass lesion.
Intraductal papilloma
Intraductal papilloma usually cause single duct discharge. The fluid is
often clear, although it may
be blood stained. If the fluid is tested with a labstix (little point in
routine practice) then it will
usually contain small amounts of blood. A microdocechtomy may be
performed.

A 2 day old baby is noted to have voiding difficulties and on closer


inspection is noted to have
hypospadias. Which of the following abnormalities is most commonly
associated with the condition?
A. Cryptorchidism
B. Diaphragmatic hernia
C. Ventricular - septal defect
D. Bronchogenic cyst
E. Atrial septal defect
Theme from January 2012 Exam
Hypospadias most commonly occurs as an isolated disorder. Associated
urological abnormalities may be
seen in up to 40% of infants, of these cryptorchidism is the most
frequent (10%).

Theme: Liver lesions


A. Cystadenoma
B. Hydatid cyst
C. Amoebic abscess
D. Mesenchymal hamartoma
E. Liver cell adenoma
F. Cavernous haemangioma
Please select the most likely lesion for the scenario given. Each option
may be used once, more than
once or not at all.
10. A 38 year old lady presents with right upper quadrant pain and
nausea. She is otherwise well and
her only medical therapy is the oral contraceptive pill which she has
taken for many years with no
ill effects. Her liver function tests are normal. An ultrasound
examination demonstrates a
hyperechoic well defined lesion in the left lobe of the liver which
measures 14 cm in diameter.
You answered Liver cell adenoma
The correct answer is Cavernous haemangioma
Cavernous haemangioma often presents with vague symptoms and
signs. They may grow to
considerable size. Liver function tests are usually normal. The lesions
are typically well defined
and hyperechoic on ultrasound. A causative link between OCP use and
haemangiomata has yet to
be established, but is possible.
11. A 37 year old lady presents with right upper quadrant pain and
nausea. She is otherwise well and
her only medical therapy is the oral contraceptive pill which she has
taken for many years with no
ill effects. Her liver function tests and serum alpha feto protein are
normal. An ultrasound
examination demonstrates a 4cm non encapsulated lesion in the right
lobe of the liver which has a
mixed echoity and heterogeneous texture.
You answered Cystadenoma
The correct answer is Liver cell adenoma
Liver cell adenomas are linked to OCP use and 90% of patients with
liver cell adenomas have
used the OCP. Liver function tests are often normal. The lesions will
typically have a mixed
echoity and heterogeneous texture.
12. A 38 year old shepherd presents to the clinic with a 3 month history
of malaise and right upper
quadrant pain. On examination he is mildly jaundiced. His liver
function tests demonstrate a mild
elevation in bilirubin and transaminases, his full blood count shows an
elevated eosinophil level.
An abdominal x-ray is performed by the senior house officer and
demonstrates a calcified lesion
in the right upper quadrant of the abdomen.
Hydatid cyst
Similar theme in September 2011 Exam
Hyatid disease is more common in those who work with sheep or dogs.
Liver function tests may
be abnormal and an eosinophilia is often present. Plain radiographs
may reveal a calcified cyst
wall.

A 5 year old boy presents to the clinic with short stature suggestive of
achondroplasia. What is the
genetic basis of this condition?
A. X linked defect
B. Y linked defect
C. YY linked defect
D. Autosomal dominant defect
E. Autosomal recessive defect
Theme from April 2013 Exam

A 58 year old man has been suffering from mechanical back pain for
several years. One morning he
awakes from sleep and feels a sudden onset of pain in his back radiating
down his left leg. Which of the
following events is most likely to account for his symptoms?
A. Prolapse of inner annulus fibrosus
B. Prolapse of outer annulus fibrosus
C. Prolapse of nucleus pulposus
D. Rupture of the ligamentum flavum
E. None of the above
Theme from 2009 Exam
Theme from September 2012 Exam

A 34 year old man presents to the surgical clinic 8 months following a


laparotomy for a ruptured spleen.
He complains of a nodule in the centre of his laparotomy wound. This is
explored surgically and a stitch
granuloma is found and excised. From which of the following cell types
do granulomata arise?
A. Polymorpho nucleocytes
B. Plasma cells
C. Reed- Sternberg cells
D. Platelets
E. Macrophages
Granulomas are organised collections
of macrophages
Theme from 2011 Exam

A 72 year old man has just undergone an emergency repair for a


ruptured abdominal aortic
aneurysm. Pre operatively he was taking aspirin, clopidogrel and
warfarin. Intra operatively he
received 5000 units of unfractionated heparin prior to application of the
aortic cross clamp. His
blood results on admission to the critical care unit are as follows:
Full blood count
Hb 8 g/dl
Platelets 40 * 109/l
WBC 7.1 * 109/l
His fibrin degradation products are measured and found to be
markedly elevated. Which of the
following accounts for these results?
A. Anastomotic leak
B. Disseminated intravascular coagulation
C. Heparin induced thrombocytopenia
D. Adverse effect of warfarin
E. Adverse effects of antiplatelet agents

Theme from April 2012 Exam

A 23 year old man presents to the surgical clinic with an inguinal hernia.
On examination he has a small
direct hernia. However, you also notice that he has pigmented spots
around his mouth, on his palms and
soles. In his history he underwent a reduction of an intussusception aged
12 years. Which of the
following lesions is most likely to be identified if a colonoscopy were
performed?
A. Hamartomas
B. Tubulovillous adenoma
C. Colorectal cancer
D. Crohns disease
E. Hyperplastic polyps
Theme from April 2012 Exam
Theme from January 2013 Exam
He is most likely to have Peutz-Jeghers syndrome which is associated
with Hamartomas.

A 56 year old surgeon has been successfully operating for many years.
Over the past few weeks she has
begun to notice that her hands are becoming blistering and weepy. A
latex allergy is diagnosed. Which
of the following pathological processes accounts for this scenario?
A. Type 1 hypersensitivity reaction
B. Type 2 hypersensitivity reaction
C. Type 4 hypersensitivity reaction
D. Type 3 hypersensitivity reaction
E. None of the above
Hypersensitivity reactions:
ACID
type 1 --Anaphylactic
type 2 --Cytotoxic
type 3 --Immune complex
type 4 --Delayed hypersensitivity
Theme from 2012 Exam

A 43 year old man presents with haemoptysis and is diagnosed as having


tuberculosis. Which of the cell
types listed below will usually internalise the tubercule bacullis?
A. Fibroblast
B. Neutrophil
C. Erythrocyte
D. Macrophage
E. Eosinophil
Theme from January 2013 Exam

Which of these tumour markers is most helpful in identifying an


individual with hepatocellular
carcinoma?
A. Serum AFP
B. Serum CA19-9
C. CEA
D. Beta HCG
E. CA125
Theme from September 2011 Exam

Theme: Renal stones


A. Calcium oxalate
B. Uric acid
C. Cystine
D. Struvite
E. Calcium phosphate
Please select the most likely stone type for each of the following urinary
tract stone scenarios. Each
option may be used once, more than once or not at all.
34. A 73 year old lady is undergoing chemotherapy for treatment of
acute leukaemia. She develops
symptoms of renal colic. Her urine tests positive for blood. A KUB x-ray
shows no evidence of
stones.
Uric acid
Chemotherapy and cell death can increase uric acid levels. In this acute
setting the uric acid
stones are unlikely to be coated with calcium and will therefore be
radiolucent.
35. A 16 year old boy presents with renal colic. His parents both have a
similar history of the
condition. His urine tests positive for blood. A KUB style x-ray shows a
relatively radiodense
stone in the region of the mid ureter.
You answered Struvite
The correct answer is Cystine
Cystine stones are associated with an inherited metabolic disorder.
36. A 43 year old lady with episodes of recurrent urinary tract sepsis
presents with a staghorn
calculus of the left kidney. Her urinary pH is 7.3. A KUB x-ray shows a
faint outline of the
calculus.
You answered Calcium phosphate
The correct answer is Struvite
Theme from April 2012 Exam
Chronic infection with urease producing enzymes can produce an
alkaline urine with formation of
struvite stone.
A pathologist is examining a histological section and identifies Hassall's
corpuscles. With what are they
most commonly associated?
A. Follicular carcinoma of the thyroid
B. Medulla of the thymus
C. Medulla of the spleen
D. Medulla of the kidney
E. Fundus of the stomach
Theme from 2010 Exam
Theme from January 2013 Exam
Theme from April 2013 Exam

A 32 year old man is involved in a house fire and sustains extensive


partial thickness burns to his torso
and thigh. Two weeks post operatively he develops oedema of both lower
legs. The most likely cause of
this is:
A. Iliofemoral deep vein thrombosis
B. Venous obstruction due to scarring
C. Hypoalbuminaemia
D. Excessive administration of intravenous fluids
E. None of the above
Theme from 2009 Exam

A 45 year old lady has recently undergone a thyroidectomy for


treatment of medullary thyroid cancer.
Which of the following tumour markers is used clinically to screen for
recurrence?
A. Free T3
B. Thyroglobulin
C. Calcitonin
D. Free T4
E. Thyroid stimulating hormone
Theme from 2011 Exam

A 22 year old man is kicked in the head during a rugby match. He is


temporarily concussed, but then
regains consciousness. Half an hour later he develops slurred speech,
ataxia and loses consciousnesses.
On arrival in hospital he is intubated and ventilated. A CT Scan is
performed which shows an extradural
haematoma. What is the most likely cause?
A. Basilar artery laceration
B. Middle meningeal artery laceration
C. Laceration of the sigmoid sinus
D. Laceration of the anterior cerebral artery
E. Laceration of the middle cerebral artery
Theme based on September 2011 Exam
Theme from April 2013 Exam

A baby is born by normal vaginal delivery at 39 weeks gestation.


Initially all appears well and then the
clinical staff become concerned because the baby develops recurrent
episodes of cyanosis. These are
worse during feeding and improve dramatically when the baby cries.
The most likely underlying
diagnosis is:
A. Choanal atresia
B. Oesophageal reflux
C. Tetralogy of Fallot
D. Oesophageal atresia
E. Congenital diaphragmatic hernia
Theme from 2011 exam
Theme from April 2013 Exam

A 20 year old man is involved in a road traffic accident. Following the


incident he is unable to extend
his wrist. However, this improves over the following weeks. Which type
of injury is he most likely to
have sustained?
A. Radial nerve neurotmesis
B. Radial nerve neuropraxia
C. Axillary nerve axonotmesis
D. Ulnar nerve neuropraxia
E. Ulnar nerve axonotmesis
Theme from April 2011 Exam
Transient loss of function makes neuropraxia the most likely injury. The
wrist extensors are innervated
by the radial nerve making this the most likely site of injury.

Theme: Thyroid neoplasms


A. Follicular carcinoma
B. Anaplastic carcinoma
C. Medullary carcinoma
D. Papillary carcinoma
E. Lymphoma
F. Hashimotos thyroiditis
G. Graves disease

For the following histological descriptions please select the most likely
underlying thyroid
neoplasm. Each option may be used once, more than once or not at all.
51. A 22 year old female undergoes a thyroidectomy. The resected
specimen shows a non
encapsulated tumour with papillary projections and pale empty nuclei.
Papillary carcinoma
Theme from April 2012
The presence of papillary structures together with the cytoplasmic
features described is strongly
suggestive of papillary carcinoma. They are seldom encapsulated.
52. A thyroidectomy specimen from a 43 year old lady shows a mass with
prominent oxyphil cells
and scanty thyroid colloid.
You answered Medullary carcinoma
The correct answer is Follicular carcinoma
Hurthle cell tumours are a variant of follicular neoplasms in which
oxyphil cells predominate.
They have a poorer prognosis than conventional follicular neoplasms
53. A 32 year old lady undergoes a thyroidectomy for a mild goitre. The
resected specimen shows an
intense lymphocytic infiltrate with acinar destruction and fibrosis.
Hashimotos thyroiditis
Lymphocytic infiltrates and fibrosis are typically seen in Hashimotos
thyroiditis. In Lymphoma
only dense lymphatic type tissue is usually present.
From which of the following cell types do giant cells most commonly
originate?
A. Neutrophils
B. Myofibroblasts
C. Fibroblasts
D. Macrophages
E. Goblet cells
Theme from September 2011 and 2009 Exam

A 43 year old lady with hypertension is suspected of having a


phaeochromocytoma. Which of the
following investigations is most likely to be beneficial in this situation?
A. Dexamethasone suppression test
B. Urinary 5-Hydroxyindoleacetic Acid (5-HIAA)
C. Histamine provocation test
D. Tyramine provocation test
E. Urinary vanillymandelic acid measurements
Theme from September 2011 Exam
Theme from September 2012 Exam

A 46 year old lady presents with symptoms of diarrhoea, weight loss of


10 Kg and a skin rash of
erythematous blisters involving the abdomen and buttocks. The blisters
have an irregular border and
both intact and ruptured vesicles. What is the most likely diagnosis?
A. Colonic adenocarcinoma
B. Pancreatic adenocarcinoma
C. Tropical sprue
D. Glucagonoma
E. Insulinoma
Theme from September 2011 Exam
Theme from September 2012 Exam
Glucagonoma is strongly associated with necrolytic migratory erythema.

A 56 year old man presents with symptoms of neuropathic facial pain


and some weakness of the
muscles of facial expression on the right side. On examination he has a
hard mass approximately 6cm
anterior to the right external auditory meatus. What is the most likely
diagnosis?
A. Pleomorphic adenoma
B. Adenocarcinoma
C. Mucoepidermoid carcinoma
D. Adenoid cystic carcinoma
E. Lymphoma
Theme from September 2011 Exam
The patient is most likely to have a malignant lesion within the parotid.
Of the malignancies listed;
adenoid cystic carcinoma has the greatest tendency to perineural
invasion.

A 45 year old women with a thyroid carcinoma undergoes a total


thyroidectomy. The post operative
histology report shows a final diagnosis of medullary type thyroid
cancer. Which of the tests below is
most likely to be of clinical use in screening for disease recurrence?
A. Serum CA 19-9 Levels
B. Serum thyroglobulin levels
C. Serum PTH levels
D. Serum calcitonin levels
E. Serum TSH levels
Theme from September 2012 Exam

A 56 year old man has undergone a radical nephrectomy. The


pathologist bisects the kidney and
identifies a pink fleshy tumour in the renal pelvis. What is the most
likely disease?
A. Renal cell carcinoma
B. Transitional cell carcinoma
C. Angiomyolipoma
D. Phaeochromocytoma
E. Renal adenoma
Most renal tumours are yellow or brown in colour. TCC's are one of the
few tumours to appear pink.
Theme from April 2012
73 year old man presents with haemoptysis and is suspected of suffering
from lung cancer. On
examination he has an enlarged supraclavicular lymph node. Which of
the following features is most
likely to be present on histological examination?
A. Increased mitoses
B. Apoptosis
C. Barr Bodies
D. Multinucleate giant cells
E. Granuloma
Theme from 2011 Exam
Increased mitoses are commonly seen in association with malignant
transformation of cells. Apoptosis is
not a common feature of metastatic cancer. Barr Bodies are formed
during X chromosome inactivation
in female somatic cells.

Which of the following pathological explanations best describes the


initial pathological processes
occurring in an abdominal aortic aneurysm in an otherwise well 65 year
old, hypertensive male?
A. Loss of elastic fibres from the adventitia
B. Loss of collagen from the adventitia
C. Loss of collagen from the media
D. Loss of elastic fibres from the media
E. Decreased matrix metalloproteinases in the adventitia
Theme from April 2012 Exam
Theme from April 2013 Exam

A 28 year old lady has a malignant melanoma removed from her calf.
Which of the following
pathological criteria carries the greatest prognostic weighting?
A. Vascular invasion
B. Abnormal mitoses
C. Breslow thickness
D. Perineural invasion
E. Lymphocytic infiltrates
Theme from April 2012 Exam

A 20 year old girl presents with a thyroid cancer, she is otherwise well
with no significant family
history. On examination she has a nodule in the left lobe of the thyroid
with a small discrete mass
separate from the gland itself. Which of the following is the most likely
cause?
A. Follicular carcinoma
B. Anaplastic carcinoma
C. Medullary carcinoma
D. Papillary carcinoma
E. B Cell Lymphoma
Theme from September 2011 Exam

A 28 year old lady is breast feeding her first child. She presents with
discomfort of the right breast.
Clinical examination demonstrates erythema and an area that is
fluctuant. Aspiration and culture of the
fluid is most likely to demonstrate infection with which of the following
organisms?
A. Clostridium perfringens
B. Staphylococcus aureus
C. Streptococcus pyogenes
D. Staphylococcus epidermidis
E. Actinomycosis
Theme from 2013, 2010 and 2009 Exam
Staphylococcus aureus is the commonest cause. The infants mouth is
usually the source as it damages
the nipple areolar complex allowing entry of bacteria.

Theme: Tumour markers


A. Invasive ductal carcinoma of the breast
B. Prostate cancer
C. Gastric cancer
D. Ovarian cancer
E. Colorectal cancer
F. Pancreatic adenocarcinoma
G. Seminoma testicular cancer
H. Non-seminomatous testicular cancer
I. Hepatocellular carcinoma
For each tumour marker please select the most likely underlying
malignancy. Each option may be used
once, more than once or not at all.
77. Raised beta-human chorionic gonadotropin with a raised alpha-feto
protein level
Non-seminomatous testicular cancer
Theme from April 2012 Exam
A raised alpha-feto protein level excludes a seminoma
78. Elevated CA 19-9
Pancreatic adenocarcinoma
79. Raised alpha-feto protein level in a 54-year-old woman
Hepatocellular carcinoma
Tumour markers
Theme from January 2013 exam

A 56 year old man is diagnosed as having a glioma. From which of the


following cell types do these
tumours usually originate?
A. Astrocytes
B. Oligodendrocytes
C. Ependymal cells
D. Squamous cells
E. Neuroglial cells
Theme from January 2012 Exam
Gliomas originate from glial (otherwise known as neuroglial) cells.
These serve a structural function in
the CNS. The tumours produced may resemble a number of CNS cell
types. Tumours are therefore
named according to the cells they resemble rather than the origin.
Where this is not possible they are
termed gliomas.
A 63 year old man finds that he has to stop walking after 100 yards due
to bilateral calf pain. He finds
that bending forwards and walking up hill helps. He is able to ride a
bike without any pain. What is the
most likely underlying cause?
A. Lumbar canal stenosis
B. Diabetic neuropathy
C. Aorto-iliac occlusion
D. Occlusion of the superficial femoral artery
E. Pelvic rheumatoid arthritis
Theme from April 2012 Exam
Theme from April 2013 Exam
The positional nature of the pain and the fact that improves with
walking uphill makes an underlying
vascular aetiology far less likely.

A 73 year old lady is admitted for a laparoscopic cholecystectomy.


During her pre-operative assessment
it is noted that she is receiving furosemide for the treatment of
hypertension. Approximately what
proportion of the sodium that is filtered at the glomerulus will be
subsequently excreted?
A. Up to 25%
B. Upt to 75%
C. Between 3 and 5%
D. <2%
E. Between 1 and 2%
Theme from 2010 Exam

A 59 year old man presents with recurrent episodes of urinary sepsis. In


his history he mentions that he
has suffered from recurrent attacks of left iliac fossa pain over the past
few months. He has also notices
bubbles in his urine. He undergoes a CT scan which shows a large
inflammatory mass in the left iliac
fossa. No other abnormality is detected. The most likely diagnosis is:
A. Ulcerative colitis
B. Crohns disease
C. Mesenteric ischaemia
D. Diverticular disease
E. Rectal cancer
Theme from 2009 and 2011 Exam

Theme: Lung cancer


A. Adenocarcinoma
B. Small cell lung cancer
C. Large cell lung cancer
D. Squamous cell carcinoma
Please select the most likely lung cancer variant for the scenario
described. Each option may be used
once, more than once or not at all.
107. A 73 year old heavy smoker presents with haemoptysis. On
examination he is cachectic and
shows evidence of clubbing. Imaging shows a main bronchial tumour
with massive mediastinal
lymphadenopathy together with widespread visceral metastases.
Small cell lung cancer
Theme from April 2012
Small cell carcinoma is associated with disseminated disease at
presentation in the majority of
cases. Most cases occur in the main airways and paraneoplastic features
are common.
108. A 68 year old female who has never smoked presents with a mass at
the periphery of her right
lung.
Adenocarcinoma
Adenocarcinomas are the most common tumour type present in never
smokers. They are usually
located at the periphery.
109. An 85 year old man presents with a cough and haemoptysis. He has
a modest smoking history of
15 pack years. He is found to have a tumour located in the right main
bronchus, with no
evidence of metastatic disease. He decides no undergo any treatment and
he remains well for a
further 12 months before developing symptomatic metastasis.
Squamous cell carcinoma
Squamous cell carcinomas are reported to be more slow growing and
are typically centrally
located. Small cell carcinomas are usually centrally located. However,
small cell carcinomas
would seldom be associated with a survival of a year without treatment.

A 63 year old male presents with several episodes of haematuria. He


suffers from COPD secondary to
long term smoking. Which is the most likely underlying cause?
A. Renal cortical adenoma
B. Renal adenocarcinoma
C. Nephroblastoma
D. Transitional cell carcinoma of the bladder
E. Adenocarcinoma of the bladder
Theme from 2009 Exam
TCC is the most common subtype and is strongly linked to smoking. The
important point to note in this
question is the term most likely as renal adenocarcinoma may produce
similar symptoms but is less
likely.

A male infant is born prematurely at 34 weeks gestation by emergency


cesarean section. He initially
appears to be stable. However, over the ensuing 24 hours he develops
worsening neurological function.
Which of the following processes is most likely to have occurred?
A. Extra dural haemorrhage
B. Sub dural haemorrhage
C. Sub arachnoid haemorrhage
D. Intraventricular haemorrhage
E. Arteriovenous malformation
Theme from April 2012 Exam
A 43 year old man presents with dyspepsia and undergoes an upper GI
endoscopy. During the procedure
diffuse gastric and duodenal ulcers are identified. A Clo test confirms
the presence of Helicobacter
pylori infection. What is the most likely explanation for the ulcers?
A. Decreased gastric motility
B. Increased urease activity
C. Decreased release of mucous and bicarbonate
D. Decreased gastrin levels
E. Increased acid production
Theme from April 2011 Exam
H-Pylori has a number of pathological effects. In this question the main
issue is by what mechanism the
organism is able to induce both gastric and duodenal ulceration.
Without modestly elevated acid levels,
the duodenum would not undergo gastric metaplasia. H-Pylori cannot
colonise duodenal mucosa and
therefore the development of ulcers at this site can only occur in those
who have undergone metaplastic
transformation (mediated by increased acidity).

A 25 year old male pedestrian is involved in a road traffic accident. He


sustains multiple injuries and is
admitted to the intensive care unit, intubated and ventilated. Over the
next week he develops adult
respiratory distress syndrome. What is the main reason for hypoxaemia
in this condition?
A. Increased lung compliance
B. Reduced diffusion
C. Reduced surfactant
D. Reduced elastase
E. Left to right shunt
Theme from 2011 Exam
Theme from January 2013 Exam
The diffuse lung injury, which is associated with loss of surfactant and
increased elastase release from
neutrophils, results in fluid accumulation. This leads to reduced
diffusion, which is the main reason for
hypoxaemia.
A 48 year old women presents with recurrent loin pain and fevers.
Investigation reveals a staghorn
calculus of the left kidney. Infection with which of the following
organisms is most likely?
A. Staphylococcus saprophyticus
B. Proteus mirabilis
C. Klebsiella
D. E-Coli
E. Staphylococcus epidermidis
Theme from April 2012 Exam
Infection with Proteus mirabilis accounts for 90% of all proteus
infections. It has a urease producing
enzyme. This will tend to favor urinary alkalinisation which is a relative
per-requisite for the formation
of staghorn calculi.

Theme: Facial nerve palsy


A. Adenoid cystic carcinoma
B. Cerebrovascular accident
C. Petrous temporal fracture
D. Warthins tumour
E. Sarcoidosis
F. Pleomorphic adenoma
G. Cholesteatoma
Please select the most likely cause of facial nerve palsy for the scenario
given. Each option may be used
once, more than once or not at all.
3. A 22 year old man presents with symptoms of lethargy and bilateral
facial nerve palsy. On
examination he has bilateral parotid gland enlargement.
You answered Adenoid cystic carcinoma
The correct answer is Sarcoidosis
Theme from 2011 Exam
Facial nerve palsy is the commonest neurological manifestation of
sarcoid. It usually resolves.
The absence of ear discharge or discrete lesion on palpation is against
the other causes.
4. A 21 year old man presents with a unilateral facial nerve palsy after
being hit in the head. On
examination he has a right sided facial nerve palsy and a watery
discharge from his nose.
Petrous temporal fracture
Nasal discharge of clear fluid and recent head injury makes a basal skull
fracture the most
likely underlying diagnosis.
5. A 43 year old lady presents with symptoms of chronic ear discharge
and a right sided facial
nerve palsy. On examination she has foul smelling fluid draining from
her right ear and a
complete right sided facial nerve palsy.
Cholesteatoma
Foul smelling ear discharge and facial nerve weakness is likely to be due
to cholesteatoma. The
presence of a neurological deficit is a sinister feature.

In patients with an annular pancreas where is the most likely site of


obstruction?
A. The first part of the duodenum
B. The second part of the duodenum
C. The fourth part of the duodenum
D. The third part of the duodenum
E. The duodeno-jejunal flexure
Theme from 2011 Exam
Theme from September 2012 Exam

A 53 year old man presents with dyspepsia. An upper GI endoscopy is


performed and Helicobacter
pylori is identified. A duodenal ulcer is present in the first part of the
duodenum. Duodenal biopsies are
taken and demonstrate epithelium that resembles cells of the gastric
antrum. Which of the following is
the most likely explanation for this process?
A. Hyperplasia of the crypts of Lieberkhun
B. Duodenal metaplasia
C. Duodenal dysplasia
D. Duodenal carcinoma
E. Hyptertrophy of Brunners glands
Metaplasia = cell type
conversion
Theme in January 2012 exam

A 73 year old man is recovering following an emergency Hartmans


procedure performed for an
obstructing sigmoid cancer. The pathology report shows a moderately
differentiated adenocarcinoma
that invades the muscularis propria, 3 of 15 lymph nodes are involved
with metastatic disease. What is
the correct stage for this?
A. Astler Coller Stage B2
B. Dukes stage A
C. Dukes stage B
D. Dukes stage C
E. Dukes stage D
Theme from September 2011 Exam

A 56 year old man is diagnosed with an abdominal aortic aneurysm and


undergoes a CT scan to asses
the size of the aorta. During the course of his investigations a lesion of
the adrenal gland is identified. It
measures 1.5 cm in diameter and the gland is otherwise normal. What is
the most likely diagnosis?
A. Adrenal gland metastasis
B. Adrenal gland arterio-venous malformation
C. Adrenal cyst
D. Phaeochromocytoma
E. Adrenal cortical adenoma
25% of all adrenal lesions >4cm in
diameter are malignant
Theme from 2011 Exam
Incidentalomas of the adrenal gland are common and represent the most
likely lesion in this scenario.
Clearly the other lesions are all possibilities but are unlikely.

Theme: Chest pain


A. Pulmonary embolism
B. Acute exacerbation asthma
C. Physiological
D. Mitral valve stenosis
E. Aortic dissection
F. Mitral regurgitation
G. Bronchopneumonia
H. Tuberculosis
I. None of the above
What is the most likely diagnosis for the scenario given? Each option
may be used once, more than once
or not at all.
47. A 28 year old Indian woman, who is 18 weeks pregnant, presents
with increasing shortness of
breath, chest pain and coughing clear sputum. She is apyrexial, blood
pressure is 140/80
mmHg, heart rate 130 bpm and saturations 94% on 15L oxygen. On
examination there is a
mid diastolic murmur, there are bibasal crepitations and mild pedal
oedema. She suddenly
deteriorates and has a respiratory arrest. Her chest x-ray shows a
whiteout of both of her
lungs.
You answered Tuberculosis
The correct answer is Mitral valve stenosis
Mitral stenosis is the commonest cause of cardiac abnormality occurring
in pregnant women.
Mitral stenosis is becoming less common in the UK population, however
should be
considered in women from countries were there is a higher incidence of
rheumatic heart
disease. Mitral stenosis causes a mid diastolic murmur which may be
difficult to auscultate
unless the patient is placed into the left lateral position. These patients
are at risk of atrial
fibrillation (up tp 40%), which can also contribute to rapid
decompensation. Physiological
changes in pregnancy may cause an otherwise asymptomatic patient to
suddenly deteriorate.
Balloon valvuloplasty is the treatment of choice.
48. A 28 year old woman, who is 30 weeks pregnant, presents with
sudden onset chest pain
associated with loss of consciousness. Her blood pressure is 170/90
mmHg, saturations on
15L oxygen 93%, heart rate 120 bpm and she is apyrexial. On
examination there is an early
diastolic murmur, occasional bibasal creptitations and mild peal
oedema. An ECG shows ST
elevation in leads II, III and aVF.
Aortic dissection
Aortic dissection is associated with the 3rd trimester of pregnancy,
connective tissue disorders
(Marfan's, Ehlers- Danlos) and bicuspid valve. Patients may complain of
a tearing chest pain
or syncope. Clinically they may be hypertensive. The right coronary
artery may become
involved in the dissection, causing myocardial infarct in up to 2% cases
(hence ST elevation
in the inferior leads). An aortic regurgitant murmur may be auscultated.
49. A 28 year old woman, who is 18 weeks pregnant, presents with
sudden chest pain. Her blood
pressure is 150/70 mmHg, saturations are 92% on 15L oxygen and her
heart rate is 130 bpm.
There are no murmurs and her chest is clear. There is signs of
thrombophlebitis in the left leg.
Pulmonary embolism
Chest pain, hypoxia and clear chest on auscultation in pregnancy should
lead to a high
suspicion of pulmonary embolism.
Theme question in September 2011 exam
Pregnant women can decompensate rapidly from
cardiac compromise.

A 63 year old man has a history of claudication that has been present for
many years. He is recently
evaluated in the clinic and a duplex scan shows that he has an 85%
stenosis of the superficial femoral
artery. Two weeks later he presents with a 1 hour history of severe pain
in his leg. On examination he
has absent pulses in the affected limb and it is much cooler than the
contra-lateral limb. Which process
best accounts for this presentation?
A. Thrombosis
B. Embolus
C. Atheroma growth
D. Sub intimal dissection
E. Anaemia
Theme from April 2012 Exam

Theme: Paediatric ano-rectal disorders


A. Ulcerative colitis
B. Juvenile polyps
C. Haemorroids
D. Intussceception
E. Rectal cancer
F. Anal fissure
G. Arteriovenous malformation
Please select the most likely cause for the condition described. Each
option may be used once, more than
once or not at all.
79. A 4 year old boy is brought to the clinic. He gives a history of
difficult, painful defecation
with bright red rectal bleeding.
Anal fissure
Theme from April 2012 Exam
Painful rectal bleeding in this age group is typically due to a fissure.
Treatment should include
stool softeners and lifestyle advice.
80. A 2 year old has a history of rectal bleeding. The parents notice that
post defecation, a cherry
red lesion is present at the anal verge.
You answered Arteriovenous malformation
The correct answer is Juvenile polyps
Theme from September 2012 Exam
These lesions are usually hamartomas and this accounts for the colour of
the lesions. Although
the lesions are not themselves malignant they serve as a marker of an
underlying polyposis
disorder.
81. A 12 year old is brought to the colorectal clinic with a history of
rectal bleeding, altered bowel
habit, weight loss and malaise. Abdominal examination is normal.
Ulcerative colitis
The systemic features in the history are strongly suggestive of
inflammatory bowel disease
rather than the other causes.

Which of the following processes facilitates phagocytosis?


A. Apoptosis
B. Opsonisation
C. Proteolysis
D. Angiogenesis
E. Necrosis
Theme from 2008 Exam

A 55 year old man undergoes a colonoscopy and a colonic polyp is


identified. It has a lobular
appearance and is located on a stalk in the sigmoid colon. Which of the
processes below best accounts
for this disease?
A. Apoptosis
B. Metaplasia
C. Dysplasia
D. Calcification
E. Degeneration
Theme from April 2012 Exam
Most colonic polyps described above are adenomas. These may have
associated dysplasia. The more
high grade the dysplasia the greater the level of clinical concern.
A 70 year old male presents with painless frank haematuria. Clinical
examination is unremarkable.
Routine blood tests reveal a haemoglobin of 18g/dl but are otherwise
normal. What is the most likely
underlying diagnosis?
A. Squamous cell carcinoma of the bladder
B. Adenocarcinoma of the prostate
C. Adenocarcinoma of the kidney
D. Wilms tumour
E. Transitional cell carcinoma of the renal pelvis
Theme from April 2012 Exam
Polycythaemia is a recognised feature of renal cell carcinoma. Wilms
tumours most commonly occur in
children.

A newborn infant is noted to have a unilateral cleft lip only. What is the
most likely explanation for this
process?
A. Incomplete fusion of the second branchial arch
B. Incomplete fusion of the nasolabial muscle rings
C. Incomplete fusion of the first branchial arch
D. Incomplete fusion of the third branchial arch
E. Incomplete fusion of the secondary palate
Theme from April 2012 Exam
Unilateral isolated cleft lip represents a failure of nasolabial ring fusion.
It is not related to branchial
arch fusion. Arch disorders have a far more profound phenotype and
malformation sequences.

Theme: Pharyngitis
A. Infectious mononucleosis
B. Acute bacterial tonsillitis
C. Quinsy
D. Lymphoma
E. Diptheria
Please select the most likely underlying cause for the following patients
presenting with pharyngitis.
Each option may be used once, more than once or not at all.
4. An 8 year old child presents with enlarged tonsils that meet in the
midline and are covered with
a white film that bleeds when you attempt to remove it. He is pyrexial
but otherwise well.
You answered Diptheria
The correct answer is Acute bacterial tonsillitis
Theme from April 2012 Exam
In acute tonsillitis the tonsils will often meet in the midline and may be
covered with a
membrane. Individuals who are systemically well are unlikely to have
diptheria.
5. A 10 year old child presents with enlarged tonsils that meet in the
midline. Oropharyngeal
examination confirms this finding and you also notice peticheal
haemorrhages affecting the
oropharynx. On systemic examination he is noted to have splenomegaly.
You answered Lymphoma
The correct answer is Infectious mononucleosis
A combination of pharyngitis and tonsillitis is often seen in glandular
fever. Antibiotics
containing penicillin may produce a rash when given in this situation,
leading to a mistaken
label of allergy.
6. A 19 year old man has had a sore throat for the past 5 days. Over the
past 24 hours he has
notices increasing and severe throbbing pain in the region of his right
tonsil. He is pyrexial and
on examination he is noted to have a swelling of this area.
Quinsy
Unilateral swelling and fever is usually indicative of quinsy. Surgical
drainage usually
produces prompt resolution of symptoms.

Which of the following tumours are most likely to give rise to para-
aortic nodal metastasis early?
A. Ovarian
B. Bladder
C. Rectal
D. Caecal
E. Cervical
Theme from 2009 Exam
Ovarian tumours are supplied by the ovarian vessels, these branch
directly from the aorta. The cervix
drains to the internal and external iliac nodes.

Theme: Thyroid disorders


A. Sick euthyroid
B. Hyperthyroidism
C. Hypothyroidism
D. Normal euthyroid
E. Anxiety state
F. Factitious hyperthyroidism
For each of the scenarios please match the scenario with the most likely
underlying diagnosis. Each
answer may be used once, more than once or not at all.
10. A 33 year old man is recovering following a protracted stay on the
intensive care unit recovering
from an anastomotic leak following a difficult Trans hiatal
oesophagectomy. His progress is slow,
and the intensive care doctors receive the following thyroid function test
results:
TSH 1.0 u/L
Free T4 8
T3 1.0 (1.2-3.1 normal)
Sick euthyroid
Theme from April 2012 Exam
Sick euthyroid syndrome is caused by systemic illness. With this, the
patient may have an
apparently low total and free T4 and T3, with a normal or low TSH.
Note that the levels are only
mildly below normal.
11. A 28 year old female presents to the general practitioner with
symptoms of fever and diarrhoea.
As part of her diagnostic evaluation the following thyroid function tests
are obtained:
TSH < 0.01
Free T4 30
T3 4.0
Hyperthyroidism
The symptoms are suggestive of hyperthyroidism. This is supported by
the abnormal blood
results; suppressed TSH with an elevated T3 and T4.
12. A 19 year old lady presents with palpitations. The medical officer
takes a blood sample for
thyroid function tests. The following results are obtained:
TSH > 6.0
Free T4 20
T3 2.0
You answered Factitious hyperthyroidism
The correct answer is Hypothyroidism
An elevated TSH with normal T4 indicates partial thyroid failure. This
is caused by Hashimotos,
drugs (lithium, antithyroids) and dyshormogenesis.

Theme: Cardiac murmurs


A. Pulmonary stenosis
B. Mitral regurgitation
C. Tricuspid regurgitation
D. Aortic stenosis
E. Mitral stenosis
F. Aortic sclerosis
What is the most likely cause of the cardiac murmur in the following
patients? Each option may be used
once, more than once or not at all.
18. A 35 year old Singaporean female attends a varicose vein pre
operative clinic. On auscultation
a mid diastolic murmur is noted at the apex. The murmur is enhanced
when the patient lies in
the left lateral position.
Mitral stenosis
Theme from September 2011 exam
A mid diastolic murmur at the apex is a classical description of a mitral
stenosis murmur. The
most common cause is rheumatic heart disease. Complications of mitral
stenosis include atrial
fibrillation, stroke, myocardial infarction and infective endocarditis.
19. A 22 year old intravenous drug user is found to have a femoral
abscess. The nursing staff
contact the on call doctor as the patient has a temperature of 39oC. He is
found to have a pan
systolic murmur loudest at the left sternal edge at the 4th intercostal
space.
Tricuspid regurgitation
Intravenous drug users are at high risk of right sided cardiac valvular
endocarditis. The
character of the murmur fits with a diagnosis of tricuspid valve
endocarditis.
20. An 83 year old woman is admitted with a left intertrochanteric neck
of femur fracture. On
examination the patient is found to have an ejection systolic murmur
loudest in the aortic
region. There is no radiation of the murmur to the carotid arteries. Her
ECG is normal.
You answered Aortic stenosis
The correct answer is Aortic sclerosis
The most likely diagnosis is aortic sclerosis. The main differential
diagnosis is of aortic
stenosis, however as there is no radiation of the murmur to the carotids
and the ECG is
normal, this is less likely.

A 33 year old man is involved in a road traffic accident. He is initially


stable and transferred to the accident and emergency department. On
arrival he is catheterised. One minute later he becomes hypotensive,
with evidence of angioedema surrounding his penis. What is the most
likely explanation for this event?
A. Type V latex hypersensitivity reaction
B. Type IV latex hypersensitivity reaction
C. Type III latex hypersensitivity reaction
D. Type I latex hypersensitivity reaction
E. Type II latex hypersensitivity reaction

A 52 year old man with dyspepsia is found to have a duodenal ulcer. A


CLO test is taken and is positive.
Which statement relating to the likely causative organism is false?
A. It is a gram negative organism
B. It lives only on gastric type mucosa
C. It may occupy areas of ectopic gastric metaplasia
D. In patients who are colonised there is commonly evidence of fundal
gastritis on endoscopy
E. It produces a powerful urease that forms the basis of the Clo test

A 13 month old boy is brought to the surgical clinic by his mother


because his left testicle is not located in the scrotum. At which of the
following sites would the testicle be located if it were an ectopic testis?
A. Canalicular
B. Inguinal
C. External inguinal ring
D. Superficial inguinal pouch
E. High scrotal

A splenectomy increases the risk of infection from all the following


organisms except?
A. Pneumococcus
B. Klebsiella
C. Haemophilus influenzae
D. Staphylococcus aureus
E. Neisseria meningitidis

An 18 month old boy presents with recurrent urinary tract infections. As


part of the diagnostic work-up he is noted to have abnormal renal
function. An ultrasound scan is performed and shows bilateral
hydronephrosis. What is the most likely underlying diagnosis?
A. Urethral valves
B. Meatal stenosis
C. Hydronephrosis
D. Pelvico-ureteric junction obstruction
E. Benign prostatic hyperplasia
67. A 52 year old male presents with tearing central chest pain. On
examination he has an aortic regurgitation murmur. An ECG shows ST
elevation in leads II, III and aVF.
A. Pulmonary embolism
B. Anterior myocardial infarction
C. Inferior myocardial infarction
D. Proximal aortic dissection
E. Distal aortic dissection

68. A 52 year old male presents with central chest pain and vomiting. He
has drunk a bottle of vodka. On examination there is some mild crepitus
in the epigastric region.
A. Pulmonary embolism
B. Inferior myocardial infarction
C. Proximal aortic dissection
D. Distal aortic dissection
E. Boerhaave Syndrome

The Mackler triad for Boerhaave syndrome: vomiting, thoracic pain,


subcutaneous emphysema.
69. A 52 year old male presents with central chest pain. On examination
he has an mitral regurgitation murmur. An ECG shows ST elevation in
leads V1 to V6. There is no ST elevation in leads II, III and aVF.
A. Pulmonary embolism
B. Anterior myocardial infarction
C. Inferior myocardial infarction
D. Proximal aortic dissection
E. Distal aortic dissection

A 78 year old lady presents with a tender swelling in her right groin. On
examination there is a tender
swelling that lies lateral to the pubic tubercle. It has a cough impulse.
What is the most likely underlying
diagnosis?
A. Thrombophlebitis of the great saphenous vein
B. Femoral hernia
C. Thrombophlebitis of saphena varix
D. Inguinal hernia
E. Obturator hernia
A 22 year old man undergoes a splenectomy for an iatrogenic splenic
injury. On the second post
operative day a full blood count is performed. Which of the following
components of the full blood
count is the first to be affected ?
A. Erythrocyte count
B. Reticulocyte count
C. Eosinophil count
D. Monocyte count
E. Lymphocyte count

1. previously healthy 54-year-old man presents to the emergency department


complaining of chest pain. His EKG shows an acute inferior wall myocardial
infarction. His blood pressure is 90/60 mm Hg. On physical examination, he has
jugular vein distention and clear lungs. You should treat him immediately with
which of the following:
a. Intravenous fluids.
b. Norepinephrine.
c. Dopamine.
d. Nesiritide.
e. Nitroprusside.

2 54-year-old man presents with two episodes of hematemesis since yesterday.


The most likely cause of this patient’s upper gastrointestinal bleeding is:
a. Gastritis.
b. Esophagitis.
c. Esophageal varices.
d. Peptic ulcer disease.
e. Mallory-Weiss tear.

3. 45-year-old man is brought to the ED after suffering 40% body surface area
partial-and full-thickness
burn. His estimated body weight is 80 kg. Using the Parkland fluid
resuscitation formula, how much
lactated Ringer’s (LR) solution would you administer to this patient in the first
hours postburn?
a. 4400 mL.
b. 5400 mL.
c. 6400 mL.
d. 7400 mL.
e. 8400 mL.

4. The recommended initial therapy for patient suffering severe allergic


reaction to bee sting is:
a. Albuterol.
b. Cimetidine.
c. Dopamine.
d. Epinephrine.
e. Norepinephrine.

5. You are managing trauma patient with flail chest who is in respiratory
distress. Indications for
mechanical ventilation would include:
a. Respiratory rate 25/min.
b. Respiratory rate 12/min.
c. Pao2 70 mm Hg at Fio2 0.5.
d. Paco2 55 mm Hg at Fio2 0.5.
e. Alveolar–arterial oxygen gradient 350.

6. In treating patient with cardiogenic shock:


a. Dopamine preserves renal blood flow but can cause reflex
bradycardia.
b. Dobutamine increases myocardial contractility and reduces
afterload.
c. You should avoid fluid challenges in all hypotensive patients in
cardiogenic shock.
d. Furosemide causes venodilatation and decreases afterload more
than preload.
e. The optimal filling pressure for patient in cardiogenic shock
ranges from 18 to 20 mm Hg.

7. The leading cause of death in patients who are resuscitated from


cardiorespiratory arrest is:
a. Myocardial contusion and right-sided heart failure.
b. Recurrent cardiopulmonary arrest.
c. Pneumonia.
d. Anoxic encephalopathy.
e. Hemorrhage from gastric perforation and liver lacerations.
8. Mechanical obstruction of the large bowel is most
commonly caused by:
a. Adhesions.
b. Carcinoma.
c. Diverticulitis.
d. Hernia.
e. Volvulus.

9. The most common cause of acute


diarrheal disease is:
a. Food intolerance.
b. Toxic contaminants.
c. Bacterial disease.
d. Parasitic contaminants.
e. Viral illness.

10. 48-year-old woman complains of epigastric and right upper quadrant pain
and jaundice. She has fever of 39.5◦(103.1◦F) with chills. She probably has:
a. Viral hepatitis.
b. Alcoholic hepatitis.
c. Pancreatitis.
d. Ascending cholangitis.
e. Acute cholecystitis.

11. Acute mesenteric ischemia is most


commonly caused by:
a. Arterial thrombosis.
b. Arterial embolus.
c. Venous occlusion.
d. Hypercoagulable state.
e. Nonocclusive vascular disease.

12. The most important process in the pathophysiology of acute


myocardial infarction is:
a. Intracoronary artery thrombosis.
b. Coronary artery embolism.
c. Coronary artery spasm.
d. Progression of coronary artery atherosclerotic narrowing.
e. Subintimal hemorrhage at the site of preexisting
atherosclerotic narrowing.

13. In evaluating patient who has suffered syncope, the most


sensitive and specific tests are:
a. Complete blood count and basic metabolic panel.
b. Electrocardiogram and echocardiogram.
c. Head CT and carotid Doppler ultrasound.
d. History and physical examination.
e. Cardiac monitor and postural (orthostatic) vital signs.

14 The most sensitive study to detect diabetic nephropathy in the


emergency department is:
a. Serum creatinine level.
b. Serum BUN.
c. Urine albumin.
d. Protein tolerance test.
e. Renal ultrasound.

15. Severe magnesium deficiency can


lead directly to:
a. Hypercalcemia.
b. Hypophosphatemia.
c. Hypokalemia.
d. Hyponatremia.
e. Hyperchloremia.

16. The eye disorder which will probably require the


patient to be hospitalized is:
a. Corneal ulcer.
b. Ultraviolet keratitis.
c. Blepharitis.
d. Hordeolum.
e. Subconjunctival hemorrhage.

17. The usual complaint of patient with


optic neuritis is:
a. Flashing lights and visual field defect.
b. Sudden, painless, diffuse monocular
vision loss.
c. Rapidly progressive loss of central
vision.
d. Gradual loss of peripheral vision.
e. None of the above.

18. 70-year-old man with paroxysmal atrial fibrillation is on chronic warfarin


therapy. The therapeutic goal
for the patient’s INR is:
a. <1.
b. 1–1.5.
c. 1.5–2.0.
d. 3–4.
e. None of the above.

19. A 58-year-old man is brought to the ED for a syncopal episode at dinner.


His wife states that he was well until she found him suddenly slumping in
the chair and losing consciousness for a minute. The patient recalls having
some chest discomfort and shortness of breath prior to the episode.His
rhythm strip, obtained by EMS, is shown below. Which of the following best
describes these findings?

a. Mobitz type I

b. Mobitz type II

c. First-degree atrioventricular (AV) block

d. Atrial flutter with premature ventricular contractions (PVCs)

e. Sinus bradycardia
20)A 21-year-old female with a history of type 1 diabetes is brought to the
emergency room with nausea, vomiting, lethargy and dehydration. She is
lethargic, has dry mucous membranes, and is obtunded. Blood pressure is
80/40, and heart rate is 122beats/min. Heart sounds are normal. Lungs are
clear. The abdomen is soft and there is no organomegaly. She is responsive
and oriented x3 but diffusely weak. Serum sodium is 126 meq/L, potassium
is 4.3 meq/L, magnesium is 1.2 meq/L, blood urea nitrogen is 76 mg/dL,
creatinine is 2.2 mg/dL, bicarbonate is 10 meq/L, and chloride is 88 meq/L.
Seum glucose is 720 mg/dL. All the following are appropriate management
steps except:

A. Arterial blood gas.

B. Intravenous insulin.

C. Intravenous potassium.

D. 3% sodium solution.

E. Intravenous fluids.

21-Which of the following is an indication for tracheal intubation?

A. Difficulty encountered by qualified rescuers in ventilating an


apneic patient with a bag-mask device.

B. A respiratory rate of less than 20 breaths per minutes in a patient


with sever chest pain.

C. Presences of premature ventricular contractions.

D. To provide airway protection in a responsive patient with an


adequate gag reflex.

E. Ischaemic stroke, with dense hemiplegia on one side

22-Which of the following is the correct initial drug and dose for treatment
of asystole?
A. Epinephrine 2 mg IV.

B. Atropine 0.5 mg IV.

C. Lidicaine 1 mg/kg IV.

D. Epinephrine 1 mg IV.

E. Amiodarone 300mg IV bolus.

23-Which of the following signs and symptoms are most likely to result from
symptomatic Bradycardia?

A. Headache, pain or pressure in the center of the chest,


palpitation.

B. Nausea, diaphoresis, pain radiating to the back and between


the shoulder blades.

C. Chest pain, shortness of breath, hypotension, dizziness or


altered level of consciousness, congestive heart failure, premature
ventricular contractions.

D. Difficulty with speech, unilateral limb weakness, severe


headache, facial droop.

E. Abdominal cramps.

24-A 75-year-old man presents to the ED complaining of having


lightheadedness and palpitation for 1 week. His heart rate is 160 bpm and
irregular; his blood pressure is 100/60 mmHg. The physical examination is
normal with no evidence of cardiac or circulatory failure. The 12-lead ECG
shows rapid atrial fibrillation but is otherwise normal. Which of the
following should be included in your initial orders for this patient?

A. Oxygen-IV-monitor.

B. Immediate defibrillation.

C. No therapy is indicated.
D. Epinephrine 1 mg IV every 3 to 5 minutes.

E. Atropine 1 mg IV.

25-Which of the following conditions can mimic the signs and symptoms of
acute stroke?

A. Hypoglycemia.

B. Cardiac arrest.

C. Pneumothorax.

D. Wolff-Parkinson-white syndrome.

E. Upper GI bleeding.

26-A 50-year-old woman is admitted with high fever and a generalized


headache. Examination reveals chemosis and cyanosis of the upper face.
There was proptosis and ophthalmoplegia of the left eye. Pin prick sensation
was lost from the left forehead. The most likely diagnosis is

A. Periorbital cellulites.

B. Osteomylelitis of the left maxillary sinus.

C. Left retro-orbital tumour.

D. Cavernous sinus thrombosis.

E. Superior sagittal sinus thrombosis.

27) A 27-year-old woman presents with a three-day history of pleuritic chest


pain. She has been taking contraceptive pills. Physical examination of the
lower limbs is normal. The 12-lead ECG shows sinus tachycardia and the
V/Q scan shows multiple mismatched segmental defects in both lung fields.
Which one of the following statement is not accurate?
A. In patients with pulmonary embolism, only 10-20% have clinical
evidence of deep venous thrombosis.

B. The S1, Q3, T3 changes on the 12-lead ECG, when identified are
diagnostic.

C. Arterial blood gases are normal in 50% of patients.

D. Warfarin should be continued for six months.

E. Thrombolysis should be considered if evidence of haemodynamic


instability is present.

28)A 24-year-old man presents to the ER with a sudden sharp pain in the left
side of the chest and shortness of breath. Examination reveals: respiratory
rate 40/minute, cyanosis, tracheal deviation to the right and a hyper-resonant
left lung. Appropriate treatment includes all the following except

A. Intubation and positive-pressure ventilation.

B. Oxygen by mask.

C. Immediate needling of the left side of his chest.

D. Immediate chest X-ray.

E. Estimation of acid-base status.

29)A 19-year old car mechanic was brought by ambulance to the ER with
recurrent tonic-clonic seizures which had been going on for three hours. He
failed to regain full consciousness between seizures, The immediate
treatment he should receive is

A. Intravenous phenytoin infusion.

B. Intravenous lorazepam.

C. Intravenous Phenobarbital infusion.


D. Intravenous sodium valproate infusion.

E. Anesthesia with thiopentone and ventilation.

30)All of the following are indicative of a severe asthmatic attack EXCEPT

A. Silent chest.

B. Hypercapnia.

C. Thoracoabdominal paradox (o\paradoxical respiration).

D. Pulsus paradoxus of 5 mmHg.

E. Altered mental status.

31)-Which of the following is most appropriately administrated to an


individual who has ingested an overdose of aspirin?

A. Acetazolamide.

B. Sodium bicarbonate.

C. N-acetylcysteine.

D. Flumazil.

E.Allopurinol.

32.Shortly after delivery, a newborn was noted to have good muscle tone
and movement of all extremities, a heart rate of 90/min, sneezing,
coughing, a loud cry, and pink color over the entire body. Her Apgar
score is:
A) 6.
B) 7.
C) 8.
D) 9.
E) 10.
33. A 4-year-old boy is in severe respiratory distress after a motor
vehicle crash. He was ejected from the vehicle and has major facial and
head trauma. When managing his airway, you should:
A) Use an appropriately sized laryngeal mask airway to limit the likelihood
of aspiration.
B) Perform a needle cricothyrotomy rather than a formal surgical
cricothyrotomy if you cannot intubate or ventilate.
C) Extend the neck to improve visualization of the airway structures.
D) Attempt blind nasotracheal intubation with an appropriate size
endotracheal tube.
E) Avoid succinylcholine because of the acute trauma.

34. A 6-month-old infant is brought to the ED because of poor feeding.


The infant is listless, but responds to pain. There are no palpable pulses
and the cardiac monitor shows a narrow complex tachycardia with a
rate of 250. The child weighs 8 kg. Your next step should be:
A) Administer verapamil 0.1 mg/kg slow IV push.
B) Administer digoxin 0.02 mg/kg IV.
C) Cardiovert immediately with 16 J.
D) Cardiovert immediately with 8 J.
E) Defibrillate immediately with 16 J.

35. A 62-year-old woman presents with an altered mental status. She is


responsive to painful stimuli and has a Glasgow Coma Scale score of 11.
Blood pressure is 100/60 mmHg and heart rate is 100/min. Physical
examination reveals poor skin turgor and dry mucus membranes. EKG
shows QT interval shortening.Laboratory findings are serum calcium
14.2 mg/dL, serum phosphorus 2.9 mg/dL, serum potassium 3.9 mEq/L,
and creatinine 1.9 mg/dL. Your initial treatment should include:
A) Magnesium sulfate 2 gm slow IV push.
B) Pamidronate 60 mg IV.
C) Calcitonin 4 IU/kg.
D) Furosemide 60 mg IV after saline resuscitation.
E) Immediate hemodialysis.

36. The initial management of a patient in hemorrhagic shock should


consist of:
A) Rapid infusion of isotonic crystalloid fluids.
B) Ensuring adequate oxygenation and ventilation.
C) Pharmacologic support to maintain stable hemodynamics.
D) Stabilization of all long bone fractures.
E) Immediate transfusion of uncrossmatched blood.

37. The benefits associated with the use of nitroglycerin in patients with
acute coronary syndromes result primarily from:
A) Pulmonary artery vasoconstriction.
B) Decreasing myocardial preload.
C) Increasing afterload.
D) Coronary vasoconstriction.
E) Inotropic support.

38- You are managing a patient who is hypotensive, refractory to


intravenous fluids. You decide to use a pure alpha-adrenergic agent, so
choose:
A) Dopamine.
B) Dobutamine.
C) Amrinone.
D) Isoproterenol.
E) Phenylephrine.

39-. Which of the following statements regarding face mask ventilation


is TRUE;
A) The sniffing position causes the tongue to be more anterior.
B) It is made more difficult when prosthetics (dentures) are left in place.
C)It should not be used for the duration of the anesthetic.
D) In the presence of normal lung compliance and an open airway, face mask
ventilation should require as much as 40 cm H2O positive pressure.
E)The sniffing position causes the esophagus to be more posterior.

40. Metabolic acidosis with a normal anion gap may be caused by:
A) Aspirin toxicity
B) Diabetic ketoacidosis
C) Chronic diarrhea
D) Uremia
E) Lactic acidosis

41. What is the best interpretation of an arterial blood gas analysis of pH,
7.35; PaCO2, 60; PO2, 80; and HCO3, 32:
A) Acute respiratory acidosis
B)Chronic respiratory acidosis with metabolic compensation
C) Chronic respiratory acidosis without metabolic compensation
D) Chronic metabolic alkalosis with respiratory compensation
E) Acute metabolic alkalosis

42. Total body water is approximately _________ percent; of total body


weight.
A) 10
B) 20
C) 40
D) 60
E) 80

43. Plasma volume is approximately _________ percent; of the


extracellular volume (ECV).
A) 10
B) 20
C) 30
D) 40
E) 50

44. The intracellular concentration of potassium (K) is approximately


_________ mEq/L.
A) 110
B) 130
C) 150
D) 4
E) 10

45. Which statement about CPR is FALSE ?


A) Immediate CPR can double or triple survival
B) CPR plus defibrillation within 3-5 min of collapse can produce survival
rates as high as 49-75%
C)Chest compressions produce blood flow by increasing the intrathoracic
pressure and by directly compressing the heart
D) Chest compressions can produce systolic BP 60-80mmHg
E) Depth of cardiac compression for infants 1-2cm

46. Which statement about ACLS is FALSE ?


A)Tracheal tube route is more preferable than intraosseous (IO) route.
B)Give adrenaline every 3-5 min when managing PEA
C)Three successive shocks may be considered if VF/VT occurs during
cardiac cath.
D)Any fluid or medication approved for IV use may be given through an
intraosseous IV
E)There is increased emphasis on the use of capnogram

47. ECG changes in Hyperkalemia include all of the following


EXCEPT:
A)Peaked T waves ;
B)Prolongation of P-R interval .
C)Disappeared P waves ;
D)Narrowing QRS complex .
E)ST segment depression .

48.Which of the following is the most likely cause of pyrexia occurring 48


hours after an abdominal operation?

f. Wound infection.

g. D.V.T.

h. Chest infection.

i. Pulmonary embolism.

j. Leaking intestinal anastomosis.

49-In tetanus infection the following statements are correct except:


a.The disease is caused by the toxin of Clostridium tetani.
b.The toxin travels along nerves to the central nervous system.
c.The spasm can stop respiration.
dIt may affect the neonate.
e.Treatment should include blood transfusion and IV aminoglycoside.

50-Which of the following tests will be affected after intake of non-


steroidal anti-inflammatory drugs:
a.Coagulation time.
b.PTT.
c.INR.
d.Bleeding time.
e.Thrombin time.
51-High cardiac output is present in:
a.Cardiogenic shock.
b.Anaphylactic shock.
c.Early septic shock.
d.Neurogenic shock.
e.Late septic shock.

52.Which of the following is not true about neurogenic shock?


a.It may follow spinal fractures.
b.There is tachycardia.
c.The extremities are warm.
d.There is bradycardia and hypotension.
e.Vasopressors may be useful.
53.The initial maneuver to establish an airway in a patient with multiple
injuries is:

a. Oropharyngeal airway.

b. Uncuffed endo-tracheal tube.

c. Suctioning foreign debris and lifting up the mandible.

d. Cuffed endo-tracheal tube.

e. Tracheostomy

54.Which of the following tests is used to monitor the adequacy of


anticoagulation with warfarin?
a.Bleeding time.
b.Clotting time.
c.Activated partial thromboplastin time (APTT).
d.Fibrinogen level.
e.International normalized ratio (INR).

55-Severe limb pain of sudden onset can be caused by all the following
conditions except:
a.Acute ischaemia.
b.Deep venous thrombosis.
c.Muscle tear.
d.Sciatica.
e.Bone fracture.

56.All of the following are presentations of acute limb ischaemia except:

a.Coldness.
b.Muscle wasting.
c.Sluggish capillary circulation.
dSkin mottling.
e.Pain.

57-The following are known to cause acute renal failure except

A Malaria

B Sickle cell trait

C Septicemia

D DIC

E Concelaled antepartum haemorrhage

58 In pregnancy complicated by bronchial asthma


A There is increased risk of congenital anomalies

B High dose of corticosteroid is contraindicated

C There is increased perinatal mortality

D Premature labor is less common

E Asthma may be improved during pregnancy.

59.Properly performed closed-chest cardiac massage provides


approximately what percentage of prearrest cardiac output?
A. 15%.
B. 25%.
C. 45%.
D. 60%.
E. 85%.

60. Vital capacity:


A. Is the volume of air expired from full inspiration to full expiration
B. Increases gradually with age in adults
C. Is greater in women than in men of similar age and height
D. Is equal to the sum of the inspiratory and expiratory reserve volumes
E. May be measured by capnogram

61.What is the maximum dose of lidocaine


A .300 mg.
B .4.5 mg/kg.
C .5mg/Kg
D. 7 mg/kg.
E. 1.5 mg/kg

62.A 1-week-old infant presents with a history of poor feeding. The child
is afebrile with a heart rate of 160 beats per minute and respirations of
40 breaths per minute. The child is noted to have fine movements of the
limbs that respond to stimuli and stop with manual stimulation. There
are no accompanying movements of the eyes or tongue. Blood glucose is
found to be 21 mg/dL. After IV access is obtained, what is the MOST
appropriate dose of IV glucose to administer?
(A) 4 cc/kg of D10W.
(B) 4 cc/kg of D25W.
(C) 2 cc/kg of D25NS.
(D) 1 cc/kg of D50.
(E) 2 cc/kg of D50

63.Which of the following statements regarding the pediatric airway is


TRUE?
A. The MacIntosh blade is preferred in children younger than 2-4 years of
age.
B A normal 4-year-old child would require a 4-mm endotracheal tube.
C. The blind finger sweep is the preferred maneuver to clear a suspected
airway
obstruction in an unconscious child when advanced airway equipment is
unavailable.
D. A child is more likely than an adult to develop airway obstruction
E. Use of cuffed tubes is preferred in children younger than 8 years to
prevent aspiration

64.Pulse oximetry is used frequently in the ED as a noninvasive


measurement of arterial oxygen saturation. In some clinical situations,
the pulse oximetry measurement will be falsely elevated and should not
be relied upon. In which of the following clinical scenarios would the
pulse oximeter reading be expected to be falsely elevated?
A. Cyanide poisoning.
B. Chronic obstructive pulmonary disease.
C. Carbon monoxide poisoning.
D. Neonate who is producing fetal hemoglobin.
E. All of the above

65.Concerning intravenous fluids in the critically ill:


(A) Approximately 20% of infused normal saline (0.9% NaCl) remains
intravascular
(B)Hartmann’s solution (Ringer’s lactate) contains approximately 20 mmol/l
potassium
(C)Normal saline has a pH of 7.4
(D)Hartmann’s solution is hypotonic
(E) Approximately
66.In cardiac arrest all of the following are true EXCEPT:
(A) Cerebral hypoxic injury begins within 3 minutes
(B) Drug delivery is optimally achieved via a central vein
(C)Epinephrine 1 mg should be administered every minute during
cardiopulmonary resuscitation
(D) Open chest cardiac massage is indicated after recent cardiothoracic
surgery
(E)Associated with trauma, the cervical spine should be protected during
airway manipulation

67. Daily fluid maintenance for 30 kg child is approximately


(A)1000 ml
(B)1300ml
(C)1700ml
(D)2000ml
(E)2300 ml

68.-Bleeding tendency following massive blood transfusion is primarily


due to:

a. Platelet depletion.

b. Decrease in fibrinogen.

c. Decrease in prothrombin.

d. Increased fibrinolytic activity.

e. Decrease of calcium.

69-The factor most commonly associated with central venous catheter


related nosocomial infection is:

a. Sterility of the insertion technique.

b. Type of solution being administered through the line.

c. Care of the catheter once inserted.

d. Proximity of the catheter to another wound.

e. Presence of another infection else where.


70-Insulin has all the following actions except:

a. Increased glycogen synthesis by the liver.

b. Decreased lipolysis.

c. Decreased proteolysis.

d. Increased gluconeogenesis.

e. Increased K uptake by the cells.

71-The recommended initial therapy for a patient suffering a severe


allergic reaction to a bee sting is:

a. Albuterol.

b. Cimetidine.

c. Dopamine.

d. Epinephrine.

e. Norepinephrine.

72-A 4-year-old girl developed a fever to ( 40°C ) and a diffuse, raised


purpuric rash this morning. Laboratory studies reveal: WBC 5,100/cu mm;
hemoglobin, 11.8 g/dL; platelets, 189 x 103/cu mm; prothrombin time,
11.8 sec; partial thromboplastin time, 28 sec; and fibrinogen, 250 mg/dL
Of the following, the MOST likely etiology of the purpura is:

a. disseminated intravascular coagulation

b. Henoch-Schِnlein purpura

c. hereditary telangiectasia
d. inherited platelet dysfunction

e. meningococcemia

73-A 1-week-old term female infant suddenly develops melena. The


mother is healthy, and the pregnancy was normal. Delivery was attended
by her family at home, but it was uncomplicated. The infant is
breastfeeding. Hemoglobin is 6.7 g/dL, white blood cell count is 13,000/cu
mm, and platelets are 599 x 103/cu mm . Of the following, the MOST likely
cause of the bleeding is:

a. factor VIII deficiency hemophilia

b. factor IX deficiency hemophilia

c. necrotizing enterocolitis

d. rectal fissure

e. vitamin K deficiency

74-Which one of the following is a reliable method of assessing


dehydration in infants: Useful indices of dehydration in infants include:

a. Assessment of intra-ocular tension by palpation

b. Skin turgor over the dorsum of the hand

c. Palpation of the anterior fontanelle

d. Blood pressure

e. Serum sodium concentration


75- Which of the following statements is true concerning: Intussussception
in children:

a. Is more common between the age of 3 months to 8 years

b. Bile stain vomiting is an early feature

c. Bleeding per rectum is the most common presenting symptom

d. Is usually caused by a polyp or pedunculated tumour

e. Hydrostatic reduction plays an important role in the management

76-Minimal change nephrotic syndrome is characterized by the following


except:

a. Edema

b. Albuminuria

c. Hypercholesterolemia

d. Hypoproteinemia

e. Hypertension

77- Opioids:

a. May cause chest wall relaxation

b. Sedation but not pruritus is reversed by naloxone

c. Morphine 3 sulphate is an unactive metabolite


d. Morphine 6 sulphate is not an active metabolite

e. Cause hypotension by vasodilatation

78.The most causes of sudden cardiac death (first 2-3 hrs) after acute
myocardial infarction?

A. Ventricular fibrillation

B. Free wall rupture

C. Low cardiac output

D. Pulmonary edema

E. Atrial fibrillation

79-In diabetic keto- acidosis which of the following is responsible for


hyperkaelemia?

a. Lack of sodium
b. Lack of insulin
c. Lack of electrolytes
d. Lack of glucose

E.increase of glucose
80. In shaft of humerus fracture, Nerve injured commonly

a. Median

b. Ulnar

c. Musculo cutaneous

d .Radial

e.posterior interosseous

81. Which of the following is TRUE regarding small

bowel obstruction?

(A) It is rarely associated with abdominal pain

(B) It usually presents with clear, nonbilious vomiting

(C) Diffuse, severe abdominal tenderness indicates secondary


peritonitis

(D) X-ray shows air-filled bowel with prominent haustrae

(E) Nasogastric tube decompression is generally ineffective.

82. Most common source of pulmonary embolus:

A.Iliofemoral veins
B. Deep veins of leg
C. Deep veins of calf
D.Popliteal vein.
E. Superfecial femoral vein.

83. Earliest sign of deep vein thrombosis is:

A.Calf tenderness
B. Rise in temperature
C. Swelling of calf muscle
D. Homan’s sign.
E.Leg oedaema.
84.Which of the following is not associated with increased likelihood of
infection after major elective surgery?
A. Age over 70 years.
B. Chronic malnutrition.
C. Controlled diabetes mellitus.
D. Long-term steroid use.
E . Anaemia .

85. Which of the following step is not a part of the primary survey in a
trauma patient?

A. Insuring adequate ventilatory support


B. Measurement of blood pressure and pulse
C. Neurologic evaluation with the Glasgow Coma Scale
D. Examination of the cervical spine.

E. Insuring adequate cervical spine support

86.The most common cause of fatal transfusion reactions is:

A. An allergic reaction.

B. An anaphylactoid reaction.

C. A clerical error.

D. An acute bacterial infection transmitted in blood.

E.Hypoclaemia

87. Major burn is defined as one of the following except.

A.Second and third degree of greater than 15% in children


B.Second and third degree of greater than 20% in adults less than 50 years
C.Third degree burn less than10%
D.Inhalation injury
E.Significant electrical injury.

88. A 63-year-old man, who weighs 65 kg, is in his 2nd postoperative day after
an abdominoperineal resection for cancer of the rectum. An indwelling Foley
catheter was left in place after surgery. The nurses are concerned because,
even though his vital signs have been stable, his urinary output in the past 2
hours has been zero. In the preceding 3 hours, they had collected 56 mL, 73
mL, and 61 mL. Which of the following is the most likely diagnosis?
A. Acute renal failure
B. Damage to the bladder during the operation
C. Damage to the ureters during the operation
D. Dehydration
E. Plugged or kinked catheter

89. A 40-year-old electrical lineman is injured on a electric pole and is


brought to the emergency department unconscious. He has an area on his
left hand that is charred, evidence of thermal injury to the arm, and an exit
wound on his right knee. Which of the following interventions is least
appropriate in the emergency department?

A. escharotomy of the left arm

B. CT scan of the abdomen and pelvis

C. ET intubation

D. CT scan of the head and neck

E. measurement of serum creatinine phosphokinase levels


90.Regarding inhalation injury which is correct ?
A. Inhalation injury should be suspected in any patient with a history of
closed–space smoke exposure ,wheezing, horseness, singed nasal
hairs, or carbonaceous sputum .

B. Bronchoscopy is the most reliable modality in the diagnosis of


inhalation injury .

C. The separation of ciliated epithelial cells from their underlying


basement membrane is a hallmark of inhalation injury .

D. Maintainig an open airway and maximizing gas exchange are key


components of management.

E.All are true

91. Nasotracheal intubation:

A. Is preferred for the unconscious patient without cervical spine injury.

B. Is preferred for patients with suspected cervical spine injury.

C. Maximizes neck manipulation.

D. Is contraindicated in the patient who is breathing spontaneously.

E.All are true.

92. Which of the following statements about head injuries is false?

A. The majority of deaths from auto accidents are due to head injuries.

B. Head injury alone often produces shock.

C. A rapid and complete neurologic examination is part of the initial


evaluation of the trauma patient.

D. Optimizing arterial oxygenation is part of initial therapy.


93.What percentage of patients with thoracic trauma require
thoracotomy?

A. 10%–15%.

B. 20%–25%.

C. 30%–40%.

D. 45%–50%.

E. 70%

94.The radiographic findings indicating a torn thoracic aorta include:

A. Widened mediastinum.

B. Presence of an apical “pleural cap.”

C. First rib fractures.

D. Tracheal deviation to the right.

E. all of the above.

95.A 28-year-old male was injured in a motorcycle accident in which he


was not wearing a helmet. On admission to the emergency room he was
in severe respiratory distress and hypotensive (blood pressure 80/40 mm.
Hg), and appeared cyanotic. He was bleeding profusely from the nose and
had an obviously open femur fracture with exposed bone. Breath sounds
were decreased on the right side of the chest. The initial management
priority should be:

A. Control of hemorrhage with anterior and posterior nasal packing.

B. Tube thoracostomy in the right hemithorax.

C. Endotracheal intubation with in-line cervical traction.

D. Obtain intravenous access and begin emergency type O blood


transfusions.
E. Obtain cross-table cervical spine film and chest film.

96.Which of the following statement(s) is true concerning the diagnosis of


a peripheral vascular injury?

a. The presence of a Doppler signal over an artery in an extremity


essentially rules out an arterial injury

b. Doppler examination is a valuable tool in the diagnosis of venous


injuries

c. A gunshot wound in the proximity of a major vessel is an absolute


indication for arteriography

d. Both the sensitivity and specificity of arteriography of the injured


extremity approaches 100%

E. none of the above

97.A 22-year-old male is hospitalized with multiple extremity fractures


including a comminuted fracture of the femur and multiple rib fractures.
Which of the following statement(s) is/are true concerning his hospital
course?
a.Low-dose heparin should not be employed during his hospital stay
a. Acute respiratory failure associated with petechiae of the head, torso,
and sclerae would suggest a pulmonary embolism

b. Early fracture fixation would decrease the incidence of fat emboli


c. The placement of a Greenfield filter should be avoided due to the risk
of lower extremity edema

e. non of the above

98. Genitourinary injuries are common with both blunt and penetrating
trauma. Which of the following statement(s) is true concerning
genitourinary trauma injuries?

a.All patients with microscopic hematuria and blunt trauma should be


evaluated with an intravenous pyelogram
b.The indications for radiographic assessment of renal injury in the face of
blunt trauma is more liberal than penetrating trauma
c.CT scan is the current imaging technique of choice for suspected renal
trauma
d. Perinephric hematomas occurring after either penetrating or blunt
trauma should not be explored

e. Extraperitoneal bladder ruptures usually treated by open surgical


exploration .

99. The most common finding of a blunt cardiac injury is:


a.Tamponade.
b.Minor ECG abnormality.
c.Coronary artery occlusion.
d.Aortic incompetence.
e.Mitral valve injury.

100..The first priority in the management of facio-maxillary injuries


is to:
a.Rule out fracture of the cervical spine.
b.Chest x-ray.
c.Ensure a patent airway
d.Stop the bleeding.
e. IV fluid resuscitation
EMERGENCY MEDICINE –Promotion Exam I and II year septemper 2012

31. All of the following are presentations of acute limb ischaemia


except:

a. Coldness.

b. Muscle wasting.

c. Sluggish capillary circulation.

d. Skin mottling.

e. Pain.

2-.A 70-year-old woman complains of several hours of lower abdominal


cramping and massive rectal bleeding. Nasogastric aspirate shows bile
but no blood. Her most likely diagnosis is:
a. Diverticulitis.
b. Peptic ulcer disease.
c. Internal hemorrhoids.
d. Diverticulosis.
e. Ischemic bowel.

3. A 48-year-old woman complains of epigastric and right upper


quadrant pain and jaundice. She has a fever of 39.5◦C with chills. She
probably has:
a. Viral hepatitis.
b. Alcoholic hepatitis.
c. Pancreatitis.
d. Ascending cholangitis.
e. Acute cholecystitis.

4. ABO matching of donor and recipient is unnecessary in the patient


receiving:
a. Whole blood.
b. Platelets.
c. Packed red blood cells (PRBCs).
d. Fresh frozen plasma.
e. Leukocyte-poor red blood cells.
5. The most important radiographs used in evaluating an adult trauma
patient are:
a. Chest, lateral cervical spine, abdomen.
b. Pelvis, abdomen, lateral and anteroposterior cervical spine.
c. Lateral cervical spine, chest, pelvis.
d. Abdomen, chest, lateral and anteroposterior cervical spine.
e. Abdomen, chest, lateral cervical spine, pelvis

6. A 23-year-old patient sustains a gunshot wound to the left fourth


intercostal space, midclavicular line. He had a palpable radial pulse at
the scene. Bilateral breath sounds are present. After rapid infusion of
2000 mL of a crystalloid solution, he has only an intermittent, thready
radial pulse. His neck veins are flat. Themonitor shows a sinus
tachycardia. This patient now requires:
a. Immediate portable chest radiograph.
b. Cricothyrotomy or tracheostomy.
c. Central line placement.
d. Left tube thoracostomy.
e. Nasotracheal intubation.

7. The most significant immediate complication associated with pelvic


fracture is:
a. Hemorrhage.
b. Rectal or vaginal lacerations.
c. Sciatic nerve injury.
d. Infection.
e. Myositis ossificans.

8.The following are known to cause acute renal failure except

A Malaria

B Sickle cell trait

C Septicemia
D DIC

E Concelaled antepartum haemorrhage

9. In pregnancy complicated by bronchial asthma

A There is increased risk of congenital anomalies

B High dose of corticosteroid is contraindicated

C There is increased perinatal mortality

D Premature labor is less common

E Asthma may be improved during pregnancy.

10.Which of the following is the most likely cause of pyrexia


occurring 48 hours after an abdominal operation?

a.Wound infection.

b.D.V.T.

c.Chest infection.

d.Pulmonary embolism.

e.Leaking intestinal anastomosis.

11.In tetanus infection the following statements are correct except:


a.The disease is caused by the toxin of Clostridium tetani.
b.The toxin travels along nerves to the central nervous system.
c.The spasm can stop respiration.
d.It may affect the neonate.
e.Treatment should include blood transfusion and IV aminoglycoside.
12.Which of the following tests will be affected after intake of non-
steroidal anti-inflammatory drugs:

a.coagulation time.
b.PTT.
c.INR.
d.Bleeding time.
e.Thrombin time.
13.High cardiac output is present in:
a. Cardiogenic shock.
b. Anaphylactic shock.
c. Early septic shock.
d. Neurogenic shock.
e. Late septic shock.

14.Which of the following is not true about neurogenic shock?

a. It may follow spinal fractures.

b. There is tachycardia.

c. The extremities are warm.

d. There is bradycardia and hypotension.

e. Vasopressors may be useful.

15.The initial maneuver to establish an airway in a patient with


multiple injuries is:

f. Oropharyngeal airway.

g. Uncuffed endo-tracheal tube.

h. Suctioning foreign debris and lifting up the mandible.


i. Cuffed endo-tracheal tube.

j. Tracheostomy

16.Which of the following tests is used to monitor the adequacy of


anticoagulation with warfarin?

a. Bleeding time.

b. Clotting time.

c. Activated partial thromboplastin time (APTT).

d. Fibrinogen level.

e. International normalized ratio (INR).

17.Severe limb pain of sudden onset can be caused by all the following
conditions except:

a. Acute ischaemia.

b. Deep venous thrombosis.

c. Muscle tear.

d. Sciatica.

e. Bone fracture.

18.The most significant immediate complication associated with pelvic


fracture is:
a. Hemorrhage.
b. Rectal or vaginal lacerations.
c. Sciatic nerve injury.
d. Infection.
e. Myositis ossificans.
19.Properly performed closed-chest cardiac massage provides
approximately what percentage of prearrest cardiac output?
A. 15%.
B. 25%.
C. 45%.
D. 60%.
E. 85%.

20.A 9 month old boy presents with an acute scrotal swelling. The
following diagnoses are likely:

f. Epididymitis
g. Orchitis
h. Torsion of the testicular appendage
i. Irreducible inguinal hernia
j. Acute idiopathic scrotal oedema

21. Vital capacity:


A. Is the volume of air expired from full inspiration to full expiration
B. Increases gradually with age in adults
C. Is greater in women than in men of similar age and height
D. Is equal to the sum of the inspiratory and expiratory reserve volumes
E. May be measured by capnogram

22.What is the maximum dose of lidocaine


A .300 mg.
B .4.5 mg/kg.
C .5mg/Kg
D. 7 mg/kg.
E. 1.5 mg/kg

23.A 1-week-old infant presents with a history of poor feeding. The child
is afebrile with a heart rate of 160 beats per minute and respirations of
40 breaths per minute. The child is noted to have fine movements of the
limbs that respond to stimuli and stop with manual stimulation. There
are no accompanying movements of the eyes or tongue. Blood glucose is
found to be 21 mg/dL. After IV access is obtained, what is the MOST
appropriate dose of IV glucose to administer?
(A) 4 cc/kg of D10W.
(B) 4 cc/kg of D25W.
(C) 2 cc/kg of D25NS.
(D) 1 cc/kg of D50.
(E) 2 cc/kg of D50

24.Which of the following statements regarding the pediatric airway is


TRUE?
A. The MacIntosh blade is preferred in children younger than 2-4 years of
age.
B A normal 4-year-old child would require a 4-mm endotracheal tube.
C. The blind finger sweep is the preferred maneuver to clear a suspected
airway
obstruction in an unconscious child when advanced airway equipment is
unavailable.
D. A child is more likely than an adult to develop airway obstruction
E. Use of cuffed tubes is preferred in children younger than 8 years to
prevent aspiration

25.Pulse oximetry is used frequently in the ED as a noninvasive


measurement of arterial oxygen saturation. In some clinical situations,
the pulse oximetry measurement will be falsely elevated and should not
be relied upon. In which of the following clinical scenarios would the
pulse oximeter reading be expected to be falsely elevated?
A. Cyanide poisoning.
B. Chronic obstructive pulmonary disease.
C. Carbon monoxide poisoning.
D. Neonate who is producing fetal hemoglobin.
E. All of the above

26.Concerning intravenous fluids in the critically ill:


(A) Approximately 20% of infused normal saline (0.9% NaCl) remains
intravascular
(B)Hartmann’s solution (Ringer’s lactate) contains approximately 20 mmol/l
potassium
(C)Normal saline has a pH of 7.4
(D)Hartmann’s solution is hypotonic
(E) Approximately

28.In cardiac arrest all of the following are true EXCEPT:


(A) Cerebral hypoxic injury begins within 3 minutes
(B) Drug delivery is optimally achieved via a central vein
(C)Epinephrine 1 mg should be administered every minute during
cardiopulmonary resuscitation
(D) Open chest cardiac massage is indicated after recent cardiothoracic
surgery
(E)Associated with trauma, the cervical spine should be protected during
airway manipulation

29.Most common early complication of central venous line is:

a. Sepsis

b. Pneumothorax

c. Thoracic duct injury

d. Thrombosis

e. subcutaneous hematoma

30. insulin induced hypoglycemia may present with any one of the
following except:

a- hemiplegia

b- night sweat

c- foot drop

d- headache

e- aggressive behavior

31. Hemorrhagic stroke can be caused by all of the following except

a. Choriocarcinoma brain secondaries.

b. Amphetamine abuse.
c. Cranial Meningioma.

d. Following secondary prophylaxis of cardio embolic stroke.

e. severe hypertension

32. Daily fluid maintenance for 30 kg child is approximately


(A)1000 ml
(B)1300ml
(C)1700ml
(D)2000ml
(E)2300 ml

33.A 57-year-old male is admitted to hospital with pneumonia.Which of the


following would be associated with a poor outcome?

a. Age 57 years
b. Blood pressure of 110/70
c. Respiratory rate of 20
d. Confusion
e. Pulse rate of 90

34-Bleeding tendency following massive blood transfusion is primarily due


to:

a. Platelet depletion.

b. Decrease in fibrinogen.

c. Decrease in prothrombin.

d. Increased fibrinolytic activity.

e. Decrease of calcium.
35-The factor most commonly associated with central venous catheter
related nosocomial infection is:

a. Sterility of the insertion technique.

b. Type of solution being administered through the line.

c. Care of the catheter once inserted.

d. Proximity of the catheter to another wound.

e. Presence of another infection else where.

36-Insulin has all the following actions except:

a. Increased glycogen synthesis by the liver.

b. Decreased lipolysis.

c. Decreased proteolysis.

d. Increased gluconeogenesis.

e. Increased K uptake by the cells.

37. A 22 year old who is known to have bronchial asthma but who does not
comply with treatment is admitted with acute severe asthma.

Which of the following would be an indication for intubation and


ventilation?

a) Respiratory rate > 25 /min

b) Pulse rate >110 beats per minute

c) Peak expiratory flow rate < 50% of predicted or best

d) Hypoxia

e Hypercapnoea

38. Which of the following is NOT a complication ofacute pancreatitis?


a. Adult respiratory distress syndrome (ARDS)
b. Myocardial depression

c. Disseminated intravascular coagulopathy (DIC)

d. Malabsorption

e. Pancreatic pseudocyst

39. All of the following statements regarding freshfrozen plasma (FFP) are
TRUE EXCEPT:-
a. it contains all factors

b. intravenous administration can lead to volumeoverload

c. viral transmission is possible with transfusion

d. it is a first-line treatment for uremic patientswith bleeding

e. it is a first-line treatment for bleeding patients with a


coagulopathy

40-The recommended initial therapy for a patient suffering a severe


allergic reaction to a bee sting is:

a. Albuterol.

b. Cimetidine.

c. Dopamine.

d. Epinephrine.

e. Norepinephrine.

41-A 4-year-old girl developed a fever to ( 40°C ) and a diffuse, raised


purpuric rash this morning. Laboratory studies reveal: WBC 5,100/cu mm;
hemoglobin, 11.8 g/dL; platelets, 189 x 103/cu mm; prothrombin time,
11.8 sec; partial thromboplastin time, 28 sec; and fibrinogen, 250 mg/dL
Of the following, the MOST likely etiology of the purpura is:
a. disseminated intravascular coagulation

b. Henoch-Schِnlein purpura

c. hereditary telangiectasia

d. inherited platelet dysfunction

e. meningococcemia

42-A 1-week-old term female infant suddenly develops melena. The


mother is healthy, and the pregnancy was normal. Delivery was attended
by her family at home, but it was uncomplicated. The infant is
breastfeeding. Hemoglobin is 6.7 g/dL, white blood cell count is 13,000/cu
mm, and platelets are 599 x 103/cu mm . Of the following, the MOST likely
cause of the bleeding is:

a. factor VIII deficiency hemophilia

b. factor IX deficiency hemophilia

c. necrotizing enterocolitis

d. rectal fissure

e. vitamin K deficiency

43-Which one of the following is a reliable method of assessing


dehydration in infants: Useful indices of dehydration in infants include:

a. Assessment of intra-ocular tension by palpation

b. Skin turgor over the dorsum of the hand

c. Palpation of the anterior fontanelle

d. Blood pressure
e. Serum sodium concentration

44. Severe pre eclampsia can result in

A Blindness

B Low cardiac output

C Thrombocytopenia

D Liver failure

E All of the above

45- Which of the following statements is true concerning: Intussussception


in children:

a. Is more common between the age of 3 months to 8 years

b. Bile stain vomiting is an early feature

c. Bleeding per rectum is the most common presenting symptom

d. Is usually caused by a polyp or pedunculated tumour

e. Hydrostatic reduction plays an important role in the management

46-Minimal change nephrotic syndrome is characterized by the following


except:

a. Edema
b. Albuminuria

c. Hypercholesterolemia

d. Hypoproteinemia

e. Hypertension

47. Which of the following signs and symptoms are most likely to result
from symptomatic Bradycardia?

E. Headache, pain or pressure in the center of the chest,


palpitation.

F. Nausea, diaphoresis, pain radiating to the back and between


the shoulder blades.

G. Chest pain, shortness of breath, hypotension, dizziness or


altered level of consciousness, congestive heart failure,
premature ventricular contractions.

H. Difficulty with speech, unilateral limb weakness, severe


headache, facial droop.

E. Abdominal cramps.

48. Which of the following conditions can mimic the signs and symptoms of
acute stroke?

E. Hypoglycemia.

F. Cardiac arrest.

G. Pneumothorax.

H. Wolff-Parkinson-white syndrome.

E. Upper GI bleeding.

49- Opioids:

a. May cause chest wall relaxation


b. Sedation but not pruritus is reversed by naloxone

c. Morphine 3 sulphate is an unactive metabolite

d. Morphine 6 sulphate is not an active metabolite

e. Cause hypotension by vasodilatation

50. A 27-year-old woman presents with a three-day history of pleuritic chest


pain. She has been taking contraceptive pills. Physical examination of the
lower limbs is normal. The 12-lead ECG shows sinus tachycardia and the
V/Q scan shows multiple mismatched segmental defects in both lung fields.
Which one of the following statement is not accurate?

F. In patients with pulmonary embolism, only 10-20% have clinical


evidence of deep venous thrombosis.

G. The S1, Q3, T3 changes on the 12-lead ECG, when identified are
diagnostic.

H. Arterial blood gases are normal in 50% of patients.

I. Warfarin should be continued for six months.

E Thrombolysis should be considered if evidence of haemodynamic


instability is present.

51-The most causes of sudden cardiac death (first 2-3 hrs) after acute
myocardial infarction?

A. Ventricular fibrillation

B. Free wall rupture

C. Low cardiac output

D. Pulmonary edema

E. Atrial fibrillation

52.A 24-year-old man presents to the ER with a sudden sharp pain in the left
side of the chest and shortness of breath. Examination reveals: respiratory
rate 40/minute, cyanosis, tracheal deviation to the right and a hyper-resonant
left lung. Appropriate treatment includes all the following except
F. Intubation and positive-pressure ventilation.

G. Oxygen by mask.

H. Immediate needling of the left side of his chest.

I.Immediate chest X-ray.

J.Estimation of acid-base status.

53-In diabetic keto- acidosis which of the following is responsible for


hyperkaelemia?

a. Lack of sodium
b. Lack of insulin
c. Lack of electrolytes
d. Lack of glucose

E.increase of glucose

54. In shaft of humerus fracture, Nerve injured commonly

a. Median

b. Ulnar

c. Musculo cutaneous

d .Radial

e.posterior interosseous
55. Which of the following is TRUE regarding small

bowel obstruction?

(A) It is rarely associated with abdominal pain

(B) It usually presents with clear, nonbilious vomiting

(C) Diffuse, severe abdominal tenderness indicates secondary


peritonitis

(D) X-ray shows air-filled bowel with prominent haustrae

(E) Nasogastric tube decompression is generally ineffective.

56. Most common source of pulmonary embolus:

A.Iliofemoral veins
B. Deep veins of leg
C. Deep veins of calf
D.Popliteal vein.
E. Superfecial femoral vein.

57. Earliest sign of deep vein thrombosis is:

A.Calf tenderness
B. Rise in temperature
C. Swelling of calf muscle
D. Homan’s sign.
E.Leg oedaema.

58.Shortly after delivery, a newborn was noted to have good muscle tone
and movement of all extremities, a heart rate of 90/min, sneezing,
coughing, a loud cry, and pink color over the entire body. Her Apgar
score is:
A) 6.
B) 7.
C) 8.
D) 9.
E) 10.
59. A 4-year-old boy is in severe respiratory distress after a motor
vehicle crash. He was ejected from the vehicle and has major facial and
head trauma. When managing his airway, you should:
A) Use an appropriately sized laryngeal mask airway to limit the likelihood
of aspiration.
B) Perform a needle cricothyrotomy rather than a formal surgical
cricothyrotomy if you cannot intubate or ventilate.
C) Extend the neck to improve visualization of the airway structures.
D) Attempt blind nasotracheal intubation with an appropriate size
endotracheal tube.
E) Avoid succinylcholine because of the acute trauma.

60.Which of the following is not associated with increased likelihood of


infection after major elective surgery?
A. Age over 70 years.
B. Chronic malnutrition.
C. Controlled diabetes mellitus.
D. Long-term steroid use.
E . Anaemia .

61. Which of the following step is not a part of the primary survey in a
trauma patient?

A. Insuring adequate ventilatory support


B. Measurement of blood pressure and pulse
C. Neurologic evaluation with the Glasgow Coma Scale
D. Examination of the cervical spine.

E. Insuring adequate cervical spine support

62.The most common cause of fatal transfusion reactions is:

F. An allergic reaction.
G. An anaphylactoid reaction.

H. A clerical error.

I. An acute bacterial infection transmitted in blood.

E.Hypoclaemia

63. Major burn is defined as one of the following except.

A.Second and third degree of greater than 15% in children


B.Second and third degree of greater than 20% in adults less than 50 years
C.Third degree burn less than10%
D.Inhalation injury
J.Significant electrical injury.

64. The benefits associated with the use of nitroglycerin in patients with
acute coronary syndromes result primarily from:
A) Pulmonary artery vasoconstriction.
B) Decreasing myocardial preload.
C) Increasing afterload.
D) Coronary vasoconstriction.
E) Inotropic support.

65. A 63-year-old man, who weighs 65 kg, is in his 2nd postoperative day after
an abdominoperineal resection for cancer of the rectum. An indwelling Foley
catheter was left in place after surgery. The nurses are concerned because,
even though his vital signs have been stable, his urinary output in the past 2
hours has been zero. In the preceding 3 hours, they had collected 56 mL, 73
mL, and 61 mL. Which of the following is the most likely diagnosis?
A. Acute renal failure
B. Damage to the bladder during the operation
C. Damage to the ureters during the operation
D. Dehydration
E. Plugged or kinked catheter
66. A 40-year-old electrical lineman is injured on a electric pole and is
brought to the emergency department unconscious. He has an area on his
left hand that is charred, evidence of thermal injury to the arm, and an exit
wound on his right knee. Which of the following interventions is least
appropriate in the emergency department?

A. escharotomy of the left arm

B. CT scan of the abdomen and pelvis

C. ET intubation

D. CT scan of the head and neck

E. measurement of serum creatinine phosphokinase levels

67.Regarding inhalation injury which is correct ?


E. Inhalation injury should be suspected in any patient with a history of
closed–space smoke exposure ,wheezing, horseness, singed nasal
hairs, or carbonaceous sputum .

F. Bronchoscopy is the most reliable modality in the diagnosis of


inhalation injury .

G. The separation of ciliated epithelial cells from their underlying


basement membrane is a hallmark of inhalation injury .

H. Maintainig an open airway and maximizing gas exchange are key


components of management.

E.All are true

68. Nasotracheal intubation:

A. Is preferred for the unconscious patient without cervical spine injury.

B. Is preferred for patients with suspected cervical spine injury.


C. Maximizes neck manipulation.

D. Is contraindicated in the patient who is breathing spontaneously.

E.All are true.

69. Which of the following statements about head injuries is false?

A. The majority of deaths from auto accidents are due to head injuries.

B. Head injury alone often produces shock.

C. A rapid and complete neurologic examination is part of the initial


evaluation of the trauma patient.

D. Optimizing arterial oxygenation is part of initial therapy.

70.What percentage of patients with thoracic trauma require


thoracotomy?

A. 10%–15%.

B. 20%–25%.

C. 30%–40%.

D. 45%–50%.

E. 70%

71.The radiographic findings indicating a torn thoracic aorta include:

A. Widened mediastinum.

B. Presence of an apical “pleural cap.”

C. First rib fractures.


D. Tracheal deviation to the right.

E. all of the above.

72.A 28-year-old male was injured in a motorcycle accident in which he


was not wearing a helmet. On admission to the emergency room he was
in severe respiratory distress and hypotensive (blood pressure 80/40 mm.
Hg), and appeared cyanotic. He was bleeding profusely from the nose and
had an obviously open femur fracture with exposed bone. Breath sounds
were decreased on the right side of the chest. The initial management
priority should be:

A. Control of hemorrhage with anterior and posterior nasal packing.

B. Tube thoracostomy in the right hemithorax.

C. Endotracheal intubation with in-line cervical traction.

D. Obtain intravenous access and begin emergency type O blood


transfusions.

E. Obtain cross-table cervical spine film and chest film.

73. You are managing a patient who is hypotensive, refractory to


intravenous fluids. You decide to use a pure alpha-adrenergic agent, so
choose:
A) Dopamine.
B) Dobutamine.
C) Amrinone.
D) Isoproterenol.
E) Phenylephrine.

74. Which of the following statements regarding face mask ventilation is


TRUE;
A) The sniffing position causes the tongue to be more anterior.
B) It is made more difficult when prosthetics (dentures) are left in place.
C)It should not be used for the duration of the anesthetic.
D) In the presence of normal lung compliance and an open airway, face mask
ventilation should require as much as 40 cm H2O positive pressure.
E)The sniffing position causes the esophagus to be more posterior.
75.Which of the following statement(s) is true concerning the diagnosis of
a peripheral vascular injury?

e. The presence of a Doppler signal over an artery in an extremity


essentially rules out an arterial injury

f. Doppler examination is a valuable tool in the diagnosis of venous


injuries

g. A gunshot wound in the proximity of a major vessel is an absolute


indication for arteriography

h. Both the sensitivity and specificity of arteriography of the injured


extremity approaches 100%

E. none of the above

76.A 22-year-old male is hospitalized with multiple extremity fractures


including a comminuted fracture of the femur and multiple rib fractures.
Which of the following statement(s) is/are true concerning his hospital
course?
a.Low-dose heparin should not be employed during his hospital stay
f. Acute respiratory failure associated with petechiae of the head, torso,
and sclerae would suggest a pulmonary embolism

g. Early fracture fixation would decrease the incidence of fat emboli

h. The placement of a Greenfield filter should be avoided due to the risk


of lower extremity edema

e. non of the above

77. Genitourinary injuries are common with both blunt and penetrating
trauma. Which of the following statement(s) is true concerning
genitourinary trauma injuries?
a.All patients with microscopic hematuria and blunt trauma should be
evaluated with an intravenous pyelogram
b.The indications for radiographic assessment of renal injury in the face of
blunt trauma is more liberal than penetrating trauma
c.CT scan is the current imaging technique of choice for suspected renal
trauma
i. Perinephric hematomas occurring after either penetrating or blunt
trauma should not be explored

j. Extraperitoneal bladder ruptures usually treated by open surgical


exploration .

78.What is the best interpretation of an arterial blood gas analysis of pH,


7.35; PaCO2, 60; PO2, 80; and HCO3, 32:
A) Acute respiratory acidosis
B)Chronic respiratory acidosis with metabolic compensation
C) Chronic respiratory acidosis without metabolic compensation
D) Chronic metabolic alkalosis with respiratory compensation
E) Acute metabolic alkalosis

79.Which of the following is the major stimulus for aldosterone


secretion?
a.Angiotensin II.
b.ACTH.
c.Hypokalaemia.
d.Hypernatraemia.
e Cortisol

80.Organisms most commonly isolated in UTIs,


a. Kleb

b. E. Coli

c. Proteus

d. Acinetobacter

e. All of the above


81. The first maneuver to correct respiratory alkalosis in a patient
on mechanical ventilation should be:

a. Paralysis.

b. Increase tidal volume.

c. Sedation.

d. Increase respiratory rate.

e. Decrease respiratory rate.

82. The most common finding of a blunt cardiac injury is:


a.Tamponade.
b.Minor ECG abnormality.
c.Coronary artery occlusion.
d.Aortic incompetence.
e.Mitral valve injury.

83.The first priority in the management of facio-maxillary injuries is


to:
a.Rule out fracture of the cervical spine.
b.Chest x-ray.
c.Ensure a patent airway
d.Stop the bleeding.
e. IV fluid resuscitation

84.Mechanical obstruction of the large bowel is most commonly


caused by:

a. Adhesions.

b. Carcinoma.

c. Diverticulitis.
d. Hernia.

e. Volvulus.

85.Total body water is approximately _________ percent; of total body


weight.
A) 10
B) 20
C) 40
D) 60
E) 80

86. Plasma volume is approximately _________ percent; of the


extracellular volume (ECV).
A) 10
B) 20
C) 30
D) 40
E) 50

87.Which of the following statements about open fractures is not correct?

A. Intravenous antibiotics should be administered as soon as possible.

B. They should be regarded as an emergency.

C. Wound closure is necessary within 8 hours.

D. Systematic wound débridement and irrigation should be performed.

E. They most often result from high-energy injuries.

88. A patient describes a fall on the outstretched hand during sports


activities. Multiple radiographic views show no distinct fracture. He is
tender to palpation in the anatomic snuffbox. The most suitable method
of management is:

A. Diagnose “sprained wrist” and apply an elastic bandage.

B. Diagnose suspected scaphoid fracture and apply a short-arm cast to


include the thumb.
C. Apply a canvas wrist splint for immobilization.

D. Prescribe salicylates and permit continued activity.

e.non of the above

89. . ECG changes in Hyperkalemia include all of the following


EXCEPT:
A)Peaked T waves ;
B)Prolongation of P-R interval .
C)Disappeared P waves ;
D)Narrowing QRS complex .
E)ST segment depression .

90. An early sign of compartment syndrome in the hand includes:

A. Pain with passive stretch of the digits.

B. Absent radial pulse.

C. Motor paralysis.

D. Swelling of the digits.

E. Stiffness of the digits

91. Which of the following statements about the diagnosis of acute


calculous cholecystitis are true?

A. Pain is so frequent that its absence almost precludes the diagnosis.

B. Jaundice is present in a majority of patients.

C. Ultrasonography is the definitive diagnostic test.

D. Cholescintigraphy is not definitive diagnostic test.

e. none of the above

92. In a hemolytic reaction caused by an incompatible blood transfusion,


the treatment that is most likely to be helpful is:
a. Promoting a diuresis with 250 ml of 50% mannitol
b. Treating anuria with fluid and potassium replacement
c. Acidifying the urine to prevent hemoglobin precipitation in the
renal tubules
d. Removing foreign bodies, such as Foley catheters, which may cause
hemorrhagic complications
e. Stopping the transfusion immediately

93. The diagnosis of acute appendicitis is most difficult to establish in:

A. Persons aged 60 and older.

B. Women aged 18 to 35.

C. Infants younger than 1 year.

D. Pregnant women.

e.all of the above

94.Which of the following statements about acute salpingitis is true?

A. The disease rarely occurs after menopause.

B. Gonococcal infection is least common.

C. There is minimal cervical tenderness to palpation.

D. Vaginal discharge occurs rarely.

e. all are true

95. The most helpful diagnostic radiographic procedure in small bowel


obstruction is:

A. CT of the abdomen.

B. Contrast study of the intestine.


C. Supine and erect x-rays of the abdomen.

D. Ultrasonography of the abdomen.

E.Barium enema

96. The most commonly used imaging method for diagnosis of acute
cholecystitis is:

A. CT of the abdomen.

B. Ultrasonography of the gallbladder.

C. Oral cholecystogram.

D. Radionuclide (HIDA) scan of the gallbladder.

e.MRCP

97. Nonsurgical causes of acute abdominal pain may include all of the
following except?

a. Hyperthyrodism

b. Adrenal insufficiency

c. Pneumonia

d. Diabetic ketoacidosis

e. Uremia

98. . Addisonian crisis, or acute adrenocortical insufficiency:

A. Occurs only in patients with known adrenal insufficiency or in those


receiving long-term supraphysiologic doses of exogenous steroids.
B. Can mimic an acute abdomen with fever, nausea and vomiting,
abdominal pain, and hypotension.

C. May cause electrolyte abnormalities, including hypernatremia,


hypokalemia, hypoglycemia, and hypercalcemia, as well as eosinophilia on
peripheral blood smear.

D. Should be diagnosed with the rapid ACTH stimulation test before steroid
replacement is instituted.

E. May be effectively treated with intravenous “stress-dose” glucocorticoid


and mineralocorticoid replacement.

99. All the following pharmacologic agents can be used in the treatment of
thyrotoxicosis to block the production of thyroid hormone except ?

a. Propylthiouracil

b. Propranolol

c. Methimazole

d. Carbimazole

e. Iodine

100.All the following conditions are associated with hypercalcemia except


?

a. Hyperthyroidism

b. Thiazide diuretics

c. Terminal ileal resection

d. Breast cancer

e. Sarcoidosis
1- In hepatocellular damage, LFTs usually shows
a- Elevated indirect bilirubin and alkaline phosphatase
b- Elevated indirect bilirubin and GGT
c- Elevated direct bilirubin and alkaline phosphatase
d- Elevated direct bilirubin and ALT
f- Elevated ALT and AST
2- 25 Years old lady presented to ER with 2 days history of right upper quadrent pain
and
fever. She has no Murphy's sign and WBC count is 7. The best management will be
a- PO Analgesia
b- IV analgesia
c- Admition and start IV antibiotics
e- Admition and start PO antibiotics
f- IV antibiotics and follow up in clinic
3- Signs and symptoms of acute cholecystitis usually include the following except:
a- Jaundice
b- RUQ pain
c- Fever
d- Elevated WBC count
e- Nausea and vomiting
4- The most important factor for gallbladder carcinoma is
a- Acute cholecystitis
b- Chronic cholecystitis
c- Porcaline gallbladder
e- Large stone
f- Diabetes
5- Which of the following can be diagnostic and therapeutic for common bile duct stones
a- US
b- CT scan
c- HIDA scan
d-.ERCP
e-MRCP
6- The most important risk factor for hepatocelluar carcinoma is
a. Alcohol
b. Hepatitis B
c. Schistosomiasis
d. Wilson'n disease
e. Hepatitis A
7- A 34 years old lady presented with RUQ pain. She has been on oral contraceptive pills
for
10 years. CT abdomen shows a 4 cm hypervascular lesion in the right lobe of the liver.
The
most likely diagnosis is
a. Hepatocellular carcinoma
b. Focal nodular hyperplasia
c. Cholangiocarcinoma
d.Adenoma .
e.. Metastatic breast carcinoma
8- The tumor marker for hepatocelluar carcinoma is
a. CEA
b. CA 19-9
c. CA 125
d. CA 15-3
e. Alph feto protein
9- The most important factor in pancreatic adenocarcinoma is
a) Alcohol
b) Smoking
c) Chronic pancreatitis
e) Diabetes
f) Gastrectomy
10- Pancreatic adenocarcinoma can present with, except
a.Hematemesis )
b.Jaundice
c) Abdominal pain
d) Abdominal mass
e) Weight loss
11- The best medical treatment for hydatid disease is
a- Mebendazole
b-Albendazole
c- Ketoconazole
d- Metronidazole
e- Steriods
12- The second most common organ that can be affected by hydatid disease is
a- Heart
b-Lung
c-Kidney
d- Pancreas
e- Brain
13- Causes of acute pancreatits include the following except
a- Viral infections
b- Gall stones
c- Trauma
d- Steroids
e- Hypocalcemia
14- The most sensitive blood test to diagnose acute pancreatits is
a- Amylase
b-Lipase
c- Bilirubin
d- GGT
e- Ca 19-9
15- In obstructive jaundice, LFTs usually shows
a- Elevated indirect bilirubin and alkaline phosphatase
b- Elevated indirect bilirubin and GGT
Elevated direct bilirubin and alkaline phosphatase - cd
- Elevated direct bilirubin and ALT
f- Elevated direct bilirubin and AST
16- Prolonged PT (INR) in obstructive jaundice is due to decrease absorption of
a- Vitamin A
b- Vitamin D
c- Vitamin E
d- Vitamin K
e- Calcium
17- Types of gallstones include any of the following, except
a- Cholesterol stones
b- Mixed stones
c-Phospholipid stones
d- Brown pigment stones
e- Black pigment stones
18- Risk factors for gallstones include all the following, except
a- Obesity
b- Contraceptive pills
c- Sickle cell anemia
d-High protein diet
e-Rapid weight loss
19- In acute cholecystitis, HIDA scan shows
a- Distended gallbladder
b- Contracted gallbladder
c-adder No filling of gallbl
d-Dilated common bile duct
e- Bile leak
20- Bile contains the following except
a- Bile salts
b- CCK
c- Bile pigments
d- Cholesterol
e- Phospholipids

7) Chronic occlusion of the popliteal artery may produce:


a. Brown discoloration of the skin over the ankle.
b. Dilated collateral vessels in calf and foot.
c. Pain in the calf that is relieved by dependency.
d. Ulceration over the medial malleolus.

8) The most common type of aneurysm is :


a. Degenerative.
b. Dissecting.
c. Post stenotic.
d. Traumatic.

9) In patient who has chronic occlusion of a common iliac artery:


a. Symptoms are usually claudication of the calf muscles only.
b. Angiography is not beneficial.
c. Impotence is a common presentation.
d. Balloon angioplasty is appropriate in some patients.

10) Atherosclerosis :
a. Is due to cholesterol deposition on the surface of the intima.
b. May cause erectile dysfunction.
c. Is the most common cause of upper limb ischemia.
d. Hormone replacement therapy in post menopausal female is a risk factor.
11) Risk factors for peripheral vascular disease include all except:
a. Smoking.
b. Diabetes.
c. Hypertension.
d. Contraceptive pills.

12) Burger's disease:


a. Has an equal sex distribution.
b. Is more seen in elderly.
c. Causes early rest pain.
d. Causes claudication of the thigh muscles.

13) Raynaud's phenomenon may be caused by :


a. Thoracic outlet syndrome.
b. Atherosclerosis.
c. Polyartritis nodosa.
d. Thrombophlebitis migrans.
14) Abdominal aortic aneurysm:
a. May cause embolization of the lower limbs.
b. Can almost always be treated with endovascular stent.
c. Should be operated upon when it is 4cm long.
d. Is more common in females .
15) Regarding haemangiomas of the skin:
a. Port wine stains show no tendency to regress.
b. Spider navei are venular in origin.
c. Cavernous haemangioma usually involutes with time.
d. They are true benign tumours.

16) Embolism may be caused by all except:


a. An aneurysm.
b. Atheromatous plaque.
c. Fractures.
d. Tumours.
e. Infective endocarditis .
f. Right side heart failure.

17) Reperfusion i n j u r y following embolectomy may include all


except:
a. Compartment syndrome.
b. Hyperkalemia.
c. Acute tubular necrosis.
d. Venous thrombosis.

18) All the following are present in acute ischemia except:


a. Pain. b. Coldness.
c. Colour changes. D.trophic changes
e. Muscle weakness.

19) Patient with claudication in calf muscles, the site of the pathology
is :
a. External iliac artery. b. Internal iliac artery. c. Profunda femoris.
d.Superficial femoral in adductor canal.

20) All the following may be present in acute ischemia except:


a. Pallor. b. Paraesthesia.
c. Paralysis. d. Loss of erectile function.
e. Coldness.
21) In diabetic foot infection:
a. Infection spreads in the submuscular plane.
b. Dry gangrene IS the commonest fate.
c. Early rest pain is a common presentation.
d. A plain x-ray of the foot is essential.

22) The commonest cause of true arterial aneurysm is :


a. Congenital. b. Mycotic.
c. Atherosclerosis. d. Arteno-venous shunt.

23) Lumbar sympathectomy is of value in the management of :


a. Intermittent claudication.
b. Diabetic neuropathy.
c. Back pain.
d. Ischemic skin trophic changes.

24) Pulsation is an item of :


a. Inspection.
b. Palpation.
c. Percussion.
d. a and b.
e. None of the above.

25) An ulcer located beneath the head of first metatarsal bone and has
thickened edges is most likely to be due to :
a. Neuropathy.
b. Ischemia.
c. Osteomyleitis.
d. All of the above.
e. None of the above

26) Normal Doppler wave of the femoral artery is usually:


a. Monophasic. b. Biphasic.
c. Triphasic. d. All of the above.
e. None of the above.

27) Which one of the following diseases will manifest by impotence:


a. Aorto-iliac disease.
b. Femoro-popliteal disease.
c. Tibio-peroneal disease.
d. Burger disease.
CHAPTER (10)
VENOUS AND LYMPHATIC DISEASES

1) Which of the following is NOT seen in phlegmasia cerulea dolens?


a. Pain. b. Edema.
c. Blanching. d. Cyanosis.

2) The first-choice diagnostic study for suspected deep venous


thrombosis of the lower extremity is:
a. Contrast sonography.
b. Impedance plethysmography.
c. Radioactive labeled fibrinogen uptake .
d. Real-time Doppler imaging.

3) The best initial therapy for deep venous thrombosis of the


common femora vein is:
a. Heparin.
b. Placement of a vena cava filter.
c. Streptokinase.
d. Venous thrombectomy.

4) Which of the following is associated with venous thrombosis:


a. Von will brand's disease. b. Vitamin C deficiency.
c. Anti-thrombin III deficiency. d. Thrombocytopenia.

5) In the prophylaxis of deep venous thrombosis:


a. Early ambulation following surgery is contraindicated.
b. Calcium-channel blockers is a useful pharmacological agent.
c. Lower limb elevation is a simple useful method.
d. Heart failure is not a major concern.

6) The following are causes of leg lymphoedema except:


a. Familial predisposition.
b. Wuchereria bancrofti infection.
c. Radiotherapy.
d. Angio-neurotic oedema.
7) The following are causes of arm lymphoedema except:
a. Radical mastectomy.
b. Axillary vein thrombosis.
C. Lymphangioma.
d. Axillary radiotherapy.

8) Virchow's triad includes all except:


a. Hypertension.
b. Endothelial damage.
c. Change in blood composition.
d. Stasis.

9) Varicose veins of the lower limbs:


a. Are commonly 2ry to DVT.
b. Can be treated with 10% ethanolamine injection.
c. If primary can cause leg ulcers.
d. Are better diagnosed with Doppler ultra sound.

10) Oral anti coagulants (Warfarin):


a. Impairs the formation of vitamin K dependent factors.
b. Inhibits synthesis of fibrinogen.
c. Dose is best monitored with APTT.
d. Requires 24 hours to become effective.

11) The followings increase the risk of post operative DVT except:
a. Anti-thrombin III deficiency
b. Increased age.
c. Malignancy.
d. Contraceptive pills.
e. Diabetes mellitus.

12) Regarding varicose veins all are true except:


a. Haemosidnn deposition causes pigmentation.
b. High venous pressure causes lipodrmato sclerosis.
c. There is a higher incidence of venous thrombosis in those who take
oral contraceptives.
d. Have a peak incidence in the third decade of life.

13) Regarding the prothrombin time all are true except:


a. Measures the activity of the extrinsic coagulation pathway.
b. Is prolonged In hemophilia A.
56 monitoring Warfarin dose.
c. Can be expressed as INR when
d. Is prolonged with vitamin K malabsorption.
14) Regarding venous thrombo embolism all are true except:
a. The majority of venous emboli arise in the leg.
b. It can be silent
c. It may cause sudden death.
d.There is increased level of protein C and S.

15) Regarding anti coagulation:


a. Warfarin affects the intrinsic pathway.
b. Heparin potentiates the action of anti thrombin III.
c. Plasmin converts fibrinogen to fibrin.
d. There is increased fibrinogen level in DIC.

16) Hodgkin's lymphoma:


a. Has Reed-Sternberg cells.
b. Has a bad prognosis than non-Hodgkin.
c. Lymphocyte depletion has a better prognosis.
d. Burkett's lymphoma is a subtype.

17) The common laboratory test used to monitor a patient receiving


heparin is :
a. Bleeding time.
b. Activated partial thromboplastin time.
c. Coagulation time.
d. INR.
e. Fibrinogen level.

18) Venous ulcers are associated with:


a. Superficial venous valve incompetence.
b. Deep venous insufficiency.
c. Micro thrombosis.
d. Flat foot.

19) A cystic hygroma:


a. A type of cavernous lymphangioma.
b. Is the remnants of the 4th branchial arch.
c. Opaque on transillumination.
d. Can undergo spontaneous resolution after puberty.
20) Which one of the following diseases will manifest by
mrornbophteb ltf migrans:
a. Polyarteritis nodosa.
b. Erythema ncdosurn.
c. Endarteritis obliterans. 57
d. Thrombangilitis obliterans.
21) The most important investigation to be ordered in cases of
varicose veins is :
a. Doppler study. b. Duplex study.
c. Venography. d. Plethysmography.

22) An 80 year old woman was hospitalized after a fall from steps
that resulted in fracture of the neck of the femur on the right side.
After surgery to replace the broken hip she was bed-ridden and
was unable to ambulate until about a month later, then she
died suddenly. Which of the following conditions is most likely to
be the immediate case of her death?
a. Carcinoma of the uterus.
b. T8.
c. Pneumococcal pneumonia.
d. Pulmonary embolism.
e. Congestive heart failure.

23) Regarding the treatment of deep venous thrombosis, which of


the following statement is wrong:
a. 1M heparin in a dose of 5000 IU every 4-6 hours.
b. Subcutaneous low molecular weight heparin in a dose of
1mg/kg/12 hours.
c. Warfarin 10mg orally followed by 5mg once daily.
d. Oral anticoagulants are continued for 6 months.

24) On aspirating a cold abscess, one has to follow the following


rules except:
a. Aspirate from a dependant site.
b. Through a health skin.
c. In a valvular manner.
d. Under aseptic precautions.
e. All of the above.

25) Regarding low molecular weight heparin, all of the following


are true except:
a. Is an alternate to IV heparin.
b. Is given subcutaneously.
c. It requires to be monitored by blood tests.
d. The dose is 1 IU/kg/12hrs.

26) Regarding thrombolytic therapy for DVT, all of the following


are true except: 58
a. Steroids are better given prior to treatment.
b. It is contraindicated In patients with peptic ulcers.
c. Their use does not decrease the chance of developing a post-phlebitic
limb
d. TPA (Tissue plasminogoen activator) is one of its members.

59
CHAPTER (I I)
ABDOMINAL WALL & HERNIA
1) Spigelia,.n hernia occur:
a. On the lateral border of the rectus abdominis.
b. In the linea alba.
c. In the medial wall of the inguinal canal.
d. In the femoral triangle.

2) A sliding hernia:
a. Is easily reducible.
b. Is apparent only with straining.
c. Occurs in more than one orifice (e.g., inguinal and femoral).
d. Has a viscus contained in the wall of the sac.

3) The cremasteric muscle is derived from:


a. External oblique aponeurosis.
b. Internal oblique muscle.
c. Transversus abdominis muscle.
d. Transversalis fascia.

4) The spermatic cord contains all of the following except:


a. Autonomic nerve fibers.
b. Genito-Femoral nerve.
c. Lymphatics.
d. Testicular artery.

5) A sliding hernia:
a. Has a high recurrence rate after repair.
b. Involves a retroperitoneal structure.
c. Is more common in the left groin.
d. Occurs almost exclusively in women.

6) Inguinal hernias:
a. More common in females.
b. A known cause of testicular atrophy.
c. Repaired only if complicated.
d In children treated only by herniotomy.

7) The differential diagnosis of a reducible groin lump may include


all except:
a. Saphena varix.
b. Psoas abscess.

c. Femoral hernia.
d. Hodgkin lymphoma

8) Regarding groin hernias:


a. Indirect inguinal hernias emerge medial to the inferior epigastric artery.
b. Sliding hernias have small bowel loop forming part of the wall of the sac.
c. Femoral hernias pass through the saphenous opening of the femoral sheat
d. Direct inguinal hernias usually emerge through the deep inguinal ring

9) Epigastric hernias:
a Usually contain small intestine.
b. Are frequently irreducible.
c. Best treated with Shouldice repair.
d. Are often more easily seen than felt.

10) Borders of Hesselbach's triangle include:


a. Medial border of rectus sheath.
b. Lacunar ligament.
c. Femoral artery.
d. The inferior epigastric artery.

11) Femoral hernia:


a. Has a high risk of strangulation.
b. Is equally common In males and females.
c. Has a neck lateral to the femoral vein.
d. May descend to the scrotum.

12) A direct inguinal hernia:


a Is a hernia through the fascia transversalis.
b. Can contain bladder as part of the wall of the sac.
c. Has a neck lateral to the inferior epigastric artery.
d. Usually descends to the scrotum.
13) A para-umblical hernia:
a. Is a form of exomphalos.
b. Does not strangulate.
c. Usually contains omentocele.
d. Is a Spigelian hernia.

14) Clinical features of a strangulated inguinal hernia includes Jill except:


a. Abdominal distention.
b. A tense and tender swelling.
c. A groin swelling with an expansile impulse on cough.
d. Abdominal pain.

15) Regarding the inguinal canal, spot the wrong statement:


a. In infant, the deep and superficial rings are superimposed.
b. In adults it measures 1.5 inches long.
c. It transmits the femoral branch of the genito-femoral nerve.
d. Inferior epigastric vessel lies medial to the deep ring.

16) Regaraing the management of drains, all are true except:


a. Drains should be brought out through a separate incision.
b. Corrugated rubber drains can be withdrawn over a period of few days
(humping) if there is little drainage.
c. Drains do not predispose to wound infection.
d. When drains are no longer needed, withdraw at once.

17) Regarding umbilical nodules, which of the following statements is wrong:


a. May indicate hepatic metastasis.
b. May be present in cases of urinary bladder carcinoma.
c. Malignant cells reach the umbilicus via the round ligament of the uterus.
d. May be mistaken for a para umbilical hernia.

18) Swellings that may be present in the midline of the abdomen include all
of the following except:
a. Divarication of the recti.
b. Fatty hernia of the linea alba.
c. Epigastric hernia.
d. Spigelian hernia.
e. Para umbilical hernia.
19) Which of the following is true for the content of Littre's
hernia:
a. Is ovary.
b. Is a diverticulum of the bladder.
- c. Is Meckle's diverticulum.
d. May be portion of the bladder.

20) Which of the following statements is untrue in obstructed


hernias:
a. Synonymous with incarcerated hernia.
b. This is an irreducible hernia containing intestine that IS
obstructed.
c. There is Interference with the blood supply.
d. The symptoms are less severe than in strangulation.
21) Differential diagnosis of an indirect inguinal hernia in the male
includes:
a. Lipoma of the cord.
b. Hydrocele of a hernia sac.
c. Encysted hydrocele of the cord.
d. All of the above.

22) Which of the following statements regarding Laugier's


femoral hernia untrue?
a. Is a hernia through a gap in Gimbernat's ligament.
b. The diagnosis is based on the unusual medial position of a small
femoral hernia
c. Nearly always the hernia is strangulated.
d. All of the above.
e. None of the above.

23) Which of the following regarding Cloquet's hernia is true?


a. The sac lies under the fascia covering the pectineus muscle.
b. Strangulation is likely.
c. The sac may co-exist with the usual type of femoral hernia.
d. All of the above.
e. None of the above.
24) All are true of obturator hernia except:
a. Six times more common in women than in men.
b. Leg remains semi flexed and its movement elicits pain.
c. Strangulation causes pain referred to the knee.
d. None of the above.
CHAPTER (12)
THYROID & PARATHYROID

1) Steps in the synthesis of thyroid hormone include all of the


following except:
a. Coupling of ioootyrosmes.
b.Ingestion of potassium iodide.
c. LInkage of iodine with tyrosine residues.
d. Oxidation of iodide to iodine.

2) Hypercalcemia can be caused by :


a. Osteoporosis.
b. Secondary hyper parathyroidism.
c. Pancreatitis.
d. Malignancy.

3) Which of the following about follicular carcinoma of thyroid gland


is true:
a. It usually present at an earlier age than papillary carcinoma.
b. It disseminates via haematogenous routes.
c. It is frequently multicentric.
d. It is the most common adenocarcinoma of the thyroid gland.

4) Which of the following is not a component of multiple


endocrine neoplash (MEN) type 2 :
a. Medullary thyroid cancer.
b. Pheochromocytoma.
c. Hyperparathyroidism.
d. Pituitary adenoma.

5) Hyperthyroidism can be caused by the following conditions except:


a. Grave's disease.
b. Plummer's disease.
c. Iatrogenic thyroxin administration.
d. Hashimoto's disease.
e. Medullary carcinoma of thyroid gland.

6) Thyroid cancer:
a. It is more common in men.
b. It is commonly a follicular type.
c. It may form part of MEN type 1 syndrome.
d. Anaplastic carcinoma carries the worst prognosis.
7) Thyroid cancer:
a. It is usually treated by hemi thyroidectomy.
b. Radio active iodine can be used in treatment of follicular
carcinomas.
c. Serum calcitonin is a useful marker for papillary carcinoma.
d. Psamoma bodies is a histoloqical finding of medullary carcinoma.

8) invasive-follicular carcinoma of the thyroid:


a. Commonly spreads to lymph nodes.
b. May be accurately diagnosed by fine needle aspiration cytology.
c.Should be treated by total thyroidectomy.
d. Has a better prognosis than papillary cancer.

9) Papillary carcinoma of the thyroid:


a. Has an equal sex incidence.
b. Arises from stromal cells.
c. Can be treated with radioactive iodine.
d. Commonly spreads to lymph nodes.

10)Anaplastic thyroid cancer:


a. Has a 90% one year survival.
b. Is usually cured by aggressive surgery.
c. Usually invades the surrounding structures.
d. Can be treated with radioactive iodine.

11)True exophthalmus in Grave's disease:


a. Is due to retro bulbar deposition of fluid and round cells.
b. May be improved by B-adrenergic eye drops.
c. Of malignant type can causes secondary deposits in the liver.
d. Is associated with low levels of IgG.

12)The causes of hypercalcemia include the following


except:
a. Multiple myeloma.
b. Bone metastasis.
c. Hyperthyroidism.
d. Acute pancreatitis.

13) In a retrosternal goiter all of the following may be


present except:
a. Dyspnea especially at night.
b. Congested puffy face.
c. Dilated veins over the chest wall.
d. Tracheal deviation.
e. Pain In the neck.
14) Regarding Grave's disease:
a. It is the commonest cause of thyrotoxicosis.
b. TSH level is elevated.
c. The thyroid gland is of normal size.
d. Patients have cold moist extremities.

15) Regarding treatment of thyrotoxicosis:


a. Surgery is contraindicated in grave's disease.
b. Radio-active iodine is indicated in pregnancy.
c. Exophthalmos is cured following sub-total thyroidectomy.
d. Propranolol controls the peripheral adrenergic features.

16) Regarding primary hyperparathyroid ism :


a. Blood calcium level is decreased.
b. Tetany is a common presentation.
c. Renal calculi are common.
d. Can be treated by vitamin 0 supplementation.

17) Elevated parathyroid hormone in blood produces all except:


a. Increased gastric acid secretion.
b. Increased calcium excretion in urine.
c. Increased incidence of renal stones.
d.Tetany.

18) Which of the following forms of thyroiditis does the


pathology extend outsid the gland:
a. De Quervain's.
b. Hashimoto.
c. Riedel's.
-
d. Acute bacterial.

19) Which of the following is not associated with thyrotoxicosis:


a. De Ouervain's.
b. Hashimoto.
c. Riedel's.
20) Regarding carbimazole in the treatment of Grave's disease, all
are true excep
a. It blocks iodine binding to tyrosine.
b. Initial dose is 30 mg per day.
c. Treatment is stopped when patient becomes euthyroid.
d. The drug starts acting after 1-2 weeks.

21) Regarding radioactive iodine in the treatment of


thyrotoxicosis, all of the following are true except:
a. It is indicated in thyro-cardiac patients.
b. It causes hypothyroidism in up to 80% of the patients treated.
c. The recommended dose is 169uCu per gram of thyroid tissue.
d. It increases the incidence of thyroid carcinoma in adults.
22) Regarding hypo parathyroid ism, all of the following are true
except:
a. Phosphorous should be increased in the diet.
b. The aim of the treatment is to control titanic convulsions.
c. Dihydro-tachysterol may be needed to control the hypocalcaemia.
d. Aluminium hydroxide is occasionally used.

23) The earliest complains of a patient with


hyperparathyroidism is all of the following except:
a. Weakness.
b. Polydepsia.
c. Tremors.
d. Constipation.

24) The treatment of primary hyperparathyroidism is mainly:


a. Medical.
b. Surgical.
c. Treatment of the cause.
d. Correction of the renal failure.

25) All of the following can be a treatment of primary toxic goiter


except:
a. Radioactive iodine.
b. Hemithyroidectomy.
c. Anti thyroid drugs.
d. Beta blockers.

26) Which of the following thyroid neoplasm has elevated serum


calcitonin level?
a. Papillary carcinoma. b. Follicular carcinoma.
c. Anaplastic carcinoma. d. Medullary carcinoma.
e. Lymphoma.
CHAPTER (13)
HEAD & NECK SURGERY

1) Swellings of the posterior triangle of the neck include:


a. cervical rib.
b. Thyroglossal cyst.
c. Sub hyoid bursa.
d. Laryngocele.

2) Salivary gland calculi:


a. Usually arise in the parotid gland .
.b. Are mostly visible on plain x-ray.
c. Are associated with hyperuracemia.
d. Are treated by medical conservative measures.

3) Midline swellings of the neck include:


a. Cystic hygroma.
b. Branchial cyst.
c. Pharyngeal pouch.

-
d. Thyroglossal cyst.

4) Pleomorhic adenoma of the parotid:


a. Is treated with radiotherapy in most cases.

-
b. Does not recur after surgical removal.
c. Is the most common tumour of the parotid.
d. Is a recognized cause of facial palsy.

5) Regarding salivary stones:


a. Stones are more commonly seen In the parotid gland.
b. Calculi are not radio-opaque.
c. There is a recurrent painful swelling associated with meals.
d. Stones are always seen within the gland substance only.

6) Pleomorhic adenoma:
a. Is mixed benign tumours of the parotid gland.
b. It affects mainly the deep lobe of the parotid.
c. Facial nerve palsy is a common complication.
d. Is treated by simple enucleation.
7) All of the following statements are correct regarding
branchial cyst except:
a. The cyst usually appears after the age of 20 years.
b. The branchial fistula is never congenital.
c. It IS susceptible to infection because its wall is surrounded by
lymphatic tissue. d. Branchial cyst appears in the carotid triangle.
e. Cold abscess IS one of the differential diagnoses.

8) All these structures are found in the parotid gland except:


a. Retro-mandibular vein.
b. Internal carotid artery .
c. Parotid lymph node.
d. The facial nerve.

9) All of the following masses move up and down with


deglutition except:
a. Thyroglossal cyst.
b. Subhyoid bursitis.
c. Laryngocele.
d. Carotid body tumor.

10) The timing of operation for cleft palate is best between:


a. 6 - 12 months.

-
b. 12 - 18 months.
c. 1 - 2 years.
d. 3 years of age.

11) Regarding the dangerous area of the face:


a. Boils in this region has to be squeezed rapidly to empty the pus.
b. It is a triangular area passing from the inner canthi to the angles of the
mouth.
c. Infection in this area can cause thrombosis of the saggital sinus.
d. Infection can reach the cavernous sinus only via the pterygoid venous
plexus.
CHAPTER (14)
OESOPHAGUS

1) Which of the following in the most effective treatment for


achalasia?
a. Repeated balloon dilatation.
b. Laparoscopic Heller's cardiomyotomy.
c. Injection of botulinum toxins.
d. Dietary changes and observation.

2) The most significant risk factor for the development of


adenocarcinoma of the esophagus is :
a. Alcohol abuse.
b. Barrett's esophagus.
c. Long-standing achalasia.
d. Smoking.

3) All of the following statements about achalasia are true except:


a. Ganglion cells in the body of the esophagus either are absent or have
degenerated.
b. Pressure in the body of the esophagus is lower than normal.
c. Affected persons usually experience more difficulty swallowing
cold fluids than warm foods.
d. Esophageal cancer is common in affected persons as in the general
population.

4) All of the following statements about para-esophageal hernia are


true except:
a. Heartburn is the usual chief complaint of affected persons.
b. Dysphagia and dyspnea are common following meals.
c. The herniated portion of the stomach may become gangrenous and
perforate.
d. Surgical repair generally is indicated.
5) The most affective treatment of achalasia is :
a. Antispasmodic medication.
b. Dilation of the lower esophageal sphincter.
c. Cardiomyotomy.
d. Resection of the gastro esophageal junction.
6) In achalasia of the cardia all of the following are present except:
a. High pressure zone at cardia.
b. Severe retrosternal discomfort.
c. Weak peristaltic waves in the body of esophagus.
d. Long standing intermittent dysphagia.
e. Recurrent chest infections.

7) Canceresophagus:
a. Is most commonly adenocarcinoma.
b. Is resectable at the time of diagnosis of majority of cases.
c. Don't spread through lymphatics.
d. Barrette's esophagus is a risk factor.

8) Regarding esophageal diseases:


a. Sliding hiatus hernias are less common than rolling hernias.
b. In rolling hernias, the gastro-esophageal junction is lifted up into the
chest.
c. In achalasia, there is degeneration of myenteric nerve plexus.
d. ln sliding hiatus hernia, reflux esophagitis is the main presenting feature.

9) Regarding clinical association of achalasia of the cardia, all are


true except:
a. Chest infection.
b. Food regurgitation.
- c. Vitamin B12deficiency.
d. Cancer esophagus.
e. Pulsion diverticulum.

10) In achalasia of the oesophagus, which statement is untrue?


a. Dysphagia is the presenting symptom.
b. Ganglion cells are often lacking from Auerbach's plexus.
c. The oesophagus show hyper motility.
d. The radiological signs are often characteristic.
e. Cardiomyotomy (Heller's operation) is the treatment of choice.

11) The most diagnostic test for oesophageal reflux is:


a. Barium swallow.
b.Oesophagoscopy.
c. Oesophageal manometry.
d. pH reflux test.
e. Epithelial biopsy.
12) Regarding post cricoid carcinoma, all of the following are true
except:
a. More common in females.
b. It usually involves the posterior wall of the hypopharynx at the level
of the cricoids cartilage.
c. Dysphagia is late.
d. Pain in the ear can be an early symptom due to involvement of the
auriculai branch of the vagus.

13) Carcinoma of the esophagus is most common in the:


a. Upper third.
-
b. Middle third.
c. Lower third.
d. Intra-abdominal part.

14) The first aid treatment of corrosive esophageal injury


includes all of the following except:
a. Ask the patient to swallow some water or milk.
b. Gastric lavage to remove the corrosive element.
c. Analgesia.
d. Look for signs of laryngeal edema or stridor.
CHAPTER(15)
STOMACH

1) The left gastroepiploic artery arises from:


a. The left gastric artery.
b. The splenic artery.
c. The right hepatic artery.
d. The gastroduodenal artery.

2) Which of the following is secreted by gastric chief celts?


a. Somatostatin. b. Gastrin.
c. Pepsinogen. d. Histamine.
e. Glucagon.

3) Which of the following is the best test to use to confirm


eradication (Helicobacter pylori) infection after treatment?
a. Serologic test.
b. Urea breath test.
c. Histology.
d. Upper GIT endoscopy.

4) Which blood group is associated with an increased risk of gastric


cancer?
a. A. b. B.
c. AB. d. O.

5) A 65-year-old woman with a known duodenal ulcer is being


treated with di, and H2 blocker therapy. She is admitted with a
major upper gastrointestin, hemorrhage. After blood replacement
is begun, the next step in h' management should be :
a. Beginning bismuth, Tetracycline, and Metronidazole.
b. Beginning Omeprazole.
c..Endoscopy and coagulation of the bleeding vessel.
d. Plyoro duodenotomy and over sewing of the bleeding vessel.
6) Patients at increased risk for gastric carcinoma include of
the followir all except:
a. Those that have undergone gastric resection for peptic ulcer.
b. Those with pernicious anemia.
c. Those have undergone gastic bypass for morbid obesity.
d. Those with blood group A.

7) Regarding benign gastric ulcers, the incorrect is that they:


a. Occur most often on the lesser curve of the stomach.
b. Are always due to hyperacidity.
c. Produce gastric pain soon after meals.
d. Require endoscopy and biopsy.
e. May relapse after medical treatment.

8) The cardinal symptoms of chronic duodenal ulcer do not include:


a. Anorexia.
b. Dyspepsia.
c. Hunger pain.
e. Heart burn
d. Periodicity.
9)An increased incidence of duodenal ulcer is associated with all
except:
a. Hyper parathyroidism.
b. Zollinger Ellison syndrome.
c. H. pylori infection.
d. Hyperthyroidism.
e. Chronic use of NSAID.
10) Perforated duodenal ulcer is best treated with:
a. Nasogastric suction tube and antibiotics.
- b. Closure over omental patch.
c. Trunkal vagotomy + qastrojulonostomy.
d. Highly selective vagotomy. e. I.V Omeprazole.

11) Chronic pyloric stenosis, produces all except


a. Metabolic alkalosis.
b. Tetany.
c.Hypercalcemia.
d. Hypokalemia.
e. Hypocholeremia.
12) Regarding peptic ulceration-all true except:
a. Helicobacter pylori has an important etiological role.
b. Is more common in smokers.
c. Can be associated with Zollinger Ellison syndrome.
d. Can present with melena
-e. Is more commonly treated by surgical intervention.

13) Perforated duodenal ulcer:


a. Usually situated posteriorly.
b. Can be treated with antibiotics.
c. Is usually found In the second part of the duodenum.
d. Many be treated by an omental patch.
e. Best diagnosed by ultra sound.

14) The following have an association with chronic duodenal ulcer


except:
a. H. pylori infection.
b. Blood group A.
c. Excess acid-pepsin secretion. d. Smoking.
e. Zollinger-Ellison syndrome. f. Hyperparathyroidism.
15) Gastric cancer may present with:
a. Curling's ulcer.
b. A positive Courvoisier's sign. c. Virchow's node.
-
d. Dysphagia to solids.
e. Dyspepsia 2 hours after meals.

16) In chronic duodenal ulcer:


a. Pain occurs soon after meal.
b. There IS considerable vomiting as an early feature.
c. There is anorexia nervosa.

-
d. Well-marked periodicity.

17) Recognized complications of chronic duodenal ulcer include all


except:
a. Pyloric stenosis.
b. Malignant transformation.
c. Penetration. d. Bleeding.
e. Perforation.

18) Regarding pyloric stenosis the untrue is :


a. Can be due to pylonc cancer.
-
b. Is due to cicatrisation of a duodenal ulcer.
c. Causes hypernatraemia.
d. Occurs in infants.
19) Risk factors of cancer stomach include:
a. Hyperacidity.
b. Plummer Vinson syndrome.
c. Blood group O.
d. Atrophic gastritis.
e. Zollinger Ellison syndrome.

20) All of the following factors are involved in the etiology of


peptic ulceration except:
a. Infection.
b. Hypergastrinemia.
c. Eating spicy food.
d. The use of non-steroidal anti-inflammatory drugs.
21) Which of the following is not a type of gastric ulcer?
a. Ulcer in the body at the lesser curvature.
b. Combined gastric and duodenal ulcer.
c. Type III gastric ulcer.
d. Type IV gastric ulcer.
CHAPTER (16)
ApPENDIX SMALL INTESTINE &
ACUTE ABDOMEN

1) The most common small bowel cancer is :


a. GIST (gastrointestinal stromal tumor).
b. Carcinoid tumours.
C. Adenocarcinoma.
d. Lymphoma.

2) Which of the following is NOT a common etiology of ileus?


a. Pneumonia.
b. Hypernatraemia.
c. Calcium channel blockers.
d. Hypothyroidism.

3) Which of the following denotes the presence of intestinal


strangulation:
a. Vomiting.
b. Constipation.
C. Abdominal rigidity.
d. Distention.
e. Exaggerated bowl sounds.

4) The most appropriate solution to infuse in a patient with


intestinal obstruction is:
a. Dextrose 5%.
b. Lactated ringer's solution. c. Dextrose 25%.
d. Na bicarbonate.
e. Plasma.

5) Regarding Meckle's diverticulum all are true except:


a. It represents the remains of the urachus.
b. Inflammation causes pain in the right iliac fossa
c. It may be attached to the umbilicus.
d. It may contain gastric mucosa.
e. It is one of the common causes of bleeding per rectum in children.
6) Resection of the terminal ileum is associated with malabsorption of :
a. Calcium.
b. Vitamin B12.
d. Folic acid.
c. Cholesterol. e. Iron.

7) Meckle's diverticulum:
a. May be seen on HIDA scan.
b. Occurs In 20% of people.
c. Usually occurs in the proximal ileum.
d. May contain ectopic adrenal tissues.
e. May presents with bleeding per-rectum.

8) Bile stained vomiting is a feature of :


a. Pyloric stenosis.
b. Intussusception.
c. Esophageal atresia.
d. Gastro-esophageal reflux. e. Achalasia of the cardia.

9) Meckle's diverticulum:
a. Is a remnant of the umbilical vein.
b. Containing ectopic gastric mucosa.
c. Lies on the mesenteric border of the small bowel.
d. Most commonly presenting with perforation.

10) Gall stone ileus:


a. Usually due to fistula following an attack of acute cholecystitis.
b. Calculi usually impact in the proximal jejunum.
c. May produce obstructive jaundice.
d. Usually seen on plain x-ray.
11) The commonest organ to be injured following fracture of the right 10th
rib mid clavicular line is :
a.Base of the right lung b. Pleura.
c. Liver. d. Kidney
e. Right supra-renal gland.

12) Pelvic abscess:


a. Can follow acute appendicitis.
b. diagnosed by pain x-ray pelvis
c. Can be treated with antibiotics.
d. Can leads to chronic constipation.
13) In acute abdomen the incorrect statement is :
a. Colicky pain indicates obstruction of a hollow viscus.
b. Rebound tenderness is characteristic of bowel strangulation.
c. Rigidity is a finding in perforated peptic ulcer.
d. Vomiting is a common symptom.
e. The abdomen is silent on auscultation in all cases.

14) In acute septic peritonitis, the untrue is :


a. There is fever and tachycardia.
b. Severe colicky abdominal pain.
c. Shifting dullness is present.
d. Diffuse tenderness and rigidity.

15) In children and adolescents, the commonest cause of intestinal


obstruction is:
a. Adhesions.
b. Intussusception.
c. Strangulated hernias.
d. Neoplasm.
e. Paralytic ileus.

16) Intussusception:
a. Is the commonest cause of bowel obstruction under the age of 18
months.
b. Presents with melena.
c. Can be treated by hydrostatic reduction in all cases.
d. Commonly presents with projectile non bilious vomiting.

17) The differential diagnosis of acute appendicitis include all


except:
a. Perforated peptic ulcer.
b. Chronic pancreatitis.
c. Acute cholecystitis.
d. Rupture ovarian cyst.
e. Disturbed ectopic pregnancy.

18) Acute appendicitis is caused by :


a. H. Pylori.
b. Following laparotomy.
c. Klebsiella.
d. Hypokalemia.
b. Streptococcus pyogens. d. E. Coli.
19) Recognized causes of paralytic ileus include all except:
a. Fracture spine.
b. Degeneration of the Auer Bach's plexus.
c. Peritonitis.

20) The best treatment for an appendicular mass is:


a. Urgent appendicectomy.
b.. Conservative treatment + delayed appendicectomy after 3 months.
c. Conservative treatment + delayed appendicectomy after 1year.
d. Right hemicolectomy.
e. Caecostomy.

21) The differential diagnosis of acute appendicitis in a young female does


not include:
a. Ruptured ovarian cyst.
b. Disturbed ectopic pregnancy.
c. Acute cholecystitis.
d. Acute diverticulitis of the colon.
e. Acute mesenteric adenitis.

22) Concerning acute appendicitis, the following statements are true except:
a. Presents with pain in the right iliac fossa as the first symptom.
b. May cause psoas spasm.
c. There is rebound tenderness in the right iliac fossa.
d. Shifting tenderness may be present.
e. May produce dysuria and tensmus.

23) Following a blunt trauma to the left upper abdomen, the commonest
organ to be injured is:
a. Spleen.
b. Splenic flexure of the colon.
c. Gastric wall.
d. Left kidney.
e. Pleura.
24) Regarding acute appendicitis all are true except:
a. Caused by strept fecal is, E coli or anaerobic organisms.
b. Can be complicated with mass formation.
c. Can be complicated with peritonitis.
d. ls treated by conservative measures + interval appendicectomy.
e. Can lead to pelvic abscess formation.

25) Septic peritonitis:


a. Has a strong association with liver cirrhosis.
b. Can be treated conservatively with antibiotics.
-
c Is commonly due to a perforated viscus.
d. Commonly associated with pancytopenia.

26) Which of the following is most commonly confused with appendicitis


in children?
a. Meckle's diverticulitis.
b Mesenteric adenitis.
c. Pelvic inflammatory disease.
d. Acute gastroenteritis.

27) Which of the following are the least frequent in positions of the appendix:
a. Retrocecal. b. Pelvic.
c. Paracecal. d. Post-ileal.

28) The most common appendicular tumor is :


a. Carcinoid Tumor. b. Adenocarcinoma.
c. Squamous cell carcinoma. d. Mixed Cellularity.

29) In strangulated intestinal obstruction, all of the following are true except:
a. Bacteria and toxins can transgress the ischemic loop to the peritoneal cavity.
b. The mucosa is the first layer to be affected by the ischemia.
c. Proximal to the strangulated loop the intestines are not dilated.
d. Unrelieved strangulation will result in perforation.
30) All the following causes of intestinal obstruction can be managed non
surgically except:
a. Adhesive intestinal obstruction.
b. Ileo-cecal intussusception.
c. Fecal impaction.
d. Sigmoid volvulus.
e. Volvulus neonatorum .
CHAPTER (17)
COLORECTAL & ANAL CANAL

1) Which of the following is a first-line agent in the treatment of mild


to moderate colitis from inflammatory bowel disease?
a. Salicylate. b. Steroids.
c. 6-Mercaptopurine. d. Methotrexate.

2) The risk of developing adenocarcinoma after 10 years of ulcerative

- colitis is :
a. < 10%. b. 10 - 20%. c. 20 - 30%. d. > 50%.

3) Which of the following is associated with increased risk of colorectal


cancer?
a. Familial adenomatous polyposis coli.
b. Inactivation of the K-ras gene.
c. Activation of the DCC (deleted in colorectal cancer) gene.
d. Activation of the p53 gene.

4) Which of the following is most likely to harbor a colorectal cancer?


a. Hamartomatous polyp.
b. Tubular adenomatous polyp.
c. Tubulo villous adenomatous polyp.
- d. Villous adenomatous polyp.

5) The most common bacterial organism present in the colon is :


- a. Bacteroides.
b. Clostridium difficile.
c. Escherichia coli.
d. Salmonella.

-
6) Regarding cancer caecum, the incorrect statement is:
a. Usually presents with acute Intestinal obstruction.
b. Associated with microcytic anemia.
c. Can presents with a palpable mass in the right Iliac fossa.
d. Right hemicolectomy is the principle treatment for operable cases.
7) Carcinoma of the rectum:
a. Is a squamous cell carcinoma.
b. Commonly spread to the inguinal lymph nodes.
c. Causes bleeding per rectum.
d. Is associated with diverticular disease.
8) Treatment of hemorrhoids include all except:
a. Laxatives
b. Band legation.
c. Lateral sphinctrotomy operation.
d. Injection sclerotherapy. e. High fiber diet.

9) Recognized complications of hemorrhoids include:


a. Fecalmcontinence.
b Strangulation.
c. Dysuria.
d. Bloody diarrhea.

10) The least common site for cancer in the gastro-intestinal tract is :
a. Esophagus.
b. Stomach.
c. Small intestine.
d. Colon and rectum. e. Liver.

11) Which of the following is a precancerous state in the large bowel:


a. Crohn's disease.
b. Diverticular disease.
c. Bilharzial colitis.
d. Peutz Jegher's syndrome.
e. Gardner's syndrome.

12) Sigmoid volvulus is characterized by the following features except:


a. Acute onset of severe cramping abdominal pain.
b. Absence of abdominal distention.
c. Absolute constipation.
d. Characteristic sign in the barium enema.
e. Feculent vomiting.

13) Regarding diverticular disease of the colon:


a. Usually involves the rectum.
b. May cause severe bleeding per-rectum.
c. Always symptomatic.
d. Is essentially due to high fiber diet.
14) Bleeding per rectum is a recognized feature in all except :
a. Cancer colon.
b. Hirschsprung disease.
c. Ulcerative colitis.
d. Diverticular disease.

15) The following are recognized causes of hemorrhoids except:


a. Rectal carcinoma.
- b. Rectal prolapse.
c. Chronic constipation.
d. Portal hypertension. e. Pregnancy.

16) Regarding full thickness rectal prolapsed all are true except:
a. Is associate with ano-rectal malformation .
b. Tends to occur at the extremes of life.
c. May be treated by direct suturing to the rectum.
d. May cause fecal incontinence.
e. Usually starts as internal intussusceptions through the pelvic floor
muscles.

17) Sigmoid volvulus:


a. Is a rare cause of colonic obstruction.
b. Is associated with long sigmoid mesenteric attachment.
c. Usually twist in a clockwise direction.
d. Usually presents with marked abdominal distention.

18) Anal fissures:


a. When chronic may be healed with xylocaine cream.
b. Are associated with chronic constipation.
c. May be treated by surgical division of the external sphincter
d. Associated with melena.

19) All the following measures can be used for the treatment
acute anal fissure except:
a. Xylocaine ointment.
b. Lateral sphinctrotomy.
c. Anal stretch.
d. Laxatives.
-e. Injection of 5% phenol in alcohol.
20) Diverticular disease of the colon:
a. Is most commonly situated in the sigmoid colon.
b. Is caused by a high fiber diet.
c. Is associated with atrophy of colonic muscle wall.
d. Is associated with Increased risk of colonic cancer.

21) Cancer caecum:


a. Is usually an annular stenosing lesion.
b. Commonly presents with intestinal obstruction.
c. Rarely causes diarrhea.
d. Is usually not palpable on abdominal examination.
e. Can presents with microcytic anemia not responding to treatment.

22) Familial polyposis coli:


a. Is an autosomal recessive disorder.
b. Rarely needs surgical intervention.
c. Is confined to the left colon.
d. Villous adenomas are more prone to malignant transformation.

23) Concerning internal piles, the following statements are correct except
that they:
a. Are due to varicosity of the internal heamorrhoidal plexus.
b. May consist of mother and daughter piles.
c. Present clinically by bleeding and prolapse.
d. Are usually associated with severe pain.
e. May resolve under conservative treatment in the early stages.

24) Which of the following is NOT a feature of ulcerative colitis?


a. Crypt abscesses.
b. Granulomas.
c. Perianal infections.
d. Pseudopolyposis.
e. Back wash ileitis.

25) A 67 years old female presented in emergency with left sided abdominal
pain, fever & shock. On emergency laparotomy, large tumor of the sigmoid
colon was found with two perforations causing peritonitis. What is the
appropriate operation?
a. Left hemicolectomy with primary anastomosis.
b. Sigmoid colectomy with primary anastomosis.
c. Hartman's procedure.
d. Abdomeno-perineal resection.

26) Hamartomatous polyps in the colon are a feature of :

-
a. Peutz-Jegher's syndrome.
b. Familial adenomatous polyposis syndrome.
c. Gardner's syndrome.
d. Ulcerative colitis.
CHAPTER (I 8)
LIVER & SPLEEN
1) Etiologies of presinusoidal portal hypertension include:
a. Alcoholism.
b. Budd-Chiari syndrome .
c. Hepatitis C.
d. Schistosomiasis.

2) Appropriate maneuvers in a patient with hepatic


encephalopathy include all of the following except:
a. Addition of glucose to the diet.
b. Administration of lactulose .
c. Construction of a side-to-side porto-caval shunt.
d. Limiting dietary protein.

3) The commonest tumour which can cause multiple metastases in


the liver is :
a. Breast cancer.
b. Bronchogenic carcinoma.
c. Renal cell carcinoma.
d. Colon cancer.
e. Follicular carcinoma of the thyroid.

4) The best treatment of an amoebic liver abscess is :


a. Urgent surgical drainage.
b. Metronidazole.
c. Emetine hydrochloride.
d. Metronidazole followed by aspiration. e. Hepatic resection.

5) Regarding hepatocellular carcinoma all true except:


a. Is associated with cirrhosis.
b. Has Alphafeto protein in the blood as a tumour marker.
c. Spreads through intra hepatic veins.
d. Has a good prognosis.
e. May present with para neoplastic syndrome.

6) Regarding post operative fever all are true except:


a. Within 48 hours, is commonly due to atelectasrs.
b. If > 41*C, it is most likely to be intectrous.
c. Pyrexia is not associated with post operative DVT.
d. AT 12 days, it is likely to be due to anastomotic leak.
7) Regarding splenectomy all are true except:
a. It is indicated for autoimmune thrombocytopenia .
b. lt is Indicated for superficial splenic tear < 2cm.
c. Patients should be vaccinated against pneumococci.
d. Patients are at a high risk of infection from encapsulated organisms
post operatively.

8) Surgical techniques of preserving a ruptured spleen include all except:


a. Partial splenectomy.
b. Induced hypotension to minimize haemorrhage.
c. Splenorraphy.
d. Wrapping of viable splenic fragments with greater omentum.
e. Conservative measures with observation in a hemodynamic stable patient.

9) Child's classification of hepatic function in cirrhosis include all except:


a. Serum bilirubin.
b. Presence of ascites.
-
c. White cell count.
d. Serum albumin.

10) Causes of huge splenomegaly include all except:


a. Chronic hepatitis C. b. Malaria.
c. Chronic myeloid leukemia.
d. Myelofibrosis.
e. Thalassemia.

11) Regarding portal vein thrombosis, the untrue statement is :


a.. Occur in patient with thrombocytopenia.
b. Occur after severe appendicitrs.
c. Is a cause of mesenteric bowel ischemia.
d. Cause splenomegally.
e. Cause portal hypertension.

12) Following splenectomy patients are at a high risk of over whelming


sepsis from:
a. Anaerobic bacteria.
b. Fungi.
c. Staph aureus.
d. Pneumococci.
e. E. Coli.

94
13) Regarding liver trauma, the untrue is :
a. May be managed conservatively.
b. May cause haemobilia.
c. Excessive bleeding is best controlled by packing gauze into liver wound.
d. Excessive bleeding may be controlled by Pringle's maneuver.

14) Hydatid cyst of the liver:


a. Is caused by entameba histoliticum.
b. Contains chocolate colored pus.
c. Respond well to metronidazole.
d. Damage of the laminated membrane leads to daughter cysts formation.
e. The ectocyst secretes the hydatid fluid.

15) Liver metastases :


a. Treated by enucleation.
b. Has a good prognosis.
c. Diagnosed only at laparotomy.
d. Sub capsular umblicated.
e. A cause of acute abdomen.

16) Treatment of traumatic injury to the liver includes all except:


a. Resection debridement of devitalized tissues.
b. Legation of exposed blood vessels and bile ducts.
c Packing with gauze.
d. Pringle's maneuver.
e. Partial hepatectomy.

17) Which of the following serum estimations in not useful in diagnosis of


chronic liver diseases:
a. Bilirubin.
b. Transaminases.
c. Alkaline phosphatase.
d. Guma glutamyl transpeptidase .
e. Creatinine phosphokinase.

18) The most common malignant tumour of the liver is:


a. Hepatocellular carcinoma.
b. Cholangio carcinoma.
c. Carcinoid tumour.
d. Metastatic deposits.
e. Lymphoma.
19) The porto-systemic shunt between the superior heamorrhoidal vein
and the middle and inferior heamorrhoidal veins will cause which of the
following pathology:
a. Internal hemorrhoids. b. External hemorrhoids.
c. Intero-external hemorrhoids. d. Anorectal varices.

20) Regarding injection sclerotherapy all of the following are true except:
a. May cause retrosternal discomfort.
b. May lead to an esophageal ulcer.

-
c. Is the second line of treatment after balloon tamponade.
d. May lead to perforation of the esophagus.

21) Segmental surgical anatomy of the liver is based on the distribution of


a. Hepatic artery.
b. Portal vein
c.Hepatic vein.
d. Biliary drainage.
CHAPTER (19)
BILIARY SYSTEM & PANCREAS

1) Cholesterol stones :
a. Are radiopaque.
b. Are caused by a decrease in phospholipids in bile salts.
c. Do not cause biliary obstruction.
d. Arise from cholesterol hyper secretion.

2) Brown gallstones:
a. Are mainly cholesterol stones.
b. Are caused by infection.
c. Arise from an increase in conjugated bilirubin.
d. Are mainly found in South America.

3) The next diagnostic test in a jaundiced patient when


ultrasonography suggests common duct dilatation is :
a. Computed tomography
b. Biliary scintigraphy.
c. Endoscopic retrograde cholangio pancreatography.
d. Percutaneous transhepatic cholangiography.

4) A biliary-enteric fistula most frequently connects the


gallbladder with the:
a. Duodenum.
b. Jejunum.
c. Stomach.
d. Hepatic flexure of the colon.

5) The most common cause of acute cholecystitis is :


a. Cystic duct obstruction.
b. Escherichia coli infection.
c. Multiple gallstones.
d. Salmonella infection.

6) The best initial procedure in defining the cause of


obstructive jaundice in a 75 year-old man is :
a. Endoscopic retrograde cholangio pancreatography.
b. Percutaneous transhepatic cholangiography.
c. Utrasonography.
d. Computed tomography scanning.
7) Abdominal ultrasonography is carried out during
evaluation for a possible abdominal aortic aneurysm. The
presence of stones in the gallbladder is identified on this study.
The patient's only abdominal symptom is a sense of fullness
after eating. The appropriate first step in managing the
gallstones is :
a. Observation.
b. Laparoscopic cholecystectomy.
c. Open cholecystectomy.
d. Ursodeoxycholic acid therapy.

8) The commonest organism that can cause ascending


cholangitis, is :
a. Staph. Aureus.
b. Strept. Pyogens.
c. Clostridia welchii.

- d. E. coli.
e. Clostridia perferang.

9) All the following measures are useful in the treatment of


acute pancreatitis except:
a. Restoration of blood volume.
b. Rest to the pancreas.
c. Relief of pain.
d. Pancreatico-gastrostomy.

10) The incidence of gall stones is increased in the following


conditions except:
a. Obesity.
b. Liver cirrhosis.
c. Thalassemia.
d. Viral hepatitis.
e. Contraceptive pills.

11) The following are well-recognized causes of right


hypochondrial pain except:
a. Right lower lobe pneumonia.
b. Acute cholecystitis.
c. Charcot's triad.
d. Amebic liver abscess.

- e. Acute colonic diverticulitis.

12) Carcinoma of pancreatic head:


a. Rarely develops after the 5th decade.
b. Responds well to chemotherapy.
c. Frequently presents with a palpable gall bladder.

d. Represents 20% of pancreatic cancer. e. Has a good prognosis.

13) Carcinoma of the pancreas:


a. Has a peak incidence in young age.
b. Has a good prognosis.
c. Commonly presents with asthenia.
d. Is more common In non smokers.

14) Regarding patient with jaundice:


a. Urobilinogen in urine suggests obstructive jaundice.
b. Bilirubin in urine suggests hemolytic jaundice.
c. Markedly elevated alkaline phosphates enzyme is an indicator of post hepatic
jaundice.
d. Raised gamma glutamyl transferase is indicator of a prehepatic jaundice.

15) Indications of ERCP include:


a. Gall bladder stones.
b. Pseudo pancreatic cyst.
c. Empyema of the gall bladder .
d. Cancer head of pancreas.

16) Regarding obstructive (post hepatic) jaundice:


a. Is milder than hemolytic jaundice.
b. Urine colour is normal.
c. Serum transaminases grossly increased.
d. Serum alkaline phosphatase is elevated.
e. Unconjugated bilirubin is usually increased.

17) Chronic cholecystitis:


a. Is usually associated with gall stones.
b. Is more common in males.
c. The gall bladder is often palpable.
d. The gall bladder has a thin wall.

18) Regarding gall stones:


a. Most are visible on plain x-ray.
b. Pure cholesterol stone form more than 10% of stones.
c. Pigment stones are formed of conjugated bilirubin.
d. Mixed stones are the commonest type.

19) The commonest organism that causes acute cholecystitis is :


a. Staph aureus.
b. Gram -ve bacilli.
d. Strept Pyogenes.
c. Bacteroids.
20) Which of the following is a contra indication to
laparoscopic cholecystectomy:

a. Mucocele of the gall bladder.


b. Large number of gall stones.
c. A patient who had previous caesarean section.
d. Liver cirrhosis.
e. Patient over 60 year of age.

21) All the following are indicated in treatment of acute


pancreatitis except:
a. I.V fluids.
b. Nasogastric tube.
c. Relief of pain.
d. Respiratory support.
e. Cysto gastrostomy.

22) Regarding acute pancreatitis all true except:


a. May be caused by regurgitation of bile into the pancreatic duct.
b. Can induce fat necrosis.
c. Has a raised serum amylase.
d. May cause carcinoma of the pancreas.

23) Cholesterol gall stones:


a. Are more common in sickle cell disease
b. Account for 25% of all gall stones.
c. Are visible on plain x-ray.
d. Usually contain a nucleus and laminated in cut section.
e. Usually single in number.

24) In gall stone disease:


a. Most of the cases are symptomatic.
b. Biliary colic is due to gall bladder ischemia.
c. Mucocele of the gall bladder contains infected bile.
d. Acute cholecystitis rarely perforates.
25) The best treatment of a residual stone in the common
bile duct after cholecystectomy operation is :
a. Re operation. b. ESWL.
c. ERCP. d. Medical treatment.
e. Percutaneous transhepatic drainage.

26) The best investigation to diagnose acute calcular cholecystitis


is:
a. CT scan. b. IV cholangiography.
c. HIDA scan. d. Ultra sound.
e. MRI.

27) The most important measure in the treatment of severe acute


pancreatitis is :
a. Early surgical drainage.
b. Administration of I.V fluids for resuscitation.
c. Glucagon administration.
d. Somatostatin administration.
e. Antibiotics.

28) A 65 year old man has developed obstructive jaundice as


a result of carcinoma of the head of the pancreas obstructing the
common bile duct. Which of the following biochemical
abnormalities will be seen:
a. Decreased bilirubin In the urine.
b. Decreased stercobilinogen in the stool.
c. Increased urobilinogen in the urine.
d. Decreased plasma direct bilirubin.

29) The most common cause of acute pancreatitis is :


a. Excess alcohol intake.
b. Bile duct stones.
c. Paracetamol.
d. Non-steroid anti-inflammatory drugs.
CHAPTER (20)
PEDIATRIC SURGERY

1) Regarding congenital diaphragmatic hernia the untrue is :


a. Is found on the left side in most cases.
b. Should be regarded as surgical emergency.
c. Associated with neonatal respiratory distress.
d. Is associated with malrotation of the gut.
e. Is associated with right to left cardiac shunt.

2) A previously healthy 8 year old boy came to the emergency


room because of hematemesis, following severe vomiting the most
probable cause is :
a. Erosive gastritis.
b. Idiopathic thrombo cytopaenic purpura (ITP).
c. Hemophilia.
d. Oesophageal varices.
e. Mallory-Weiss syndrome .

3) Congenital hypertrophic pyloric stenosis is characterized by


all the following except:
a. The usual age of presentation is 3-6 months.
b. Recurrent attacks of projectile non bilious vomiting.
c. Constipation.
d. Metabolic alkalosis.
e. A small pyloric lump may be palpable.

4) Hirschsprung's disease:
a. Only involves the sigmoid colon.
b. Is characterized by an excess of ganglion cells in the myenteric
plexus.
c. Is best diagnosed by rectal mucosal biopsy.
d. Can involves the terminal ileum.
5) In Hirschsprung's disease:
a. There is absent sympathetic nerve supply to the bowel.
b. Enterocolitis may be a complication.
c. The lumen of the Aganglionic segment is grossly dilated.
d. The rectum is not affected.
6) The most common form of esophageal atresia is :
a. Pure esophageal atresia (no fistula).
b. Pure tracheo-esophageal fistula (no atresia).
c. Esophageal atresia with distal tracheo-esophageal fistula.
d. Esophageal atresia with proximal tracheo-esophageal fistula.

7) Undescended testes are usually repaired at which age?


a. When diagnosed.
b. In infancy.
c. Before 2 years of age.
d. Before 6 years of age.

8) A double bubble on an air contrast upper gastrointestinal


series in an infant is characteristic of :
a. Duodenal atresia.
b. Jejunal atresia.
c. Meconium ileus.
d. Pyloric stenosis.

9) The most common cause of mortality in congenital


diaphragmatic hernia is :
a. Pulmonary hypertension.
b. Intestinal obstruction.
c. Respiratory failure.
d. Ipsilateral lung hypoplasia.

10) Regarding Hirschsprung's disease of the following are true except:


a. Incidence is 1 : 5000 births.
b. Male: female ratio is 4 : 1.
c. The familial incidence is 6%.
d. It starts in the upper part of the rectum
CHAPTER (21)
UROLOGY

1) All the following drugs may be used for treatment of prostatic


cancer except:
a. Ethyil estradiol.
b. Androgens.
c. Luteinizing hormone release hormone.
d. Anti androgens.
e. Stilbesterol.

2) Which of the following lymph node groups is the site of


metastases of testicular neoplasm:
a. Internal iliac.
b. Inguinal.
c. External iliac.
d. Para-aortic.
e. Common iliac.

3) The commonest presentation of a nephroblastoma is :


a. Abdominal mass.
b. Haematuria.
c. Renal pain.
d. Pulmonary metastases.
e. Polycythaemia.

4) The recommended age to operate for a patient with an undescended


testis is:
a. 2 months. b. At puberty.
c. 8 years. d. 2 years.
e. 15 years.

5) The best procedure to diagnose injury of the urethra is :


a. IV pyelography.
b. Passage of a urethral catheter.
c. Ascending urethrography.
d. Micturation cysto-urethrography.
e. Ultrasound.
6) A 10 years old boy, involved in a road traffic accident and
diagnosed to have fracture pelvis he is unable to pass urine
because of :
a. Penile urethra rupture.
b. Posterior urethra rupture.
c. Rupture of membranous urethra.
d. Intra peritoneal rupture bladder.

7) Regarding urinary stones, the untrue statement is :


a.They are usually radiolucent.
b. Are most commonly calcium oxalate stones.
c If they are phosphate stones. give rise to stag horn calculus
d. May cause hydronephrosis.
e. May cause urine retention.

8) Regarding hypernephroma, all of the following are true except:


a Is the commonest abdominal malignancy in children.
b. Is associated with polycythaemia.
c. May extend into the renal vein.
d. May cause haematuria.
e. May spread to lung.

9) Regarding carcinoma of the prostate:


a. It is usually a squamous cell carcinoma.
b It spreads to the lumbar spine and sacrum.
c. Bone metastases are seen as osteolytic lesions.
d. Normal PSA level excludes the presence of malignancy.
e. Radical prostatectomy is the treatment of choice in all cases.

10) Regarding testicular tumours, the untrue is :


a. Seminoma and teratoma are more common than non-germ cell
tumours.
b. Seminoma sends pulmonary deposits.
c. They have an increased incidence in undescended testis.
d. Alpha-fetoprotein and Beta-HCG are tumour markers.
e. The treatment of choice is radical orchidectomy.

11) The following are complications of benign prostatic hyperplasia


except:
a. Urinary calculi.
b. Bladder diverticulum.
c. Cancer prostate.
d. Renal failure.
e. Urinary infections.

12) Torsion testis:


a. Can be managed conservatively.
b. Associated with raised serum - HCG.
c. May be diagnosed by colour flow Doppler.
d. Ectopic testis is a risk factor.

13) Bladder irrigation during TUR prostatectomy may produce


a. Hemolysis.
b. Hyponatraemia.
c. Hypercalcemia.
d. Hyperkalemia.
e. Alkalosis.

14) A 60 year old male presenting with 3 attacks of painless


haematuria during the last 6 weeks. He had no dysuria or
frequency. He should be suspected to be suffering from:
a. Polycystic kidney.
b. Renal calculi.
C.Hypernephroma.
d. Cancer bladder.
e. Benign prostatic hyperplasia.

15) Regarding hypernephroma, all are true except:


a. It arises from renal tubular epithelium.
b. May leads to 2ry varicocele formation.
c. Commonly arise in one pole of the kidney.
d. May cause renovascular hypertension.
e. May form a palpable mass.
16) The cardinal features of ectopia vesicae include the following
except:
a. The pelvic bones are widely separated.
b. The scrotum is often split.
c. Ascending urinary infection is a common complication.
d. The testes may be undescended.
e. The penis is normally developed.

17) Intra peritoneal rupture bladder, it is untrue that it :


a. May cause peritonitis.
b. Requires immediate laparotomy.
c. Occurs in over distended bladder.
d. Fracture pelvis is the common underlying cause.
e. May be due to external trauma or intra-vesical instrumentation

18) Regarding extra peritoneal rupture bladder:


a. Causes urine extravasationIn the perivesicalspace and inside the
peritonealcavity.
b. May cause shifting dullness.
c. Causes intense desire to micturate and supra pubic pain.
d. It is a complication of TUR prostatectomy operation

19) The incorrect statement about bilharzial cystitis is that it :


a. Predispose to 2ry infection and stone formation.
b. Rarely causes bladder neck obstruction.
c. May produce calcified shadow in the plain x-ray.
d. One of the common causes of haematuria in Egypt.
e. May require surgical treatment.

20) In penile hypospadius :


a. The urethra opens midway between the scrotum and the anus.
b. The prepuce is deficient superiorly.
c. The penis is curved downwards.
d. No treatment is required till the age of 10years.
e. Circumscion should be done during the neonatal period.

21) In extrapelvic rupture urethra, the following statements are true


except that it :
a. Usually affects the bulbous urethra.
b. Causes bleeding from the meatus.
c. Results in urine retention.
d. Requires urgent catheterization.
e. May be partial or complete.

22) Regarding intra pelvic rupture urethra all are true except:
a. Always involves the membranous urethra.
b. Is a complication of fracture pelvis.
c. Usually associated with rupture of the puboprostatic ligaments.
d. Causes urethral bleeding and inability to micturate.
e. Produces no signs on per-rectal examination.

23) The earliest symptom of benign prostatic hyperplasia is :


a. Dysuria.
b Nocturnal frequency.
c. Hesitancy.
d. Post micturation dripping.
e. Urine retention.

24) The following statements about ectopic testis are true except that it :
a. Is probably due to rupture of the scrotal tail of the gubernaculum.
b. May lie in the groin or pubic region.
c.Doesnot develop normally.
d. Has a normal long spermatic cord.
e. Can be easily replaced in the scrotum.

25) The complications of undescended testis include the following except:


a. Torsion of the spermatic cord.
b. Malignancy.
c. Loss of 2ry sex characters.
d. Infertility.
e. Oblique inguinal hernia.
26) Concerning testicular seminoma:
a. Is a rare tumour of the testis.
b. Treated initially by radiotherapy .
c. Arises from the cells of seminiferous tubules.
d. Serum Beta-HCG is a tumour marker.
e. Spreads to deep inguinal nodes.

27) Regarding primary varicocele:


a. Should always be treated surgically.
b. Occurs in association with renal cell carcinoma.
c. Forms a soft compressible swelling in the spermatic cord.
d. Is translucent.
28) In an elderly male, a prostatic mass was discovered on rectal
examination. Features suggestive of malignancy include the following
except:
a. An irregular surface.
b. Fixation of the overlying rectal wall.
c. Hard in consistency .
d. Deepening of the posterior sulcus.
e. Elevated irregular border.

29) In a 20 years female patient, gross haematuria is most likely to be due


to :
a. Cystitis. b. Renal calculi.
c. Polycystic kidney. d. Cancer bladder.
e. Hemophilia.

30) The contraindication to Percutaneous Nephrolithotomy (PCNL) is :


a. Hydronephrosis.
b. Bleeding disorders.
c. Stage horn stone.
d. Multiple calcyeal calculi.
e. Urinary tract infections.
31) The indications for prostatectomy in benign prostatic hyperplasia
include all except:
a. Profuse bleeding. b. Diverticulum formation.
c. Stone formation.
d. Residual urine more than 200c.c.
32) Regarding squamous cell carcinoma of the bladder:
a. Commonly arises from the trigone.
b. Is radiosensitive.
c. Bllharzias is the commonest predisposing factor.
d. Can be treated with partial cystectomy.
e. More common among old ages.

33) The complications of unilateral hydronephrosis include the following


except:
a. Stone formation. b. Uremia.
c. Rupture. d. Infection.

34) The following calculi are formed in infected urine:


a. Calcium oxalate stones.
b.Triple phosphate stones.
c. Urate stones.
d. Cystine stones.
e. Pigment stones.

35) A 40 year old male was admitted to the ER in severe hypovolemic


shock following a road traffic accident. Abdominal examination
revealed tenderness and rigidity in the left loin and in the supra pubic
region. He should be suspected to have sustained
a. Rupture spleen.
b. Rupture left kidney.
c. Retroperitoneal heamatoma.
d. Intra peritoneal rupture bladder.
e. Intra pelvic rupture urethra.

36) Prostate cancer occurs most commonly in which zone of the prostate?
a. Central zone. b. Urethral zone.
c. Transitional zone.
d Peripheral zone.
37) Transurethral resection syndrome after endoscopic resection of the
prostate occurs because of :
a. Air embolism during the procedure.
b. Blood loss from postoperative irrigation.
c. Absorption of irrigating fluid.
d. Relaxation of the urethral sphincter form spinal anesthesia.
38) The most common urinary diversion after total cystectomy is:
a. End ureterostomy with external skin stoma.
b. Uretero-sigmoid anastomosis.
c. Ileal conduit.
d. Colonic conduit.

39) Initial therapy for metastatic prostate cancer is:


a. Total prostatectomy (sphincter-preserving).
b. Radical prostatectomy.

-
c. Chemotherapy.
d. Bilateral orchidectomy.

40) Extra peritoneal bladder rupture is usually treated by :


a. Placement of a Foley catheter and expectant management.
b. Supra pubic cystostomy.
c. Endoscopic repair.
d. Laparotomy.

41) All of the following signs are present in torsion testis except:
a. The affected testis is elevated.
b. The skin of the scrotum is normal.
c. There IS vomiting.
d. The contra lateral testis has a transverse lie.

42) All of the following lines are used in the treatment of benign
prostatic hyperplasia except:
a. Alpha blockers.
b. Diuretics.
c. Endoscopic laser ablation.
d. Trans urethral prostatectomy.
43) Regarding Wilms' tumor, all of the following are true except:
a. It is capsulated.
b. There is early infiltration of the renal pelvis.
c. Sometimes is assocrated with hemihypertorphy.
d. Postoperative Actinomycin D, Vrcristine and Adriamycm are
required if there is no residual tumor.
44) During physical examination for the purpose of life insurance, a 38
years old man is found to have a left inguinal mass. The right testis is
palpated in the scrotum and is of normal size, but a left testis cannot
be palpated in the scrotum. Ultrasonography shows that the inguinal
mass is consistent with a cryptorchid testis. Which of the following
approaches is most appropriate to deal with this patient's testicular
abnormality?
a. Perform orchidopexy.
b. Remove both testes.
c. Commence testosterone therapy.
d. Remove only the cryptorchid testis.
e. Perform a chromosome analysis.

45) A 78 year old healthy man is noticed on a routine annual check-up to


have a hard nodule palpable in the prostate on digital rectal
examination. Microscopic examination of the biopsies of this nodule show
small, crowded glands containing cells with prominent nucleoli. This
man is most likely to have which of the following conditions?

a. Benign prostatic hyperplasia.


b. Chronic prostatitis.
c. Metastatic urothelial carcinoma.
d. Infarction of the prostate.
e. Adenocarcinoma of the prostate.

46) The renal angle lies between:


a. The costal margin and the spines of the vertebrae.
b. The sacrospinalis and the midline.
c. The costal margin and the midline.
d. The costal margin and the sacrospinalis.
CHAPTER (22)
ORTHOPEDIC SURGERY

1) The recommended treatment for a compound fracture of


the tibia is :
a. Internal fixation by a plate and screws.
b. Plaster fixation.
c. External skeletal fixator.
d. Intra medullary nail fixation. e. Skeletal traction.

2) Osteoclastoma is characterized by all except:


a. The usual age of presentation is 20-30years.
b. The lesion is metaphyseal.
c. The usual presentation is a bony mass.
d. The commonest site of the lesion is around the knee.
e. Plain x-ray reveals a soap bubble appearance.

3) The recommended treatment for a closed fracture mid shaft


femur in an adult is:
a. Reduction and fixation in a plaster cast.
b. External skeletal fixator.
c. Intra-medullary nail.
d. Skeletal traction.
e. Thomas splint.

4) All of the followings may be radiological findings of an


osteosarcoma except:
a. The lesion is metaphyseal.

b. Sunray appearance.
c. Bone destruction.
d. New bone formation.
e. Soap bubble appearance.

116
5) The usual causativeorganism that leads to acute
haematogenous osteomyleitisis:
a. E. Coli. b. Strept. Pyogens.
c. Staph. Aureus. d. Salmonella.
e. Hemolytic streptococci.

6) All of the following are correct about giant cell tumour of


the bone except:
a. It causes expansion of the affected area.
b. It usually arises in the diaphysis of the affected bone.
c. Surgery and radiotherapy have got a role in the treatment.
d. The most commonly affected bones are around the knee.
e. Characterized by the radiological appearance of medullary plug.

7) Which of the following lesions does not commonly cause


metastasis in the bones:
a. Renal cell carcinoma.
b. Prostatic cancer.
c. Follicular carcinoma of the thyroid .
d. Rectal carcinoma.
e. Breast cancer.

8) A 10 year old boy involved in an accident. X-ray shows fracture


neck fibula, on examination he is unable to dorsiflex the ankle, the
nerve involved is :
a. Popliteal nerve.
b. Anterior tibial.
c. Lateral popliteal (common peroneal) nerve.
d. Superficial peroneal nerve.
e. Sciatic nerve.

9) Immobilization of fractures of long bones should include the:


a. Fractured bone only.
b. Proximal joint.
c. Both proximal and distal joints.
d. Joint involved in fracture.
10) The local complications of closed fractures does not include:
a. Malunion.
b. Non union.
c. Osteomyleitis.
d. Sudek's atrophy.
e. Joint stiffness.

11) Non-union in closed fractures may be due to any of the


following except:
a. Inadequate immobilization.

b. Soft tissue interposition.


c. Impaired blood supply.
d. Impaction of the fragments.
e. Wide separation of the fragments.

12) Which statement is untrue concerning Sudek's atrophy:


a. Occurs most often after wrist and ankle injuries.
b. Is characterized by severe pain and stiffness.
c. Followed by osteoarthritis of the near joint.
d. May require sympathectomy
e. Is a type of osteodystrophy.
13) Regarding fracture clavicle, the untrue is :
a Is usually due to direct trauma.
b. Commonly Involves the middle third.
c. Is often associated with over riding of fragments.
d. Causes dropping and deformity of the shoulder.
e. Is usually treated by figure of eight bandage.

14) Regarding anterior shoulder dislocation, the incorrect


statement is :
a. There is a flattened shoulder contour.
b. The arm is abducted and externally rotated.
c. All movements of the shoulder are limited.
d. The anterior and posterior folds of the axilla are elevated.
e. Recurrent dislocation is a famous complication.

15) Closed fracture of the shaft of the humerus is best treated by :


a. Closed reduction and shoulder spica.
b. Intra medullary nail.
c. Plate and screws fixation.
d U-shaped plaster cast and arm to neck sling.
e. Skeletal traction.

16) The most vulnerable structure to be injured in supra condylar


fracture of the humerus is:
a. Brachial artery. b. Median nerve. c. Ulnar nerve.
d. Radial nerve. e. Elbow joint.

17) The commonest complications of fracture neck femur is :


a. Thromboembolism. b. Coxa vera deformity.
c A vascular necrosis of the head femur.
d. Delayed union. e. Malunion.

18) Regarding Cole's fracture:


a. It is a fracture of the distal ulna and radial styloid process.
b. Commonly seen between the age of 20-30 years.
c Produces a dinner fork deformity.
d. Usually treated by plate and screw fixation.
e. A vascular necrosis is a famous complication.
19) The following statements about osteochondroma (exostosis) are
true except:
a. Never affects flat bones.
b. Can leads to mechanical block of the near joint.
c. Affects the diaphysis of long bones.
d. May be associated with dwarfism.
e. Can be multiple.

20) Generalized osteitis fibrosa cystica is characterized by the


following features except:
a Hypo-parathyroidism.
b. Diffuse rarefaction and decalcification of bones.
c. Multiple cysts of the bones.
d. Urinary symptoms due to renal calculi.
e. Pathological fracture of the affected bones.

21) The treatment of osteoclastoma includes the following


measures except:
a. Curettage of tumour tissue and packing the cavity with bone chips.
b. Excision with safety margin of bone.
c. Radiotherapy.
d. Chemotherapy.
e. Replacement of the affected part by prosthesis.

22) The following statements about multiple myeloma are true


except that it :
a. Is a primary malignant tumour of bone marrow.
b. Characterized by the presence of Benc Johns protein in urine.
c . ls rarely associated with anemia.
d. May cause paraplegia.
e. Can produce abnormal immunoglobulin.

23) Regarding chondrosarcoma:


a..Most commonly affects the flat bones.
b. Is radio sensitive.
c. Has onion peel appearance as a characteristic radiological
appearance.
d. Commonly spread to the regional draining lymph nodes.
e. Occurs most often in children.
24) Ewing's sarcoma is characterized by the following except:
a. Is a tumour of children.
b. Always arises in the metaphysic of long bones
c. Gives onion peel appearance in plain x-ray.
d. Presents with fusiform swelling with inflammatory changes in the
overlying soft tissue.
e. Sends pulmonary metastasis.

25) The followings are a differential diagnosis of acute osteomyleitis


except:
a. Ewing's sarcoma.
b. Osteoclastoma.
c. Septic arthritis.
d. Rheumatic arthritis.
e. Cellulitis.

26) The most common osteolytic metastases in bones are derived


from the:
a. Lung. b. Stomach.
c. Breast. d. Kidney.
e. Prostate.

27) The treatment of acute septic arthritis includes the following


except:
a. Broad spectrum systemic antibiotics.
b. Splint in the position of function.
c. Aspiration of pus.
d. Arthrotomy and drainage.
e. Local corticosteroids injection in the affected joint.

28) The most characteristic feature of Volkmann's contracture is :


a. Wrist drop.
b. Sensory loss on the medial aspect of the forearm.
c. Extension of the metacarpo-phalangeal joints with flexion of the
inter-phalangeal joints.
d. Wasting of the forearm extensor muscles.
e. Skin trophic changes.
29) Complications of fracture tibia include all except:
a. Leg shortening. b. Compartment syndrome
c. Delayed union. d. Soft tissue injury.
e. Fat embolism.
30) The nerve commonly injured with fracture neck of humerus is :
a. Axillary nerve.
b. Subscapular nerve.
c. Nerve to latismus dorsi.
d. Long thoracic nerve of bell. e. Radial nerve.

31) Regarding fracture Pelvis all are true except:


a. It is unstable, if there is disruption of the true pelvic ring.
b. Commonly associated with visceral injury.
c. Open book type is due to vertical shear trauma.
d. Always treated with skeletal traction.
e. Is usually due to road-traffic accidents.

32) The incorrect statement concerning chronic osteomyleitis is:


a. Usually following inadequately treated acute attack.
b. Can be complicated by pyogenic septic arthritis.
c. Can leads to pathological fracture.
d. Sequestrum is a dead separated piece of bone.
e. There is subperiosteal new bone formation.

33) Regarding TB of the dorsal spine:


a. Usually due to direct spread from a near by focus.
b. Lumbar lordosis is a common feature.
c. The inter vertebral discs commonly spared.
d. Can leads to the formation of a psoas abscess.
e. Can be treated with skeletal traction.
34) Intertrochanteric fracture neck femur:
a. Usually complicated by a vascular necrosis.
b. Hip hemi arthroplasty is the treatment of choice.
c. Intra medullary nail can be a line of treatment.
d. is an extra capsular fracture.
e. The limb is usually adducted and externally rotated.

35) Regarding supra-condylar fracture of the humerus all are true


except:
a. 50% of the fracture is of the green stick type.
b. Malunion can lead to a cubitus verus deformity.
c. Brachial artery injury is a famous complication.
d. Is commonly seen among children.
e. Usually treated by a full arm plaster cast.
36) Which of the following is the most common malignant lesion of the
bone?
a. Chondroblastoma. b. Fibrosarcoma.
c. Ewing's sarcoma. d. Osteosarcoma.

37) Which of the following does NOT frequently metastasize to bone?


a. Renal cell carcinoma.
b. Thyroid cancer.
c. Lung cancer.
d. Colon cancer.

38) Which of the following is the most common causative


organism in osteomyleitis?
a. Salmonella. b. Pseudomonas.
c. Staphylococcus. d. Escherichia coli.

39) An 82 year old woman with no major medical problems is


evaluated in the emergency room after a fall with a painful left hip and
an inability to ambulate. Radiographs show not only a fracture
of the left femoral head, but also a compressed fracture of the
T10 vertebra. Which of the following conditions is she most likely to
have?
a. Acute osteomyleitis.
b. Osteogenesis imperfecta.
d. Polyostotic fibrous dysplasia.
c Osteoporosis.
e. Metastatic breast carcinoma.

40) Regarding healing of bone fractures all of the following


conditions are true except:
a. Children better than in adults.
b Transverse better than in oblique fractures.
c. Impacted better than in distracted fractures.
d. Internal better than in external fixation.

41) Regarding acute hematogenous osteomyleitis, all of the following are true
except:
a Is caused by staphylococcus aureus in 100% of the cases.
b. Horizontal spread of infection results in subperiosteal sequestra.
c. Has to be differentiated from Ewing's sarcoma.
d. The plain x-ray is normal initially.

TRANSPLANTATION AND ONCOLOGY


1) Li-Fraumeni syndrome is associated with a mutation of which of
the following genes?
a. BRCA-1.
b. APC.
c. p53.
d. CA 19-9.

2) Aflatoxin exposure is associated with which one of the following


cancers?
a. Lung cancer.
b. Vaginal carcinoma.

-
c. Endometrial cancer.
d. Liver cancer.

3) The origin of carcino embryonic antigen is embryonic:

-
a. Endoderm.
b. Mesoderm.
c. Ectoderm.
d. Mesenchyme.

4) Elevation of cancer antigen 15-3 is useful in detecting metastatic:


a. Liver cancer.
b. Pancreatic cancer.
c. Thyroid cancer.
d. Breast cancer.

5) Which of the following tumor markers is the best to monitor


recurrence of colon cancer?
a: α-fetoprotein.
b. Carcino embryonic antigen.
c. Cancer antigen 15 - 3.
d. 5 -hydroxyl indole acetic acid.

6) Epstein-Barr virus has been associated with the development of


each of the following malignancies except:
a. B-cell lymphoma.
b. Burkett's lymphoma.
c. Cancer of the pancreas.
d. Nasopharyngeal carcinoma.
7) All of the following conditions are known to have a familial pattern
associated with a predisposition to cancer except:
a. Colonic polyposis.
b. Breast cancer.

-
c. Peutz-Jegher's syndrome.
d. Gardner's syndrome.

8) In a 20-year-old woman with an a symptomatic 2cm mass in the


lower pole of the right thyroid lobe, the most appropriate initial
maneuver is:
a. Excisional biopsy.
b. Incisional biopsy.
c. Needle aspiration cytology.
d. Radioiodine uptake study.

9) Which of the following tumors do not cause elevated level of carcino


embryonic antigen (CEA):
a. Breast cancer.
b. Colorectal cancer.
c. Gastric cancer.
_d. Renal cell carcinoma.

10) Hyper acute rejection of renal transplant:


a. Is cell mediated reaction.
b. Can be treated by the use of cyclosporine
c. Respond well to corticosteroids.
d. Is best treated by removal of the transplant.

11) Recognized complications of immune-suppressive therapy for renal


transplantation include all except:
a. Hypertension.
b. Nephrotoxicity.
c. Increases in incidence of malignancy.

-
d. Pancytopenia.
e. Hypercalcemia.

12) All the following may be an indication for liver transplantation except:
a. Liver cirrhosis.
b. Acute fulminant hepatitis.
c. Congenital biliary atresia.

-
d. Wilson's disease.
e. Malignant obstructive jaundice due to preiampullary carcinoma.
13) Which one of the following plays an important role in the
prevention of neoplasia?
a. Proto-oncogenes.

-
b. Hypertrophy and hyperplasia.
c. Apoptosis.
d. Metaplasia.

14) All of the following are common sites of hematogenous spread of


carcinoma except:
a. Spleen.
b. Brain.
c. Suprarenal glands.
d. Liver.
15) Which of the following statements about spread of malignant
tumours is correct?
a. Permeation means the cancer cells that invade a lymphatic vessel can
break away and be carried by lymph circulation to a regional node.
b. Malignant melanoma spreads by permeation alone.
c. 'Kiss cancer' of the lips is an example of spread of malignant tumours
by implantation.
d. Krukenberg's tumour IS an example of haematogenous spread. e. Rodent
ulcer spreads via the lymphatics.
SKIN AND SUBCUTANEOUS TISSUES

1) Which of the following is the most common type of melanoma?


a. Acral lentiginous.
b. Superficial spreading.
c. Nodular.
d. Lentigo maligna.

2) Clark level IV melanoma extends to which of the following structures?


a. Epidermis.
b. Papillary dermis.
c. Reticular dermis.
d. Subcutaneous tissue.

3) Malignant melanoma:
a. Superficial spreading type is the most common.
b. Chemotherapy has been shown to be an effective treatment.
c. prolonged exposure to tar derivatives is a risk factor.
d. Hutchinson melanotic freckle carry the worst prognosis.

4) Regarding skin grafts:


a. Split thickness skin grafts contain epidermis only.
b. Split thickness graft maintains their own blood supply from the donor area.
.c.Thinner split skin grafts are more likely to "take" in the recipient area
than thicker ones.
d. Split thickness grafts have a less incidence of contracture than full thicknes:
grafts.

5) Squamus cell carcinoma of the skin:


a. Is a locally malignant tumour.
b. Metastasize early by blood.
c. Can be treated with radiotherapy.
d. Can be staged with Berslow thickness involvement.
~.
6) Basal cell carcinoma:
a. Undergo frequent lymph node spread.
b. Are usually radio resistant.
c. Are the commonest malignant skin lesion.
d. Are frequently invasive.
7) Prognosis of malignant melanoma is related to all except:
a. Clinical stage.
b. Ulceration.
c. Berslow thickness infiltration.
d. Degree of pigmentation.

8) Regarding cutaneous squamous cell carcinoma all are true except:


a. Are associated with xeroderma pigmentosa.
b. May complicate chronic venous ulcer.
c. Has a rolled in beaded edges.
d. Metastasize to lymph nodes.

9) A sequestration dermoid cyst is :


a. Due to squamous cells being implanted in by a needle.
b. A sebaceous cyst.
c. Due to cells being buried over lines of fusion.
d. Mostly occurs in the limbs.

10) A hamartoma is :
a. Any collection of blood clot.
b. A developmental malformation.
c. A tumour of blood vessels.
d. A tumour of muscles.

11) A basal cell carcinoma of the skin:


a. Is radio resistant.
b. Metastasizes to the regional lymph nodes.
c. Has raised everted edges .
d. Most commonly occurs at the outer canthus of the eye.

12) All of the following are premalignant skin lesions except:


a. Xeroderma pigmentosa.
b. Solar keratosis.
c. Seborrheic dermatitis.
d. Bowen's disease.

13) Flaps can be all of the following except:


a. Musculo-cutaneous (Myocutaneous).
b. Fascio-cutaneous.
c. Spilt thickness.
d. Free microvascular.
41
14) Which one of the following is a complete statement for
description of a size of the swelling:
a. 5 x 2 cm.
b. 5 x 2 inches.
c. 0.5 x 1 inch.
d. 2 x 3 x 4 cm.

15) Sloping edge of an ulcer is characteristic of:


a. Septic ulcer.
b. Syphilitic ulcer.
c. Carcinomatous ulcer.
d. Tuberculous ulcer.

16) The best dressing is :


a. Swab.
b. Gauze.
c. Skin.
d. Aerosol plastic spray.
CHAPTER (8)
THE BREAST

1) The treatment of choice for a 1.8cm in diameter, NO, MO invasive


breast cancer is :
a. Lumpectomy alone.
b. Lumpectomy, sentinel node biopsy, and radiation.
c. Mastectomy with sentinel node biopsy and radiation.
d. Mastectomy with axillary node dissection and radiation.

2) Which of the following is associated with an increased risk of in situ


carcinoma of the breast?
a. Sclerosing adenosis.
b. Intraductal papilloma.
c. Fibroadenoma.
d. Atypical lobular hyperplasia.

3) Which of the following increases the risk of breast cancer?


a. Multiple gestations.
b. Late menarche.
c. Late menopause.
d. Prolonged and multiple episodes of lactation.

4) The most appropriate treatment for a woman with duct carcinoma


in situ (DCIS) involving 2 quadrants of the breast is :
a. Observation.
b. Lumpectomy.
c. Lumpectomy with radiation therapy.
d. Mastectomy.

5) Lobular carcinoma in situ (LCIS) :


a. Can be diagnosed on mammogram by micro calcifications adjacent to a
mass.
b. Can occur in men, although It is rare.
c. Is usually diagnosed in the sixth decade of life.
d. Progresses to lobular carcinoma in 50% of women.
6) Paget's disease:
a. Is associated with underlying duct carcinoma.
b. Can be differentiated from melanoma by carcino embryonic antigen
staining.
c. Is self-limiting and requires no treatment.
d. Is diagnosed by characteristic skin changes in the axilla.
7) Mondor's disease:
a. Is usually self-limiting and spontaneously resolves in 4-6 weeks.
b. Is indicative of an increased risk of breast cancer.
c. Requires surgical treatment for cure.
d. Is often bilateral.

8) Lev II axillary nodes:


a. Are located around the axillary vein.
b. Are medial to or above the pectoralis minor muscle.
c. Are made up of the central and interpectoral nodes.
d. Are made up of the external mammary and scapular nodes.

9) The BRCA genes are:


a. Tumor suppressor genes.
b. Proto-oncogenes.
c. Regulators of RNA transcription.
d. Regulators of protein modification.

10) Drugs that may produce gynecomastia include all of the


following EXCEPT:
a. Cimetidine.
b. Diazepam.
c. Furosemide .
d. Tamoxifen.

11) Breast cancer:


a. Is common among young girls .
b.lnvasive lobular type is bilateral in 25% of cases.
c. Decreases In incidence after the age of 65years.
d. Estrogen receptor +ve tumour has a bad prognosis.

12) Risk factors for breast cancer include all except:


a. Nullipanty.
b. Contra lateral breast cancer.
c. Increasing age.
d. Mammary duct ectasia.

13) Mammography features of breast cancer include:


a. Diffuse coarse macro calcification.
b. Calcification of mammary ducts.
c. Dense lesion With irregular outlines.
d. Nipple erosion.
14) Cyst sarcoma phylloides of the breast:
a. Are mostly malignant.
b. Commonly spread to regional lymph nodes.
c. Have a fleshy and lobulated appearance.
d. Are called breast mice.

15) Breast cancer:


a. Has worse prognosis if it is estrogen receptor positive.
b. Is more common with late menarche.
c. Has a genetic predisposition.
d. Is more common in multipara.

16) Regarding invasive breast cancer, the wrong statement is :


a. Involvement of supra clavicular nodes denotes distant metastasis.
b. Presence of estrogen receptors denotes better prognosis.
c.T3 denotes tumour fixed to skin and N3 means fixed ipsilateral nodes.
d. Lobular carcinomas have tendency to affect both breast.

17) All the following are precancerous to the breast except:


a. Duct papilloma.
b. Duct ectasia.
c. Florid hyperplasia.
d. Atypical hyperplasia.
e. Duct carcinoma in situ.

18) The most important prognostic index in breast cancer is :


a. site of the lesion.
b. Level of CEA.
c. Age of the patient.
d. Presence of micro calcification.
c. Presence of lymph node metastases.

19) A blood stained nipple discharge indicates:


a. Fibroadenoma.
b. Duct papilloma.
c. Duct ectasia.
d. Fat necrosis of the breast.

20) A breast abscess :


a. Can follow mumps.
b. Is caused by hemolytic streptococcus.
c. Goes through a stage of lactation mastitis.
d. Is necessarily treated by aspiration.
21) Retraction of the nipple:
a. Is always unilateral.
b. Can be caused by fat necrosis.
c. Is associated with duct ectasia.
d. Is a feature of duct papilloma .

22) The Mammographic criteria of a malignant lesion includes all of


the following except:
a. Speculated mass.
b. Retracted nipple.
c. Skin edema.
d. Macro calcification.

23) The main presentation of Paget disease of the nipple is :


a. Mass located just under the nipple.
b. Bleeding per nipple.
c. Unilateral red scaly nipple.
d. Unilateral itchy red vesicles affecting the nipple.

24) In modified radical mastectomy all of the following structures


are removed except:
a. Nipple and areola.
b. Pectoralis major muscle.
c. Axillary lymph nodes.
d. Intercosta-brachial nerve.

25) The most common site of breast cancer is the:


a. Upper medial quadrant.
b. Upper lateral quadrant.
c. Lower medial quadrant.
d. Lower lateral quadrant.

26) Conservative breast surgery is contraindicated in all of the


following except:
a. Palpable mobile axillary nodes.
b. Tumor > 4cm.
c. Inflammatory carcinoma.
d. Multiple tumors in the same breast.

27) A patient with a breast cancer that measures Scm with fixed axillary
nodes and no distant metastasis is considered:
a. T2, N2, M
b. T3, N2, M0.
c. T2, N1, M0. d.
T3, N1, M0.
CHAPTER (9)
ARTERIAL DISEASES

1) Abdomi.n. al aortic aneurysms should be repaired if their diameter is larger


than:
a.3·cm.
b.4cm.
c. 5 cm.
d. 6 cm.

2) The most common peripheral artery aneurysm is:


a. Brachial. b. Radial.
c. Popliteal. d. Tibial.

3) The compartment most commonly affected in alower leg


compartment syndrome is the:
a. Anterior compartment.
b. Lateral compartment.
c. Deep posterior compartment.
d. Superficial posterior compartment.

4) A patient who develops dizziness, drop attacks, and diplopia


with exercise most likely has:
a. Carotid stenosis.
b. Subclavian steal syndrome.
c. Coronary subclavian steal syndrome.
d. Coronary artery disease.

5)An ankle-brachial index of 0.7 :


a. Is normal.
b. Indicates an increased risk of cardiovascular events.
c. Is indicative of moderate Ischemia with rest pain.
d. Is Indicative of severe ischemia with a risk for gangrene.
6) The most common presenting symptom of acute arterial occlusion
is :
a. Pain.
b. Pallor.
c. Paraesthesia.
d. Pulselessness

FLUID ELECTROLYTES AND NUTRITION

CHOSE THE SINGLE BEST ANSWER:

1) Metabolic acidosis with a normal anion gap (AG) occurs with:

a. Diabetic acidosis. b. Renal


failure.
c. Severe diarrhea.
d. Starvation.
2) serum sodium of 129 seen in the immediate postoperative period:
a. Warrants aggressive treatment with hypertonic saline to prevent seizures. b.
Should be treated with boluses of 0.9% NaCI until corrected.
c. Is a self-limiting problem due to transient Increase in antidiuretic hormone
secretion.
d. Is due to excessive fluids given intra-operatively.
3) Which of the following is an early sign of hyperkalemia?
a. Peaked T waves. b.
Peaked P waves.
c. Wide QRS complex. d.
Peaked U waves.
4) the next most appropriate test to order in a patient with a pH of 7.1, PC02 of
40, sodium of 132,a potassium of 4.2, and a chloride of 105 is
a . Serum bicarbonate.
b. Serum magnesium. c.
Serum ethanol.
d. Serum salicylate.
5) Normal saline is:
a. 135mEq NaCI/L. b. 145
mEq NaCI/L. c. 148mEq
NaCI/L.
d. 154mEq NaCI/L.
6) Water constitutes what percentage of total body weight?
a.30-40%. c.50-60%.
b.40-50%.
d.60-70%.
7) A patient who has spasms in the hand when a blood pressure cuff is blown up
most likely has:
a. Hypercalcemia.
b. Hypocalcaemia.
c. Hypermagnesaemia. d.
Hypomagnesaemia.
8) Metabolic acidosis with a normal anion gap is found in a patient with:
a. Alcohol intoxication. b. Aspirin
ingestion.
c. Diabetic Ketoacidosis. d. Small
bowel fistula.
9) the effective osmotic pressure between the plasma and interstitial fluid
compartments is primarily controlled by :
a. Bicarbonate. b.
Chloride ion.
c. potassium ion. d.
Protein.
10) The most common fluid disorder in the surgical patient is :
a. Extracellular fluid deficit.
b. Hyperkalemia.
c. Hyponatraemia.
d. Metabolic acidosis. e. Metabolic alkalosis.
11) Symptoms and signs of extracellular fluid volume deficit include all of the
following EXCEPT:
a. Anorexia. b.
Apathy.
c. Decreased body temperature.
d. High pulse pressure.
12) The osmolarity of the extracellular fluid space is determined primarily by the
concentration of:
a. Bicarbonate. b.
Chloride ion.
c. Phosphate radicals.
d. Sodium ion.

2
13) When lactic acid is produced in response to injury, the body minimizes pH
change by:
a. Decreasing production of sodium bicarbonate in tissues.
b. Excreting carbon dioxide through the lungs.
c. Excreting lactic acid through the kidneys.
d. Metabolizing the lactic acid in the liver.
14) The simplest effective method of estimating the degree of acidosis in a patient in
shock is the measurement of:
a. Arterial pH.
b. End tidal C02 concentration. c. pH of
mixed venous blood.
d. Serum CO2 level.
15) A decrease in intracellular water can be precipitated by :
a. A decrease in sodium in extracellular fluid.
b. An increase in sodium in extracellular fluid.
c. An increase in sodium in intracellular fluid. d. An
isotonic decrease in extracellular fluid.
16) The first step in the management of acute hypercalcemia should be:
a. Correction of deficit of extracellular fluid volume.
b. Hemodialysis.
c. Administration of furosemide. d.
Administration of vitamin D.
17) Enteral nutrition :
a. Results in a reduction of infectious complications i n
critically ill patients.
b. Is more expensive than parenteral nutrition.
c. Results in faster return of bowel function in healthy patients after
gastrointestinal surgery.
d. Has a higher complication rate than parenteral nutrition.
18) Adrenocorticotropic hormone:
a. Is synthesized in the hypothalamus.
b. Is secreted in response to pain, anxiety, and injury.
c. Continues to be released in a circadian pattern In injured patients.
d. Causes the release of mineralocorticoids from the adrenal in a circadian pattern.
19) Which of the following are the most potent mediators of the inflammatory
response?
a. Corticosteroids. b.
Catecholamine .
c. Cytokines.
d. Prostaglandins.
20) The most frequent trace mineral deficiency developing in a patient receiving
parenteral nutrition is a deficiency of:
a. Calcium.
b. Chromium. c.
Magnesium. d. Zinc.

21) A primary action of aldosterone is to


a. Convert angiotensinogen to angiotensin.
b. Decrease chloride reabosrption In the renal tubule. c.
Decrease potassium secretion in the renal tubule .
d. Increase sodium reabsorption in the renal tubule.
e. Increase rennin release by the juxtaglomerular apparatus.
22) Regarding enteral nutrition :
a. Is the route of choice if the gastro intestinal tract is intact and functioning. b. Is
indicated in patients with intestinal obstruction.
c. Feeding gastrostomy reduce the risk of pulmonary aspiration. d.
Formed mainly of glucose 25%.
23) Recognized complications of central venous line insertion include all except:
b. Pneumothorax. a. Air embolism. c. Hemothorax.
d. Horner's syndrome.
24) Central venous pressure (CVP) :

a. Is affected by posture.
b. Is elevated in right ventricular failure.
c. Is normal in septic shock.
d. Accurately reflects cardiac output

25) The normal serum electrolytes in mEq/L are as follows except for:
.
a. Sodium : 135-145. b. Potassium: 3.5-4.5.
c. Calcium : 9 -11. e.Bicarbonate:
d. Chloride : 102-105.
23-27.
26) Hyponatraemia may result from any of the following except:
a. Adrenal insufficiency.
b.Cirrhosis of the liver. c.
Nephrotic syndrome. d. Diabetes
insipidus. e. Diuretic abuse.
27) Which statement among the following is wrong about hyperkalemia:
a. Results form renal failure, acidosis and over treatment with potassium salts.
b. Causes tachycardia.
c. May produce cerebral symptoms.
d. Causes peaking of the T-waves in the ECG. e. May
cause cardiac arrest.
28) Acidosis may be produced by the following except:
a. Prolonged shock. b.
Hypoventilation.
c. Uncontrolled diabetes. d. Open
heart surgery.
e. Cirrhosis of the liver.
29) Concerning metabolic acidosis, which of the following statements is untrue:
a. Occurs in diabetes, starvation, shock and anuria. b. May
be due to diarrhea and intestinal fistulas.
c. Is often associated with air hunger.
d. Causes bradycardia and hypotension.
e. Always follows cardiac arrest.
30) Alkalosis is often associated with:
a.Hypokalemia.
b. Hypocalcaemia.
c. Hypomagnesaemia. d. All of
the above.
e. None of the above.
31) Metabolic acidosis can be seen with the following except:
a. Septic peritonitis. b.
Septic shock.
c. Pyloric stenosis.
d. Diabetic coma.
32) Essential components of total parenteral nutrition include all except:
a. Nitrogen.
b. Carbohydrates.
c. Essential fatty acids.
d. Acetic acid.
33) The following electrolytes are significantly altered by changes in pH except:
a. Sodium.
b. Potassium. c.
Calcium .
d. Magnesium e.
Chloride.
34) The earliest sign of hypocalcaemia is :
a. Carpopedal spasms.
b. Positive Chvosteks' sign. c. Positive
Trousseau's sign.
d. Tingling of fingers and circumoral region. e.
Defective blood coagulation.
35) post-operative fever and tachycardia with arterial PO2 80mmHg pco2
35mmHg is most likely to be due to :

6
b.Atelectasis. a. Hypoventilation. c. Fat embolism.
d. Pulmonary oedema. e. Pulmonary embolism.
36) Concerning hypokalemia, the incorrect statement among the following is :
a. Follows loss of gastrointestinal secretions. b. May
be produced by diuretics .
c. Is often associated with acidosis.
d. Predisposes to cardiac arrhythmias.
e. Produces severe muscular weakness.
37) During the insertion of a subclavian catheter for hyper alimentation in a
patient with cancer esophagus the patient became dyspneic with a
respiratory rate of 32/min pulse rate of 120/min and drop of the B.P. to
80/60. The appropriate immediate action is :
a. Chest x-ray. b. Lung
scan.
c. Intubation and mechanical ventilation.
d. Chest tube.
e. Vasopressors.

7
38) All of the following are true regarding CVP measurement except:
a. It is equal to the end-diastolic pressure in the right ventricle. b. It
roughly corresponds to the blood volume.
c.The normal pressure is 5-10mm Hg.
d. A chest x-ray is necessary to confirm the position.
#>

39) A patient with pure metabolic acidosis can have all of the following except:
a. A pH of 7.36.
b. A p02 of 100mmHg. c. A
pC02 of 38mmHg.
d. A HC03 of 16mmol/L.
40) The causes of hypercatabolism in the surgical patient include all of the
following except:
b a. Major trauma. c. Peritonitis.
. e. None of the above.
A
c
u
t
e

p
a
n
c
r
e
a
ti
ti
s
.
d
.
A
ll
o
f
t
h
e

a
b
o
v
e
.

41) The complications of enteral nutrition includes all of the following except:
a. Pharyngitis.
b. Pulmonary aspiration.
c. Venous thrombosis. d.
Hyperglycemia.
42) The effects of malnutrition on the outcome of surgery include all of the
following except:
a. Impairment of wound healing.
b. Higher susceptibility to infection. c.
increased hospital stay.
d. All of the above.
e. None of the above.
43) Sodium is considered to be the main:
a. Extracellular anion. b. Intracellular anion .
c. Extracellular cation. d. Intracellular cation
44) The main role of the kidney in regulation of the acid-base balance is by :
a. Reabsorbing the filtered bicarbonate. b.
Reabsorbing H+ ions.
c. Excreting bicarbonate.
d. All of the above.
e. None of the above.
45) A 34year old male, a known case of ulcerative colitis, presented to the
emergency room with sever diarrhea, weakness, and his ECG showed

8
prolongation of the QT wave. This patient is suffering mainly from:
a. Hyponatraemia . b. Hypokalemia .
c. Metabolic alkalosis. d. All of the above. e. None
of the above.
46) The blood gasses of a patient revealed a pH of 7.31, PaO2 of 75 mmHg (N=80-
110), a PC02 of 54mmHg (N=36-44) and an HC03 of 28mmollL (N=22-26), this
patient is suffering from:
a. Metabolic acidosis.
b. Compensated metabolic acidosis.
c.Respiratory acidosis.
d. Compensated respiratory acidosis. .
47) The normal extracellular pH is :
a. 7.4 ± 0.03. b. 7.4 ± 0.04.
c. 7.4 ± 0.05. d. 7.4 ± 0.6.
48) To assess the nutritional state of a patient, all of the following is included
except:
a. Body weight. b. Height.
c. Forearm circumference. d. Serum albumin.
49) Complications of enteral nutrition includes all of the following except:
a. Gastro-esophageal reflux. b.
Hyperglycemia.
c. Distension colic and diarrhea.
d. Gut barrier failure.
50) postoperative fluid replacement should be based on the following, except:
a. Maintenance requirements.
b. Extra needs resulting from systemic factors (e.g. fever, burns). c. Sex
of the patient.
d. Losses from drains.
51) The main treatment of respiratory acidosis is :
a. IV sodium bicarbonate 1mEq/kg body weight.
b. Improve the ventilation. c.
Sedation.
d. Re breathe in a bag.

9
HEMOSTASIS AND SURGICAL BLEEDING
1) Which of the following is NOT one of the four major physiologic events of
hemostasis?
a. Fibrinolysis.
b. Vasodilatation.
c. Platelet plug formation.
d. Fibrin production.
2) The half-life of platelets is :
a. 2-3 days.

b. 7-10 days. c. 14-21


days.
d. 30 - 40 days.
3) The primary defect in von Willebrand's disease is :
a. Inadequate production of fibrin. b.
Excessive fibrinolysis.
c. Failure of platelet aggregation. d.
Failure of vessel constriction.
4) Bank blood is appropriate for replacing each of the following, except:
a. Factor I (fibrinogen).
b. Factor II (prothrombin).
c. Factor VII (proconvertin).
d. Factor VIII (antihemophilic factor).
5) Exsanguinating hemorrhage is most likely to follow which of the following
injuries in a previously healthy young adult?
a. Closed fracture of the femur.
b. Open fracture of the tibia and fibula.
c. Partial transection of the artery from a sharp injury. d.
Severe crush injury of the foot.
6) A prolonged bleeding time may be anticipated in patients with each of the
;
following problems, except:
a. Aspirin ingestion in the past week.
b. Classic hemophilia.
c. Qualitative platelet dysfunction. d. Von
Willebrand's disease.

1
0
7) The most common cause for a transfusion reaction is :
a. Air embolism.
b. Contaminated blood. c.

-
Human error.
d. Unusual circulating antibodies.
;-

8) Each of the following is a symptom of a hemolytic transfusion reaction except:


a. Constricting chest pain. b.
Flushing of the face.
c. Lumbar pain.
d. Syncope.
,
9) The most common clinical manifestation of a hemolytic transfusion reaction is:

a. Flank pain. b.
Jaundice.

-
c. Oliguria.
d. A shaking chill.

10) The most common fatal infectious complication of a blood transfusion is :


a. Acquired immunodeficiency syndrome. b.
Cytomegalovirus.
c. Malaria.
d. Viral hepatitis .
11) Each of the following factors requires vitamin K for its production except:
a. Factor VIII. b.
Factor X.
c. Factor IX (Christmas factor).
d. Proconvertin (factor VII).

12) The hemorrhagic disorder of true hemophilia is best corrected by transfusion-


of :
a. Banked blood.
b. Fresh blood.
c. Fresh citrated plasma. d.
Frozen plasma.
e. Cryoprecipitate.

11
13) Consumption coagulopathy due to disseminated intravascular clotting (DIC)
is treated by the following except:
a. Fresh blood transfusion,
b. Ringer's lactate infusion. c. Heparin,
d. Fibrinogen,
e. Fresh frozen plasma,

14) In acute emergency if blood has to be given immediately without cross


matching it is best to give:
a. Group 0 Rh -ve,
b. Group 0 Rh +ve, c. Group
AS Rh -ve. d. Group AS Rh
+ve. e, Group S Rh +ve.

15) Patients with DIC have all the following except:


a. Prolongation of the coagulation time. b.
Increased fibrinogen level.
c. Thrombocytosis.
d. Increased fibrin degradation products. e.
Increased bleeding time.

16) Complications of massive blood transfusion include:


a, Hypokalemia.
b.Hypothermia.
c. Thrombocytosis. d.
Hypercalcemia.

17) Haemolytic anemia:


a. Is characteristically microcytic.
b. May lead to gall stones formation.
c. Is associated with conjugated hyper bilirubinemia. d.
Causes elevated serum haptoglobin levels.
18) The following are recognized complications of blood transfusion except:
a. Citrate toxicity.
b. Hepatitis C virus transmission.
c. Hypersplenism. d.
Hyperkalemia.
e. Febrile reaction.

12
19) The following may cause a fall in the platelet count in peripheral blood except:
a. DIC. b. Massive blood transfusion. c.
Warfarin. d. Heparin.
20) Disseminated intravascular coagulation (DIC):
a. Produces rises in platelet count.
b. Is associated with decreased fibrinolysins. c. May
present with thrombosis.
d. Is best managed by whole blood transfusion.
21) Hereditary spherocytosis is characterized by all the following except:
a. Recurrent hemolytic crisis. b.
Micro spherocytosis.
c. Gall stones may be present. d.
Thrombocytopenia.
e.Increased fragility of RBCs.
22) Complications of blood transfusion includes all of the following except:
a. Transmission of HIV infection. b.
Hyperkalemia.
c. Citrate intoxication. d. All

-
of the above.
e. None of the above.
23) Primary hemostasis includes all of the following except:
a. Vasoconstriction of the bleeding vessel. b.
Formation of a platelet plug.
c. Tamponade of the bleeding by the surrounding tissues. d. Fibrin

-
deposition.
24) Which of the following is not a type of hemorrhage as related to the timing of
surgery:

a. Primary. b. Secondary.

13
d. Reactionary. c. Tertiary.
25) A patient with massive bleeding from a peptic ulcer is expected to have all of
the following except:

a. Blood pressure of 90/60 mmHg.


b. Pulse of 120/ M.
c. Respiratory rate of 26.
d. Temperature of 37.5*C.

14
26) In cases of hemorrhage, blood transfusion is needed in which of these
classes:
a. Class I.
b. Class II.
c.Class III.
d. None of the above.
27) The blood volume of the newborn is :
a.20-40ml/kg.
b. 40-60ml/kg.
c. 80-85ml/kg.
d. 100-120ml/kg.
SHOCK
1) A patient with acute respiratory distress syndrome (ARDS) exhibits all of the
following findings except:
a. Decreased pulmonary compliance.
b. Hypercarbia.
c. Hypoxia.
d. Patchy infiltrates in chest X-ray.
2) Neurogenic shock is characterized by the presence of :
a. Cool moist skin.
b. Increased cardiac output.
c.Decreased peripheral vascular resistance.
d. Decreased blood volume.
3) A patient has a blood pressure of 70/50 mmHg and a serum lactate level of
30mg/100mL (normal: 6 to 16). His cardiac output is 1.9Umin, and his central venous
pressure is 2 cm H20. The most likely diagnosis is :
a. Congestive heart failure. b. b.Cardiac
tamponade.
c.Hypovolemic shock .
d. Septic shock.
4) In septic shock:
a.Causes may include peritonitis and burns.
b. Metabolic alkalosis may develop.
c. Nitrous oxide may be used for treatment of vasoconstriction.
d. Caused by gram negative exotoxins.
5) Grade III hemorrhagic shock:
a.It occurs after the sudden loss of 1.5-2 liters blood.
b. Is associated with a near normal heart rate.
c. Systolic blood pressure is normal.
.;
d. The respiratory rate is usually below 20 breathl min.
6) Recognized complications of septic shock include all except:
a. Warm extremities. b. Hypothermia.
_d., Polyuria.
7) A rapid loss of 25% of the blood will :
a. Decrease ADH levels.
b. Produces hyperkalemia.
c.Activate the rennin-angiotensin system. d.
Inhibits thirst sensation.
8) All of the followings may be present in septic shock except:
a. Mental confusion. b.
Tachycardia. C.yolyuria.
d. Jaundice.
e. Fever.
9) Haemorrhage:
a. Is arterial if bright red and spurting in time with pulse.
b. Is reactionary, if occurring 4 hours after injury. c. Is
secondary if ccurring 24 hours after Injury. d.
Bradycardia is a major sign.
10) In management of hypovolemic shock:
a. Morphine is a good drug in patients with head trauma.
b.Febrile reaction to blood transfusion is usually due to pyrogens.
c. Vasoconstrictor drugs should be given.
d. 100% oxygen administration is given in grade II hemorrhage.

11) The essential pathophysiological disorder in all types of shock is :


a. Hypotension. b.
Tachycardia.
c. Viscero-cutaneous vasoconstriction .
d.impaired tissue perfusion. e.
Reduced blood volume.

12) The most important factor in the treatment of a shocked patient is restoration
of:
a. Arterial blood pressure.
b. Central venous pressure. c.
Circulating blood volume.
d. Cardiac output.
e. Tissue perfusion.
----

13) The best emergency intravenous infusion in hypovolemic shock is :


a. Normal saline.
b. Hypertonic saline.
c Lactated Ringer's solution. d. 5%
glucose.
e. Hypertonic glucose.

14) The hyperdynamic phase of septic shock (septicemia) is characterized by the


following signs except:
a. Hyperventilation. b.
Hypotension.
c. Tachycardia.
d.Pale cold extremities. e.
Oliguria.

15) Septic shock responds best to :


a. Massive antibiotics. b.
Intravenous infusion. c.
Corticosteroids.
d.Drainage of septic collections. e.
Vasopressors and inotropics.

16) The best guide for the required blood transfusion in haemorrhagic shock is the:
a. Arterial blood pressure. b. Pulse
rate.
c. Haematocrit.
d.Central venous pressure (CVP). e. Urine
output per minute.

17) In the adult respiratory distress syndrome (ARDS) the most important
therapeutic measure is :
a. Oxygen inhalation.
b. Mechanical ventilation.
c. Large doses of IV steroids.
d. Massive antibiotics.
e. Intravenous diuretics.
18) An elderly patient with a history of congestive heart failure developed a
perforated peptic ulcer. He was taken to the operating room after the
administration of 500ml of 5% glucose over 6 hours. Upon induction of
anaesthesia, the B.P. dropped to 80mmHg.The most probable cause is :
a. Cardioqenlc shock.
b. Septic shock .
c. Neurogenic shock.
d. Anaphylactic shock.
e. Internal haemorrhage.
19) The followings are clinical manifestations of multi organ system failure except:
a. Hypocapnea.
b. Metabolic acidosis.
c. Gastro intestinal hemorrhage.
d. Adult respiratory distress syndrome. e. Acute
renal failure.
20) Physiological responses to hemorrhage include all except:
a. Peripheral vasoconstriction. b.
Tachycardia.
c. Increased ADH.
d.Hemoconcentration.
21) Post operative day 5, following sigmoid colectomy for cancer sigmoid, a
patient developed tachycardia, warm peripheries, WBC = 17000.Hb% = 8gm/dL.
The underlying cause is :
b. Hypovolemic shock. a. Anaphylactic shock. c. Cardiogenic shock.
d. Septic shock.
22) The best solution used for resuscitation of a patient with hypovolemic shock
is:
b. a.Lactated Ringer solution. c.
Glucose Glucose 25%.
5%. d.
Kadalex.
23) Management of a patient with septic shock includes all of the following except:
a. Fluid replacement.
b. Oxygen administration. c.
Renal support.
d. Delay management of the septic focus till the patient recovers.
24) All of the following can be used to resuscitate a shocked patient except:
b. Dextran. a. Lactated Ringer's solution. c. Blood
transfusion .
d. Glucose 5%.
25) In cases of shock, inotropic agents are used:
a. In all cases of anaphylactic shock.
b.When the vital signs fail to improve inspite of adequate fluid replacement. c. In the
initial phases of septic shock.
d. In all cases of neurogenic shock.
SURGICAL INFECTIONS. ANTIBIOTICS WOUNDS
AND WOUND HEALING

1) The proliferative phase of wound healing occurs how long after the injury?
a. 1 day. b. 2 days.
c. 7 days. d. 14 days.
2) The tensile strength of a wound reaches normal (pre-injury) levels:
a. 10 days after injury. b. 3 months after injury. c. 1 year after injury.
d.Never.
3) Steroids impair wound healing by :
a. Decreasing angiogenesis and macrophage migration.
b. Decreasing platelet plug integrity.
c. Increasing release of lysosomal enzymes.
d. Increasing fibrinolysis.
4) The treatment of choice for keloids is :
a. Excision alone.
b. Excision with adjuvant therapy (e.g. radiation).
c. Pressure treatment.
d. Intralesional injection of steroids.
5) The major cause of impaired wound healing is:
a. Anemia.
b. Diabetes mellitus.
c. Local tissue infection.
d. Malnutrition.
6) Which of the following antibiotics is the best for treatment of infections
caused by Staphylococci:
a. Aminoglycosides.
b. Quinolones.
c. Amoxycillin-Clavulanic acid.
d. Metronidazole.
7) A diabetic male patient presented with pain in the back of the neck and fever
of 1 week duration. On examination, the back of the neck showed an indurated
red area of 7x5cm diameter with a soft center and the surface showed 4
sinuses. The treatment of this case includes all of the following except:
a. Control of the diabetes.
b. Ampicillin-Sulbactam antibiotic (Sensitive towards staphylococci).
c. Glycerine magnesia dressings.
d. Excision of sloughs.
e. Isolation of the case.
8) A 17 year old boy fell from his bicycle in a dusty street 12 hours ago. He had
a 2cm wound on his right knee. On interrogation of his parents we were told
that his last tetanus vaccination was when he was 6 years old. The best
treatment of this case is all of the following except:
a. Clean the wound.
b. Single dose of 0.5ml Tetanus Toxoid 1M.
c. 250 units Tetanus immunoglobulin.
d. Delayed primary closure of the wound.
9) The treatment of necrotizing fascitis of the thigh includes all of the following
except:
a. Rest of the patient and the part.
b. Elevation.
c. Penicillin and an Aminoglycoside antibiotic.
d.Blood transfusion.
e. Aspiration of the pus and continuous Irrigation with betadine solution.

10) A 47 year old male with extensive sepsis in the form of necrotizing fasciitis
will be expected to have:
a. Carpopedal spasm.
b. Kusmaul's respiration
c. Normal or elevated pC02.
d. Restlessness and cyanosis.
11) A carbuncle is a :
a. Mixed staphylococcal and streptococcal infection of sweat glands.
b. Infective gangrene of subcutaneous tissues usually by staphylococcus aureus in a
diabetic patient.
c. Rapidly spreading non-suppurative Infection of lymphatics of the skin caused by
strains of Hemolytic streptococci.
d. An invasive non-suppurative infection of the loose connective tissue.
12) The most appropriate prophylactic antibiotic to use in a patient undergoing
surgery for a small bowel obstruction is :
a. Cephazolin. b. Ceftriaxone.
c. Ampicilln-sulbactam. d. Aminoglycoside.
13) A patient with a localized wound infection after surgery should be treated with:
a. Antibiotics and warm soaks to the wound.
b. Antibiotics alone.
c. Antibiotics and opening the wound.
d.lncision and drainage alone.
14) Infections that require operative treatment include all of the following, except:
a. Empyema.
b. Infected ascites.
c. Necrotizing fasciitis of the thigh.
d. Vascular graft infection.
15) Which of the following is the most commonly acquired infection in
hospitalized surgical patients?
a. Lower gastrointestinal tract.
b. Lower respiratory tract.
c. Nasopharynx.
d. Surgical wound.
16) Which of the following is the most effective way to prevent post-operative
wound infection in an obese patient after an open appendectomy?
a. Leaving the subcutaneous tissue and skin open to heal by secondary intention.
b. Closing the wound over a rubber drain.
c. Closing the wound with a closed suction drain.
d. Closing the wound with multiple sutures in the subcutaneous tissue.
17) Staphylococcus aureus produces each of the following, except:
a. Cell wall peptidoglycan.
b. Enterotoxin.
c. Epidermolytic toxin.
d. Neuroexotoxin.
18) An exotoxin plays an important part in the pathogenicity of infection with
each of the following, except:

a. Clostridium botulinum.
b. Clostridium tetani.
c. Escherichia coli.
d.Staphylococcus aureus.

-
19) The drug of choice for Colstridial myonecrosis is:
a. Penicillin G.
b. Ampicillin.
c. Amikacin.
d. Cephalosporin.
20) Cefuroxime is a :
a. 1st generation cephalosporin.
b. 2nd generation cephalosporin.
c. 3rd generation cephalosporin.
d. 4th generation cephalosporin.
21) All of the followings can delay wound healing except:
a. Malnutrition.
b. Infections.
c. Anemia.
d. Advanced age.
e. Hypocalcaemia.

22) Regarding surgical wound infections:


a. Appendicectomy for appendicitis is a "clean-contammated" wound.
b. An elective uncomplicated right hemicolectomy is a "contammated" wound.
c.Compound fracture is a "contaminated" wound.
d. Inguinal hernia repair is a clean-contaminated wound.
23) Staphylococcus aureus is usually responsible for abscess formation in
the following site:

a. Breast.
b. Appendix.
c. Gall bladder.
d. Perianal.
24) Factors predisposing to wound infection include:

25
a.Vitamin
C
deficiency

b.anemia

c.ischemia
of the
wound

d. Hypo-
albuminem
ia.

26
25) Regarding skin infections and abscesses:
a. Breast abscess is most commonly caused by staph. aureus.
b. A fruncle is an Infection of a sweat gland.
c. Erysipelas is caused by clostridia.
d. Gas gangrene is cause by streptococcus pyogens.
26) About wound management, one statement only is correct:
a. Debridement is the removal of dead tissues and foreign bodies.
b. Debridement is necessary for incised wounds to get a good drainage.
c. Nerve repair in lacerated crushed wounds is of first priority.
d. In fresh clean-cut wounds mark the cut tendons for delayed treatment.
e. Deep fascia closure after muscle debndement prevents infection.
27) Cellulitis :
a. Is a non suppurative inflammation of the subcutaneous tissues.
b. Caused by staph. aureus infection.
c. Surgical drainage is the principle treatment.
d. Highly infectious.
28) Unfavorable factors in wound healing are the following except:

27
b. Presence of foreign a. Infection.
material. c. Suture under tension.
d. Wound drainage.
e.wound hematoma .

29) The following statements are correct except:


a. Hypo proteinaemia Impairs wound healing.
b. Immobilization of the injured part helps wound healing.
c. Corticosteroids impair wound healing.
d. Irradiation of a wound promotes healing.
e. Cancer chemotherapy delays wound healing.

30) Concerning post-operative wound infection, which of the following statement


is wrong:
a. Is most often due to wound hematoma.
b.Can always be prevented by prophylactic antibiotics.
c. Frequently takes the form of an abscess.
d. May precipitate fatal secondary haemorrhage.
e. May end fatally by septic shock.

28
31) The treatment of acute tetanus include the following except:
a. Administration of antitoxin. b. High-dose penicillin.
c. Sedatives, muscle relaxants and mechanical ventilation.
d. Suturing the wound.
e. Nutritional and nursing care.
32) Most hands infections are caused by :
a. Streptococci.
b. Staphylococci.
c. E. Coli.
d. Anaerobes.
e. Pseudomonas.
33) Most hospital acquired infections involve the:
a. Surgical wound.
b. Intravenous sites.

c. Respiratory tract.

d. Urinary tract.
e. Deep veins of the leg.
34) Which of the following are types of wound healing:
a. Healing by primary intention.
b. Healing by secondary intention.
c. Healing by tertiary intention.
d. All of the above.
e. None of the above.
35) Treatment of incised wounds of less than 6 hours duration includes:
a. Primary closure.
b. Delayed primary closure.
c. Debridement.
d. None of the above.
36) Which of the following are components of wound healing?

27
27
a. Wou
nd
cont
racti
on.
b. Gra
nula
tion
tissu
e
form
atio
n.
c. Epithelial
ization.

d. All of the
above.
e. None of
the above.

28
28
37) Granulation tissue refers to :
a. Platelets and fibrin.
b. Inflammatory cells and damaged tissues.
c. Budding capillaries and fibroblasts.
d. Slough and blood.
38) Debridement means :
a. Excising 1mm of skin from the edge of the wound.
b. Amputation.
c. Excision of devitalized tissue.
d. Suturing of the wound.
39) The earliest manifestations of surgical wound infection are:
a. Swollen, tender and red wound.
b. Fluctuant areas and/or crepitus.
c. Wound pain and fever.
d. Discharge from the wound.
40) Regarding the dangerous area of the face, all of the following are true except:
a. The outer canthus forms one of its boundaries.
b. The main risk is cavernous sinus thrombosis.
c. The angular vein communicates with the ophthalmic veins.
d. The anterior facial vein communicates through the pterygoid venous plexus which
communicates with the cavernous sinus by an emissary vein that enters
the skull through the foramen rotundum.

41) Regarding lacerated wounds, all of the following are true except:
a. Devitalized skin edges are removed.
b. Deep fascia is opened widely.
c. Dead muscles are excised.
d. Cut nerves are sutured primarily.

29
29
MULTIPLE TRAUMA AND BURN

1) A patient with a palpable carotid pulse but no femoral pulses has an


approximate systolic blood pressure of :
a. 60 mmHg.
b. 70 mmHg.
c. 80 mmHg .
d. 90 mmHg.
2) What percentage of blood volume must be lost in healthy patients before
hypotension occurs?

31
31
3) A patient with spontaneous eye opening, who is confused and localizes pain
has a Glasgow Coma Score of :
a. 9. b. 11.
c. 13.
- d. 15.
4) A 19 year-old man is admitted to the emergency department with a stab wound
just below the right inguinal ligament. There is profuse bleeding from the wound, and
he is in shock. The first step in local wound control should be to:
a. Apply compression of the bleeding vessel with a gloved finger.
b. Place a tourniquet on the right thigh above the wound.
c. Use clamps and ligatures to control the bleeding.
d. Wrap the wound and upper thigh in a bulky pressure dressing.
5) A 20 year-old man has an injury to the posterior urethra. After appropriate
initial management, the most common late complication is :
a. Ascending unnary tract infection.
b. Retrograde ejaculation.
c. Sterility.
d.Urethral stricture.

6) An early sign of anterior compartment syndrome in the calf is :


a. Absence of pulses in the foot.
b. Firm calf muscles.
c. Foot drop.
d. Paraesthesia between the great and second toes.

31
7)What percentage burn does a patient have who has suffered burns to one leg
(circumferential), one arm (circumferential), and anterior trunk?
a.18%.
b.27%.
c.36%.
d.45%.
8)The appropriate management of a deep partial-thickness burn is :
a. Early excision and grafting.
b. Surgical debridement and dressings.
c. Dressings only.
d. Observation.
9)Major burns:
a. Result in a decrease in the metabolic rate.
b. Are associated with cardiogenic shock.

-
c. Can cause adult respiratory distress syndrome.
d. To chest do not require escharotomy.
10) The following nerves are at risk with the operation described:
a. External laryngeal nerve-thyroidectomy.
b. Ilio-inguinal nerve - hydrocele repair.
c. Sural nerve - femoro - popliteal by pass.
d. Facial nerve - submandibular gland surgery.
11) Partial thickness burn :
a. Have a pale white appearance.
b. Are painless and Insensitive.
c. Are sensitive to air.
d. Commonly result from contact with concentrated chemicals.
12) Complications of major burn include all except:
a.DIC.
b. Myocardial suppression.
c. Septic shock.
d. Oliguria.

-
e. Cushing's ulcer of the duodenum.
13) Gastro intestinal complications of major burns include all except:
a. Paralytic
. ileus
b. Acute
gastric
dilatation

c. Curling's ulcer.
d. Hepatic
dysfunction.
14) What is the first priority in the treatment of a patient who had a burn of the face
due to inhalation injury in a closed space?
a. Covering the burnt area with sulphadiazine cream.
b. Insertion of wide bore canula + start I.V resuscitation.
c. I.V antibiotics.
d.Endotracheal intubation.
e. Protection of the eyes by an ointment.

15) The management of a patient with a burn of 30% of the trunk includes all except:
a. I.V ringer's lactate solution.
b. Haematocrit readings.
c. Monitoring urine output.
c. Urgent tracheostomy.

16) The primary survey of a seriously injured multi-trauma patient include:


a. Airway maintenance with cervical spine control.
b. Abdominal assessment.
c. Rectal examination.
d. Musculo-skeletal assessment.
e. Diagnostic peritoneal lavage.

17) Patients with major burns are liable to develop the following except:
a. Acute peptic ulceration.
b. Paralytic ileus.
c. Cerebral oedema.
d. Septicemia.
e. Mesenteric vascular occlusion.

18) Metabolic changes after burn are due to the following except:
a. The endocrine responses to injury.

-
b. Local fluid loss into the burnt area.
c. Reduced heat loss.
d. Increased insensible water loss.
e. Bacterial infection.
19) The most urgent measure in the management of a severely injured patient
in the reception room is:
a. Control of active bleeding.
b. Taking blood sample for grouping and cross matching.
c. Establishing an intravenous line.

-
d. Securing clear airway and adequate pulmonary ventilation.
e. Providing tetanus prophylaxis.

20) The most serious risk of surgery on a diabetic patient is :


a. Urinary tract infection.
b. Chest infection.
c. Ketoacidosis.
d. Anuria.
e. Wound sepsis.

21) Post-operative pulmonary complications are most often due to :


a. Hypercapnea.
b. Emphysema.
c. Atelectasis.
d. Thromboembolism.
e. Chloride.

22) The possibility of cervical spine injury should be considered in which of


the following conditions:
a. Bony abnormalities over the cervical spine.
b. the presence of maxillofacial trauma.
c. Altered level of consciousness.
d. Multisystem trauma.
e. All of the above.

23) A burned victim has involvement of his face, anterior part of the neck the
anterior surface of the chest and abdomen and the anterior parts of both upper
limbs will have a :
a. 25% burn.
b. 31.5% burn.
c. 36% burn.
d. 40.5% burn.

33
24) This same victim is considered to have:
a. A minor burn.

-
b. An Intermediate burn.
c. A major burn.
d. An extensive burn

25) A 62Kg 18 year old female sustained a burn of all her back and the backs of
both lower limbs. This is considered which extent of burns:
a. Minor.
b. Intermediate.
c. Major.
d. Superficial.

26) This same patient will require this volume of saline (in ml) during the first
24 hours according to Evan's formula:
a.3233.

b. 1894.
c.2232.
d.2408.

27) The body mass index of a 105Kg patient who has a height of 168cm is :

34
a.34.
3.

35
b. 37.2.
c.32.7.
d.24.7.

28) The possibility of cervical spine injury should be considered in which of the following
conditions:
a. Bony abnormalities over the cervical spine.
b. The presence of maxillofacial trauma.
c. Altered level of consciousness. d. Multisystem trauma.
e. All of the above.

29) Which of the following modalities cannot be used to obtaln and maintain a patent airway
in a patient with suspected injury of the cervical spine?
a. Naso-pharyngeal tube.

-
b. Endotracheal intubation.
c. Tracheostomy.
d. Cricothyroidotomy.

1. Which of the following is not a prognostic factor of the severity of acute pancreatitis?

a. Serum amylase.

b. Age of the patient.

c. Blood sugar level.

d. Base excess.

e. Total leucocytic count.

2. The following statements about amoebic liver abscess are all correct except:

a. Infection comes from amoebic colitis.

b. It produces chocolate-coloured pus.

c. May produce pleural effusion.

d. Primary line of treatment is incision and drainage.

e. Differential diagnosis includes hepatocellular carcinoma in black Africans

3. The following statements about gall bladder stones are all correct except:

a. Most of these stones are radio-opaque.

b. In many cases these stones produce no symptoms.

c. Gall stones are present in most cases of gall bladder cancer.


36
d. Ultrasonography is the preferred imaging study.

e. Removal of the gall balder is the standard treatment of symptomatic cases

4. The following statements about acute cholecystitis are all correct except:

a. The great majority of cases are caused by stones.

b. The condition is more serious in diabetics.

c. Differential diagnosis includes acute pancreatitis.

d. The main presentation is jaundice.

e. Initial treatment may be conservative.

5. Complications of common bile duct stones include the following except:

a. Jaundice.

b. Cancer of the head of pancreas.

c. Acute cholangitis.

d. Acute pancreatitis.

e. Secondary biliary cirrhosis.

6. Risk factors for brown gallstone formation include all of the following except:

a. Ascaris lumbricoides.

b. Clonorchis sinensis.

c. Rapid weight loss.

d. Periampullary diverticulum.

e. Choledochal cyst.

7. During laparoscopic cholecystectomy for symptomatic gallbladder stones in a 38 year-old-


female patient, a bleeding occurred from a spurter while the surgeon is trying to dissect the
cystic duct , the field of surgery is obscured, the next management step is:

a. Conversion to open cholecystectomy.

b. Suction-irrigation and control of the bleeder under direct vision.

37
c. Quick clipping of the bleeder.

d. Coagulation of the bleeder.

e. Pringle’s manoeuvre.

8. All the following statements about verticular disease of the colon are correct except:

a. A low fiber diet is a predisposing factor.

b. There is hypertrophy of the circular muscular coat.

c. The sigmoid colon is the commonest site.

d. It is premalignant.

e. It may lead to massive colonic bleeding.

9. Regarding septic shock all of the following are true, except:

a. Cold shock has a better outcome than warm shock.

b. The gastrointestinal tract plays a major role in the multi-organ failure syndrome.

c. Is caused by Candida in around 10% of cases.

d. Hyperventilation is an early sign of pre-shock septic state.

e. The respiratory tract is the primary focus in ICU’s in the majority of cases.

10. Which of the following is not true about neurogenic shock?

a. It may follow spinal fractures.

b. There is tachycardia.

c. The extremities are warm.

d. There is bradycardia and hypotension.

e. Vasopressors may be useful.

11. All of the following can be treated conservatively in a stable trauma patient except:

a. Lung contusion.

b. Liver laceration.

38
c. Kidney laceration.

d. Splenic hematoma.

e. Perforation of the small intestine

12. Mediastinal widening after deceleration type chest trauma is a pathognomonic sign of:

a. Cardiac injury.

b. Aortic rupture.

c. Bronchial rupture.

d. Thymic injury.

e. Sternal frature.

13. The initial maneuver to establish an airway in a patient with multiple injuries is:

a. Oropharyngeal airway.

b. Uncuffed endo-tracheal tube.

c. Suctioning foreign debris and lifting up the mandible.

d. Cuffed endo-tracheal tube.

e. Tracheostomy.

14. For which of the following organ injuries is diagnostic peritoneal lavage (DPL) least likely to
be helpful?

a. Pancreas

b. Small intestine.

c. Spleen.

d. Sigmoid colon.

e. Liver.

15. The first priority in the management of facio-maxillary injuries is to:

a. Rule out fracture of the cervical spine.

b. Chest x-ray.

39
c. Ensure a patent airway.

d. Stop the bleeding.

e. I.V. fluid resuscitation.

16. A 16-year-old male driver was involved in car accident. The patient was hemodynamically
unstable upon extrication and was transported to a trauma center. While in transport the
patient became severely hypotensive, unresponsive, with impending respiratory distress.
Which of the following is considered an immediate life-threatening injury in this patient?

a. Pneumothorax secondary to rib fractures.

b. Aortic intimal tear.

c. Diaphragmatic rupture.

d. Tension pneumothorax.

e. Myocardial contusion.

17. One of the following is not an absorbable suture:

a. Catgut.

b. Polydioxanone.

c. Polyamide.

d. Polyglycolic acid.

e. Polygalactin.

18. The proper treatment of post-operative hyponatraemia following an uneventful operation:

a. Hemodialysis.

b. I.V. frusemide.

c. Restriction of free water.

40
d. I.V. hypertonic saline.

e. I.V. mannitol.

19. The best time for surgery on uncomplicated inguinal hernia in infants is:

a. As soon as possible.

b. At the age of one year.

c. At the age of two years.

d. Pre-school age.

e. When the baby is 10 Kg or more.

20. Which statement of the following is true regarding a strangulated hernia?

a. Arterial obstruction precedes venous obstruction.

b. It is rare in paraumbilical hernias

c. It is not necessarily accompanied by intestinal obstruction.

d. Manual reduction is successful in most cases.

e. Absence of gangrenous bowel will obviate the risk of future stricture

21. The most frequent cause of post-operative pyrexia within 48 hrs of surgery is:

a. Subphrenic abscess.

b. Acute parotitis.

c. Urinary tract infection.

d. Atelectasis.

e. Blood transfusion.

22. Which of the following will immediately delay or cancel an elective surgical case if not
obtained appropriately preoperatively?

a. CBC.

b. Urine analysis.

c. CXR.

d. Informed consent.

41
e. ECG.

23. Which of the following is a valid reason to delay an elective operation?

a. Hypertension.

b. Hyperlipidemia.

c. Cigarette smoking.

d. Coronary artery bypass surgery 6 months ago.

e. Jugular venous distension.

24. Of the following which is the most effective way in preventing surgical infection:

a. Surgical technique.

b. Skin preparation.

c. Antibiotic prophylaxis.

d. Bowel preparation.

e. Drains and irrigation.

25. All the following statements about exotoxin are true except:

a. Most are polypeptides.

b. Tetanous toxoid is an example of an exotoxin.

c. Exotoxins are more heat labile than endotoxins.

d. Botulinum exotoxin is an example of an exotoxins.

e. The lipopolysaccharide of the cell wall of gram-negative bacteria is an example of an


exotoxins.

26. Which of the following is not true about systemic inflammatory response (SIRS):

a. It may be caused by peritonitis, burns and acute pancreatitis.

b. The white leucocytic count may be reduced below 4000.


42
c. There is metabolic acidosis.

d. The temp may drop to 36 ºC.

e. There is reduced output of catecholamines.

27. Which of the following is an early manifestation of sepsis:

a. Decreased cardiac output.

b. Hypoglycaemia.

c. Cutaneous vasodilatation.

d. Increased arteriovenous O2 difference.

e. Respiratory acidosis.

28. Which of the following is true about gas gangrene:

a. The commonest responsible organism is clostridium difficile.

b. The infection is mainly in the subcutaneous tissues.

c. Anti-gas gangrene serum is the most important item of treatment.

d. The disease is characterized by rigidity and muscle spasms.

e. The disease usually follows lacerations involving the gluteal area and thigh.

29. Which of the following is not true about gas gangrene:

a. It is mainly caused by Cl. welchii.

b. There is usually crepitus and colour changes in the affected area.

c. The main effects are due to bacterial endotoxin.

d. Antigas gangrene serum is no more used in the treatment.

e. Debridement of the affected tissues is vital in the treatment.

30. Factors that make lacerated wounds more liable to infection include the following except:

a. Presence of foreign bodies.

43
b. Ischaemic or dead tissues.

c. Suture of the wound under tension.

d. Leaving the wound open after debridement

e. .The use of internal fixation for accompanying fractures.

31. The main source of staphylococci in the body is:

a. The urinary tract.

b. The stomach, duodenum, and upper jejunum.

c. The colon.

d. The nasal cavity and skin.

e. None of the above.

32. The following is correct about erysipelas except:

a. Caused by methicillin resistant Staph. aureus (MRSA).

b. Sensitive to penicillin.

c. Infectious.

d. Spreading infection with no pus formation.

e. Rose pink skin patches with vesicles at the edge.

33. In pseudomembranous enterocolitis the causing organism is:

a. Clostridium perfringens.

b. Clostridium septicum.

c. Clostridium difficle.

d. Clostridium tetani.

e. Clostridium oedematiens.

34. All are factors that predispose to wound infection, except:

a. Inadequate hemostasis.
44
b. Prolonged operation.

c. Diabetes.

d. Anemia.

e. Malnutrition.

35. Regarding necrotizing fasciitis:

a. Skin manifestations may be totally absent.

b. It is commonly a single microbial infection.

c. The muscles are usually involved.

d. Broad spectrum antibiotics are the most essential measure of treatment.

e. It commonly occurs in previously healthy people.

36. What is most important in the management of a lacerated contaminated wound?

a. Removal of foreign and dead tissues.

b. Local antibiotics powder.

c. Insertion of a drain.

d. Skin graft.

e. Hyperbaric O2.

37. Which of the following is most important in the treatment of necrotizing fascitis?

a. Wide surgical debridement.

b. Hyperbaric O2.

c. Antitaxin.

d. Antifungal agents.

e. Immunoglobulins.

38. Which of the following operations is a clean contaminated operation?

45
a. Inguinal herniectomy

b. Colectomy for a prepared colon.

c. Perforated appendicitis.

d. Thyroidectomy.

e. Drainage of a pericolic abscess.

39. Risk factors for breast cancer include the following except:

a. Prior breast cancer.

b. Mammary duct ectasia.

c. Breast cancer has affected a sister.

d. Early menarche and late menopause.

e. In situ lobular or duct carcinoma

40. Haematogenous spread of breast cancer may produce the following except:

a. Pathological fractures.

b. Brain metastases.

c. Pleural effusion.

d. Jaundice.

e. Hypocalcemia.

41. The following statements about early breast cancer are all correct except:

a. Means a T2N1M0 tumour or less.

b. Means that cure is possible.

c. Means that microscopic metastases are not present.

d. Primary treatment is by surgery.

e. Adjuvant chemotherapy is indicated in the presence of axillary node deposits.

42. All the following statements about follicular thyroid cancer are true except:

a. It present at a later age than papillary carcinoma.

b. It disseminates via hematogenous route.


46
c. It is less common than papillary carcinoma.

d. It is frequently multicentric.

e. Bone is a favorite metastatic site.

43. In a patient with head injury, which of the following can be a cause of
hypovolaemic shock?

a. Extradural haematoma.

b. Subdural haematoma.

c. Scalp lacerations.

d. Subarachnoid haemorrhage.

e. Intracerebral haematoma.

44. Which of the following is true about femoral hernia?

a. It is more common in males.

b. Strangulation is infrequent.

c. It is lateral to the femoral vein.

d. Surgery should always be recommended as treatment.

e. It is more frequent than inguinal hernia in females.

45. Which of the following is true about Meckel's diverticulum?

a. It is a false diverticulum.

b. It is present near the jejunoileal junction.

c. It should always be excised once diagnosed.

d. It is a common cause of bleeding per rectum in children.

e. Incidence of complications is high.

46. Which of the following is not true about anal fissure?

a. The commonest presentation is fresh bleeding per rectum.

b. If situated in a lateral position, there is usually a specific pathology.

c. The differential diagnosis includes carcinoma of the anal canal.


47
d. It may lead to pruritis ani.

e. Glycerine trinitrate may be used for treatment.

47. Which of the following is a common cause for massive lower gastrointestinal
bleeding?

a. Crohn’s colitis.

b. Rectal polyp.

c. Angiodysplasia of the colon.

d. Carcinoma of the caecum.

e. Solitary rectal ulcer.

48. During conservative treatment of a patient with adhesive intestinal obstruction,


which of the following is an indication to proceed to surgery?

a. Nasogastric suction of more than 2 L/24 hours.

b. Exaggerated bowel sounds.

c. Abdominal rigidity.

d. Abdominal distension.

e. Serum K level below 2.5 mEq/L.

49. The commonest organisms responsible for post-appendicectomy wound


infection are:

a. Clostridia.

b. Staph. albus.

c. Gram negative bacilli + bacteriodes.

d. Strept. viridans.

e. Pseudomonas aeroginosa.

50. The most common etiology for acute osteomyelitis in children is?

a. Hematogenous inoculation.

b. Poor immune defense mechanisms.

48
c. Direct inoculation.

d. Poor hygiene.

e. Following of a fracture.
51. Thromboembolism after pelvic surgery is usually from the veins
a) iliac b) Calf
c) Femoral d) Pelvic
52. An obese patient develops acute oedematous lower limb following a Pelvic
surgery. Deep vein thrombosis is suspected . The most useful investigation in
this case would be
a) Doppler imaging b) Fibrinogen uptake
c) Venography d) Plethysmography
53. Which of these is not a risk factor for thromboembolism
a)Myocardial infarction
b)Hypertension
c)Estrogen therapy
d)Superficial thrmbophlebitis
54. Most common cause of death in patients with
Burger’s disease is
a) Gangrena b) Pulmonary embolism
c) Myocardial infarction d) Carcinoma lung
55. Which of the following best responds to sympathectomy
a) Burger’s disease b) Hyperhydrosis
c) Raynaud’s disease d) Acrocyanosis
56. Intermittent claudication at the level of the hip indicates
a)Popliteal artery occlusion
b)Bilateral iliac artery occlusion
c)Common femoral occlusion
d)superficial femoral artery occlusion
57. The artery commonly involved in cirsoid aneurysm is
a) Occipital b) Superficial temporal
c) Internal carotid d) External carotid
58. Plusating tumours include all except
a)Bone sarcoma
b)Osteoclastoma
c)Secondaries from hyper nephromas
d)Secondary from prostate
59. Preferred material for femoro popliteal bypass –
a) Dacron b) PTFE
c) Saphenous vein d) Gortex
49
60. Most common cause of aneurysm of abdominal aorta is
a) Trauma b) Atherosclerosis
c) Syphilis d) Cystic medial necrosis
61. The most common complication of an aortic aneurysm size 8 cm is
a) Rupture b) Intramural thrombosis
c) Embolism d) Calcification
62. Management of a cause of iliac artery embolism requires
a)Embolectomy
b)Injection of vasodilators
c)Hypotensive therapy
d)Sympathectomy
63. In the abdomen, aneurysms of the ………. commonly occur next only to the
aorta.
a)Internal iliac artery
b)External iliac artery
c)Splenic artery
d)Inferior mesentric artery
64. In extraperitoneal approach, to left sympathectomy the following may be injured

a) Ureter b) Gonadal vessels
c) A+B d) IVC
65. Commonest site of throboangitis obliterans is –
a) Femoral artery b) Popiteal artery
c) iliac artery d) Pelvic vessels
66. Treatment of acute femoral embolus is
a)Warfarin
b)Heparin
c)Immediate embolectomy
d)Embolectomy after 5 days bed rest
67. Ganglion which is spared in Lumbar sympathetomy is
a) Ll b) L2
c) L3 d) L4
68. Vessels most commonly involved in thrombo angitis obliterans
a)ilio-femoral
b)Aorto-iliac
c)Femora popliteal
d)Anterior and Posterior tibial
69. The Following are used in treatment of Buergers disease except
a) Trental b) Anticoagulation
c) Sympathectomy d) Antiplatelets
50
70. Buerger’s disease affects all except
a) Small arteries b) Small veins
c) Medium -size arteries d) Multiparity
71. A 45-year-old male having a long history of cigarette smoking presented with
gangrene of left foot. An amputation of the left foot was done. Representative
sections from the specimen revealed presence of arterial thrombus with
neutrophilic infiltrate in the arterial wall. The inflammation also extended into the
neighbouring veins and nerves. The most probably diagnosis is
a)Takayasu arteritis
b)Giant cell arteritis
c)Hypersensitivity angiitis
d)Thromboangiitis obliterans
72. The commonest cause of aneurysm formation
is
a) Gun shot injury b) Syphilis
c) Congenital factors d) Atherosclerosis
73. A knitted Dacron artery graft
a)Is not porous
b)Is eventually dissolved by tissue reaction
c)Never gets infected
d)Can be easily incised and the opening resutured
74. The sequence of symptoms in pulmonary embolism is
a)Fever, pain, dyspnoea
b)Fever, dyspnoea
c)Dysponea, pain, haemoptysis
d)Dysponea, cough, purulent sputum
75. Temporary improvement in a patient’s ischaemic foot can be attained by giving
intravenously
a) 10% Mannitol b) 10% Dextrose
c) Dextran 40 d) Dextran 100
76. Diabetic gangrene is due to
a)Ischemia
b)Increased blood glucose
c)Altered defence by host and neuropathy
d)All of the above
77. All are true about Embolic Arterial occlusion except-
a)No previous history
b)Muscles are unaffected
c)Pulse is absent
d)Anaesthesia is present
51
78. All are true about Raynauds phenomena except-
a)Exposure to cold aggravate
b)Spasm of vessels
c)More common is females
d)Atherosclerosis of vessels
79. The commonest site of lodgement of a pulmonary embolus is in the territory at
a) Rt. lower lobe b) Rt. upper lobe
c) Lt. lower lobe d) Lt. upper lobe
80. Kaposi sarcoma is commonly seen in
a) Upper limbs b) Lower limbs
c) Head and Neck d) Trunk
81. Which of the following causes maximum bleeding-
a)Partial arterial severing
b)Complete arterial severing
c)Artery caught between fractured ends of bones
d)Intimal tear
82. In a 40 years old male thrombus in the common femoral artery is because of
a)Atheroma
b)Thrombangits obliterans
c)Reynauds disease
d)Abdominal mass
83. One of the following is not indicated for arterial
leg ulcer
a) Debridement b) Elevation of limb
c) Head end of bed is raised d) Low dose aspirin

84. The commonest cause of arterio-venous fistula is-


a)Penetrating injury
b)Congenital
c)Neoplasmic invasion of an artery and adjacent vein
d)Aneurysm of the artery eroding a vein
85. AV fistula leads to all except
a)Sinus tachycardia
b)Increased preload
c)Cardiac arrythmias
d)Increased cardiac output
86. Commonest peripheral aneurysm is

52
a) Popliteal b) Femoral
c) Carotid d) iliac

87. Popliteal aneurysm-All are true except-


a)Presents as a swelling behind the knee
b)Presents with symptoms due to complication
c)Surgery is indicated in case of complication
d)Uncommon among peripheral aneurysm
88. Graft used in infra inguinal by pass is
a) PTFE b) Dacron
c) Autologous vein d) Autologous artery
89. True about Erythrocyanosis except
a)Affects young girls
b)Cold peripheries
c)Palpable pulses
d)Ulceration & gangrene of fingers
90. Pseudo aneurysms are most commonly due-

a) Atherosclerosis b) Trauma
c) Congenital deficiency d) Infections

91. Bilateral pulseless disease in upper limbs in caused


by
a) Aortoarteritis b) Coarctation of aorta
c) Fibromuscular dysplsia d) Buerger’s disease
92. Dissection of which artery is seen in pregnancy
a) Carotid artery b) Aorta
c) Coronary A d) Femoral artery

93. Peripheral arterial occlusion (Sudden onset) is characterized byall except


a) Paresthesia b) Rubor
c) Pallor d) Pain
94. In which one of the following conditions Dactylitis CANNOT be see
a) Sickle – cell anemia b) Beta thalassemia
c) Congenital syphilis d) Tuberculosis
e) Sarcoidosis
95. Pseudoarterial aneurysm in drug abuser’s seen in –

53
a) Radial b) Brachial
c) Femoral
d) Pedal

96. Lumbar sympathectomy is of value in the management of-


a)Intermittent claudication
b)Distal ischaemia affecting the skin of the toes
c)Arteriovenous Fistula
d)Back pain
97. The most common cause of acquired arteriovenous fistuala is
a) Bacterial infection b) Fungal infection
c) Blunt trauma d) Penetrating trauma
98. Etiopathogenesis of diabetic foot include the following except
a)Myelopathy
b)Osteoarthropathy
c)Microangiopathy
d)Infection
99. Fogarty’s catheter is used for
a)Drainage of urinary bladder
b)Parenteral hyperalimentation
c)Removal of embolus from blood vessels
d)Ureteric catheterisation
100. Perforators are not present at
a) Ankle b) Medial calf
c) Distal to calf d) Below inguinal ligament

101. White leg is due to


a)Femoral vein thrombosis and lymphatic obstruction
b)Deep femoral vein thrombosis
c)Lymphatic obstruction only
d)None of the above
102. All of the following are seen in deep vein thrombosis except
a) Pain b) Discolouration
c) Swelling d) Claudication
103. The following is the commonest site for venous ulcer
a)Instep of foot
54
b)Lower 1/3 leg and ankle
c)Lower 2/3 of leg
d)middle 1/3 of leg
104. The most important perforator of the Lower limb is between
a)Long saphenous and posterior tibial vein
b)Short saphenous and posterior tibial vein
c)Short saphenous and popliteal vein
d)Long saphenous and femoral vein
105. Best method for diagnosis of Deep vein thromvosis is
a)Doppler examination
b)Plethysmography
c)Contrast phlebography
d)1131 Fibrinogen studies
106. Commonest complication of varicose vein stripping is-
a) Thrombo embolism b) Hemorrhage
c) Ecchymosis d) Infection
107. Investigation of choice for diagnosis of deep vein thrombosis
a) Venogram b) Doppler
c) Isotope scan d) Homans sign
108. Pulsating varicose vein in ayoung adult is due to-
a)Arteriovenous fistula
b)Sapheno femoral incompetence
c)Deep vein thrombosis
d)Abdominal tumour

109. Which is not used in treatment of Superficial venous thrombosis


a)Immediate anticoagulation
b)Rest and elevation
c)Analgesics
d)Treat assosiated malignancy
110. An operated case of varicose veins has a recurrence rate of
a) About 10% b) About 25%
c) About 50% d) Over 60%

55
111. Operations for varicose veins are best accomplished by
a)Stripping
b)Multiple subcutaneos ligatures
c)Subfascial ligatures
d)Division and ligation at the superficial venous system
112. A 60-years old male has been operated for carcinoma of caecum and right
hemicolectomy has been done. On the fourth post – oprative day, the patient
develops fever and pain in the legs. The most important clinical entity one should
look for is
a)Urinary tract infection
b)Intravenous line infection
c)Chest infection
d)Deep vein thrombosis
113. All of following may be predisposing factors for deep vein thrombosis except
a) Oral contrceptives b) Nephrotic syndrome
c) Sickle cell anemia d) Thrombocytosis
114. The duration of heparin therapy in deep vein thrombosis is
a) 7 – 10 days b) 15-20 days
c) 3-4 days d) 1 month
115. Cocket & Dodd’s operation is for
a)Saphenofemoral flush ligation
b)Subfascial ligation
c)Deep vein thrombosis
d)Diabetic foot
116. In obstruction of inferior vena cava there is
a)Prominent thoraco epigastric vein
b)Caput medusa
c)Hemorrhoids
d)Esophageal varices
117. Most accurate & non invasive method for diagnosing deep vein thrombosis
a)Doppler duplex
b)Plethesmography
c)Radioactive labelled fibrinogen
d)Angiography
118. Most common site for venous thrombosis –

a) Popliteal vein b) Soleal vein


c) Femoral vein d) Internal iliac vein

119. Deep vein thrombosis is caused by all except –


a)Lower limb trauma
56
b)Hip and pelvic surgery
c)Subungual melanoma
d)Cushing’s syndrome
120. Which of the following test is used to detect perforator incompetence in varicose
a) Trendelenberg test b) Fegan’s test- (localise)
c) Morissey’s test d) Homan’s test
121. The deficiency of all of the following factors increases the incidence of thrombus
formation except
a) Lipoprotein A b) Protein – C
c) Anti – thrombin III d) Protein – S
122. The most common vein to get thrombosed is the
a) Long saphenous b) Short saphenous
c) Both d) Posterior tibial
123. Brodie -Trendlenburg test demonstrates-
a)Mid – thigh perforation
b)Deep vein thrombosis
c)Sapheno — femoral incompetence
d)Calf perforators
124. An intern was doing saphenous cannulation for a burns patient. Then the patient
developed sudden onset of pain along the medial border of the corresponding
foot. Which nerve must have been accidentally ligated

a) Sural nerve b) Deep peroneal nerve


c) Saphenous nerve d) Genicular nerve

125. In DVT all are seen except


a)High fever
b)Increased temperature at site
c)Pain
d)Tenderness
126. An obese patient develops acute oedematous lower limb following a Pelvic
surgery. Deep vein thrombosis is suspected . The most useful investigation in
this case would be
a) Doppler imaging b) Fibrinogen uptake
c) Venography d) Plethysmography
127. In a patient on anticoagulant therapy, the INR is maintained at
a)1.5 to 2.5 times the normal
b)2.5 to 3.5 times the normal
c)3.5 to 4.5 times the normal
d)4.5 to 5.5 times the normal

57
128. The initial therapy of documented deep venous thrombosis in a post operative
case is –
a)Subcutaneous heparin therapy
b)Intravenous heparin therapy
c)Thropmbolytic therapy with urokinase
d)Aspirin therapy
129. Which of the following statements is true regarding fat embolism
a)Most patients with major trauma involving long bones have urinary fat
globules
b)All patients with urinary fat globules develop fat embolism
c)Peak incidence of respiratory insufficiency for pulmonary fat embolism is
around day 7 after injury
d)Heparin as an anticoagulant decreases mortality and morbidity in fat embolism
syndrome
130. Commonest cause of hemobilia is
a) Gall stones b) Trauma
c) Cholangitis d) Hepatoma
131. Which of the following statements related to gastric injury is not true?
A.Mostly related to penetrating trauma
B.Treatment is simple debridement and suturing
C.Blood in stomach is always related to gastric injury
D.Heals well and fast
132. Ten days after a splenectomy for blunt abdominal trauma, a 23-year-old man
complains of upper abdominal and lower chest pain exacerbated by deep
breathing. He is anorectic but ambulatory and otherwise making satisfactory
progress. On physical examination , his temperature is 38.2°C(108°C)rectally,
and he has decreased breath sounds at the left lung base. His abdominal wound
appears to be healing well, bowel sound are active and there are no perito-neal
signs. Rectal examination is negative. The W.B.C. count is 12,500 mm3with a
shift to left. Chest X-rays show platelike atelectasis of the left lung field.
Abdominal X-rays show a nonspecific gas pattern in the bowel and an air-fluid
level in the left upper quadrant. Serum amylase is 150 Somogyi units dl
(normal 60 to 80). The most likely diagnosis is AI 2002
A.Subphrenic abscess
B.Pancreatitis
C.Pulmonary embolism
D.Subfascial wound infection
133. A case of blunt trauma is brought to the emergency, in a state of shock; he is not
responding to IV crystal-loids; next step in his management would be:
A.Immediate laparotomy
58
B.Blood transfusion
C.Albumin transfusion
D.Abdominal compression
134. Which of the following is true about renal trauma
A.Urgent IVP is indicated
B.Exploration of the kidney to be done in all cases
C.Lumbar approach to kidney is preferred
D.Renal artery aneurysm is common
135. A patient sustained Traumatic injury to major abdominal vessels. It has been
planned to explore the
Suprarenal Aorta, the Caeliac Axis, the Superior Mesentric Artery, and the Left
Renal Artery. What maneuvre for exposure is recommended:
A.Cranial visceral Rotation
B.Caudal visceral Rotation
C.Left Medial Visceral Rotation
D.Right Medical Visceral Rotation
136. 30 year old person met with a roadside accident.On admision his pulse rate was
120/minute, BP was 100/60 mmHg.Ultrasonagraphy examination revealed
laceration of the lower pole of spleen and haemoperitoneum. He was
resuscitated with blood and fluid. Two hours later, his pulse was 84/minute and
BP was 120/70 mm Hg. The most appropriate course of management in this
case would be
a)Exploring the patient followed by splenectomy
b)Exploring the patient followed by excision of the lower pole of spleen
c)Splenorrhaphy
d)Continuation of conservatve treatment under close monitoring system
and subsquent surgery if further inficated
137. Which one of the following is not a part of the Revised Trauma score
a)Glasgow coma scale
b)Systolic blood pressure
c)Pulse rate
d)Respiratory rate
138. Following trauma, which hormone is not released—
a) Thyroxine b) Glucagon
c) ADH d) GH
139. In penetrating injury of the abdomen commonly affected is
a) Liver b) Large bowel
c) Duodenum d) Small intestines
140. Death in blunt trauma chest is due to
a) Rupture oesophagus
59
b) Tracheobronchial injury c)Pulmonary contusions
d)Chylothorax
141. A 12 hour old bullet injury to the left colon is ideally treated by
a)Primary closure without drainage
b)Primary closure with drainage
c)Resection of affected segment with upper segment colostomy and lower
segment as a mucus fistula
d)Primary repair with loop colostomy
142. True about burns
a)Hyperglycemia is seen in early burns
b)Child with burns should have damp dressing
c)Chemical powder burns should be kept dry
d)3rd degree burns are painfull
143. Metabolic derangements in severe burns are all except
a)cortico steroid secretion
b)Hyperglycaemia
c)secretion of HCl
d)Neutrophil dysfunction
144. A third degree cirumferential burn in the arm and forearm region, which of the
following is most important for monitoring
a)Blood gases
b)Carboxy-oxygen level
c)Macroglobiunria cryoglobinuria
d)Peripheral pulse and circulation
145. What is the most important aspect of management of burn injury in the first 24
hours ?
a) Fluid resuscitation b) Dressing
c) Escharotomy d) Antibiotics
146. The initial colonization of a burn is by which micro organisms
a) Proteus b) Pseudomonas
c) Staphylococcus d) E. coli
147. Cardiac arrest, ECG changes occurs in
a) Thermal burn b) Electrical burn
c) Cold burn d) Ionising radiation injury
148. Stress ulcers seen in burns are
a) Curling’s ulcer b) Cushing’s ulcer
c) Meleney’s ulcer d) Rodent ulcer
149. ‘Sterile needle test’ helps in differentiating -
a)Healing process
b)Depth of burns
60
c)Degenerative proces
d)Infection
150. Head & neck involvement in burns in infant is -
a) 9% b) 18%
c) 27% d) 32%

151. An adult whose both lower limbs are charred along with genitalia has – - –
-burns
a) 18% b) 19%
c) 36% d) 37%
152. Generalised diffuse peritonitis has been compared to second and third degree
burns of
a) 13% b) 30 %
c) 45% d) 60 %
153. In treatment of hand injuries, the greatest priority is
a)Repair of tendons
b)Restoration of skin cover
c)Repair of nerves
d) Repair of blood vessels
154. During the surgical procedure
a)Tendons should be repaired before nerves
b)Nerves should be repaired before tendons
c)Tendons should not be repaired at the same time
d)None is true
155. In hand injuries first to be repaired is
a) Bone b) Tendon
c) Muscle d) Nerve
156. Which one of the following surgical procedures is considered to have a clean-
contaminated wound ?

a),Elective open cholecystectomy for cholelithiasis


b)Hemiorrhaphy with mesh repair
c)Lumpectomy with axillary node dissection
d)Appendectomy with walled off abscess

157. The Vitamin which has inhibitory effect on wound healing is


a) Vitamin-A b) Vitamin-E
c) Vitamin-C d) Vitamin B-complex

61
158. Golden period for treatment of open wounds is
….hours
a) 4 b) 6
c) 12 d) 24
159. Cause of persistance of a sinus or fistulae includes-
a)Foreign body
b)Non dependentt drainage
c)Unrelieved Obstruction
d)Presence of malignancy
e)All of the above
160. The term universal tumour refers to
a) Adenoma b) Papilloma
c) Fibroma d) Lipoma
161. Which is the commonest incidentaloma detected in the liver
a)Focal nodular hyperplasia
b)Haemangioma
c)Hepatocellular adenoma
d)Hydatid cyst
162. "Crumbled egg appearance" in liver seen in –
a) Hepatic adenoma
b) Chronic amoebic liver abscess c)Hydatid liver disease
d)Haemangioma
163. Earliest tumour to appear after bith is
a) Sternomastoid tumour b) Cystic hygroma
c) Branchial cyst d) Lymphoma
164. All are true statement about malignant melanoma except
a)Clark’s classification used for prognosis
b)Women have better prognosis
c)Acral lentigenous have better prognosis
d)Limb perfusion is used for local treatment
165. Prognosis of malignant melanoma depends on
a) Grade of tumor b) Spread of tumor
c) Depth of invasion d) Metastasis
166. Worst prognosis in Melanoma is seen in the subtype-
a)Superficial spreading
b)Nodular Melanoma
c)Lentigo Maligna Melanoma
d)Amelanotic Melanoma

62
167. Least malignant melanoma is
a) Lentigo maligna b) Superifcial spreading
c) Nodular d) Amelanotic
168. True about melanoma of the anal canal is
a)Present usually as anal bleeding
b)AP resection gives better result than local excision
c)Local recurrence at the same site after resection
d)Radiosensitive

Good Luck

DR NASER RADWAN

63

You might also like